700 105 20MB
English Pages 496 [493] Year 2023
Any screen. Any time. Anywhere. Activate the eBook version of this title at no additional charge.
Elsevier eBooks+ gives you the power to browse, search, and customize your content, make notes and highlights, and have content read aloud.
Unlock your eBook today. 1. Visit http://ebooks.health.elsevier.com/ 2. Log in or Sign up 3. Scratch box below to reveal your code 4. Type your access code into the “Redeem Access Code” box 5. Click “Redeem”
It’s that easy!
Place Peel Off Sticker Here
For technical assistance: email [email protected] call 1-800-545-2522 (inside the US) call +44 1 865 844 640 (outside the US) Use of the current edition of the electronic version of this book (eBook) is subject to the terms of the nontransferable, limited license granted on http://ebooks.health.elsevier.com/. Access to the eBook is limited to the first individual who redeems the PIN, located on the inside cover of this book, at http://ebooks.health.elsevier.com/ and may not be transferred to another party by resale, lending, or other means. 2022v1.0
Review of Surgery for
ABSITE
AND
BOARDS
Review of Surgery for
ABSITE
AND
BOARDS
THIRD EDITION
EDITORS
Christian de Virgilio, MD, FACS
Areg Grigorian, MD
Chair Department of Surgery Harbor-UCLA Meical Center Torrance, California; Co-Chair College of Applie Anatomy; Professor of Surgery UCLA School of Meicine Los Angeles, California
Assistant Clinical Professor of Surgery Department of Surgery Division of Trauma, Burns an Critical Care University of California, Irvine Orange, California
ASSOCIATE EDITORS
Amanda C. Purdy, MD
Eric O. Yeates, MD
Naveen Balan, MD
Surgical Resient Physician Department of Surgery Harbor-UCLA Meical Center Torrance, California
Resient Physician Department of Surgery University of California, Irvine Orange, California
Surgical Resient Department of Surgery Harbor-UCLA Meical Center Torrance, California
ILLUSTRATOR
Stephanie Cohen, MD Surgical Resient Beth Israel Deaconess Meical Center Boston, Massachusetts
1600 John F. Kenney Blv. Ste 1800 Philaelphia, PA 19103-899
REVIEW OF SURGERY FOR ABSITE AND BOARDS, THIRD EDITION
ISBN: 978-0-33-87054-
Copyright © 2023 by Elsevier Inc. All rights reserved. No part of this publication may be reprouce or transmitte in any form or by any means, electronic or mechanical, incluing photocopying, recoring, or any information storage an retrieval system, without permission in writing from the publisher. Details on how to seek permission, further information about the Publisher’s permissions policies, an our arrangements with organizations such as the Copyright Clearance Center an the Copyright Licensing Agency, can be foun at our website: www.elsevier.com/permissions This book an the iniviual contributions containe in it are protecte uner copyright by the Publisher (other than as may be note herein).
Notice Practitioners an researchers must always rely on their own experience an knowlege in evaluating an using any information, methos, compouns, or experiments escribe herein.Because of rapi avances in the meical sciences, in particular, inepenent verication of iagnoses an rug osages shoul be mae. To the fullest extent of the law, no responsibility is assume by Elsevier, authors, eitors, or contributors for any injury an/or amage to persons or property as a matter of proucts liability, negligence or otherwise, or from any use or operation of any methos, proucts, instructions, or ieas containe in the material herein.
Previous eitions copyrighte 018 an 010.
Content Strategist: Jessica McCool Content Development Specialist: Shweta Pant Publishing Services Manager: Shereen Jameel Project Manager: Beula Christopher Design Direction: Ryan Cook
Printe in the Unite States of America. Last igit is the print number: 9 8 7
6 5 4 3
1
To my family, who always support me, and to all students of surgery, who motivate and inspire me to always keep learning the art and science of medicine. —Christian de Virgilio
I would not be where I am today if it wasn't for my mentors. Dr. de Virgilio—you are the reason I love surgical education.Dr. Demetriades, you taught me trauma surgery but more importantly, you taught me how to be an effective and inspiring teacher. Dr. Inaba, you have taught me how to be an effective leader both inside and outside the operating room. Dr.Nahmias, you have taught me how to be an academician and researcher. And to my loving wife, Rebecca Grigorian—a superhero mom and physician! Thank you all! —Areg Grigorian
Contributors Mark Archie, MD Assistant Clinical Professor of Surgery Department of Surgery Harbor-UCLA Medical Center David Geffen School of Medicine at UCLA Los Angeles, California Naveen Balan, MD Surgical Resident Department of Surgery Harbor-UCLA Medical Center Torrance, California Jeremy M. Blumberg, MD Chief of Urology Harbor-UCLA Medical Center; Associate Professor of Urology David Geffen School of Medicine at UCLA Los Angeles, California Nina M. Bowens, MD Assistant Professor Department of Surgery David Geffen School of Medicine at UCLA; Associate Program Director, Vascular Surgery Program Division of Vascular and Endovascular Surgery Harbor-UCLA Medical Center Torrance, California Caitlyn Braschi, MD Resident Physician Department of Surgery Harbor-UCLA Medical Center Torrance, California Formosa Chen, MD, MPH Health Sciences Assistant Clinical Professor Department of Surgery David Geffen School of Medicine at UCLA Los Angeles, California Kathryn T. Chen, MD Assistant Professor Department of Surgery Harbor-UCLA Medical Center Torrance, California Christine Dauphine, MD, FACS Vice Chair, Education Department of Surgery Harbor-UCLA Medical Center Torrance, California; Professor of Surgery David Geffen School of Medicine at UCLA Los Angeles, California
Christian de Virgilio, MD, FACS Chair Department of Surgery Harbor-UCLA Medical Center Torrance, California; Co-Chair College of Applied Anatomy; Professor of Surgery UCLA School of Medicine Los Angeles, California Benjamin DiPardo, MD Resident Department of Surgery UCLA Los Angeles, California Richard Everson, MD Assistant Clinical Professor of Surgery Department of Surgery Harbor-UCLA Medical Center David Geffen School of Medicine at UCLA Los Angeles, California Mytien Goldberg, MD Assistant Clinical Professor of Surgery Department of Surgery Harbor-UCLA Medical Center David Geffen School of Medicine at UCLA Los Angeles, California Areg Grigorian, MD Assistant Clinical Professor of Surgery Department of Surgery Division of Trauma, Burns and Critical Care University of California, Irvine Orange, California Joseph Hadaya, MD, PhD Resident Physician Department of Surgery David Geffen School of Medicine at UCLA Los Angeles, California Danielle M. Hari, MD, FACS Division Chief, Surgical Oncology Department of Surgery Harbor-UCLA Medical Center; Associate Professor Department of Surgery David Geffen School of Medicine at UCLA Los Angeles, California
vii
viii
Contributors
Dennis Kim, MD Trauma Medical Director Island Health Trauma Services Victoria, British Columbia, Canada Catherine M. Kuza, MD, FASA Assistant Professor Department of Anesthesiology, Division of Critical Care Keck School of Medicine of the University of Southern California Los Angeles, California Steven L. Lee, MD, MBA Professor and Chief Pediatric Surgery UCLA Mattel Children’s Hospital Los Angeles, California John McCallum, MD, MPH Assistant Professor Department of Surgery Harbor-UCLA Medical Center Torrance, California Michael A. Mederos, MD Resident Physician Department of Surgery UCLA Los Angeles, California Alexandra Moore, MD Surgery Resident Department of Surgery UCLA Los Angeles, California Jeffry Nahmias, MD, MHPE Associate Professor Department of Surgery University of California, Irvine Orange, California Kristofer E. Nava, MD Department of General Surgery Western Michigan University Homer D. Stryker School of Medicine Kalamazoo, Michigan Junko Ozao-Choy, MD, FACS Vice Chair, Research Department of Surgery Harbor-UCLA Medical Center Torrance, California; Associate Professor of Surgery David Geffen School of Medicine at UCLA Los Angeles, California Joon Y. Park, MD Surgery Resident Department of Surgery David Geffen School of Medicine at UCLA Los Angeles, California
Beverley A. Petrie, MD, FACS, FASCRS Professor of Surgery Department of Surgery David Geffen School of Medicine at UCLA Los Angeles, California; Assistant Chief Division of Colon and Rectal Surgery, Department of Surgery Harbor-UCLA Medical Center Torrance, California Amanda C. Purdy, MD Surgical Resident Physician Department of Surgery Harbor-UCLA Medical Center Torrance, California Shonda L. Revels, MD, MS Assistant Professor Department of Surgery UCLA Los Angeles, California Jordan M. Rook, MD Resident Physician Department of Surgery David Geffen School of Medicine at UCLA Los Angeles, California Saad Shebrain, MBBCh, MMM, FACS Program Director, Associate Professor of Surgery Department of Surgery Western Michigan University Homer Stryker M.D. School ofMedicine Kalamazoo, Michigan Eric R. Simms, MD Chief Division of General, Bariatric and Minimally Invasive Surgery, Assistant Program Director of Surgery Residency Department of Surgery Harbor-UCLA Medical Center Torrance, California Veronica Sullins, MD Assistant Clinical Professor of Surgery Department of Pediatric Surgery David Geffen School of Medicine at UCLA Los Angeles, California Maria G. Valadez, MD General Surgery Resident Department of Surgery Harbor-UCLA Medical Center Torrance, California Luis Felipe Cabrera Vargas, MD, MACC, FACS, MACCVA, MFELAC President of the Future Surgeons Chapter of the Colombian Surgery Association, Professor of the Universidad Javeriana and Universidad El Bosque, Fellow of Vascular Surgery of the Universidad Militar Nueva Granada Bogotá, Colombia
Contributors Zachary N. Weitzner, MD Resident Physician Department of Surgery UCLA Los Angeles, California
Eric O. Yeates, MD Resident Physician Department of Surgery University of California, Irvine Orange, California
James Wu, MD Assistant Clinical Professor of Surgery Department of Surgery UCLA Medical Center David Geffen School of Medicine at UCLA Los Angeles, California
Amy Kim Yetasook, MD MIS and Bariatric Surgeon Department of Surgery Harbor-UCLA Medical Center Torrance, California; Assistant Professor Department of Surgery David Geffen School of Medicine at UCLA Los Angeles, California
Tajnoos Yazdany, MD Vice Chair of Education Program Director, Obstetrics and Gynecology Residency, Chief and Program Director, Female Pelvic Medicine and Reconstructive Surgery, Associate Professor David Geffen School of Medicine at UCLA Harbor-UCLA Medical Center Torrance, California
ix
Foreword It is an honor to write the forewor to the thir eition of Review of Surgery for ABSITE and Boards by one of the foremost surgical eucators of our time, Dr. Christian e Virgilio. This book grew out of his initial informal attempts to improve ABSITE scores among his own resients at Harbor-UCLA. Over the years, this effort has grown an expane, incluing collaborators from multiple institutions, to prouce a book that has become an essential tool in the surgical resient’s armamentarium. The most valuable aspect of this book, in my humble opinion, is that in aition to questions testing pure “iactic” knowlege—factois the resient is expecte to learn by rote an memorize—there are many clinical questions that require an avance level of cognitive effort. Here, the learner is expecte to synthesize anatomic an physiologic knowlege within a clinical context an exercise surgical jugment base on probabilities of ifferent outcomes. Too often, books specically targete at passing multiple-choice examinations ten to skip the latter, in favor of questions that have easy answers—hence the common surgical aphorism that there are more exam questions on the clinical presentation of MEN- synrome than patients with this isease! Writing questions that test esoteric minutiae is easy; writing questions that promote further reaing an stuy of complex surgical scenarios is much harer. I applau Dr. e Virgilio an his colleagues for reaching this higher goal, while still incluing the “knowlege-regurgitation” questions that are an inevitable part of the stanarize exam process. Each question is followe by a thoughtful explanation of the right answer, with accompanying
references, to provie a brief summary of essential relevant knowlege. The newest eition also inclues a summary of “high-yiel” principles at the beginning of each chapter, which will further enhance the goal of rapi issemination of essential information on a given topic. In aition to serving as a valuable training tool for the in-service examination, it is our hope that this book will also inspire the resient to augment their learning by elving into relevant sections of textbooks an online resources, incluing vieos an pocasts—all part an parcel of the total eucational package freely available to moern surgical trainees. The breath an epth of multimeia eucation available toay is enormous, compare to what I ha as a resient; conversely, the volume of knowlege an technical skills new surgeons are expecte to learn an master has also increase signicantly. The oubling of scientic knowlege, in meicine an surgery, is now occurring at an exponential pace, an we nee all the help we can get to keep up! I am grateful to Dr. e Virgilio an his colleagues for continuing to invest the effort necessary to upate this wonerful book, so it can continue to serve as a vital resource for present an future surgeons. Sharmila Dissanaike, MD, FACS, FCCM Peter C. Canizaro Chair, University Distinguishe Professor of Surgery, Texas Tech University Health Sciences Center Lubbock, Texas
xi
Preface We are thrille about this thir eition of Review of Surgery for ABSITE and Boards, create to help stuents of surgery prepare for the American Boar of Surgery In-Training (ABSITE) an the American Boar of Surgery (ABS) Qualifying (written) Examination. The original inspiration for the book stemme from a surgery review program we evelope at Harbor-UCLA Meical Center, esigne to stimulate the resients to rea, improve performance on the ABSITE, an enhance their likelihoo of passing the ABS examinations on the rst try. We were inspire to hear that the rst two eitions prove to be a valuable resource. With that in min, we have strive to make the 3r eition even better with some exciting upates an changes. Areg Grigorian an I have ae three new Assistant Eitors to our team, Drs. Amana Pury, Eric Yeates, an Naveen Balan. All are surgical resients; Drs. Pury an Balan at Harbor-UCLA an Dr. Yeates at UC Irvine. We hanpicke them because of their outstaning recor of accomplishment in test taking an question writing an their emonstrate strong interest in surgical eucation. We have also ae numerous resients an surgical eucators from aroun the country (an even one from Colombia) as contributing authors. Another important new feature is that we ae a summary of high-yiel information at the beginning of each chapter. We feel this will serve as a rapi-re way to brush up on key points. We have also ae new, high-yiel questions to remain up-to-ate with the ever-changing an ynamic el of surgery.
Finally, we have ae illustrations from an increibly talente surgical illustrator, Dr. Stephanie Cohen, who is a surgical resient at Beth Israel Deaconess. We love her work so much that we aske her to make a rawing for the cover! The cover illustration, which combines elements of art, music, an anatomy, remins us that Surgery is both an art an a science. To master the arts requires tremenous eication. Excellent surgical knowlege is one characteristic that is paramount to becoming an outstaning surgeon. This requires a lifelong commitment to reaing an then testing your knowlege. We believe that the ieal way to acquire knowlege is to create a year-roun reaing program. Strive to rea aily, even if just for 15 minutes. As with the original version, we believe that the greatest value of our book lies in the esign of the questions an the robust responses. The questions are intene to make you think (try not to get frustrate if you miss many of them!). We provie in-epth explanations for why we feel the correct answer is right an why the incorrect answers are wrong. Please be aware that no textbook or review book has all the answers. Some questions an answers may be controversial. If you isagree with a question or think you foun an error, we woul love to hear back from you (our emails are cevirgilio@lunquist.org an [email protected]u). We sincerely hope you n our review book useful. Christian de Virgilio and Areg Grigorian
xiii
Acknowledgments We would like to acknowledge the efforts of Elsevier for the timely preparation and publication of this review book, in particular Jessica McCool, Content Strategist, who helped with the development of this book and supported it throughout production, and the contributions made by Shweta Pant, Senior Content Development
Specialist, Beula Christopher, Senior Project Manager, and Ryan Cook, Book Designer. In addition, we would like to thank the surgery faculty and residents at HarborUCLA and UC Irvine Medical Centers who assisted in the production and inspiration of this project.
xv
Contents
PArt i: PATIENT CARE 1.
Abdomen—General, 1 Naveen Balan, Areg Grigorian, and Christian de Virgilio
2.
Abdomen—Hernia, 9 Amanda C. Purdy and Amy Kim Yetasook
3.
Abdomen—Biliary, 19 Amanda C. Purdy and Danielle M. Hari
4.
Abdomen—Liver, 33 Naveen Balan, Kathryn T. Chen, and Danielle M. Hari
5.
Abdomen—Pancreas, 47 Joon Y. Park and Danielle M. Hari
6.
Abdomen—Spleen, 65 Maria G. Valadez, Benjamin DiPardo, and Eric R. Simms
7.
Alimentary Tract—Esophagus, 75 Amanda C. Purdy and Eric R. Simms
8.
Alimentary Tract—Stomach, 87 Naveen Balan, Amy Kim Yetasook, and Kathryn T. Chen
9.
Alimentary Tract—Small Bowel, 105 Zachary N. Weitzner, Formosa Chen, and Beverley A. Petrie
10. Alimentary Tract—Large Intestine, 123 Joseph Hadaya, Formosa Chen, and Beverley A. Petrie
11. Alimentary Tract—Anorectal, 143 Michael A. Mederos, Formosa Chen, and Beverley A. Petrie
12. Breast, 155 Naveen Balan, Junko Ozao-Choy, and Christine Dauphine
13. Endocrine Surgery, 175 Michael A. Mederos and James Wu
14. Skin and Soft Tissue, 197 Eric O. Yeates, Areg Grigorian, and Christian de Virgilio
15. Surgical Critical Care, 205 Eric O. Yeates and Dennis Kim
16. Trauma, 219 Naveen Balan, Caitlyn Braschi, and Dennis Kim
xvii
xviii
ContEnts
17. Vascular—Arterial, 245 Amanda C. Purdy and Nina M. Bowens
18. Vascular—Venous, 265 Amanda C. Purdy and John McCallum
19. Vascular—Access, 275 Luis Felipe Cabrera Vargas, Mark Archie, and Christian de Virgilio
20. Transplant, 283 Joseph Hadaya, Areg Grigorian, and Christian de Virgilio
21. Thoracic Surgery, 291 Jordan M. Rook and Shonda L. Revels
22. Pediatric Surgery, 305 Alexandra Moore, Veronica Sullins, and Steven L. Lee
23. Plastic Surgery, 321 Amanda C. Purdy and Mytien Goldberg
24. Genitourinary, 327 Amanda C. Purdy and Jeremy M. Blumberg
25. Gynecology, 339 Amanda C. Purdy and Tajnoos Yazdany
26. Head and Neck, 347 Zachary N. Weitzner and James Wu
27. Nervous System, 357 Eric O. Yeates and Richard Everson
PArt ii: MEDICAL KNOWLEDGE 28. Anesthesia, 365 Eric O. Yeates and Catherine M. Kuza
29. Fluids, Electrolytes, and Acid-Base Balance, 379 Jordan M. Rook, Areg Grigorian, and Christian de Virgilio
30. Immunology, 391 Kristofer E. Nava and Saad Shebrain
31. Infection and Antimicrobial Therapy, 397 Eric O. Yeates and Jeffry Nahmias
32. Nutrition and Metabolism, 409 Eric O. Yeates, Areg Grigorian, and Christian de Virgilio
33. Oncology and Tumor Biology, 417 Alexandra Moore, Areg Grigorian, and Christian de Virgilio
34. Pharmacology, 427 Eric O. Yeates, Areg Grigorian, and Christian de Virgilio
35. Preoperative Evaluation and Perioperative Care, 437 Naveen Balan, Areg Grigorian, and Christian de Virgilio
36. Transfusion and Disorders of Coagulation, 451 Caitlyn Braschi, Joon Y. Park, and Eric R. Simms
37. Wound Healing, 465 Eric O. Yeates, Areg Grigorian, and Christian de Virgilio
PART I
PATIENT CARE
Abdomen—General NAVEEN BALAN, AREG GRIGORIAN, AND CHRISTIAN DE VIRGILIO
1
ABSITE 99th Percentile High-Yields I. Enhance recovery after surgery (ERAS) – associate with a lower overall complication rate, although there is no ifference in surgical complications or mortality A. Preoperative optimization 1. Inclues preamission patient eucation on analgesia management after OR, control of meical comorbiities, smoking cessation, prehabilitation, nutritional care, an correction of anemia . Ieal patient is ASA 1 or , ambulatory, goo nutritional status; absolute contrainication is urgent surgery, ASA 4–6, severely malnourishe, or immobile B. Intraoperative management 1. Stanar anesthesia protocol, minimizing intraoperative uis, preventing intraoperative hypothermia, maintain normal serum glucose, minimally invasive approach (when feasible), avoi routine use of rains C. Postoperative care 1. Avoi routine use of nasogastric (NG) tubes, multimoal analgesia to minimize opioi use, use of epiurals in laparotomy cases, use of TAP (transversus abominis plane) blocks, early urinary catheter iscontinuation, an early mobilization
QUESTIONS 1. A 56-year-ol male unergoes laparoscopic peritoneal ialysis (PD) catheter placement. Several months later the patient comes to the emergency epartment reporting problems with his PD catheter. He reports that he can instill ialysate without ifculty but is unable to withraw ui through the catheter. His abomen is istene an he has mil abominal pain. He is afebrile an not tachycaric. What is the next best step? A. Prompt removal of PD catheter B. Abominal x-ray C. Instill tPA through the catheter D. Intraperitoneal antibiotics E. Intravenous antibiotics
2. A 4-year-ol male unergoes laparotomy for an anterior abominal stab woun with peritoneal violation. A small perforation of the transverse colon is repaire primarily. While examining the small bowel, an antimesenteric iverticulum is foun 10 cm proximal from the ileocecal junction. It is 3 cm in iameter, 3 cm in height, an there is a brous ban extening from the iverticulum to the abominal wall. There is no palpable abnormality ajacent to the iverticulum an no evience or history of GI bleeing. What is the appropriate management of the iverticulum? A. Obtain aitional imaging postoperatively B. Diverticulectomy C. Biopsy D. Observation E. Segmental resection 1
2
PArt i Patient Care
3. Which of the following is true about intraabominal hypertension (IAH) an abominal compartment synrome (ACS)? A. Diagnosis of ACS is establishe when intraabominal pressure is greater than 0 mmHg B. Intraabominal hypertension is ene as intraabominal pressure >1 mmHg C. Neuromuscular blockae reuces mortality in patients with ACS D. Paracentesis is contrainicate in patients with IAH E. Cerebral perfusion is increase in ACS 4. Which of the following is true regaring omental torsion? A. Seconary torsion is more common than primary B. If surgery is necessary, management consists of etorsion an omentopexy C. Treatment is usually observation with pain control D. The pain is usually in the left lower quarant of the abomen E. It typically prouces purulent-appearing peritoneal ui 5. The most common organism isolate from the infecte peritoneal ui of a patient with a PD catheter is: A. Beta-hemolytic streptococcus B. Enterococcus C. Escherichia coli D. Coagulase-negative staphylococcus E. Coagulase-positive staphylococcus 6. A 70-year-ol woman presents with progressive abominal pain an abominal istention with nonshifting ullness. A CT scan emonstrates loculate collections of ui an scalloping of the intraabominal organs. At surgery, several liters of yellowish-gray mucoi material are present on the omentum an peritoneal surfaces. Which of the following is true about this conition? A. There is no role for surgical resection B. It is most commonly of ovarian origin C. There is a strong genetic inuence D. It is more common in males E. Cytoreuctive surgery may be of benet 7. The most common cause of a retroperitoneal abscess is: A. Diverticulitis B. Appenicitis C. Renal infection D. Tuberculosis of the spine E. Hematogenous sprea from a remote location
8. A 50-year-ol male with cirrhotic ascites seconary to hepatitis C presents with fever, elevate white bloo cell count, an abominal pain. He has a history of esophageal varices. He has been on the liver transplant list for 6 months. Paracentesis was performe an cultures were sent. A single organism grows from the culture. Which of the following is true regaring this conition? A. It is most likely ue to appenicitis B. Prophylactic use of uoroquinolone can be use to prevent this conition C. In aults, nephrotic synrome is the most common risk factor D. In chilren, E. coli is the most common isolate E. He will likely nee an exploratory laparotomy 9. A 74-year-ol male presents to clinic hoping to have his reucible umbilical hernia repaire seconary to increasing but intermittent pain an iscomfort. Two ays before his clinic visit, he ha been ischarge from the hospital for unstable angina, for which he unerwent balloon angioplasty with placement of a bare metal coronary artery stent (BMS). When shoul his surgery be scheule? A. weeks B. 1 month C. months D. 6 months E. 1 year 10. Which of the following is true regaring abominal incisions an the prevention of incisional hernias? A. A 4:1 suture:woun length is the current recommene closure length B. There is no ifference in hernia occurrence between a running closure an an interrupte closure C. A permanent monolament suture is preferre in the closure of the fascia in a running fashion D. Prophylactic use of mesh after open aortic aneurysm surgery is not efcacious E. A 1-cm bite between each stitch is the recommene istance uring abominal closure
CHAPtEr 1 Abdomen—General
11. A 55-year-ol obese male presents to the hospital for his bariatric sleeve gastrectomy proceure. His comorbiities inclue iabetes an hypertension, an he states he was iagnose with “walking pneumonia” weeks ago an place on antibiotics, which he has nishe. Which of the following woul not be benecial if the SCIP measures for preoperative an postoperative care are followe? A. Placing the patient on an insulin sliing scale to keep glucose levels between 80 an 10mg/L B. Clipping the patient’s abominal hair with an electric shaver before operating C. Aministering anticoagulation on postoperative ay 1 D. Aministering antibiotics within 1 hour of surgery E. Discontinuing antibiotics by postoperative ay 1 12. A 3-year-ol female who is 4 weeks pregnant presents to the emergency epartment with acute onset of abominal pain, fever, an vomiting. She states that the pain woke her up in the mile of the night with suen onset of epigastric pain that is now iffuse. She has no vaginal bleeing an fetal monitoring emonstrates normal vitals for the fetus. Upon physical exam, the patient has iffuse tenerness with guaring throughout the abomen, worse in the epigastric region. Pelvic examination is normal. She has a leukocytosis of 15,000 cells/L. Abominal x-ray series shows some ilate bowel loops but no other nings. What is your next step in management of this patient? A. Abominal ultrasoun B. CT scan of the abomen/pelvis with contrast C. Amit an observe with serial abominal exams D. Exploratory laparotomy E. Diagnostic laparoscopy 13. Which of the following is true regaring a rectus sheath hematoma? A. If locate above the umbilicus, it is more likely to resemble an acute intraabominal process B. If locate below the umbilicus, it is more likely to cause severe bleeing C. The majority are associate with a history of trauma D. Operative rainage is the treatment of choice in most cases E. Angiographic embolization is not useful
3
14. A woman presents with a rm, enlarging mass on her abominal wall. After appropriate workup, she is iagnose with a esmoi tumor. Which of the following is true about this conition? A. There is a high rate of metastasis without proper treatment B. The chance of local recurrence is low after appropriate intervention C. These tumors ten to enlarge uring menopause D. They occur most commonly in women after chilbirth E. These tumors arise from proliferative chonroblastic cells 15. Which of the following is true regaring retroperitoneal sarcomas? A. They are best manage by enucleation B. Prognosis is best etermine by histologic grae C. Fibrosarcomas are the most common type D. Lymph noe metastasis is common E. Raiation therapy is often curative for small sarcomas 16. A 75-year-ol female with recently iagnose atrial brillation, for which she was given an anticoagulant, presents with suen onset abominal pain unrelate to oral intake. Surgical history is remarkable for a total hip arthroplasty 3 years ago. Her physical exam is signicant for a tener, palpable abominal wall mass above the umbilicus that persists uring exion of abominal wall muscles. The mass is most likely relate to which of the following? A. A malignancy B. Bleeing from the superior epigastric artery C. Occult trauma D. An intraabominal infection E. Bleeing from the inferior epigastric artery
4
PArt i Patient Care
ANSWERS 1. B. PD catheters can become malpositione postopera-
3. B. IAH is ene as an intraabominal pressure
merman D. Fluoroscopic manipulation of peritoneal ialysis catheters: outcomes an factors associate with successful manipulation. Clin J Am Soc Nephrol. 01;7(5):795–800.
much oes ecompressive laparotomy reuce the mortality rate in primary abominal compartment synrome?: a singlecenter prospective stuy on 66 patients. Medicine (Baltimore). 017;96(5):e6006.
2. B. This patient has a Meckel iverticulum. This is a
4. A. It is important to be aware of omental torsion because
ticulum in aults: surgical concerns.Front Surg. 018;5:55.
omental torsion in chilren. J Pediatr Surg. 1995;30(6):816–817.
tively espite intraoperative conrmation of proper placement. Instilling ialysate in the peritoneal cavity without the ability to remove it may lea to abominal istention an mil pain. The rst step for a suspecte malpositione PD catheter that may have been ippe or kinke is to obtain a KUB. If the catheter appears malpositione, then a reasonable next step woul be to return to the OR for iagnostic laparoscopy to reposition the catheter. For catheters that are clogge (resistance to instilling ialysate through the catheter or inability to instill ui), tPA can be use (C). Omentopexy or omentectomy can also be helpful in cases of a malfunctioning catheter ue to obstruction. Peritonitis is a common complication of PD an accounts for 50% of technical failures. This complication presents with abominal pain, fever, an clouy ialysate. The initial management involves intraperitoneal antibiotics, most commonly vancomycin, which cures 75% of cases without iscontinuation of PD (D). Patients who continue to become increasingly septic may require intravenous (IV) antibiotics as well (E). Any fungal infection of PD requires prompt removal of the catheter (A). Reference: Miller M, McCormick B, Lavoie S, Biyani M, Zim-
true intestinal iverticulum that results from the failure of the vitelline uct to obliterate uring the fth week of fetal evelopment. It is the most common congenital anomaly of the GI tract. Pancreatic heterotopia is foun in a minority of cases. The most common heterotopic tissue foun in resecte specimens is gastric mucosa, which can lea to ulcer formation an GI bleeing. Meckel with gastric mucosa is locate at the antimesenteric borer; however, ulceration occurs in the opposite mesenteric borer of the ileum. Symptomatic cases require surgical intervention. The management of an incientally iscovere asymptomatic Meckel iverticulum uring abominal exploration is a controversial topic. Recently, it has been suggeste to selectively intervene on patients with risk factors, namely age cm in iameter, presence of heterotopic tissue, palpation of abnormal noules, or presence of brous bans. This patient has three inications for removal incluing age 1 mmHg. This is assesse by measuring the blaer pressure while the patient is paralyze. ACS is ene by IAH >0 mmHg AND evience of en-organ malperfusion (i.e., oliguria) (A). Patients who are mechanically ventilate often have high peak pressures. Primary ACS occurs most commonly after surgical proceures associate with massive resuscitation an tense fascial closure. Seconary ACS is ue to meical conitions such as ascites or conitions requiring resuscitation without an abominal proceure (i.e., signicant burn injury). Nasogastric ecompression an neuromuscular blockae are conservative measures to treat IAH but neither has been proven to signicantly reuce mortality (C). Reucing IAH with paracentesis shoul be performe rst in seconary ACS ue to ascites (D). In refractory cases an all other cases of ACS, ecompressive laparotomy shoul be performe expeitiously to lower mortality. The pathophysiology of ACS involves compression of the IVC, which can lea to elevate SVC pressures, an in turn increase intracranial pressures resulting in ecrease cerebral perfusion pressures (E). Reference: Muresan M, Muresan S, Brinzaniuc K, et al. How
it reaily mimics an intraabominal perforation. Because it is typically very ifcult to iagnose preoperatively, the iagnosis is most often mae at surgery. Torsion of the omentum escribes a twisting of the omentum aroun its vascular peicle along the long axis. Primary torsion, in which case there is no unerlying pathology, is extremely rare. Seconary torsion is much more common, an the torsion is usually precipitate by a xe point such as a tumor, an ahesion, a hernia sac, or an area of intraabominal inammation. Omental torsion is much more common in aults in their fourth or fth ecae of life. Chilren with torsion are typically obese, likely contributing to a fatty omentum that preisposes to twisting. Other factors that preispose a patient to torsion inclue a bi omentum an a narrowe omental peicle. In primary omental torsion, the twiste omentum tens to be localize to the right sie; thus, it is most commonly confuse with acute appenicitis, acute cholecystitis, an pelvic inammatory isease (D). Complicating the iagnosis is the fact that the omentum itself tens to migrate an envelop areas of inammation. Laparoscopy is ieal for establishing the iagnosis an excluing other etiologies. Treatment is to resect the twiste omentum, which can often be infarcte at the time of surgery, an to correct any other relate conition that may be ientie (B, C). The ning of purulent ui woul suggest another iagnosis because it is not consistent with omental torsion. The ui usually seen is serosanguinous (E). References: Chew DK, Holgersen LO, Frieman D. Primary
CHAPtEr 1 Abdomen—General Sánchez J, Rosao R, Ramírez D, Meina P, Mezquita S, Gallaro S. Torsion of the greater omentum: treatment by laparoscopy. Surg Laparosc Endosc Percutan Tech. 00;1(6):443–445. Young TH, Lee HS, Tang HS. Primary torsion of the greater omentum. Int Surg. 004;89():7–75.
5. D. Coagulase-negative staphylococci (Staphylococcus epi-
dermidis) is by far the most common cause of peritoneal catheter–relate infections (A–C). Staphylococcus aureus is coagulase positive (E). Another ening feature of S. aureus is that it is catalase positive. The iagnosis is mae by a combination of abominal pain, evelopment of clouy peritoneal ui, an an elevate peritoneal ui white bloo cell count greater than 100/mm3. Initial treatment consists of intraperitoneal antibiotics, which seem to be more effective than IV antibiotics for a total of weeks. If the infection fails to clear base on abominal examination, clinical picture, or persistent peritoneal ui leukocytosis, then the catheter nees to be remove an a temporary hemoialysis catheter will nee to be inserte. S. aureus an gram-negative organism infections are less likely to respon to antibiotic management alone.
6. E. Pseuomyxoma peritonei is a rare process in which
the peritoneum becomes covere with semisoli mucus an large loculate cystic masses. There is no familial preisposition (C). A useful classication erive from a large series uses two categories: isseminate peritoneal aenomucinosis (DPAM) an peritoneal mucinous carcinomatosis (PMCA). DPAM is histologically a benign process an is most often ue to a rupture appenix. In one large series, appeniceal mucinous aenoma was associate with approximately 60% of patients with DPAM. In patients classie as PMCA, the origin was either a well-ifferentiate appeniceal or intestinal mucinous aenocarcinoma (B). Pseuomyxoma peritonei is most common in women age 50 to 70 years (D). It is often asymptomatic until late in its course. Symptoms are often nonspecic, but the most common symptom is increase abominal girth. Physical examination may emonstrate a istene abomen with nonshifting ullness. Management is surgical, with cytoreuction of the primary an seconary implants, incluing peritonectomy an omentectomy (A). If there is a clear origin at the appenix, a right colectomy shoul also be performe. If the origin appears to be the ovary, total abominal hysterectomy with bilateral salpingo-oophorectomy is recommene. The recurrence rate is very high (76% in one series). References: Gough D, Donohue J, Schutt AJ, et al. Pseuomyxoma peritonei: long-term patient survival with an aggressive regional approach. Ann Surg. 1994;19():11–119. Hinson FL, Ambrose NS. Pseuomyxoma peritonei. Br J Surg. 1998;85(10):133–1339. Ronnett BM, Zahn CM, Kurman RJ, Kass ME, Sugarbaker PH, Schmookler BM. Disseminate peritoneal aenomucinosis an peritoneal mucinous carcinomatosis: a clinicopathologic analysis of 109 cases with emphasis on istinguishing pathologic features, site of origin, prognosis, an relationship to “pseuomyxoma peritonei.” Am J Surg Pathol. 1995;19(1):1390–1408.
7. C. Primary retroperitoneal abscesses are seconary to
hematogenous sprea while seconary retroperitoneal abscesses are relate to an infection in an ajacent organ. The most common source of retroperitoneal abscesses is seconary, with renal infections accounting for nearly 50% of all
5
cases. Hematogenous sprea is not a signicant contributing factor for seconary retroperitoneal abscesses (E). Other common causes inclue retrocecal appenicitis (B), perforate uoenal ulcers, pancreatitis, an iverticulitis (A). In rare cases, patients may have Pott isease, which is a isseminate form relate to tuberculosis (D). Patients typically present with back, pelvic, ank, or thigh pain with associate fever an leukocytosis. Flank erythema may be present. Kiney infections often have gram-negative ros such as Proteus an E. coli. Treatment consists of broa-spectrum antibiotics an rainage, an ientication of the source. If the abscess is simple an unilocular, then CT-guie rainage is the treatment of choice. Operative rainage may be require for complex abscesses.
8. B. Spontaneous (primary) bacterial peritonitis (SBP) is
ene as bacterial infection of ascitic ui in the absence of any surgically treatable intraabominal infection. Patients usually present with fever, iarrhea, an abominal pain, but if severe enough, they will also have altere mental status, hypotension, hypothermia, an a paralytic ileus. However, 13% of patients will be completely asymptomatic. Treatment is with antibiotics alone. Prophylactic antibiotics (with uoroquinolones) to prevent SBP shoul be consiere in high-risk patients with cirrhosis, ascites, an history of gastrointestinal bleeing (as in the present case). Patients with cirrhosis who have low ascitic ui protein (10 ras) exposure early in pregnancy (within the rst 4 weeks) can lea to fetal emise. However, such a high exposure excees the ose of typical imaging (abominal x-ray is 00 mra while abominal an pelvic CT is about 3–4 ras). Between 8 an 15 weeks’ gestation, high-ose (>10 ras) raiation can lea to intrauterine growth retaration an central nervous efects. Beyon 15 weeks (as in the present case), there o not appear to be any eterministic effects (ose-epenent events such as fetal loss, congenital efects) on the fetus. Stochastic effects (those that are not ose epenent), such as the subsequent risk of cancer or leukemia, are increase with exposure of 1 ra or more. The risk is about 1 cancer for every 500 exposures. Conversely, if the pregnant patient with an acute abomen progresses to peritonitis an bowel perforation, the risk of fetal emise is very high. Thus, the risk of fetal miscarriage is higher with visceral perforation than with raiation exposure, an therefore all measures shoul be taken for an accurate iagnosis. Magnetic resonance imaging (MRI) is consiere a goo imaging option in pregnancy; however, its use in the emergent setting may be limite by its availability. Ultrasoun is also useful but woul be more useful if the patient presente with right upper quarant pain (suspecte biliary isease) or right lower quarant pain (suspecte appenicitis).
CHAPtEr 1 Abdomen—General Reference: Khanelwal A, Fasih N, Kielar A. Imaging of acute abomen in pregnancy. Radiol Clin North Am. 013;51(6):1005–10.
13. B. Rectus sheath hematomas are clinically signicant
because of the fact that they can easily be mistaken for an intraabominal inammatory process. The etiology is an injury to an epigastric artery within the rectus sheath. In most cases, there is no clear history of trauma (C). Particularly in the elerly who are taking oral anticoagulants, these typically occur spontaneously. Patients frequently escribe a suen onset of unilateral abominal pain, sometimes precee by a coughing t. In one series, 11 of 1 patients were women, an in another series, all 8 were women, with an average age in the sixth ecae. Below the arcuate line, there is no aponeurotic posterior covering to the rectus muscle. Therefore, hematomas below this line can cross the miline, causing a larger hematoma to form, an then cause bilateral lower quarant pain resembling a perforate viscus. On physical examination, a mass is often palpable. The Fothergill sign is the ning of a palpable abominal mass that remains unchange with contraction of the rectus muscles. This helps istinguish it from an intraabominal abscess, which woul not be palpable with rectus contraction. The iagnosis is best establishe with a CT scan, which will emonstrate a ui collection in the rectus muscle. The hematocrit shoul be closely monitore. Once the iagnosis is establishe, management is primarily nonoperative an consists of resuscitation, monitoring of serial hemoglobin/hematocrit levels, an reversal of anticoagulation (D). However, one shoul be cautious with reversal of anticoagulation, as stable patients may benet from continue anticoagulation (e.g., recent mechanical valve). On rare occasions, angiographic embolization may be necessary (E). Surgical management, while rarely necessary, woul involve ligation of the bleeing vessel an evacuation of the hematoma. References: Berná JD, Zuazu I, Marigal M, García-Meina V, Fernánez C, Guirao F. Conservative treatment of large rectus sheath hematoma in patients unergoing anticoagulant therapy. Abdom Imaging. 000;5(3):30–34. Zainea GG, Joran F. Rectus sheath hematomas: their pathogenesis, iagnosis, an management. Am Surg. 1988;54(10):630–633.
14. D. Desmoi tumors are unusual soft-tissue neoplasms
that arise from fascial or bro-aponeurotic tissue. They are proliferations of benign-appearing broblastic cells with abunant collagen an few mitoses (E). Desmoi tumors o not metastasize (A); however, they are locally aggressive an have a very high local recurrence rate reaching almost 50% (B). They have been associate with Garner synrome (intestinal polyposis, osteomas, bromas, an epiermal or sebaceous cysts) an familial aenomatous polyposis (FAP), which is why patients shoul be scheule for a colonoscopy soon after iagnosis. In sporaic cases, surgical trauma appears to be an important cause. Desmoi tumors may evelop within or ajacent to surgical scars. Patients with FAP have a 1000-fol increase risk of the evelopment of esmoi tumors. Desmois are more common in women of chilbearing age, ten to occur after chilbirth, an may be linke to estrogen. Oral contraceptive pills (OCP) have also been foun to be associate with the occurrence of these tumors, whereas antiestrogen meications may lea to shrinkage. They’ve been reporte to shrink after menopause (C).
7
Patients are typically in their thir or fourth ecae of life an present with pain, a mass, or both. They are classie as either extra abominal (extremities, shouler), abominal wall, or intraabominal (mesenteric an pelvic). There are no typical raiographic nings, but MRI may elineate muscle or soft-tissue inltration an is require in larger tumors to elineate anatomic relations before surgical intervention. Core neele biopsy often reveals collagen with iffuse spinle cells an abunant brous stroma, which may suggest a low-grae brosarcoma; however, the cells lack mitotic activity. An open incisional biopsy of lesions larger than 3 to 4 cm is often necessary. Wie local excision with negative margins is inicate for symptomatic esmoi tumors. Nonresectable or incientally foun, asymptomatic, intraabominal esmoi tumors (even if resectable) shoul be treate with nonsteroial antiinammatory agents (e.g., sulinac) an antiestrogens, which have met with objective response rates of 50%. In regar to ajuvant therapy, recent retrospective reviews have seen signicant reuctions in recurrence with raiation combine with surgery an even with raiation alone. More research is necessary for the use of chemotherapy agents, but it has been seen that when cytotoxic chemotherapy agents are use in inoperable esmoi tumors, there is a 0% to 40% positive response. The aggressive nature of these tumors an high rate of occurrence make esmoi tumors the secon most common cause of eath in patients with FAP, after colorectal carcinoma. References: Ballo MT, Zagars GK, Pollack A, Pisters PW, Pollack RA. Desmoi tumor: prognostic factors an outcome after surgery, raiation therapy, or combine surgery an raiation therapy. J Clin Oncol. 1999;17(1):158–167. Hansmann A, Aolph C, Vogel T. High ose tamoxifen an sulinac as rst-line treatment for esmoi tumors. Cancer. 004;100(3):61–60. Janinis J, Patriki M, Vini L, Aravantinos G, Whelan JS. The pharmacological treatment of aggressive bromatosis: a systematic review. Ann Oncol. 003;14():181–190. Nuyttens JJ, Rust PF, Thomas CR Jr, Turrisi AT 3r. Surgery versus raiation therapy for patients with aggressive bromatosis or esmoi tumors: a comparative review of articles. Cancer. 000;88(7):1517–153.
15. B. Most retroperitoneal tumors are malignant an com-
prise approximately half of all soft-tissue sarcomas. The most common sarcomas occurring in the retroperitoneum are liposarcomas, malignant brous histiocytomas, an leiomyosarcomas (C). Approximately 50% of patients will have a local recurrence an 0% to 30% will en up having istant metastases. Lymph noe metastases are rare (D). Retroperitoneal sarcomas present as large masses because they o not typically prouce symptoms until their mass effect creates compression or invasion of ajacent structures. Symptoms may inclue gastrointestinal hemorrhage, early satiety, nausea, vomiting, an lower extremity swelling. Retroperitoneal sarcomas have a worse prognosis than nonretroperitoneal sarcomas. The best chance for long-term survival is achieve with an en bloc, margin-negative resection. Tumor stage at presentation, high histologic grae, unresectability, an grossly positive resection margins are strongly associate with increase mortality rates. Tumor grae is the most signicant preictor of outcome. Complete surgical resection is the most effective treatment for primary or recurrent retroperitoneal sarcomas (A, E). Surgical cure
8
PArt i Patient Care
can be limite because the margins are often compromise by anatomic constraints. There is no ifference in survival between those who ha a resection with a grossly positive margin an those with inoperable tumors. Unlike extremity sarcomas, external beam raiation therapy is limite for retroperitoneal malignancies because there is a low tolerance for raiation to surrouning structures. Postoperative an intraoperative raiation therapy have been shown to reuce local recurrence, but further stuies are neee to etermine if this leas to improve survival. Reference: Lewis JJ, Leung D, Wooruff JM, Brennan MF. Retroperitoneal soft-tissue sarcoma: analysis of 500 patients treate an followe at a single institution. Ann Surg. 1998;8(3):355–365.
16. B. This patient was recently iagnose with atrial
brillation an starte on oral anticoagulants. One shoul suspect a rectus sheath hematoma in oler patients taking anticoagulants who present with the clinical tria of acute abominal pain, an abominal wall mass, an anemia. The
mass is palpable even uring exion of abominal wall muscles, helping to ifferentiate this from an intraperitoneal process (Fothergill sign) (D). In a review of 16 patients by Mayo Clinic, anticoagulation was associate with 70%. Above the arcuate line, the etiology is often relate to a lesion to the superior epigastric artery within the rectus sheath (E). In most cases, there is no clear history of trauma (C). In particular, in the elerly who are taking oral anticoagulants, they typically occur spontaneously. The most common treatment for patients with rectus sheath hematomas is rest, analgesics, an bloo transfusions as necessary. In general, coagulopathies are correcte; however, continuing anticoagulation may be pruent in select patients (e.g., biomechanical valve, recent sale embolus). In extreme cases, angioembolization may be require. References: Alla VM, Karnam SM, Kaushik M, Porter J. Spontaneous rectus sheath hematoma. West J Emerg Med. 010;11(1):76–79. Cherry WB, Mueller PS. Rectus sheath hematoma: review of 16 cases at a single institution. Medicine (Baltimore). 006;85():105–110.
Abdomen—Hernia AMANDA C. PURDY AND AMY KIM YETASOOK
2
ABSITE 99th Percentile High-Yields I. Abominal Wall Hernia a. From skin to peritoneum: skin → fascia of Camper → fascia of Scarpa → external oblique → internal oblique → transversus abominis → transversalis fascia → preperitoneal fat → peritoneum b. Superior to arcuate line: 1. Anterior sheath comprise of aponeurosis of external oblique an the anterior half of the aponeurosis of internal oblique . Posterior sheath comprise of aponeurosis of transversus abominis an aponeurosis of the posterior half of internal oblique; posterior sheath not present inferior to arcuate line c. Ten to 15% of all incisions will evelop into ventral (incisional) hernia; woun infection after surgery oubles risk of a hernia evelopment . Miline epigastrium is a physiologic area of weakness in the abomen where patients can evelop iastasis recti an/or epigastric hernia; risk factors inclue pregnancy an weight gain e. Diastasis recti: attenuation of linea alba causing rectus muscle separation; when the rectus contract, a bulge appears in the upper miline abomen; no fascial efect, not a hernia II. Umbilical Hernias a. Peiatric umbilical hernias 1. Congenital efect, repair by age 5 or sooner if symptomatic b. Umbilical hernias in aults 1. Acquire efect, increase intraabominal pressure causes weakening of transversalis fascia an of the umbilical ring . Women are 3 times more likely to evelop umbilical hernias than men (ue to pregnancy), an up to 90% of women evelop an umbilical hernia uring pregnancy; incarceration occurs more in men 3. Cirrhotic patients with uncomplicate umbilical hernias shoul be meically optimize before unergoing elective surgical repair; this inclues free water restriction, iuretics, an large volume paracentesis (with infusion of albumin); mesh can be use III. Inguinal Hernias a. Cremaster muscle bers arise from internal oblique muscle, inguinal ligament from external oblique muscle; the internal oblique an transversalis fascia form the internal ring of the inguinal canal; the conjoint tenon is the lower common aponeurosis of the internal oblique an the transversus abominis b. Though wiely believe as true, there is little evience to support physical activity with inguinal hernia evelopment; inguinal hernias have increase type 3 collagen an ecrease type 1 collagen c. During issection, an inirect hernia sac is foun on the anteromeial aspect of spermatic cor . The genital branch of the genitofemoral nerve supplies sensation to the mons an labia majora in women, an in men it supplies motor to the cremaster an sensation to the scrotum; it runs within the spermatic cor an exits via the eep inguinal ring 9
10
PArt i Patient Care
e. The iliohypogastric nerve arises from the rst lumbar branch an travels between the transversus abominis an the internal oblique muscles f. The ilioinguinal nerve runs anterior to the spermatic cor in men or roun ligament in women an passes through the supercial inguinal ring; supplies sensation to the upper meial thigh g. Peiatric inguinal hernias (ue to a congenital failure of the processus vaginalis to close): 1. Repair only requires high ligation of the hernia sac (ligation at the internal ring) IV. Hernia Repair a. Open repair i. Open repair with mesh (5%) no matter which metho is chosen 10. One hour after laparoscopic repair of a left inguinal hernia, the patient complains of severe burning groin pain. Which of the following is the most appropriate recommenation? A. Immeiate return to the OR for laparoscopy B. Nonsteroial antiinammatory rugs C. Neurontin D. Opioi analgesia E. Inject groin region with local anesthetic 11. Ischemic orchitis after inguinal hernia repair is most often ue to: A. Too tight a reconstruction of the inguinal ring B. Preexisting testicular pathology C. Inavertent ligation of the testicular artery D. Completely excising a large scrotal hernia sac E. Anomalous bloo supply to the testicle 12. A 45-year-ol man presents with an asymptomatic right inguinal hernia. It is easily reuce with gentle pressure. Which of the following is true about this conition? A. The likelihoo of strangulation eveloping is high without surgery B. Without surgery, intractable pain will most likely evelop C. Waiting until symptoms evelop is a reasonable alternative to surgery D. Laparoscopic repair is the best option E. If the hernia is small, there is a lower chance of incarceration
14
PArt i Patient Care
13. A 5-month-ol previously full-term male infant presents with a tener left groin mass that has been present for the past several hours. There is slight erythema over the skin. He is afebrile an his labs are normal. Which of the following is the best next step? A. Attempt manual reuction, an if successful, scheule surgical repair when infant reaches 1 year of age B. Attempt manual reuction, an if successful, immeiately take to the operating room for surgical repair C. Attempt manual reuction, an if successful, scheule repair in ays D. Attempt manual reuction, an if successful, scheule left-sie surgical repair with contralateral groin exploration in ays E. Take immeiately to the operating room for operative repair 14. Which of the following best escribes umbilical hernias in chilren? A. They have a signicant risk of incarceration. B. Repair is inicate once an umbilical hernia is iagnose C. Repair shoul be performe if the hernia persists beyon 6 months of age D. Most close spontaneously E. Repair shoul be performe only if the chil is symptomatic 15. Which of the following is true regaring umbilical hernias in aults? A. Most are congenital B. Repair is contrainicate in patients with cirrhosis C. Strangulation is less common than in chilren
D. Small, asymptomatic hernias can be clinically observe E. Primary closure has recurrence rates similar to those of mesh repair 16. Which of the following is true regaring femoral hernias? A. They are the most common hernia in females B. The Cooper ligament is consiere the anterior borer of the femoral canal C. They are lateral to the femoral vein D. Repair involves approximating the iliopubic tract to the Cooper ligament E. A Bassini operation is consiere an appropriate surgical option 17. A 55-year-ol woman presents with a painless abominal wall bulge. She reports a successful iet an exercise program an has lost almost 40 kg over the past years. However, she is worrie because yesteray when she was sitting up in be, she notice an upper miline abominal bulge that looks like a large rige between her rib cage an belly button. On physical exam the bulge becomes visible when she lifts her hea off the be. Which of the following is true regaring her conition? A. Surgical repair shoul be one immeiately before signs of incarceration evelop B. There are both congenital an acquire etiologies C. A strict regimen of abominal wall exercises usually results in complete resolution D. The efect is limite to the transversalis fascia E. Typically these efects contain only preperitoneal fat
ANSWERS 1. D. This patient has a symptomatic ventral incisional her-
nia. The best option for repair in this patient with multiple risk factors for perioperative infection (iabetes an obesity) is laparoscopic hernia repair with mesh. Compare to open incisional hernia repair, laparoscopic repair has a lower incience of surgical site infection an is the best option for patients at risk for postoperative infection (C–D). Open an laparoscopic ventral hernia repairs with mesh have similar recurrence rate. Component separation is a technique where the anterior rectus sheath is incise cm lateral to the semilunar line in orer to primarily close large efects while
minimizing tension. This is unnecessary in this case, as the efect is only 3 cm wie, an a minimally invasive technique is more appropriate (E). Abominal wall strengthening exercises are the primary repair for rectus iastasis, which is an attenuation of the linea alba in the superior abominal wall without a true hernia. This patient has a hernia, as evience by fascial efect on physical exam (A). Reference: Guielines for laparoscopic ventral hernia repair. SAGES. Publishe June 7, 016. https://www.sages.org/publications/ guielines/guielines-for-laparoscopic-ventral-hernia-repair
CHAPtEr 2 Abdomen—Hernia
2. C. This patient has a parastomal hernia. Although the inci-
ence of parastomal hernias is higher with en ostomies than with loop ostomies, this may simply be ue to loop ostomies getting reverse more often, an sooner than en ostomies that are more often permanent. The majority of parastomal hernias are asymptomatic an o not require intervention. However, this patient is experiencing symptoms with intermittent bowel obstruction an shoul unergo repair (E). The best option for management of a symptomatic parastomal hernia is to take the ostomy own if appropriate. Unfortunately, this is not an option for this patient with a prior abominoperineal resection (APR). The next best option is repair of the hernia with synthetic mesh using the Sugarbaker technique, where intraperitoneal mesh covers the entire efect, an the bowel leaing to the ostomy enters laterally between the mesh an abominal wall. Biologic mesh is associate with higher recurrence rates compare to prosthetic mesh (D). It may be consiere for patients with signicant contamination. Primary repair of parastomal hernias has been largely abanone ue to unacceptable recurrence rates of up to 70% (A). Ostomy relocation solves the problem at han (the current symptomatic parastomal hernia); however, it is inferior to repair with mesh as there is a high risk of eveloping another parastomal hernia at the new ostomy site (B). Reference: Hansson BM, Slater NJ, van er Velen AS, et al. Surgical techniques for parastomal hernia repair: a systematic review of the literature. Ann Surg. 01;55(4):685–695.
3. B. After vertical miline abominal incision, approx-
imately 10% to 0% of patients evelop incisional hernias. Ranomize controlle trials have shown that small (5 mm) fascial bites 5 mm apart have a signicantly lower rate of eveloping incisional hernia than large (1 cm) bites 1 cm apart (A, C). Also, a suture to woun length ratio of at least 4:1 is associate with less tension an a ecrease incience of incisional hernia evelopment (D, E). References: Deerenberg EB, Harlaar JJ, Steyerberg EW, et al. Small bites versus large bites for closure of abominal miline incisions (STITCH): a ouble-blin, multicentre, ranomise controlle trial. Lancet (London, England). 015;386(10000):154–160. Millbourn D, Cengiz Y, Israelsson LA. Effect of stitch length on woun complications after closure of miline incisions: a ranomize controlle trial. Arch Surg. 009;144(11):1056–1059.
4. D. Conitions that increase intraabominal pressure
(cystic brosis, chronic lung isease, ventriculoperitoneal shunts, constipation, an peritoneal ialysis) are associate with higher risk for eveloping an inguinal hernia. Patients with small asymptomatic hernias are at risk for eveloping symptoms as their hernias enlarge uring peritoneal ialysis. Therefore, everyone unergoing peritoneal ialysis shoul be examine for presence of abominal hernias preoperatively. If a hernia is foun, the patient shoul unergo concurrent herniorrhaphy at the time of peritoneal ialysis catheter placement (A–C). Hernia repair shoul be one with mesh, as mesh is associate with ecrease recurrence rates an are safe in patients unergoing peritoneal ialysis (E). Reference: Chi Q, Shi Z, Zhang Z, Lin C, Liu G, Weng S. Inguinal hernias in patients on continuous ambulatory peritoneal ialysis: is tension-free mesh repair feasible? BMC Surg. 00;0(1):310.
5. D. The genitofemoral nerve arises from the L1-L level.
The genital branch innervates the cremaster muscle an
15
sensation to the sie of the scrotum an the labia. It is responsible for the cremasteric reex. In women, it accompanies the roun ligament of the uterus. The genital branch of the genitofemoral nerve is part of the cor structures. It lies on the iliopubic tract an accompanies the cremaster vessels (B). The ilioinguinal nerve lies on top of the spermatic cor (A). It innervates the internal oblique muscle an is sensory to the upper meial thigh ajacent to the genitalia. The nerve can sometimes splay out over the cor, making issection ifcult. The iliohypogastric an ilioinguinal nerves arise from the T1-L1 level an intermingle. They provie sensation to the skin of the groin, the base of the penis, an the upper meial thigh. The iliohypogastric nerve lies on the internal oblique muscle (C), provies sensory innervation from the skin overlying the pubis, an oes not intermingle with the genitofemoral nerve because they cross ifferent paths (E). Reference: Wantz GE. Testicular atrophy an chronic resiual neuralgia as risks of inguinal hernioplasty. Surg Clin North Am. 1993;73(3):571–581.
6. A. Poupart ligament is another name for the inguinal lig-
ament. The inguinal ligament is forme from the anteroinferior portion of the external oblique aponeurosis foling back on itself. It extens from the anterosuperior iliac spine to the pubic tubercle, turning posteriorly to form a shelving ege. The cremaster muscle bers arise from the internal oblique muscle an surroun the spermatic cor (B). The genital branch of the genitofemoral nerve passes through the eep ring (C), whereas the ilioinguinal nerve passes through the supercial ring. The genital branch innervates the cremaster muscle, whereas the femoral branch controls sensation to the upper lateral thigh (D). Inirect hernias arise lateral to the inferior epigastric vessels, whereas irect hernias arise meial to the inferior epigastric vessels. The lateral borer of the rectus muscle, inferior inguinal ligament, an inferior epigastric artery ene the borers of Hesselbach triangle an ene the location of a irect hernia (E).
7. C. The arcuate line is locate below the umbilicus, typ-
ically one-thir the istance to the pubic crest (A). Between the costal margin an the arcuate line, the anterior rectus sheath is mae up of a combination of the aponeurosis of the external an internal oblique muscles. The posterior sheath is mae up of a combination of the aponeuroses of the internal oblique an transverse abominal muscles. Below the arcuate line, the anterior sheath is mae up of the aponeuroses of all three abominal muscles (E). The internal oblique aponeurosis splits above the arcuate line to envelop the rectus abominis muscle (B). There is no posterior sheath below the arcuate line (D), an the transversalis fascia therefore makes up the posterior aspect of the rectus abominis muscle.
8. E. This patient has bilateral inguinal hernias, one of
which is recurrent an shoul be offere a laparoscopic repair. The avantages of this inclue the ability to visualize both sies through a single incision an a potentially easier surgery in the setting of recurrence. It also protects the patient from eveloping a femoral hernia since the femoral canal is covere by the mesh. Of note, femoral hernias are known to evelop after open inguinal hernia repair. They evelop on average sooner than a typical recurrence, suggesting that the original hernia was in fact a femoral one an
16
PArt i Patient Care
was misse at the original surgery. The two laparoscopic approaches inclue TEP an TAPP. TEP involves issecting a plane in the preperitoneal space, which may actually be avantageous when compare to TAPP because intraabominal ahesions are avoie (A). This oes not hol true for prior pelvic surgery as the preperitoneal space may be obliterate in these patients, necessitating a TAPP. If the peritoneum is violate uring TEP, it is important to repair the efect to prevent ahesion formation postoperatively, but it is not manatory to convert to a ifferent technique (C). Though there are few absolute contrainications to laparoscopic hernia surgery, bowel ischemia with perforation or sepsis preclues the use of mesh, which is require in both TEP an TAPP. Tacking of the mesh in either laparoscopic approach can reuce mesh migration but shoul be avoie lateral to the epigastric vessels an inferior to the iliopubic tract to avoi placement in the “triangle of oom” or the “triangle of pain,” which contains the external iliac vessels an several nerves (lateral femoral cutaneous an femoral branch of genitofemoral, respectively) (B). Injury to these nerves is relatively specic to laparoscopic repairs (D). Reference: Fischer JE. Fischer’s mastery of surgery. Wolters Kluwer Health/Lippincott Williams & Wilkins; Chicago, IL, 01.
9. C. The preferre initial approach for an uncomplicate
inguinal hernia is still actively ebate within the surgical community. The LEVEL-trial specically compare TEP repair versus open mesh repair an emonstrate reuce pain in the immeiate postoperative perio an earlier return to work. However, this came at the expense of longer operating room times an higher intraoperative complication rates (B, D). This seems to be consistent with the results of a New England Journal of Medicine (NEJM) stuy from 004 comparing open mesh repair to all methos of laparoscopic mesh repair. However, they iverge on reporte recurrence rates, with the NEJM stuy favoring open repair (recurrence of 4% versus 10.1%) while the LEVEL-Trial showe equivalent recurrence rates (3.0% for open an 3.8% for TEP) (E). The LEVEL-Trial also inicate an equivalent prevalence of chronic pain, which was not one of the outcomes in the NEJM article (A). References: Langevel HR, van’t Riet M, Weiema WF, et al. Total extraperitoneal inguinal hernia repair compare with Lichtenstein (the LEVEL-Trial): a ranomize controlle trial. Ann Surg. 010;51(5):819–84. Neumayer L, Giobbie-Hurer A, Jonasson O, et al. Open mesh versus laparoscopic mesh repair of inguinal hernia. N Engl J Med. 004;350(18):1819–187.
10. A. Severe groin pain eveloping in the recovery room
following laparoscopic hernia repair is most likely ue to a stapling/tacking injury to a nerve. If this complication is suspecte, the patient shoul return to the operating room to remove the offening tack. Acute groin pain is most likely from injury to the ilioinguinal nerve. However, the most commonly injure nerve uring laparoscopic hernia repair is the lateral femoral cutaneous nerve (provies sensation to the lateral thigh). Injecting the groin with local anesthetic may not relieve the pain an if it works, it will only be a temporary measure (E). Meical therapy is not appropriate if the suspecte etiology is irritation of the nerve seconary to stapling/tacking (B–D). Chronic groin pain may occur in 10% to 5% of patients 1 year after surgery. The etiology is
thought to be entrapment of the nerve uring surgery or postoperative scarring. Chronic groin pain is best worke up with MRI. If conservative management oes not resolve the pain, operative exploration an ivision of the nerve(s) have met with success. The ieal approach in the setting of hernia reoperation after open repair is to enter a space in which the tissue planes have not been violate. The preferre management is a laparoscopic retroperitoneal triple neurectomy, which allows a single stage approach to access the ilioinguinal, iliohypogastric, an genitofemoral nerves.
11. D. Ischemic orchitis is thought to evelop as a result of
thrombosis of veins of the pampiniform plexus, leaing to testicular venous congestion. It has thus been terme congestive orchitis. The precise etiology of ischemic orchitis is unclear. The most commonly ientie risk factor is extensive issection of the spermatic cor. This occurs particularly when a patient has a large hernia sac, an the entire istal sac is issecte an excise. As such, it is recommene that the sac instea is ivie an the istal sac left in situ. In aition, the cor shoul never be issecte past the pubic tubercle. The presentation is that of a swollen, tener testicle, usually to 5 ays after surgery. The testicle is often high riing. This may eventually progress to testicular atrophy. Scrotal uplex ultrasonography has been shown to be useful in evaluating the perfusion of the testicle after hernia repair. However, it oes not change the management of ischemic orchitis. Management is expectant. In the past, attempts to reexplore the groin were unertaken to try to loosen the inguinal ring, but this was not successful (A). The bloo supply to the testicle is via the testicular artery, but there are rich collaterals incluing the external spermatic artery an the artery to the vas. Thus, inavertent ligation of the testicular artery oes not typically lea to this complication (C). Preexisting testicular pathology (B) or anomalous bloo supply (E) to the testicle is not thought to contribute to ischemic orchitis following inguinal hernia repair. However, ischemic orchitis can occur more frequently in recurrent inguinal hernia surgery using the anterior approach; thus, the laparoscopic approach shoul be consiere for recurrent hernias. References: Holloway B, Belcher HE, Letourneau JG, Kunberger LE. Scrotal sonography: a valuable tool in the evaluation of complications following inguinal hernia repair. J Clin Ultrasound. 1998;6(7):341–344. Wantz GE. Testicular atrophy an chronic resiual neuralgia as risks of inguinal hernioplasty. Surg Clin North Am. 1993;73(3):571–581.
12. C. A large prospective ranomize stuy in men
emonstrate that watchful waiting for patients with asymptomatic or minimally symptomatic inguinal hernias is an acceptable option for surgery (D). The patients were followe for as long as 9 years. Acute hernia incarceration without strangulation evelope in only one (0.3%) patient, an acute incarceration with bowel obstruction evelope in only one (A). Approximately one-fourth of the watchful waiting group eventually crosse over to receive surgical repair ue to increase hernia-relate pain (B). Smaller hernias ten to have a smaller neck, placing them at higher risk for eveloping incarceration (E). Reference: Fitzgibbons RJ Jr, Giobbie-Hurer A, Gibbs JO, etal. Watchful waiting vs repair of inguinal hernia in minimally symptomatic men: a ranomize clinical trial. JAMA. 006;95(3):85–9.
CHAPtEr 2 Abdomen—Hernia
13. C. The vast majority of inguinal hernias in chilren are
the inirect type ue to a persistent patent processus vaginalis. Approximately 1% to 5% of chilren can evelop an inguinal hernia. However, the incience increases in preterm infants an those with a low birth weight. Right-sie hernias are more common, an 10% of hernias iagnose at birth are bilateral. Incarceration is a more serious problem in peiatric patients than in aults. Emergent operation on an infant with an incarcerate hernia can be very challenging. Thus, it is preferable to try to reuce the hernia, which is successful in 75% to 80% of cases, allow the inammation to subsie over several ays, an then perform the repair semielectively. The routine use of contralateral groin exploration is not wiely supporte (D). For elective cases, one option is to perform laparoscopy via the hernia sac to look for a contralateral hernia an, if foun, procee to repair. If there are any signs of strangulation (e.g., leukocytosis, fever, elevate lactate), then manual reuction shoul be avoie, an the patient shoul be taken immeiately to the operating room for surgical intervention (E). In the patient escribe, though the skin is erythematous, there are no signs of systemic toxicity. Methos to achieve reuction inclue the use of intravenous (IV) seation, Trenelenburg positioning, ice packs, an gentle irect pressure. Reuction without subsequent surgery is not appropriate. That being sai, infants with anemia an history of prematurity are at signicantly increase risk of postoperative apnea an woul require overnight monitoring. Reference: Özemir T, Arıkan, A. Postoperative apnea after inguinal hernia repair in formerly premature infants: impacts of gestational age, postconceptional age an comorbiities. Pediatr Surg Int. 013;9(8):801–804.
14. D. In chilren, umbilical hernias are congenital. They
are forme by a failure of the umbilical ring to close, causing a central efect in the linea alba. Most umbilical hernias in chilren are small an will close by years of age, particularly if the efect is less than 1 cm in size. As such, repair is not always inicate at the time of iagnosis (B). Aitionally, the ecision to perform an elective repair is not solely etermine by the presence of symptoms (E). If closure oes not occur by age 4 or 5 years, elective repair is then consiere a reasonable option (C), even if the patient is asymptomatic. If the hernia efect is large (> cm) or the family is bothere by the cosmetic appearance, repair shoul be consiere. Although umbilical hernias in chilren can incarcerate, this is very rare (A). If the chil presents with abominal pain, bilious emesis, an a tener, har mass protruing from the umbilicus, immeiate exploration an hernia repair are inicate.
15. D. Unlike in chilren, umbilical hernias in aults are
usually acquire (A). Risk factors are any conitions that increase intraabominal pressure, such as pregnancy, obesity, an ascites. Overall strangulation of umbilical hernias in aults is uncommon, but it occurs more often than in chilren (C). Small, barely palpable an asymptomatic hernias can be followe clinically. Larger or symptomatic hernias shoul be repaire. In patients with cirrhosis an ascites, the markely increase pressure causes the skin overlying the hernia to become thin an eventually ischemic. One of the most catastrophic complications in this setting is rupture of
17
the hernia through the ischemic skin, leaing to peritonitis an eath. Thus, patients with cirrhosis an ascites shoul unergo repair if there is evience that the skin overlying the hernia is thinning or becoming ischemic (B). However, repair shoul be elaye until after meical management of the ascites. If meical management fails an the skin over the hernia is thinne an tense, then a transjugular portosystemic shunt shoul be consiere before repair. Alternatively, if the patient is a caniate for liver transplant, the hernia can be repaire uring the transplantation. Umbilical hernias have historically all been repaire by primary closure. Borrowing from the low recurrence rates using mesh for inguinal hernias, umbilical hernias are now more frequently being repaire using mesh, particularly those with large efects. A recent prospective, ranomize stuy compare primary closure with mesh repair. The early complication rates such as seroma, hematoma, an woun infection were similar in the two groups. However, the hernia recurrence rate was signicantly higher after primary suture repair (11%) than after mesh repair (1%) (E). Some authors are now avocating for the routine use of mesh for all ault umbilical hernias in the absence of bowel strangulation. References: Arroyo A, García P, Pérez F, Anreu J, Canela F, Calpena R. Ranomize clinical trial comparing suture an mesh repair of umbilical hernia in aults. Br J Surg. 001;88(10):131–133. Belghiti J, Duran F. Abominal wall hernias in the setting of cirrhosis. Semin Liver Dis. 1997;17(3):19–6.
16. D. Femoral hernias occur more commonly in females
an have a high risk of incarceration. However, the most common overall hernia in females is an inirect inguinal hernia (A). Bowel entering a femoral hernia passes own a narrow femoral canal. This is because the femoral ring, which serves as the entrance for the femoral canal, is very rigi an unyieling. Thus, the xe neck of a femoral hernia is prone to pinching off the bowel, putting the patient at risk for incarceration. The borers of the femoral canal are as follows: inguinal ligament (anterior) (B), Cooper ligament (posterior), femoral vein (lateral), an Poupart ligament (meial). Femoral hernias occur most commonly lateral to the lymphatics an meial to the femoral vein, within the empty space (C). It is important to recognize that femoral hernias pass eep (posterior) to the inguinal ligament. As such, repairs to the inguinal ligament (such as a Bassini operation an stanar mesh repair) will not obliterate the efect (E). The femoral hernia can be xe either through a stanar inguinal approach or irectly over the bulge using an infrainguinal incision. The essential elements of femoral hernia repair inclue issection an removal of the hernia sac an obliteration of the efect in the femoral canal. This can be accomplishe by either approximation of the iliopubic tract to the Cooper ligament or by placement of prosthetic mesh. Reference: e Virgilio C, Frank PN, Grigorian A, es. Surgery: a case based clinical review. Springer; 015.
17. B. It is important to unerstan the ifference between
epigastric hernias an iastasis recti because the former is a true hernia, which shoul be repaire, an the latter is a benign conition. Diastasis recti is cause by increase separation of the rectus abominis muscles an a relative thinning of the linea alba, which can mimic a hernia. The
18
PArt i Patient Care
conition can be acquire, such as in multiparous women where the repeate stretching of the abominal wall causes the rectus muscles to separate, or congenital, seconary to more lateral attachment of the rectus muscles at birth. Classically, patients present after recent weight loss because this allows for the lesion to be visible. There is no risk for strangulation in iastasis recti because all of the facial layers are intact (A, D). Though several methos of surgical repair have been escribe, these are mainly cosmetic. In general, all that is require is reassurance an abominal wall exercises to help strengthen the musculature—though complete resolution in aults is unlikely (C). In contrast, epigastric hernias are true hernias an represent a true efect in the linea alba. They are generally small an contain either preperitoneal fat or part of the falciform ligament (E). They arise from efects in the fascia in locations where neurovascular bunles
perforate through. Though small, they can cause signicant pain because of compression of the nerves traveling through the efect. There is some evience to suggest that iastasis rectus may increase the risk for evelopment of an epigastric hernia an will make primary repair of epigastric hernias more challenging. Of note, patients with iastasis recti are at increase risk of abominal aortic aneurysms. References: Brunicari FC, Anersen DK, Schwartz SI. Schwartz’s principles of surgery. 10th e. McGraw-Hill Eucation. Köhler G, Luketina RR, Emmanuel K. Suture repair of primary small umbilical an epigastric hernias: concomitant rectus iastasis is a signicant risk factor for recurrence. World J Surg. 015;39(1):11–16. Townsen CM, Jr, Beauchamp RD, Evers BM, Mattox KL, es. Sabiston textbook of surgery: the biological basis of modern surgical practice. 17th e. Philaelphia, PA: W.B. Sauners; 004.
Abdomen—Biliary AMANDA C. PURDY AND DANIELLE M. HARI
3
ABSITE 99th Percentile High-Yields I. Physiology A. Bile consists of water, bile salts, phospholipis, an cholesterol B. Primary bile acis (cholic & chenoeoxycholic acis) become seconary bile acis when ehyroxylate by gut bacteria (lithocholate an eoxycholate acis) C. Mechanism of bile concentration in the gallblaer: active transport of NaCl into gallblaer mucosal cells, passive absorption of water II. Cholecystitis, Choleocholithiasis, an Cholangitis A. Acute calculous cholecystitis: gallblaer inammation ue to gallstone impacte in the neck; management is NPO, IV antibiotics, resuscitation, laparoscopic cholecystectomy B. Acute acalculous cholecystitis: usually seen in critically ill patients in the ICU; US emonstrates gallblaer wall thickening, pericholecystic ui, with no stones; HIDA if US is equivocal, tx is IV antibiotics, resuscitation, an percutaneous cholecystostomy tube if critically ill versus laparoscopic cholecystectomy if stable for surgery C. Choleocholithiasis: may have obstructive jaunice; elevate irect bilirubin, may have transaminitis; US emonstrates gallstones, ilate CBD, +/− stone in the CBD (sensitivity only 50%); tx is ERCP followe up by laparoscopic cholecystectomy 1. If intraoperative cholangiogram is positive for choleocholithiasis: rst attempt to ush the stone with saline; if it oesn’t work try ushing after giving 1 mg IV glucagon (relaxes sphincter of Oi); if stones on’t clear, options inclue: a) Postoperative ERCP b) Transcystic CBD exploration—best for small stones, large cystic uct, or small CBD; generally preferre over transuctal CBD exploration because it avois a CBD incision c) Transuctal CBD exploration—best for large stones (>8–10 mm), large CBD, proximal stones (above cystic uct), choleochotomy mae anterior to avoi vasculature laterally D. Acute ascening cholangitis: Charcot tria (RUQ pain, fever, jaunice) presents in about 0%; Reynols penta as hypotension an confusion; RUQ US: +/− gallstones, ilate CBD, elevate irect BR; tx is IV antibiotics, ui resuscitation, pressors if in septic shock, followe by ERCP or percutaneous transhepatic cholangiography (PTC) tube placement after resuscitation; laparoscopic cholecystectomy uring same amission if ue to gallstones E. Mirizzi synrome: large stone in the gallblaer neck compresses the common hepatic uct (CHD), can cause CHD stricture or stula between the gallblaer an CHD; usually presents similar to cholecystitis an iagnose uring cholecystectomy; manage with cholecystectomy III. Choleochal cysts A. Due to an anomalous pancreaticobiliary junction, with a fuse, long common pancreaticobiliary channel allowing pancreatic enzymes to reux into the biliary tree leaing to inammation an cystic egeneration 19
20
PArt i Patient Care
B. More common in females an those of Asian escent, 60% iagnose before age 10 C. First step in workup is US but MRCP is best for iagnosis an preop planning D. Associate with cholangiocarcinoma an gallblaer cancer; type III has very low risk of malignancy; management for all types besies type III is surgical to ecrease subsequent malignancy risk 1. Management is base on location (escribe by Toani Classication): . Type I (fusiform ilation, most common): cyst excision, Roux-en-Y hepaticojejunostomy, cholecystectomy 3. Type II: cyst excision, primary closure, cholecystectomy 4. Type III: enoscopic sphincterotomy an cyst unroong 5. Type IVa: cyst excision, partial hepatectomy, Roux-en-Y hepaticojejunostomy, cholecystectomy 6. Type IVb: cyst excision, Roux-en-Y hepaticojejunostomy, cholecystectomy 7. Type V (Caroli isease): if only in one lobe of the liver—hepatic resection an cholecystectomy; if bilobar or unresectable—liver transplant IV. Gallblaer Polyps A. Polypoi lesions of the gallblaer: cholesterolosis (most common, cholesterol-laen macrophages in the lamina propria), aenomatous polyp (risk for gallblaer cancer) B. Inications for cholecystectomy for gallblaer polyps: symptomatic, polyp >10 mm, primary sclerosing cholangitis, an polyp of any size C. If cholecystectomy is not inicate, shoul follow patient with serial US in 6 to 1 months V. Gallblaer Aenocarcinoma (most common biliary malignancy) A. Risk factors: gallstones, gallblaer polyp >10 mm, porcelain gallblaer with selective mucosal calcication (as oppose to transmural calcication) B. May present similarly to cholecystitis, often iagnose on pathology after cholecystectomy C. Management: 1. T1a (into lamina propria) → cholecystectomy . >T1b (into muscularis) OR >N1 → cholecystectomy, segment IVb & V hepatectomy, portal lymphaenectomy (port site resection not inicate); followe by ajuvant chemotherapy (gemcitabine an cisplatin) 3. If positive cystic uct margin, nee extrahepatic bile uct resection an hepaticojejunostomy VI. Cholangiocarcinoma A. Risk factors: primary sclerosing cholangitis, ulcerative colitis, choleochal cyst, liver uke infection B. Can present with painless jaunice; suspect in patient with focal bile uct stenosis without history of biliary surgery or pancreatitis; best imaging is MRCP C. Unresectable if istant metastasis, which inclues multifocal hepatic isease an lymph noe mets beyon the porta hepatis D. For potentially resectable cholangiocarcinoma, start with iagnostic laparoscopy; goal of surgery is negative margins; all surgery inclues portal lymphaenectomy; management epens on location:
Location/Classięcation
Management
Lower 1/3 of extrahepatic bile duct
Whipple
Middle 1/3 of extrahepatic bile duct
Resection, hepaticojejunostomy
Upper 1/3 of bile duct AKA Klatskin tumor Further classięed with the Bismuth classięcation:
Type I
CHD (not to the conĚuence)
Type II
CHD to the conĚuence
Type IIIa
CHD + RHD
Type IIIb
CHD + LHD
Type IV
CHD + RHD + LHD
If localized to one side—hemihepatectomy, extrahepatic bile duct excision, Roux-en-Y hepaticojejunostomy If unresectable hilar tumor ≤3cm without nodal disease or distant mets—evaluate for transplant
CHAPtEr 3 Abdomen—Biliary
21
VII. Bile uct injuries (incience 0.3%–0.8%, most commonly ue to cystic uct stump leak) A. Risk of bile uct injury higher with laparoscopic cases an elective (not emergent/urgent cases) B. Principles of management: control sepsis, rain bile collections, an establish secure biliary rainage C. Marke laboratory abnormalities are not typical; bilirubin may be elevate ue to systemic resorption; US is initial imaging stuy, +/− HIDA D. In immeiate postop perio, treat with IV antibiotics, ui resuscitation, percutaneous rainage an ERCP with stent placement an/or sphincterotomy as this is sufcient for majority of cases; if not, percutaneous transhepatic catheter require; if leak has not heale in 6 to 8 weeks, biliary reconstruction is consiere with Roux-en-Y hepaticojejunostomy E. If iscovere intraoperatively, repair only inicate if aequate hepatobiliary surgical experience is available; otherwise, wie rainage an referral to higher level of care
Fig. 3.1 Biliary Cysts.
22
PArt i Patient Care
Questions 1. A 10-year-ol boy with sickle cell isease presents with right upper quarant pain, nausea, vomiting, fever, an yellowing of the eyes for the past ay. He enies ark urine or light stool. On exam, he is febrile, hemoynamically stable, an has a positive Murphy sign. He has leukocytosis, elevate alkaline phosphatase, an elevate unconjugate bilirubin. On ultrasoun, there are gallstones, pericholecystic ui, an gallblaer wall thickening, an CBD iameter is 4 mm. After starting IV ui resuscitation an IV antibiotics, what is the next step? A. MRCP B. ERCP C. Percutaneous transhepatic cholangiography D. Laparoscopic cholecystectomy E. Cholecystostomy tube 2. A 5-year-ol woman is unergoing elective laparoscopic cholecystectomy for symptomatic cholelithiasis. When removing the gallblaer from the fossa, a -mm tubular structure is completely transecte an is leaking bile. The structure appears to come from the liver fossa an enter irectly into the gallblaer. What is the most appropriate management? A. Laparoscopic clip placement B. Repair over a T-tube C. Roux-en-Y hepaticojejunostomy D. Immeiate transfer to a hospital with a hepatobiliary surgeon E. Complete cholecystectomy an plan for postoperative ERCP 3. A 45-year-ol male presents with hematemesis two weeks after a motor vehicle accient in which he suffere a liver injury that was manage nonoperatively. Laboratory values are signicant for an elevate total bilirubin an alkaline phosphatase, as well as signicant anemia. This patient is most likely to have which of the following? A. Arterioportal vein stula B. Arteriohepatic vein stula C. Arterial pseuoaneurysm D. Portal venous pseuoaneurysm E. Cavernous hemangioma
4. Which of the following patients shoul be offere a cholecystectomy? A. A 40-year-ol woman with an incientally iscovere 6-mm gallblaer polyp B. A 30-year-ol man with asymptomatic gallstones unergoing gastric bypass C. A 65-year-ol woman with asymptomatic gallstones an an incientally iscovere porcelain gallblaer with selective mucosal calcication D. A 50-year-ol man with a history of iabetes an asymptomatic gallstones E. A 1-year-ol boy with sickle cell isease an asymptomatic gallstones 5. Which of the following is true regaring bile an gallstones? A. The primary bile acis are eoxycholic an lithocholic aci B. The primary phospholipi in bile is lecithin C. Cholecystectomy ecreases bile salt secretion D. Brown pigmente gallstones are more likely to be foun in the gallblaer versus the CBD E. Bile consists of an equal part of bile salts, phospholipis, an cholesterol 6. Which of the following is true regaring the gallblaer? A. It passively absorbs soium an chlorie B. In the setting of cholelithiasis, cholecystokinin (CCK) can cause gallblaer pain that waxes an wanes C. It harbors an alkaline environment D. Glucagon can help empty the gallblaer E. Its contraction is inhibite by vagal stimulation
CHAPtEr 3 Abdomen—Biliary
7. A 75-year-ol woman presents to the emergency epartment with a -ay history of nausea, feculent vomiting, an obstipation. Her bloo pressure on amission is 80/60 mm-Hg, an her heart rate is 10 beats per minute, both of which normalize after uis. Plain lms reveal istene loops of small bowel with air–ui levels an air in the biliary tree. Which of the following is the best management option? A. Small bowel enterotomy with removal of the gallstone plus B. Small bowel enterotomy with removal of the gallstone C. Small bowel enterotomy with removal of the gallstone followe 8 weeks later by cholecystectomy an takeown of stula D. Small bowel resection to inclue area of impacte gallstone E. Small bowel resection to inclue area of impacte gallstone plus cholecystectomy an takeown of the stula 8. Jaunice with absent urine urobilinogen is most consistent with: A. Hepatitis B. Cirrhosis C. Hemolysis D. Biliary obstruction E. Sepsis 9. Which of the following is true regaring bile an gallblaer isease? A. Primary bile acis are forme by econjugation B. Bile acis are passively absorbe in the terminal ileum C. Bile acis are responsible for the yellow color of bile D. Bile uct stones occurring 1 year after cholecystectomy are consiere primary common uct stones E. In between meals, gallblaer emptying is stimulate by motilin 10. Which of the following is true regaring biliary anatomy? A. The right hepatic uct tens to be longer than the left an more prone to ilation B. Venous return from the gallblaer is most often via a cystic vein to the portal vein C. Heister valves have an important role in the gallblaer’s function as a bile reservoir D. The CBD an pancreatic uct typically unite outsie the uoenal wall E. The arterial supply to the CBD erives primarily from the left hepatic an right gastric arteries
23
11. Ultrasonography of the gallblaer reveals a polypoi lesion. This most likely represents: A. a cholesterol polyp B. aenomyomatosis C. a benign aenoma D. aenocarcinoma E. an inammatory polyp 12. Which of the following is the correct pairing of anatomic structure an irection for retraction uring a laparoscopic cholecystectomy? A. Gallblaer funus laterally B. Gallblaer infunibulum laterally C. Gallblaer boy laterally D. Gallblaer infunibulum cephala E. Gallblaer funus meially 13. Hyrops of the gallblaer: A. Poses a signicantly increase risk of malignancy B. Is ue to a stone impacte in the cystic uct C. Typically starts with an enteric bacterial infection D. Is associate with marke right upper quarant tenerness E. Results in the gallblaer getting lle with bile-staine ui 14. During a laparoscopic cholecystectomy for symptomatic cholelithiasis, the surgeon inavertently transects the CBD. An experience hepatobiliary surgeon is available. The best choice for operative repair is: A. En-to-en CBD anastomosis B. Choleochouoenostomy C. Choleochojejunostomy D. Hepaticouoenostomy E. Hepaticojejunostomy 15. The most common cause of benign bile uct stricture is: A. Ischemia from operative injury B. Chronic pancreatitis C. Common uct stones D. Acute cholangitis E. Sclerosing cholangitis
24
PArt i Patient Care
16. A 45-year-ol man has a 50% total boy surface area thir-egree burn. Fever, marke leukocytosis, an right upper quarant pain evelop on hospital ay 7. His bloo pressure is 130/80 mm-Hg, an his heart rate is 110 beats per minute. Ultrasonography shows a istene gallblaer with gallblaer wall thickening an sluge. However, it is negative for gallstones. Antibiotics are initiate. The next step in management woul consist of: A. Laparoscopic cholecystectomy B. Compute tomography C. Hepatobiliary iminoiacetic aci (HIDA) scan D. Percutaneous cholecystostomy E. Upper enoscopy 17. During laparoscopic cholecystectomy, bile appears to be emanating near the junction of the CBD an cystic uct. Upon conversion to open cholecystectomy, the injury is note to be a 3-mm longituinal tear in the anterolateral istal common hepatic uct. The uct itself measures 7 mm in iameter. Management consists of: A. Primary repair of the injury without a T tube B. Primary repair of the injury over a T tube C. Primary repair of the injury with a T tube place through a separate choleochotomy D. Hepaticojejunostomy E. Choleochouoenostomy 18. Which of the following statements is true regaring the use of intraoperative cholangiography (IOC) uring laparoscopic cholecystectomy? A. It helps prevent inavertent incision of the common bile uct (CBD) B. It is the best way to ientify clinically signicant common uct stones C. Routine use is justie because of its ability to ientify anatomic anomalies of the hepatic ucts D. Routine use is helpful to ensure complete removal of the gallblaer an cystic uct E. Routine use is unnecessary
19. An 80-year-ol patient presents with nausea, fever, an right upper quarant pain an tenerness. Ultrasonography reveals gallstones as well as air in the wall of the gallblaer. His temperature is 103.5°F an bloo pressure is 70/40 mm-Hg. Meical therapy is initiate, an pressors are neee to maintain bloo pressure. Which of the following is true regaring this conition? A. Metroniazole is an important antibiotic choice B. Emergent cholecystectomy is inicate C. Urgent percutaneous rainage is preferre over cholecystectomy D. The most common organism is an anaerobic gram-negative ro E. Perforation of the gallblaer is rare 20. Which of the following best escribes the role of preoperative biliary rainage before a Whipple proceure in a patient with obstructive jaunice? A. It has been shown to ecrease the rate of cholangitis B. It has been shown to increase the rate of woun infections C. It shoul be performe routinely if the bilirubin level is greater than 8 mg/L D. It has been shown to shorten the hospital stay E. It has been shown to ecrease the mortality rate 21. A 35-year-ol Chinese man presents with a fever of 103.5°F, right upper quarant pain, an jaunice. Laboratory values are signicant for a white bloo cell count of 15,000 cells/L, an alkaline phosphatase level of 400 U/L, an a serum bilirubin level of 3.8 mg/L. Magnetic resonance cholangiopancreatography (MRCP) emonstrates a markely ilate CBD, markely ilate intrahepatic ucts with several intrahepatic uctal strictures, an multiple stones throughout the uctal system. Which of the following is true regaring this conition? A. It is associate with close contact with ogs an sheep B. It is more commonly associate with black pigment stones versus brown pigment stones C. It more commonly affects males D. Metroniazole is able to resolve the majority of cases E. Initial treatment is with enoscopic retrograe cholangiopancreatography an transhepatic cholangiography
CHAPtEr 3 Abdomen—Biliary
22. A 65-year-ol woman presents with symptoms an signs of acute cholecystitis an unergoes an uneventful laparoscopic cholecystectomy. On postoperative ay 7, the pathology report inicates a supercial gallblaer carcinoma that invaes the perimuscular connective tissue. There is no evience of istant metastasis on subsequent imaging. Which of the following woul be the best management? A. Raiation an chemotherapy B. Observation C. Reoperation with resection of liver segments IVB an V D. Reoperation with resection of liver segments IVB an V an regional lymph noe issection E. Reoperation with resection of liver segments IVB an V, regional lymph noe issection, an resection of all port sites 23. A 4-year-ol male presents with acholic stools an cola-colore urine. Alkaline phosphatase is 000 IU/L, AST is 78 IU/L, ALT is 88 IU/L, an total bilirubin is .1 mg/L. Liver biopsy emonstrates periuctal concentric brosis aroun macroscopic bile ucts. He is positive for perinuclear antineutrophil cytoplasmic antiboy (p-ANCA). Which of the following is true about this conition? A. It is more commonly associate with Crohn isease than it is with ulcerative colitis B. Cancer antigen (CA) 19-9 levels shoul be etermine C. Enoscopic retrograe cholangiopancreatography (ERCP) will preominantly emonstrate irregular narrowing of the intrahepatic biliary tree D. Symptoms are often well controlle with meical management E. It is more common in females 24. Which of the following is a feature of gallblaer cancer? A. Speckle cholesterol eposits are foun on the gallblaer wall B. There are thickene noules of mucosa an muscle C. Gallblaer cancer is more common in males D. It is more likely to be accompanie by large gallstones compare with smaller ones E. Cancer invaing muscularis layer is manage with cholecystectomy alone
25
25. Choleochal cyst isease is thought to be cause by an abnormality of the: A. Bile uct smooth muscle B. Bile composition C. Bile uct aventitia D. Pancreaticobiliary uct junction E. Bile uct mucosa 26. On CT scan, a type I choleochal cyst appears to be aherent to the posterior wall of the portal vein. Management consists of: A. Partial excision of the cyst, leaving posterior wall behin, an cholecystectomy with Rouxen-Y hepaticojejunostomy B. Complete excision of the cyst, cholecystectomy, an hepaticojejunostomy C. Partial excision of the cyst, fulguration of posterior cyst mucosa, an cholecystectomy with Roux-en-Y hepaticojejunostomy D. Observation E. Roux-en-Y cyst jejunostomy 27. Which of the following is the best management of a localize Klatskin tumor? A. Pancreaticouoenectomy (Whipple proceure) B. Resection of the entire extrahepatic biliary tree with hepatic resection if necessary C. Resection of the mile thir of the biliary tree with hepaticojejunostomy D. Chemotherapy E. Raiation followe by chemotherapy 28. Which of the following is true regaring cholangiocarcinoma? A. The majority are intrahepatic B. Bismuth-Corlette type I cholangiocarcinoma occurs above the conuence of the right an left hepatic ucts C. Most patients benet from ajuvant chemoraiation after surgical intervention D. It arises from malignant transformation in hepatocytes E. Resection with biliary-enteric bypass is consiere appropriate management in patients with early isease
26
PArt i Patient Care
Answers 1. D. This patient with sickle cell isease has acute calculous cholecystitis an shoul unergo laparoscopic cholecystectomy after ui resuscitation an initiation of antibiotics. Signs that point to acute cholecystitis in this case inclue: fever, positive Murphy sign, leukocytosis, an ultrasoun nings of gallstones, gallblaer wall thickening, an pericholecystic ui. MRCP is reasonable if there is concern for possible choleocholithiasis. However, it is important to istinguish obstructive jaunice from jaunice from hemolytic anemia (as seen in this patient) (A). Although this patient has jaunice, his labs show an increase unconjugated bilirubin. He also oes not have ark urine or acholic stools, an CBD iameter is normal. This is more consistent with hemolytic anemia than with obstructive jaunice (in which you woul expect conjugate bilirubinemia, ark urine, acholic stools, an CBD ilation). This young patient with sickle cell isease has chronic hemolysis, which likely le to evelopment of pigmente gallstones, an now cholecystitis. Sepsis can trigger increase hemolysis in patients with sickle cell isease an is responsible for his perceive increase jaunice since symptom onset. ERCP woul be an appropriate choice if there is a very high suspicion for choleocholithiasis or ascening cholangitis; however, there is no evience of biliary obstruction in this case (B). Percutaneous transhepatic cholangiography can also be use to ecompress the biliary tree, which is not inicate in this case (C). Cholecystostomy tube can be consiere in patients with cholecystitis that are too unstable to unergo cholecystectomy, which is not true in this case (E). 2. A. Ducts of Luschka are small ucts that originate in the gallblaer fossa an rain irectly into the gallblaer, as escribe in this question. When transecte, they can cause bile leaks. When iscovere intraoperatively, the uct shoul be clippe or oversewn. More commonly these are iagnose postoperatively as a ui collection at the gallblaer fossa (biloma) an shoul be raine percutaneously an an ERCP with sphincterotomy an stent placement shoul be performe to encourage bile ow into the uoenum (E). Primary repair over a T-tube an Roux-en-Y hepaticojejunostomy are the appropriate treatment for common bile uct injuries (with 50% luminal injury, respectively), which is not what is escribe in this case (B, C). If a common uct injury occurs at a hospital without a surgeon who is experience in biliary reconstruction, the surgeon shoul place wie rains an then arrange transfer to a referral center. However, that is not necessary in this case (D). References: Mercao MA, Domínguez I. Classication an management of bile uct injuries. World J Gastrointest Surg. 011;3(4):43–48. Spanos CP, Syrakos T. Bile leaks from the uct of Luschka (subvesical uct): a review. Langenbecks Arch Surg. 006;391(5):441–447.
3. C. Hemobilia is a rare conition an presents with a classic (Quinke) tria of upper gastrointestinal bleeing (hematemesis), combine with jaunice an right-sie upper abominal pain. It is most often a result of iatrogenic injury of the right hepatic artery (more common if there is an aberrant right hepatic artery off the superior mesenteric
artery) uring laparoscopic cholecystectomy but may also occur following blunt an penetrating traumatic injuries. The unerlying lesion is typically an arterial pseuoaneurysm that has a connection with the biliary tree (hence the jaunice). It can also occur in association with gallstones, tumors, inammatory isorers, an vascular isorers. Treatment in most instances involves angiographic embolization of the artery (thus angiography is most likely to be the therapeutic stuy of choice). Enoscopy may show bloo coming from the ampulla of Vater but will not typically be therapeutic (because the bleeing is coming from a hepatic artery pseuoaneurysm). The remaining answer choices are not thought to play a role in hemobilia (A, B, D, E). References: Ahrent SA, Pitt HA. Biliary tract. In: Townsen CM, Jr, Beauchamp RD, Evers BM, Mattox KL, es. Sabiston textbook of surgery: the biological basis of modern surgical practice. 17th e. Philaelphia: W.B. Sauners; 004:1597–164. Bloechle C, Izbicki JR, Rashe MY, et al. (1994). Hemobilia: presentation, iagnosis, an management. Am J Gastroenterol. 1994;89(9):1537–1540. Croce MA, Fabian TC, Spiers JP, Kusk KA. Traumatic hepatic artery pseuoaneurysm with hemobilia. Am J Surg. 1994;168(3):35–38. Nicholson T, Travis S, Ettles D, etal. Hepatic artery angiography an embolization for hemobilia following laparoscopic cholecystectomy. Cardiovasc Radiol. 1999;(1):0–4.
4. C. Asymptomatic patients who are incientally iscovere to have gallstones usually o not require surgery because the lifetime risk of eveloping symptoms is 70 years). The most specic stuy to help conrm iagnosis is a CT scan showing air in the biliary tree. It usually results from a large gallstone (>.5 cm) that has eroe through the gallblaer into the
27
ajacent uoenum an causing air in the biliary tree, creating a cholecystouoenal stula (the most common type of biliary stula). Less commonly, the stula can be between the gallblaer an the colon (hepatic exure) or the stomach. The stone typically loges in the narrowest portion of the gastrointestinal tract—the istal ileum, near the ileocecal valve. The iagnosis of gallstone ileus is mae preoperatively in only approximately half of cases because a history of biliary isease may be absent, pneumobilia may not be seen, the gallstone may not be visualize, or the abominal raiographic nings may be nonspecic. Because many of these patients are elerly, have other major comorbiities, an are often markely ehyrate, initial surgical management shoul focus on relieving the obstruction. This is best accomplishe by a transverse enterotomy proximal to the palpable stone an stone removal (C–E). It is also important to run the small bowel because a signicant portion of patients will have more than one gallstone. Leaving the stula oes not seem to lea to signicant morbiity on long-term follow-up. Most surgeons woul not recommen taking the patient back at a later time for stula takeown. A resection of the small bowel is usually not necessary. References: Ahrent SA, Pitt HA. Biliary tract. In: Townsen CM, Jr, Beauchamp RD, Evers BM, Mattox KL, es. Sabiston textbook of surgery: the biological basis of modern surgical practice. 17th e. Philaelphia: W.B. Sauners; 004:1597–164. Roríguez-Sanjuán JC, Casao F, Fernánez MJ, Morales DJ, Naranjo A. Cholecystectomy an stula closure versus enterolithotomy alone in gallstone ileus. Br J Surg. 1997;84(5):634-637. Tan YM, Wong WK, Ooi LLPJ. A comparison of two surgical strategies for the emergency treatment of gallstone ileus. Singapore Med J. 004;45():69–7. Halabi WJ, Kang CY, Ketana N, Lafaro KJ, Nguyen VQ, Stamos MJ, Imagawa DK, Demirjian AN. Surgery for gallstone ileus: a nationwie comparison of trens an outcomes. Ann Surg. 014;59():39–35.
8. D. Bilirubin is the result of the breakown of ol re bloo cells into heme. Heme is broken own into biliverin an then bilirubin. Bilirubin is boun to albumin in the circulation, but as it reaches the liver, it is conjugate an eventually enters the gastrointestinal tract. In the gastrointestinal tract, it is econjugate into urobilinogen by bacteria. Some urobilinogen gets reabsorbe in the gut, returns to the liver, an is excrete in the urine, where it is eventually converte to urobilin, giving urine its yellow appearance. The remaining urobilin is oxiize to stercobilin in the intestines, giving stool its brown appearance. In the presence of biliary obstruction, less bilirubin enters the gut, less urobilinogen is mae, an therefore less appears in the urine. Less stercobilin is mae an therefore the stools turn pale. Hemolysis woul generate an increase in bilirubin an a corresponing increase in urobilinogen in the gut an in the urine (C). The remaining answer choices o not play a signicant role in bilirubin metabolism (A, B, E). Reference: Ahrent SA, Pitt HA. Biliary tract. In: Townsen CM, Jr, Beauchamp RD, Evers BM, Mattox KL, es. Sabiston textbook of surgery: the biological basis of modern surgical practice. 17th e. Philaelphia: W.B. Sauners; 004:1597–164.
9. E. Cholesterol that has been conjugate with taurine or glycine is consiere a primary bile (cholic an chenoeoxycholic aci). Seconary bile acis are a result of bacterial
28
PArt i Patient Care
econjugation in the gastrointestinal tract (A). Although bile acis are passively absorbe along the entirety of the small intestine, they are actively absorbe only in the terminal ileum (B). Bile acis are colorless, an the yellow hue of bile is a result of the pigmente biliverin (breakown prouct of bilirubin) that is also foun in bile (C). Bile uct stones occurring after years are consiere primary common uct stones an are often pigmente (D). During the fasting state, gallblaer emptying is stimulate by motilin. Reference: Luiking YC, Peeters TL, Stolk MF, et al. Motilin inuces gall blaer emptying an antral contractions in the faste state in humans. Gut. 1998;4(6):830–835.
10. D. The left hepatic uct is longer than the right an is more likely to be ilate in the presence of istal obstruction (A). The spiral Heister valves within the cystic uct o not have any true valvular function (C). In approximately threefourths of iniviuals, the CBD an the main pancreatic uct unite outsie the uoenal wall an traverse the uoenal wall as a single uct. The bloo supply to the CBD runs along the lateral an meial walls at the 3 an 9 o’clock positions an comes from the right hepatic artery an retrouoenal artery (off gastrouoenal artery) (E). Thus, a transverse hemitransection of the uct will likely interrupt the bloo supply an rener a repair prone to ischemia an stricture. Venous return of the gallblaer is typically raine irectly to the parenchyma of the liver (B). Reference: Osottir, M, Hunter, JG. Gallblaer. In: Brunicari FC, Anersen DK, Billiar TR, etal., es. Schwartz’s principles of surgery. 8th e. New York: McGraw-Hill; 005:1187–100.
11. A. Most polypoi lesions of the gallblaer are benign, an of these, cholesterol polyps are the most common. They are usually small (10 mcU/mL with elevate C-peptie (≥.5 ng/mL),
CHAPtEr 5 Abdomen—Pancreas
iii.
iv.
v.
vi.
49
fasting insulin to glucose ratio >0.4, no sulfonylurea or meglitinie etecte; less likely to be etecte with octreotie scan; if can’t localize with CT, can try 18-F-DOPA PET scan 1. Management: b. Manage symptoms with small, frequent meals c. If mm from the pancreatic uct: enucleate . If > cm OR 1000 (while off PPI ×7 hours at least), gastric pH cm OR 1000; most in istal pancreas so present late without obstructive jaunice, management: formal resection (inclue splenectomy if oing istal pancreatectomy because of malignancy risk) VIpoma: presents with high-volume watery iarrhea, ehyration, muscle cramping, cutaneous ushing; most are malignant; labs that support the iagnosis: high VIP levels, hypokalemia, achlorhyria, metabolic aciosis, hypercalcemia, hyperglycemia; management: formal resection (inclue splenectomy if oing istal pancreatectomy) Somatostatinoma: from elta cells; mostly malignant (90%); can be associate with neurobromatosis 1; presents with steatorrhea, iabetes, gallstones, hypochlorhyria; iagnose with high fasting somatostatin levels; management: formal resection
V. Pancreatic aenocarcinoma a. Risk factors: increase age, smoking, obesity, new-onset iabetes in elerly b. Workup an staging: CT pancreas protocol, CA19-9, CT chest/abomen/pelvis c. Consier iagnostic laparoscopy to assess for M1 isease prior to resection or neoajuvant therapy . Pancreatic aenocarcinomas with istant metastases (M1) are consiere unresectable an treate with systemic therapy; those not associate with istant metastases are further classie below:
Classięcation
Arterial Contact
Venous Contact
Initial Management
Resectable
None
SMV/PV: ≤180° AND no contour irregularity IVC: no contact
Surgery*
Borderline Resectable
SMA/Celiac: ≤180°
SMV/PV: >180° and/or contour irregularity, appears reconstructable IVC: contact
Neoadjuvant therapy, restage, surgery if appropriate
Locally Advanced
SMA/Celiac: >180° (encasement)
Unreconstructable SMV/PV
Neoadjuvant therapy, restage, surgery if appropriate
SMV = superior mesenteric vein, PV = portal vein, IVC = inferior vena cava, SMA = superior mesenteric artery. *Can also consider neoadjuvant therapy especially for high-risk masses, but most common ABSITE answer is still surgery for resectable disease.
50
PArt i Patient Care
e. If patient has biliary obstruction on presentation an cannot procee irectly to surgery, can have ERCP an stent placement; stent associate with increase risk of perioperative infection; shoul obtain a new CA19-9 level after biliary ecompression f. Generally, if a patient has a symptomatic pancreatic hea mass, you can procee with Whipple without a biopsy g. If planning on neoajuvant therapy, neeEUS-guie biopsy prior to treatment; after completion of neoajuvant therapy, restage with CT an CA19-9 an resect if appropriate h. All pancreatic aenocarcinoma gets ajuvant therapy i. Benets of chemotherapy versus chemoraiation are not clear; either are appropriate
Fig. 5.1 Geography of Pancreatic Neuroenocrine Tumors.
CHAPtEr 5 Abdomen—Pancreas
Fig. 5.2 The Puestow Proceure.
Fig. 5.3 The Beger Proceure.
51
52
PArt i Patient Care
Fig. 5.4 The Frey Proceure.
CHAPtEr 5 Abdomen—Pancreas
53
Questions 1. A 50-year-ol man with a history of Roux-en-Y gastric bypass presents with epigastric pain an fullness two months after an episoe of acute pancreatitis. CT scan reveals an 8-cm pancreatic pseuocyst that abuts the gastric funus. What is the most appropriate management? A. Enoscopic cystogastrostomy via the gastric remnant B. Percutaneous rainage C. Surgical cystogastrostomy via the gastric remnant D. Surgical Roux-en-Y cyst-jejunostomy E. Repeat imaging in 4 weeks 2. A 55-year-ol woman with a history of coronary artery isease is iagnose with a resectable 3-cm insulinoma in the tail of the pancreas. She ha a percutaneous angioplasty with rug-eluting stent placement three weeks ago an is on aspirin an clopiogrel. Despite eating small, frequent meals, she continues to have signicant, intermittent light-heaeness, palpitations, an iaphoresis aily. What is the most appropriate next step in management? A. Octreotie B. Diazoxie C. Neoajuvant chemotherapy D. Continue aspirin an clopiogrel, procee with enucleation E. Hol clopiogrel, continue aspirin, procee with istal pancreatectomy 3. A 60-year-ol man with chronic pancreatitis is presenting for follow-up. Despite alcohol an smoking cessation, oral analgesic meication, celiac axis nerve block, an ERCP with stent placement, he continues to have severe pain an foul-smelling iarrhea. Imaging reveals pancreatic calcication, an enlarge pancreatic hea, an pancreatic uct iameter is 5 mm. What is the most appropriate management to help resolve his symptoms? A. Puestow proceure B. Frey proceure C. Beger proceure D. Minor papilla sphincterotomy E. Whipple
4. Which of the following is true regaring the role of enoscopic retrograe cholangiopancreatography (ERCP) an/or timing of surgery for acute biliary pancreatitis? A. In mil pancreatitis, laparoscopic cholecystectomy can be safely performe within 48 hours of amission B. ERCP with sphincterotomy shoul be use routinely before surgery C. If the total bilirubin fails to normalize, ERCP with sphincterotomy shoul be performe preoperatively D. In severe pancreatitis, early cholecystectomy reuces morbiity an mortality E. There is minimal risk of worsening the pancreatitis with the performance of ERCP 5. Which of the following is true regaring pancreatic cysts? A. Serous cystaenoma has malignant potential B. Asymptomatic patients with mixe-type intrauctal papillary mucinous neoplasm (IPMN) shoul unergo conservative management C. Weight loss in patients with IPMN is mostly attribute to an elevate level of TNF-alpha D. Mucinous cystaenoma usually occurs in women an in the boy or tail of the pancreas E. Asymptomatic mucinous cystaenoma can be manage with repeat imaging in 6 months 6. A 55-year-ol man presents with a 1-hour history of epigastric pain, nausea, an vomiting. He has iffuse mil abominal tenerness to palpation. Laboratory values are signicant for serum amylase of 800 U/L, serum glucose of 130mg/L, chlorie of 104 mEq/L, white bloo cell count of 1,000 cells/μL, serum soium of 15mEq/L, an triglycerie levels of 1800mg/L. The most likely explanation for the hyponatremia is: A. Excessive ui loss B. Inappropriate antiiuretic hormone response C. Excessive free water replacement D. Pseuohyponatremia E. Arenal insufciency
54
PArt i Patient Care
7. Management of pancreatic lymphoma is by: A. Pancreaticouoenectomy B. Chemotherapy C. Pancreaticouoenectomy with postoperative chemotherapy D. Raiation therapy E. Preoperative chemoraiation followe by pancreaticouoenectomy 8. Which of the following is true regaring pancreas ivisum? A. The uct of Santorini ens in a blin pouch B. The inferior portion of the pancreatic hea rains through the uct of Santorini C. The majority of the pancreas rains through the uct of Santorini D. The uct of Wirsung rains through the minor papilla E. The ucts of Wirsung an Santorini fail to evelop 9. The preferre enitive treatment for recurrent acute pancreatitis ue to pancreas ivisum is: A. Lateral pancreaticojejunostomy (Puestow proceure) B. Pancreaticouoenectomy (Whipple proceure) C. Minor papilla sphincterotomy D. Major papilla sphincterotomy an pancreatic uctal septotomy E. Distal pancreatectomy 10. A 50-year-ol male with chronic pancreatitis has faile meical management an is being consiere for more invasive treatment. Which of the following is true regaring potential interventions? A. Pancreaticouoenectomy (Whipple proceure) is inappropriate for chronic pancreatitis B. Enoscopic proceures have been shown to be superior to surgical treatment C. Lateral pancreaticojejunostomy (Puestow proceure) is appropriate if the pancreatic uct is larger than 6 mm D. The most common inication for invasive intervention in chronic pancreatitis is poor exocrine an enocrine function E. Long-term pain control is similar to either the Puestow, Beger, or Frey proceure 11. Aenocarcinoma of the pancreas arises most often from which anatomic site? A. Main pancreatic uct B. Branch pancreatic uct C. Pancreatic acinus D. Ampulla of Vater E. Pancreatic islet
12. A 60-year-ol man presents with obstructive jaunice, acholic stools, an weight loss. An abominal ultrasoun scan emonstrates a ilate biliary tree an no gallstones. A ynamic contrast-enhance CT scan emonstrates a soli mass localize to the hea of the pancreas without evience of istant metastasis, or aenopathy. Vascular involvement can’t be exclue. The patient is otherwise in goo health. Laboratory values are normal. Which of the following is the next step in the management? A. Exploratory laparotomy B. Diagnostic laparoscopy C. MRI D. Enoscopic ultrasoun E. Positron emission tomography (PET) scan 13. Which of the following is true regaring alcohol an its relation to the pancreas an/or pancreatitis? A. It inuces spasm of the sphincter of Oi B. It ecreases pancreatic secretion C. A single episoe of binge rinking cannot lea to pancreatitis D. The type of alcohol consume is an important risk eterminant E. It inhibits chymotrypsin 14. A 48-year-ol male presents with vague abominal pain of weeks uration. He was recently ischarge for an episoe of alcohol-relate pancreatitis. Laboratory exam is remarkable for a milly elevate serum amylase. A compute tomography (CT) scan emonstrates a 4-cm wellcircumscribe peripancreatic ui collection with homogenously low attenuation. The borers of the collection appear to be ill-ene. The patient is afebrile an hemoynamically stable. What is the most appropriate next step? A. Intravenous (IV) antibiotics an uis B. Amit an place the patient on nothing by mouth (NPO) C. Percutaneous aspirate for carcinoembryonic antigen (CEA) level D. Exploratory laparotomy E. Observe 15. Which of the following is the least favorable management option for a chronic large pancreatic pseuocyst? A. Enoscopic transpapillary rainage using a stent B. Laparoscopic cystogastrostomy C. CT-guie rainage with a pigtail catheter D. Open Roux-en-Y cystojejunostomy E. Enoscopic transgastric cystogastrostomy
CHAPtEr 5 Abdomen—Pancreas
55
16. A 65-year-ol man presents with a persistent skin rash of the lower abomen an perineum, accompanie by intermittent vague left upper quarant pain an recent weight loss. A chemistry panel reveals serum glucose to be 160 mg/L, but results are otherwise unremarkable. CT reveals a large mass in the pancreas. Which of the following is true regaring the most likely conition? A. This patient is at higher risk for venous thromboembolic isease B. The mass is most commonly in the hea of the pancreas C. The secretory peptie responsible for the symptoms also stimulates exocrine pancreatic ow D. Patients often have associate hypokalemia E. These are often benign lesions
20. A 41-year-ol female presents with palpitations, trembling, iaphoresis, an confusion. Serum glucose is 48 mg/L an C-peptie level is elevate. Her symptoms resolve with the aministration of a carbohyrate loa. Which of the following is true regaring the most likely conition? A. Elevate C-peptie an hypoglycemia rule out an exogenous source B. Patients will often have a mass in the neck of the pancreas C. The most sensitive stuy for localization is a high-resolution CT scan D. Recurrent lesions can be manage with streptozocin an 5-FU E. It is the least common functional pancreatic enocrine neoplasm
17. The most common cause of chronic pancreatitis worlwie is: A. Gallstones B. Alcohol abuse C. Hereitary D. Hypertriglyceriemia E. Infectious
21. Octreotie scanning is most useful for localization of which of the following tumors? A. VIPoma B. Glucagonoma C. Pancreatic polypeptie-secreting tumor D. Gastrinoma E. Insulinoma
18. A 35-year-ol cachectic woman presents with episoic severe watery iarrhea that has le to multiple hospital amissions for replacement of uis an electrolytes over the course of several months. Stool cultures are repeately negative an she has no history of travel abroa. On examination, a mass is palpate in the epigastrium/right upper quarant. CT reveals a large, bulky pancreatic mass with extension into the superior mesenteric vein an ajacent organs. The best palliative management option for this patient’s symptoms is: A. Octreotie B. Streptozotocin C. Embolization D. Chemotherapy E. Raiation therapy
22. Which of the following is true regaring pancreatogenic (type 3) iabetes? A. Ketoaciosis is common B. The iabetes is easily controlle C. Peripheral insulin sensitivity is ecrease D. Glucagon an pancreatic polypeptie (PP) levels are low E. Hyperglycemia is usually severe
19. A 65-year-ol male presents for evaluation of yellowing skin. Review of systems is signicant for loose-tting clothes, fatigue, an night sweats. Laboratory evaluation is remarkable for elevate total bilirubin. CT scan reveals a pancreatic mass. Which of the following is least likely to contribute to this conition? A. History of cholecystectomy B. Diabetes C. Smoking D. BRCA E. Coffee consumption
23. A 30-year-ol nurse presents with intermittent iaphoresis, trembling, an palpitations. Her fasting bloo sugar is 50 mg/L. Her insulinto-C peptie ratio is greater than 1. Which of the following is the next step in management? A. CT scan of the abomen B. Psychiatric counseling to iscuss sulfonylurea abuse C. Psychiatric counseling to iscuss exogenous insulin abuse D. Octreotie scan E. Magnetic resonance imaging
56
PArt i Patient Care
24. A 60-year-ol alcoholic man presents with chronic, vague abominal pain. He enies a history of pancreatitis an is otherwise in goo health. CT reveals a 6-cm multiloculate, septate cyst at the tail of the pancreas. FNA of the cyst is noniagnostic. Flui amylase an CEA are in the high normal range. Management consists of: A. Distal pancreatectomy with possible splenectomy B. CT-guie rainage of the cyst C. Enoscopic cystogastrostomy D. Roux-en-Y cystojejunostomy E. Repeat imaging in 6 months
28. Which of the following is true regaring anatomy or the embryologic evelopment of the pancreas? A. The most commonly injure vessel uring issection behin the neck of the pancreas is the celiac vein B. The pancreas receives its arterial supply from only the celiac artery C. The ventral pancreas constitutes the hea an part of the boy of the pancreas D. Venous rainage of the pancreas is to the inferior vena cava E. The uncinate process is orsal to the portal vein an superior mesenteric artery
25. After a motor vehicle accient, persistent ascites evelops in a 55-year-ol man. Other than the ascites, CT nings are unremarkable. Paracentesis reveals clear ui with an amylase level of 5000 U/L. The patient fails an attempt at bowel rest, parenteral nutrition, an paracentesis. Denitive management woul consist of: A. Distal pancreatectomy B. Placement of pigtail catheter C. Roux-en-Y pancreaticojejunostomy D. Pancreaticouoenectomy E. Placement of a transuoenal pancreatic uct stent
29. A 35-year-ol man presents with severe abominal pain an iffuse abominal tenerness. CT scan with IV contrast emonstrates areas of hypoattenuation in the pancreas. His vitals are stable. His temperature is 38.4°C. Which of the following is true regaring his conition? A. Fine-neele aspiration (FNA) for culture shoul be performe B. Early IV antibiotics have emonstrate improve survival C. Early necrosectomy ecreases morbiity an mortality when compare with elaye intervention D. The patient shoul be observe with repeat imaging if he eteriorates clinically E. Percutaneous rainage shoul be performe
26. A 60-year-ol man presents with chronic epigastric abominal pain an jaunice. CT reveals iffuse swelling of the pancreas with compression of the intrapancreatic common uct. Neele biopsy of the pancreas reveals iffuse brosis an a plasma an lymphocytic inltrate. Serum IgG levels are increase. Primary management consists of: A. Whipple proceure B. Sterois C. Chemotherapy D. Hepaticojejunostomy E. ERCP with stenting 27. A 61-year-ol female unergoes a pancreaticouoenectomy (Whipple) operation. On postoperative ay ve she becomes hypotensive, tachycaric, an has severe abominal pain. Nasogastric tube emonstrates bilious output. She receives L of uis an BP improves to 110 mmHg. A CT scan reveals a signicant amount of free (with HU [hounsel units] of 5). The next step in her management is: A. Angiography with embolization B. Immeiate take back to the OR C. IV octreotie rip D. Transfuse bloo an transfer to ICU E. Upper enoscopy
30. A 60-year-ol woman presents with gallstone pancreatitis. Which of the following is the best preictor of a resiual gallstone persisting in the common bile uct? A. Persistent elevation of the total bilirubin level B. A ilate common bile uct on amission C. Persistent elevation of the alkaline phosphatase level D. Persistent elevation of the serum amylase level E. Persistent abominal pain 31. Which of the following pancreatic cystic lesions is almost exclusively foun in a young female? A. Serous cystic aenoma B. Mucinous cystic neoplasm C. Sie-uct IPMN D. Main-uct IPMN E. Soli pseuopapillary epithelial neoplasm
CHAPtEr 5 Abdomen—Pancreas
57
Answers 1. C. This patient has a symptomatic, large pancreatic
pseuocyst. Since it has been at least 6 weeks after his episoe of acute pancreatitis, an the pseuocyst is >6 cm, he shoul be offere enitive treatment (E). The majority of pancreatic pseuocysts are manage with enoscopic cystogastrostomy as it is minimally invasive an has a high success rate. In orer to perform enoscopic cystogastrostomy, the pseuocyst must abut the gastric wall. However, the above patient has a history of Roux-en-Y gastric bypass. The gastric funus is part of the remnant stomach an is not easily accessible enoscopically. As such, an enoscopic cystogastrostomy woul not be routinely offere as this woul require the expertise of a highly skille enoscopist using ouble-push balloon enoscopy techniques (A). Percutaneous rainage is not an ieal option because there is a high rate of pancreaticocutaneous stula formation an shoul be reserve for infecte pancreatic pseuocysts in patients too unstable for enoscopy or surgery (B). Surgical options inclue cystogastrostomy or Roux-en-Y cyst-jejunostomy. In this case, the patient alreay has a Roux-en-Y bypass an cystogastrostomy is the more appropriate option to rain the pseuocyst without signicantly altering the anatomy (D). Reference: Nealon WH, Walser E. Surgical management of complications associate with percutaneous an/or enoscopic management of pseuocyst of the pancreas. Ann Surg. 005;41(6):948–957.
2. B. This patient is experiencing symptoms of hypoglyce-
mia seconary to her insulinoma espite ahering to eating frequent, small meals. The treatment of choice for insulinomas is surgical removal. If the insulinoma is small ( mm away from the pancreatic uct, enucleation can be performe. Choice (D) is incorrect for two reasons: (1) it woul be inappropriate to procee with major pancreatic surgery while on ual antiplatelet therapy; an () because this patient’s tumor is 3 cm, the treatment of choice is resection with istal pancreatectomy an not enucleation (D). However, because of her history of percutaneous coronary intervention with rug-eluting stent placement less than 1 month ago, she shoul continue ual antiplatelet therapy (E). After rug-eluting stent placement, ual antiplatelet therapy shoul ieally be continue for 6 months to minimize stent thrombosis. If urgent surgery is neee, clopiogrel can be temporarily hel prior to the 6-month mark but shoul not be hel within the rst 4 to 6 weeks when stent thrombosis risk is the highest. Therefore, this patient’s treatment shoul be focuse on symptom management until she is reay for surgery. Diazoxie is the initial meication of choice to control symptoms in patients with insulinomas. It works by inhibiting the release of insulin from beta islet cells. While octreotie is a goo option to control symptoms from VIPomas an glucagonomas, it oes not work reliably for insulinomas as insulinomas o not always contain somatostatin receptors. Octreotie shoul only be consiere as symptom management for insulinomas if octreotie scanning is positive (inicating that the tumor contains somatostatin receptors). Otherwise, octreotie will inhibit glucagon an actually worsen hypoglycemia (A). Chemotherapy is generally only
consiere for metastatic insulinomas an is inappropriate in this case as the majority of insulinomas are benign (C). References: Valgimigli M, Bueno H, Byrne RA, et al. 017 ESC focuse upate on ual antiplatelet therapy in coronary artery isease evelope in collaboration with EACTS: The Task Force for ual antiplatelet therapy in coronary artery isease of the European Society of Cariology (ESC) an of the European Association for Cario-Thoracic Surgery (EACTS). Eur Heart J. 018;39(3):13–60. Gill GV, Rauf O, MacFarlane IA. Diazoxie treatment for insulinoma: a national UK survey. Postgrad Med J. 1997;73(864):640–641.
3. C. This patient has chronic pancreatitis with persistent
pain espite meical management with celiac axis nerve block an is therefore a surgical caniate. When etermining which proceure to perform, there are two main factors to consier: (1) if the pancreatic uct is ilate (≥6 mm), an () if the pancreatic hea is involve. In the case of an enlarge pancreatic hea an a normal-size main pancreatic uct (4 mg/L). Otherwise, an intraoperative cholangiogram shoul be performe, an if a common bile uct stone is etecte, either a laparoscopic common uct exploration or a postoperative ERCP shoul be performe (C).
58
PArt i Patient Care
References: Chang L, Lo S, Stabile BE, Lewis RJ, Toosie K, e Virgilio C. Preoperative versus postoperative enoscopic retrograe cholangiopancreatography in mil to moerate gallstone pancreatitis: a prospective ranomize trial. Ann Surg. 000;31(1):8–87. Kelly TR, Wagner DS. Gallstone pancreatitis: a prospective ranomize trial of the timing of surgery. Surgery. 1988;104(4):600–605. Rosing DK, e Virgilio C, Yaghoubian A, et al. Early cholecystectomy for mil to moerate gallstone pancreatitis shortens hospital stay. J Am Coll Surg. 007;05(6):76–766.
5. D. Serous cystaenoma is a benign true cyst that most
commonly occurs in women an in the pancreatic hea. It is often asymptomatic, but large cysts (>4 cm) may cause vague abominal pain. They o not nee to be resecte unless they are symptomatic (A). Mucinous cystaenoma is consiere premalignant, has a female preominance, occurs commonly in the boy or tail of the pancreas, an shoul always unergo resection (E). IPMN is ivie into three types base on pancreatic uct involvement: main-uct, sie-branch, an mixe-type. Main-uct IPMN carries up to a 50% risk of harboring malignant cells an shoul always be resecte in surgically appropriate caniates. Mixe-type IPMN also has a higher risk an shoul be remove (B). Sie-branch IPMN has a lower risk of malignancy an can be observe unless it is symptomatic, larger than 3 cm, or associate with mural noules. The weight loss in patients with IPMN is mostly attribute to exocrine insufciency from uct blockage an not TNF-alpha cachexia (C).
6. D. Severe hypertriglyceriemia leas to a falsely low
soium level. Water is isplace in the serum by lipis, resulting in an error in measurement. The anger is that the clinician who is unaware may try to correct the hyponatremia with hypertonic saline, leaing to severe hypernatremia. Similarly, a signicantly elevate level of serum glucose can also result in pseuohyponatremia. Excess volume loss seconary to emesis can lea to a hypovolemic hyponatremia but is accompanie by a hypochloremic metabolic alkalosis (A). Patients with gastrointestinal (GI) losses can have hyponatremia exacerbate by excessive free water replacement (C). Arenal insufciency may lea to hyponatremia seconary to the loss of action of alosterone at the istal convolute renal tubules but is accompanie by severe refractory hypotension an marke hyperkalemia (E). Reference: Howar J, Ree J. Pseuohyponatremia in acute hyperlipemic pancreatitis: a potential pitfall in therapy. Arch Surg. 1985;10(9):1053–1055.
7. B. Primary pancreatic lymphoma is extremely rare. Thus,
the management approach is base on case series an experience with lymphoma at other sites. Patients with pancreatic lymphoma may present with symptoms an CT nings suggestive of pancreatic aenocarcinoma, an as such, it may be ifcult to iagnose preoperatively. However, suspicion of lymphoma shoul be raise in the presence of a large bulky pancreatic tumor or with more iffuse pancreatic involvement. This is one situation in which CT-guie neele biopsy of the mass is inicate because the majority of stuies inicate that pancreatic lymphoma respons to chemotherapy as the primary moality. Surgery or raiation is not typically use in the management of pancreatic lymphoma (A, C–E).
References: Arcari A, Anselmi E, Bernuzzi P. Primary pancreatic lymphoma: a report of ve cases. Haematologica. 005;90(1), ECR09. Bouvet M, Staerkel GA, Spitz FR, et al. Primary pancreatic lymphoma. Surgery. 1998;13(4):38–390. Grimison P, Chin M, Harrison M. Primary pancreatic lymphoma-pancreatic tumors that are potentially curable without resection: a retrospective review of four cases. BMC Cancer. 006;6.
8. C. In pancreatic ivisum, the ucts of Wirsung an San-
torini fail to fuse (E). The result is that the majority of the pancreas rains through the uct of Santorini an through the lesser papilla. The inferior portion of the pancreatic hea an uncinate process rains through the uct of Wirsung an the major papilla (B, D). It is consiere a normal anatomic variant an is seen in 10% of iniviuals. It is thought to lea to an increase risk of pancreatitis because the minor papilla sometimes cannot hanle the higher ow of pancreatic juices. In another more common variant, the uct of Santorini ens in a blin pouch but still fuses with the Wirsung uct (A).
9. C. Pancreas ivisum can lea to recurrent episoes of
acute pancreatitis as well as chronic pancreatitis with intractable pain. Unlike other forms of chronic pancreatitis, however, marke ilation of the orsal uct is unusual. As such, surgical ecompressive proceures are not successful (A, B). For patients with recurrent attacks of acute pancreatitis, the best option is sphincterotomy of the minor papilla because the uct of Santorini is proviing the primary rainage to the pancreas. A stuy from Marseille foun a ecrease rate of acute pancreatitis in 4 patients after minor papilla sphincterotomy an orsal uct stenting. The complication rate was lower with sphincterotomy than with stent insertion. Major papilla sphincterotomy woul not likely be helpful because it rains a minority of the pancreas in pancreatic ivisum (D). Distal pancreatectomy is typically not neee (E). Reference: Heyries L, Barthet M, Delvasto C, Zamora C, Bernar JP, Sahel J. Long-term results of enoscopic management of pancreas ivisum with recurrent acute pancreatitis. Gastrointest Endosc. 00;55(3):376–381.
10. C. The most common inication for surgical interven-
tion in patients with chronic pancreatitis is chronic pain (D). Surgical rainage of a ilate pancreatic uct with istal obstruction is more effective than enoscopic approaches in patients with chronic pancreatitis (B). The Puestow proceure involves cutting open the length of the main pancreatic uct an anastomosing a Roux limb of jejunum to the uct but requires a ilate uct (>6 mm). Both the Whipple proceure (for inammation limite to the pancreatic hea) an total pancreatectomy are options for the treatment of intractable chronic pancreatitis, although they are associate with greater morbiity than a rainage proceure (A). The Beger proceure is another option, which resects the pancreatic hea but spares the uoenum, stomach, an bile uct, but this is a technically challenging proceure. The Frey proceure is similar to Beger but easier to perform since it avois the transection of the pancreatic neck over the superior mesenteric vessels. The best long-term pain control is achieve with longituinal pancreaticojejunostomy with limite resection of the hea of the pancreas, which Beger an Frey both satisfy, with Frey being the preferre option (E). However, Frey requires a ilate uct an pancreatic hea.
CHAPtEr 5 Abdomen—Pancreas References: Cahen DL, Gouma DJ, Nio Y, et al. Enoscopic versus surgical rainage of the pancreatic uct in chronic pancreatitis. N Engl J Med. 007;356(7):676–684. DiMagno MJ, DiMagno EP. Chronic pancreatitis. Curr Opin Gastroenterol. 01;8(5):53–531. Jawa ZAR, Kyriakies C, Pai M, et al. Surgery remains the best option for the management of pain in patients with chronic pancreatitis: a systematic review an meta-analysis. Asian J Surg. 017;40(3):179–185. Roch A, Teysseou J, Mutter D, Marescaux J, Pessaux P. Chronic pancreatitis: a surgical isease? Role of the Frey proceure. World J Gastrointest Surg. 014;6(7):19–135.
11. A. The majority of aenocarcinomas of the pancreas
arise from the main pancreatic uct. Approximately 66% of pancreatic aenocarcinomas evelop within the hea or uncinate process of the pancreas. The remaining answer choices can lea to pancreatic aenocarcinoma, but it occurs less frequently (B, C, E). Carcinoma at the ampulla of Vater is most commonly uoenal aenocarcinoma (D). Reference: Albores-Saavera J, Schwartz AM, Batich K, Henson DE. Cancers of the ampulla of Vater: emographics, morphology, an survival base on 5,65 cases from the SEER program: Cancer of the Ampulla of Vater. J Surg Oncol. 009;100(7):598–605.
12. D. In a patient with obstructive jaunice, the rst stuy
to perform is an abominal ultrasoun scan. In the absence of abominal pain an in the presence of weight loss, it is highly likely that the iagnosis is malignancy. A ynamic, contrast-enhance CT scan is highly effective in etermining the resectability of the mass. In cases where vascular involvement is not clear, enoscopic ultrasonography has aie in etermining resectability. Pancreatic cancer is consiere unresectable if the tumor is encasing or occluing the superior mesenteric vein or portal vein an causing vein contour irregularity, as this is consiere unreconstructable. Aitionally, pancreatic cancer is consiere unresectable if the tumor is abutting or encasing the superior mesenteric artery, hepatic artery, or celiac trunk by more than 180°. More frequently, enoscopic guie biopsy is being performe. The avantage of this approach is that there is no risk of tumor seeing because the area through which the neele is passe becomes part of the Whipple specimen. That being sai, in the situation in which the mass appears to be resectable, percutaneous or enoscopic ultrasonography–guie biopsy is not consiere necessary. Neele biopsy is prone to sampling error; therefore, a negative biopsy ning woul not alter the plan to perform a Whipple proceure (A). Likewise, a positive biopsy ning woul not alter the operative ecision. Operative morbiity an mortality after the Whipple proceure are sufciently low that one woul accept the low likelihoo (∼5%) that the lesion is benign. Biopsy shoul be reserve for situations in which the lesion appears to be unresectable because it may guie chemotherapy. It is also inicate in situations in which the appearance of the mass suggests other less common pathologies such as pancreatic lymphoma. Diagnostic laparoscopy is often one before proceeing with a Whipple to conrm there are no obvious hepatic or peritoneal lesions (B). Suspecte lesions are sent for a frozen sample. MRI may be a useful ajunct in patients with equivocal nings on CT or in cases where hepatic metastasis is suspecte (C). The role of PET in cancer workup continues to evelop but as of now it is unclear if PET as
59
any aitional information beyon what is provie with CT (E). References: Small W, Hayes JP, Suh WW. ACR appropriateness criteria [r] borerline an unresectable pancreas cancer. Oncology. 016;30(7):619–619. Tummala P, Junaii O, Agarwal B. Imaging of pancreatic cancer: an overview. J Gastrointest Oncol. 011;(3):168–174. Wang WL, Ye S, Yan S, et al. Pancreaticouoenectomy with portal vein/superior mesenteric vein resection for patients with pancreatic cancer with venous invasion. Hepatobiliary Pancreat Dis Int. 015;14(4):49–435.
13. A. The exact mechanism by which alcohol inuces pan-
creatitis is unclear. Ethanol inuces spasm of the sphincter of Oi, an this may lea to an increase in uctal pressure with a simultaneous brief stimulation of pancreatic secretion (B). It also increases pancreatic uct permeability, ecreases pancreatic bloo ow, an inappropriately activates chymotrypsin (E). Most patients with alcohol-relate pancreatitis have a longstaning history of heavy rinking. The type of alcohol consume is not important but rather the quantity an uration (D). The mean amount consume in patients in whom pancreatitis evelops is 100 to 175 g/ay, although it can rarely evelop after just one binge (C). Aitionally, the risk of pancreatitis seems to be higher in patients who have a iet high in protein an fat.
14. E. The history of recent pancreatitis combine with the
history of vague abominal pain, elevate serum amylase, an CT scan emonstrating a peripancreatic ui collection most likely represents pancreatic pseuocyst. Most patients with pseuocyst o not nee amission an can continue to eat, although a low-fat iet is recommene. Amission an total parenteral nutrition (TPN) woul only be recommene if they were unable to tolerate an oral iet (B). There is no reason to start IV antibiotics because he is not presenting with an infecte pseuocyst (A). Initial management of pseuocysts is conservative via observation because most spontaneously resolve. Pancreatic cyst CEA level is consiere the most accurate tumor marker for iagnosing a mucinous pancreatic cystic lesion. However, in the present setting, given the high suspicion for a pseuocyst, it woul not be neee (C). Invasive interventions are inappropriate because most pseuocysts resolve spontaneously (D). Preictors of failure for conservative management inclue pancreatic pseuocysts larger than 6 cm or those that have persiste for more than 6 weeks. CT or ultrasoun can be use to characterize interval changes in pancreatic pseuocysts.
15. C. Internal rainage is usually preferre to external
rainage for a symptomatic pancreatic pseuocyst that has faile to resolve with conservative therapy. External rainage is associate with a higher rate of complications, incluing infection an pancreaticocutaneous stula. The only inication for percutaneous rainage is in a patient with a ocumente or clinically apparent infected pancreatic pseuocyst that is unstable for a surgical or enoscopic proceure. Pseuocysts communicate with the pancreatic uctal system in 80% of cases. Internal rainage can be achieve enoscopically via a transmural approach or a transpapillary approach. This is gaining popularity making it the new rst-line treatment for pancreatic pseuocyst. If there
60
PArt i Patient Care
is portal hypertension (e.g., splenic vein thrombosis, unerlying cirrhosis, esophageal or gastric varices), then surgical open internal rainage may be more appropriate. Options inclue a cystogastrostomy, a Roux-en-Y cystojejunostomy, an a cyst uoenostomy (A–B, D–E). Cystogastrostomy can be performe enoscopically, laparoscopically, or with a combine approach. Failure of the enoscopic approach can be preicte by the ning of major uctal isruption or stenosis on enoscopic retrograe cholangiopancreatography (ERCP) or magnetic resonance cholangiopancreatography. Regarless of the approach, biopsies of the cyst wall must be one to rule out malignancy. References: Cantasemir M, Kara B, Kantarci F, Mihmanli I, Numan F, Erguney S. Percutaneous rainage for treatment of infecte pancreatic pseuocysts. South Med J. 003;96():136–140. Nealon WH, Walser E. Surgical management of complications associate with percutaneous an/or enoscopic management of pseuocyst of the pancreas. Ann Surg. 005;41(6):948–957. Yusuf TE, Baron TH. Enoscopic transmural rainage of pancreatic pseuocysts: results of a national an an international survey of ASGE members. Gastrointest Endosc. 006;63():3–7.
16. A. Glucagonoma can be remembere by the 4 Ds: ia-
betes, ermatitis, eep vein thrombosis, an epression. The rash is terme necrolytic migratory erythema an tens to manifest on the lower abomen or perineum. The mass characteristically appears in the tail of the pancreas along with VIPoma (a neuroenocrine tumor that secretes vasoactive intestinal polypeptie [VIP]). The responsible hormone, glucagon, inhibits exocrine pancreatic ow (C). The iagnosis of glucagonoma is conrme by measuring fasting glucagon levels. Because the tumors are in the istal pancreas, the patient oes not usually present with jaunice; as such, the iagnosis is often mae late when the tumor is large. Because glucagonoma is most commonly malignant, it shoul be remove with enucleation (if 0.3) an an elevate C peptie level. However, the avent of newer antiiabetic meications such as sulfonylureas can also present with a similar biochemical prole (A). Localization is achieve by CT scan an enoscopic ultrasonography. On occasion, they cannot be localize preoperatively, in which case, intraoperative ultrasonography is useful an is consiere the most sensitive imaging stuy. In contrast to the other functional enocrine pancreatic neoplasms, an octreotie scan is poor at localizing insulinoma owing to the fact that these lesions may not express sufcient somatostatin receptors (C). They are evenly istribute throughout the hea, boy, an tail of the pancreas. There is no pancreatic tumor that characteristically appears in the neck of the pancreas (B). The majority of insulinomas are benign (90%). They can be treate with enucleation. Diazoxie inhibits insulin release an is
CHAPtEr 5 Abdomen—Pancreas occasionally use for preoperative control of symptoms relate to hypoglycemia symptoms. For patients with recurrent or metastatic malignant insulinoma, tumor ebulking may be benecial as is the use of streptozocin an 5-FU. References: Dewitt CR, Hear K, Waksman JC. Insulin an C-peptie levels in sulfonylurea-inuce hypoglycemia: a systemic review. J Med Toxicol. 007;3(3):107–118. Halfanarson TR, Rubin J, Farnell MB, Grant CS, Petersen GM. Pancreatic enocrine neoplasms: epiemiology an prognosis of pancreatic enocrine tumors. Endocr Relat Cancer. 008;15():409–47.
21. D. Many pancreatic enocrine tumors have high con-
centrations of somatostatin receptors an can therefore be image with a raiolabele form of the somatostatin analogue octreotie (inium-111 pentetreotie). Octreotie scanning has the avantage of whole-boy scanning, which is useful in gastrinomas because they can present in a wie area. Use in combination with enoscopic ultrasonography, it etects more than 90% of gastrinomas. It is also useful for localizing carcinoi tumors. As many as 90% of gastrinomas are foun in the Passaro triangle, an area ene by the junction of the cystic uct an common bile uct, the secon an thir portions of the uoenum, an the neck an boy of the pancreas. Although a CT scan is also useful, an octreotie scan is particularly helpful in localizing gastrinomas smaller than 1 cm. Somatostatinoma an VIPoma ten to be large bulky tumors an are thus reaily seen by CT (A). Glucagonoma may present with a mass seen in the pancreatic tail (B). Octreotie scanning will miss as many as 40% of insulinomas because they may not express sufcient somatostatin receptors (E). Pancreatic polypeptie (PP) seems to have an important role in glucose metabolism. PP regulates the expression of the hepatic insulin receptor gene. PP-secreting tumor is rare an often asymptomatic but can be establishe by the presence of an enhancing solitary pancreatic hea tumor on CT imaging with elevate fasting PP level (C). Reference: e Herer WW, Kwekkeboom DJ, Valkema R, et al.
61
to insulin is proinsulin. Proinsulin is package in the pancreatic B cell, where it is cleave to insulin an C peptie, which are then release into the circulation at an equal ratio. Insulin is cleare by the liver, whereas C peptie is cleare by the kiney an is cleare more slowly than insulin, such that the normal insulin-to-C peptie ratio is less than 1 uring fasting. With a true insulinoma, both insulin an C peptie levels woul be elevate; however, the ratio woul still be less than 1. Factitious hypoglycemia will present with an insulin-to-C peptie ratio greater than 1 only if the patient is using exogenous insulin. In contrast, sulfonylurea abuse will have a ratio of less than 1 since it stimulates proinsulin release from the pancreas (B). Factitious hypoglycemia has been reporte more frequently in health-care workers an is associate with a higher incience of suicie, epression, an personality isorers. Thus, the patient shoul be referre for psychiatric counseling. Octreotie scan (D) is not useful in the workup for insulinoma but CT, MRI, or enoscopic ultrasoun may emonstrate a pancreatic mass (A, E). References: Lebowitz M, Blumenthal S. The molar ratio of insulin to C-peptie: an ai to the iagnosis of hypoglycemia ue to surreptitious (or inavertent) insulin aministration. Arch Intern Med. 1993;153(5):650–655. Waickus CM, e Bustros A, Shakil A. Recognizing factitious hypoglycemia in the family practice setting. J Am Board Fam Pract. 1999;1():133–136.
24. A. It is important to be aware that not all ui-lle pan-
pancreatic resection is terme type 3 diabetes. It iffers from type 1 an iabetes in that it is associate with ecrease glucagon an PP levels an insulin ue to pancreatic loss or estruction. Because all three of these hormones regulate glucose levels, the ensuing iabetes is consiere to be ifcult to control (B). Furthermore, peripheral insulin sensitivity is increase, whereas hepatic insulin sensitivity is ecrease (C). The result is that patients are prone to the evelopment of hypoglycemia, but ketoaciosis an marke hyperglycemia are rare (A, E). For iabetes to evelop as a result of pancreatitis, extensive estruction of the pancreas must occur. In fact, resections involving up to 80% of an otherwise normal glan can be one without enocrine insufciency. This may help explain why not all post-Whipple patients evelop poor glucose control.
creatic abnormalities in a patient with a history of rinking represent pseuocysts (B–E). Some of these lesions may represent cystic neoplasms of the pancreas. Suspicion of a cystic neoplasm shoul be particularly increase in the absence of a history of pancreatitis, as in this patient. A cystic neoplasm shoul also be suspecte when the CT scan emonstrates a soli component (septation) in the cystic lesion. The ifferential iagnosis inclues serous cystaenoma, mucinous cystic neoplasm, intrauctal papillary-mucinous aenoma, an soli pseuopapillary neoplasm. On a CT scan, a central scar is characteristic of a serous cystaenoma (although present in only 0%), whereas the ning of peripheral eggshell calcications, although rare, is iagnostic of mucinous cystic neoplasm an highly suggestive of cancer. In the patient presente, the proceure of choice is surgical resection with istal pancreatectomy an splenectomy. This is base on several factors: the patient is having symptoms; he is a goo caniate for surgery; the lesion is reaily amenable to resection; an the lesion is large, has septations, an has multiple loculations. If, conversely, a patient has an incientally iscovere pancreatic cyst without symptoms, surgery is generally recommene if the risk of surgery is low. Before surgery, further stuies are recommene to attempt to etermine the malignant potential. The workup may inclue MRI, enoscopic ultrasonography to better elineate the mass, an CT-guie aspiration of the ui for amylase level an tumor markers (carcinoembryonic antigen, CA 19–9, CA 15, CA 7–4, CA 15–3).
23. C. Although the patient has symptomatic hypoglycemia,
25. E. After surgery, trauma, or bouts of pancreatitis, per-
Neuroenocrine tumors an somatostatin: imaging techniques. J Endocrinol Invest. 005;8(11 Suppl International):13–136.
22. D. Diabetes in the setting of chronic pancreatitis or after
seemingly consistent with an insulinoma, her insulin-to-C peptie ratio is greater than 1. This combination, particularly in a health-care worker, is highly suggestive of factitious hypoglycemia with exogenous insulin abuse. The precursor
sistent ascites or pleural effusions can evelop. These are generally cause by a isruption of the pancreatic uct, with free extravasation of pancreatic ui, leaing to the evelopment of an internal pancreatic stula, which is rare.
62
PArt i Patient Care
More commonly, the extravasate ui leas to the formation of a containe ui collection known as a pseuocyst. Management of pancreatic ascites or effusion rst requires establishing the iagnosis by obtaining a sample of the ui an emonstrating a markely elevate amylase level an a protein level greater than 5 g/L. Serum amylase may be elevate from reassertion across the peritoneal membrane. The recommene management is a stepwise progression, rst with conservative management with bowel rest, parenteral nutrition, placing the patient NPO, an paracentesis to completely rain the ui. If this fails to resolve the internal stula, ERCP with pancreatic stenting is recommene. If this fails, surgery is inicate an shoul be tailore to the location of the uctal injury (B). For istal uct isruptions, a istal pancreatectomy is recommene (A), whereas for isruption of the boy, a Roux-en-Y pancreaticojejunostomy is performe (C). Whipple proceure (pancreaticouoenectomy) is not neee (D). Conservative therapy incluing somatostatin is successful in only approximately 50%, so nearly one-half will require an invasive proceure. References: Gómez-Cerezo J, Barbao Cano A, Suárez I, Soto A, Rios JJ, Vazquez JJ. Pancreatic ascites: Stuy of therapeutic options by analysis of case reports an case series between the years 1975 an 000. Am J Gastroenterol. 003;98(3):568–577. O’Toole D, Vullierme MP, Ponsot P, et al. Diagnosis an management of pancreatic stulae resulting in pancreatic ascites or pleural effusions in the era of helical CT an magnetic resonance imaging. Gastroenterol Clin Biol. 007;31(8–9 Pt 1):686–693.
26. B. Autoimmune pancreatitis is a form of chronic pan-
creatitis that is increasingly being recognize an can be confuse with pancreatic lymphoma or pancreatic cancer. It presents most often as a iffusely enlarge hypoechoic pancreas. A CT scan often shows iffuse narrowing of the main pancreatic uct without the typical calcications seen with chronic alcoholic pancreatitis. Pathology reveals a plasma cell an lymphocytic inltrate. Laboratory values reveal increase levels of IgG an often iabetes. Antiboies against lactoferrin an carbonic anhyrase have been reporte, but they are not a specic ning. The treatment of choice is steroi therapy, an the isease respons well to this management. Chemotherapy or invasive surgical/enoscopic proceures are not necessary (A, C–E). References: Ketikoglou I, Moulakakis A. Autoimmune pancreatitis. Dig Liver Dis. 005;37(3):11–15. Okazaki K. Autoimmune-relate pancreatitis. Curr Treat Options Gastroenterol. 001;4(5):369–375.
27. A. This presentation is concerning for elaye blee-
ing following a pancreaticouoenectomy (Whipple) proceure. This is most often ue to a gastrouoenal artery stump leak. Flui with HU >5 is most consistent with bloo. CT may show a pseuoaneurysm, but this may not always be present. On hospital ay 5, the tissue planes are often fragile, making it ifcult to control bleeing in the operating room (B). After resuscitation with bloo proucts, the most appropriate next step involves performing an angiography with embolization. A bleeing ulcer is also in the ifferential, but less likely in the absence of blooy nasogastric tube output an with CT nings, so upper enoscopy is not likely to be helpful (E). Esophagogastrouoenoscopy (EGD) nees to be selectively performe this early after surgery because the scope may compromise the freshly mae
gastrojejunostomy anastomosis if the afferent/efferent limbs are to be evaluate. Transfusion of bloo is appropriate but transport interventional suite shoul be next, as the patient may have a heral blee followe by exsanguination (D). Octreotie has no role in the management of gastrouoenal artery stump bleeing (C). One stuy emonstrate that wrapping the gastrouoenal artery stump using the falciform ligament uring surgery may ecrease the risk of this complication. References: Xu C, Yang X, Luo X. Wrapping the gastrouoenal artery stump uring pancreatouoenectomy reuce the stump hemorrhage incience after operation. Chin J Cancer. 014;6(3):99–308. Han GJ, Kim S, Lee NK, et al. Preiction of late postoperative hemorrhage after Whipple proceure using compute tomography performe uring early postoperative perio. Korean J Radiol. 018;19():84–91.
28. E. The ventral pancreas constitutes the uncinate
process an inferior portion of the hea of the pancreas, leaving the remainer the embryologic remnant of the orsal pancreas (C). The uncinate process lies ventral to the aorta but orsal to the portal vein an superior mesenteric artery. The most commonly injure vessel uring issection behin the neck of the pancreas is the superior mesenteric vein (A). The pancreas receives bloo supply from two sources: the celiac axis (superior pancreaticouoenal artery) an superior mesenteric artery (inferior pancreaticouoenal artery) (B). Venous rainage of the pancreas is to the portal system (D).
29. D. CT scan with IV contrast emonstrating areas of
hypoattenuation (nonperfuse) in the pancreas in a patient with this presentation is concerning ue to necrotizing pancreatitis. It is important to note that the necrotic pancreas is not usually infecte initially. Thus, initial management of necrotizing pancreatitis is conservative with the avoiance of early invasive interventions. FNA with culture might be consiere later (because infecte necrosis typically evelops weeks later) in the course of the hospitalization if the patient were to manifest evience of sepsis such as leukocytosis, tachycaria, refractory abominal pain, bacteremia, an/or persistent fevers (A). Prophylactic antibiotics for severe pancreatitis shoul not be routinely aministere (B). In patients with proven (via neele aspiration) infecte necrosis, minimally invasive percutaneous or enoscopic interventions (step-up approach) followe by vieo-assiste retroperitoneal ebriement with the goal of postponing or obviating the nee for open surgery is preferre (E). Furthermore, early necrosectomy has been shown to increase morbiity an mortality when compare with elaye intervention (C). In a patient that oes not appear to have an infecte necrotizing pancreatitis, it is appropriate to approach management conservatively with meical optimization an repeat CT scan if there is a eterioration in clinical status. It is best to allow the patient to manifest the severity of the isease before invasive interventions. References: Bugiantella W, Ronelli F, Boni M, et al. Necrotizing pancreatitis: a review of the interventions. Int J Surg. 016;8 Suppl 1:S163–S171. Mier J, León EL, Castillo A, Robleo F, Blanco R. Early versus late necrosectomy in severe necrotizing pancreatitis. Am J Surg. 1997;173():71–75.
CHAPtEr 5 Abdomen—Pancreas
63
30. A. Although elevation of alkaline phosphatase can be
31. E. Soli pseuopapillary epithelial neoplasm is A
rubin is a useful preictor of persisting common bile uct stone in gallstone pancreatitis. Am Surg. 008;74(10):977–980. Chang L, Lo SK, Stabile BE, Lewis RJ, e Virgilio C. Gallstone pancreatitis: a prospective stuy on the incience of cholangitis an clinical preictors of retaine common bile uct stones. Am J Gastroenterol. 1998;93(4):57–531.
lary epithelial neoplasm—A rare but curable pancreatic tumour in young women. S Afr J Surg. 011;49():78–81.
seen with a resiual common bile uct stone, the best preictor is a persistent elevation of the total bilirubin (C). Amylase is not typically elevate in this patient population (D). Because the pathophysiology of gallstone pancreatitis is transient obstruction of the ampulla of Vater by a gallstone, a signicant number of patients will have some egree of common bile uct ilation on amission; as such, common bile uct ilation is not a specic ning (B). This iffers from patients with symptomatic cholelithiasis, in which uctal ilation is frequently associate with common uct stones. Persistent abominal pain can occur as a result of multiple etiologies an shoul be appropriately worke up with history an physical, laboratory stuies, an/or imaging, if necessary (E). References: Chan T, Yaghoubian A, Rosing D, et al. Total bili-
rare tumor occurring almost exclusively in young women. It has low malignant potential an for the majority of patients, the tumor can be resecte with curative intent regarless of the size. Metastasis an recurrence are uncommon. Serous cystic aenoma also occurs most commonly in women, but this has no malignant potential an oes not nee to be resecte unless it is causing mass effect (A). Mucinous cystic neoplasm is consiere a premalignant lesion, has a female preominance, occurs commonly in the boy or tail of the pancreas, an shoul always unergo resection (B). Main-uct IPMN has a high risk of harboring malignant cells an shoul be resecte (D). Sie-uct IPMN can be manage conservatively unless it is symptomatic, larger than 3 cm, or associate with mural noules (C). Reference: Frost M, Krige JE, Bornman PC. Soli pseuopapil-
Abdomen—Spleen MARIA G. VALADEZ, BENJAMIN DIPARDO, AND ERIC R. SIMMS
6
ABSITE 99th Percentile High-Yields I. Anatomy an Physiology A. White pulp contains macrophages an both B an T lymphocytes B. Re pulp is responsible for removing eforme or abnormal RBCs an nuclear remnants foun in RBCs C. Splenocolic, gastrosplenic, splenorenal, an phrenicosplenic ligaments 1. Short gastric arteries are foun in the gastrosplenic ligament an can be a source of postoperative hemorrhage . Splenic artery lies anterior an superior to the splenic vein 3. Lack of normal peritoneal attachments results in a wanering spleen D. Accessory spleen 1. Suspecte if peripheral bloo smear not consistent with asplenia after splenectomy or if recurrence of primary pathology; splenic hilum is the most common location followe by tail of pancreas II. Splenic masses A. Most common benign splenic tumor: hemangioma B. Most common primary splenic tumor: non-Hogkin lymphoma C. Parasitic cysts (most common worlwie but rare in the Unite States): 1. Majority are hyati cysts seconary to echinococcus; treate with partial or total splenectomy ue to risk of rupture D. Nonparasitic cysts: 1. Cysts can be true cysts or pseuocysts, but this ifferentiation is ifcult to make preop; true cysts or primary cysts have epithelial lining an are congenital; pseuocysts or seconary cysts lack an epithelial lining an typically result from traumatic hematoma formation . If asymptomatic, can be observe with serial imaging regarless of size; if symptomatic, treate with partial or total splenectomy E. Splenic abscesses 1. Etiology: bacteremia, trauma, hemoglobinopathies, splenic artery embolization, following acute pancreatitis, immunosuppression, or trauma . Most common organism: Streptococcus pneumoniae 3. Treatment: IV antibiotics followe by splenectomy (gol stanar) 4. If poor surgical caniate with thick-walle unilocular abscess, treat with percutaneous rainage III. Splenectomy A. Vaccinations 1. Vaccinate against encapsulate organisms: Streptococcus pneumoniae, Neisseria meningitidis, an Haemophilus inuenzae; ieally weeks before surgery; weeks after if emergent
65
66
PArt i Patient Care
. Pneumococcal (PPSV3) vaccine shoulbegiven at least 8 weeks after the PCV13 vaccine with revaccination at 5 years; meningococcal (MenACWY) vaccination shoul be given 8 weeks after initial ose with revaccination every 5 years; also require yearly inuenza an COVID vaccination B. Postsplenectomy consierations 1. Overwhelming postsplenectomy sepsis (OPSI) (5 cm) an causing mass effect, resection recommene b) Enoscopic esophagoiverticulostomy: uses stapler or energy evices to ivie the cricopharyngeus enoscopically; ONLY for large iverticula >3 cm (if iverticulum too small, the cricopharyngeal myotomy will be incomplete) c) Open surgery has lower recurrence rates an slightly higher complication rates than enoscopic management V. Esophageal Tumors A. Leiomyoma (hypoechoic mass on enoscopic ultrasoun, more common in males) 1. Most common benign tumor of the esophagus, arise from the smooth muscle cells (mesenchymal); if 5 cm, surgically enucleate (via VATS or laparoscopy)
CHAPtEr 7 Alimentary Tract—Esophagus
77
B. Esophageal cancer 1. Aenocarcinoma an squamous; aenocarcinoma more common in the Unite States . Use enoscopic ultrasoun to etermine T stage, CT for N stage 3. Localize: limite to the mucosa or invaing lamina propria (an N0, M0) a) Enoscopic resection for T1a = within the mucosa (to the lamina propria or muscularis mucosa) b) Esophagectomy for T1b = within the submucosa c) Neoajuvant chemoraiation rst if >T = to the muscularis propria 4. Regional: (noal isease but M0) a) Neoajuvant chemoraiation rst, followe by esophagectomy b) Nee 15 noes for proper oncologic staging 5. Distant (palliative care) 6. Esophagectomy, transhiatal approach (left cervical an abominal incisions) or transthoracic approach (right thoracotomy an abominal incision); similar outcomes, transhiatal may be associate with shorter length of hospital stay 7. Conuit choices after esophagectomy inclue the stomach (most common, nee to preserve the right gastroepiploic artery for perfusion), the colon, an the jejunum 8. Special circumstances a) Upper esophageal cancer (within 5 cm of the upper esophageal sphincter); chemoraiation as primary treatment moality b) Distal esophageal cancer (within 5 cm of GE junction) (1) Some may originate in esophagus an others in stomach () Depening on location, may sprea to meiastinal or abominal noes (3) Most receive neoajuvant or perioperative chemoraiation prior to surgery VI. Esophageal Perforation A. Etiologies: iatrogenic by instrumentation (most common at cricopharyngeus), spontaneous (Boerhaave, occurs 5 cm above GE junction on left), trauma, caustic ingestion B. Workup: CXR (may see left-sie effusion), esophagram with gastrogran → if negative or inconclusive: esophagram with thin barium 1. Gastrogran major side effect: pneumonitis if aspirated . Barium major side effect: severe peritonitis/mediastinitis C. Management: start with NPO, IV ui resuscitation, broa-spectrum antibiotics incluing antifungals D. Nonoperative management: if patient hemoynamically stable, not septic, with mil symptoms, an a containe perforation (minimal meiastinal contamination) E. Surgical approach: right posterolateral thoracotomy for proximal or mile one-thir, left posterolateral thoracotomy for lower one-thir F. Management epens on etiology of perforation 1. Malignant obstruction a) Early cancer: perform esophagectomy b) Avance cancer: esophageal stenting . Benign obstruction a) If ue to achalasia, perform myotomy on contralateral sie 3. Normal esophagus G. Repair of perforation a) Exten myotomy to expose full length of mucosal injury (musical injury often longer than muscle injury) b) Debrie all nonviable tissue c) Two-layer closure (mucosa with absorbable suture an muscle with nonabsorbable suture) ) Reinforce repair with intercostal ap
78
PArt i Patient Care
Fig. 7.1
CHAPtEr 7 Alimentary Tract—Esophagus
79
Questions 1. A 40-year-ol woman with a history of Raynau isease presents for evaluation of ysphagia. Barium esophagram shows reux but no structural abnormalities, an upper enoscopy shows reux esophagitis. Manometry shows absent peristalsis of the istal esophagus an ecrease tone of the lower esophageal sphincter. Initial management consists of: A. Laparoscopic Heller myotomy with partial funoplication B. Proton pump inhibitor an metoclopramie C. Pneumatic ilation D. Calcium channel blocker E. Nissen funoplication 2. A 63-year-ol man with a history of GERD presents for progressive ysphagia over the past 4months. Esophagram shows an irregular lesion in the istal esophagus. Enoscopy with biopsy an enoscopic ultrasoun conrms a cm irregular mass with invasion into the submucosa. Imaging oes not show any istant masses or abnormal lymph noes. What is the rst step in treatment? A. Chemotherapy B. Chemoraiation C. Enoscopic resection D. Enucleation E. Esophagectomy 3. A 0-year-ol man presents to your ofce after being hospitalize for lye ingestion. Esophagoscopy reveale a high-grae esophageal caustic injury, an the patient was treate with supportive care. The patient asks about the longterm complications after caustic injury to the esophagus. Which of the following is true? A. He is at increase risk for esophageal aenocarcinoma B. He shoul unergo enoscopic surveillance beginning 6 weeks after ingestion C. The most common complication is an esophageal stricture D. Ault caustic ingestion is less severe than peiatric ingestion E. Early use of a neutralizing agent ecreases the risk of subsequent stricture formation
4. A 35-year-ol woman is in the recovery room after enoscopic ilation of a peptic stricture in the mi-thoracic esophagus. She begins to complain of chest pain an oynophagia. She is hemoynamically stable an chest raiograph oes not show any free air or pleural effusion. Esophagram with water-soluble contrast is normal. What is the most appropriate next step in management? A. CT scan of the chest, abomen, an pelvis with IV an oral contrast B. Esophagram with thin barium C. Nasogastric tube placement D. Left posterolateral thoracotomy E. Right posterolateral thoracotomy 5. Which of the following is true regaring the surgical approach, anatomy, or bloo supply to the esophagus? A. Outer longituinal muscle is an extension of the cricopharyngeus muscle B. Cervical esophagus is supplie by the inferior thyroi artery C. The narrowest point in the esophagus is at the aortic arch D. Branches off the intercostal arteries are the major bloo supply to the thoracic esophagus E. The stanar surgical approach to the miesophagus is a left thoracotomy 6. Which of the following statements is true about Mallory-Weiss synrome? A. The chief pathologic ning is spontaneous perforation of the esophagus B. It typically occurs on the right sie C. It is usually associate with air in the meiastinum D. Enoscopy shoul be performe to conrm the iagnosis E. Esophageal balloon tamponae is an appropriate option in cases of persistent bleeing
80
PArt i Patient Care
7. A 40-year-ol female has been using a proton pump inhibitor (PPI) to control gastroesophageal reux isease (GERD) for the past 7 years. She is otherwise healthy. She was seen in clinic an eeme a suitable caniate for enitive surgical intervention. During the operation, after the phrenoesophageal ligament is mobilize, her istal esophagus is inspecte, an it appears shortene. Preoperative upper gastrointestinal stuy i not ientify a hiatal hernia. Which of the following will most likely nee to be one? A. Procee with a stanar Nissen funoplication B. Procee with a Dor funoplication C. Perform Collis gastroplasty D. Abort the operation an initiate management with sterois E. Take several biopsies before aborting the operation
11. Esophageal manometry performe in a patient with a true paraesophageal hernia will emonstrate that the LES is: A. Above the normal position B. At the normal position C. Hypertensive D. Hypotensive E. Short
8. A 51-year-ol male has been unergoing yearly enoscopy with biopsy for Barrett esophagus (BE). His most recent biopsy emonstrates highgrae ysplasia without noules. Which of the following is the best next step in management? A. Esophagectomy with reconstruction B. Repeat enoscopy with biopsy in 3 months C. Enoscopic raiofrequency ablation D. Antireux operation E. Oncology referral for consieration of neoajuvant chemotherapy
13. Which of the following will preispose a patient to the evelopment of esophageal isease? A. LES length of 3 cm B. Resting LES pressure of 8 mm Hg C. Resting upper esophageal sphincter (UES) pressure of 70 mm Hg D. Abominal length less than 1 cm E. Relaxation of LES with swallowing
9. Barrett esophagus: A. Is a congenital abnormality B. Occurs more frequently in women C. When iagnose, shoul be treate with an antireux proceure to prevent cancer D. Diagnosis requires replacement of a 3-cm segment of the squamous cells by columnar epithelium E. Features the presence of goblet cells 10. Which of the following is true regaring Barrett esophagus? A. PPIs are consiere a more effective treatment option than H blockers B. Dietary restrictions such as those use for patients with GERD are not useful C. Patients with short- an long-segment Barrett esophagus have a similar risk of high-grae ysplasia D. Use of high-ose PPIs with aspirin is contrainicate E. Photofrin is a useful treatment moality
12. Which of the following statements about a paraesophageal hernia is true? A. It is associate with anemia B. It oes not pose a risk for incarceration an strangulation C. Diagnosis is not reaily mae with upper enoscopy D. It is usually cause by a traumatic injury E. It rarely requires operative repair
14. A 5-year-ol male with cirrhosis an known esophageal varices presents with a large amount of hematemesis. Which of the following statements is true? A. Beta blockae is ineffective for preventing rebleeing B. The most important next step is enoscopy for both iagnostic an therapeutic interventions C. Prophylactic antibiotics o not improve survival D. Early aministration of vasoactive rugs oes not improve outcomes E. Enoscopic ban ligation has been emonstrate to be superior to enoscopic sclerotherapy
CHAPtEr 7 Alimentary Tract—Esophagus
15. A 59-year-ol iabetic male with a history of chronic obstructive pulmonary isease (COPD) an prior congestive heart failure presents with a -year history of progressively ifcult swallowing. Esophagram emonstrates a ilate proximal esophagus with abrupt tapering istally. Manometry shows high pressure in the lower esophageal sphincter (LES) at rest an failure of the LES to relax after swallowing. Upper enoscopy is negative. Which of the following is true regaring this patient? A. The unerlying conition is characterize by high-amplitue peristaltic waves of the esophagus B. Laparoscopic esophagomyotomy with complete funoplication is the treatment of choice C. A trial of calcium channel blockers shoul be starte D. Esophageal pneumatic ilation is the next step in management E. Peroral enoscopic myotomy (POEM) is the treatment of choice 16. During the course of an upper enoscopy for manometry conrme achalasia, the enoscopist thinks he may have cause an inavertent perforation of the left lower istal esophagus. The patient is stable an shows no signs of sepsis. Esophagogram conrms a markely ilate esophagus with a istal-free perforation. Management consists of: A. Intravenous (IV) antibiotics, placing patient NPO (nothing by mouth), an close observation B. Left thoracotomy, primary repair, longituinal myotomy on the contralateral sie C. Laparoscopic primary repair an longituinal myotomy on the ipsilateral sie D. Esophagectomy with immeiate reconstruction E. Esophageal stent placement 17. A 36-year-ol male presents for consultation regaring an inciental esophageal mass seen on compute tomography (CT) scan. This was performe after he was involve in a motor vehicle collision (MVC). He ha no serious injuries an was ischarge the same ay. Barium swallow emonstrates a smooth, crescent-shape lling efect. Which of the following is true regaring this mass? A. Resection with a 1-cm margin is the treatment of choice B. They most commonly present with satellite tumors C. They have no risk of malignant egeneration D. Esophageal ultrasonography may be useful E. A preoperative enoscopic biopsy shoul be performe
81
18. Which of the following statements is true about Zenker iverticulum? A. It is a true iverticulum B. It is best iagnose with esophagoscopy C. It is unlikely to cause aspiration D. It is a pulsion iverticulum E. Small iverticula (5 cm). Otherwise, they are incientally iscovere uring the course of other stuies. They have a characteristic appearance on barium swallow of a smooth, crescent-shape lling efect that encroaches on the lumen. On enoscopy, the mucosa is usually intact, an the tumor moves up an own with swallowing. If it has the characteristic appearance, the tumor shoul not unergo biopsy because of an increase risk of mucosal perforation. This can create scarring that may affect later efforts at resection (E). Esophageal ultrasonography is very useful in the iagnosis of leiomyomas because it will emonstrate a homogeneous region of hypoechogenicity. Treatment is to enucleate the mass, which can be one via a vieoscopic approach with intraoperative esophagoscopy (A). The cell of origin of these tumors is mesenchymal. The average age at presentation is 38 years, an they are twice as common in males an most commonly locate in the lower two-thirs of the esophagus. Leiomyomas are usually solitary, but multiple tumors are seen in as many as 10% of patients (B). Reference: Aurea P, Grazia M, Petrella F, Bazzocchi R. Giant leiomyoma of the esophagus. Eur J Cardiothorac Surg. 00;(6):1008–1010.
18. D. A Zenker iverticulum is a false esophageal iver-
ticulum that oes not contain all layers of the esophagus; it is also a type of pulsion iverticulum (A). A pulsion iverticulum forms at a point of weakness an is ue to alterations in luminal pressure. Conversely, a traction iverticulum is from external pulling on the esophageal wall, such as from iname lymph noes with tuberculosis. Zenker iverticulum is the most common type of esophageal iverticulum. It usually presents in oler patients (>60 years). It characteristically arises at a point of weakness, most commonly at the Killian triangle, which is forme by the inferior bers of the inferior constrictor muscle an the superior borer of the cricopharyngeus muscle. Patients typically present with ysphagia, regurgitation of unigeste foo, halitosis, episoes
86
PArt i Patient Care
of aspiration, an salivation (C). With the characteristic history, the rst iagnostic stuy is a barium swallow. In the absence of other pathology (such as an irregular mucosa), enoscopy is not neee (B). Treatment is surgical by either open or enoscopic techniques. The open technique involves cervical esophagomyotomy with stapling an amputation of the iverticulum. The enoscopic technique involves ivision of the common wall between the iverticulum an the esophagus. Stuies have shown that results with the enoscopic technique are better with larger iverticula (E). Diverticula smaller than 3 cm are too short to accommoate one cartrige of staples an to allow complete ivision of the sphincter; therefore, this size is consiere a contrainication to this technique. References: Bonavina L, Bona D, Abraham M, Saino G, Abate E. Long-term results of enosurgical an open surgical approach for Zenker iverticulum. World J Gastroenterol. 007;13(18):586–589. Collar JM, Otte JB, Kestens PJ. Enoscopic stapling technique of esophagoiverticulostomy for Zenker’s iverticulum. Ann Thorac Surg. 1993;56(3):573–576. Narne S, Cutrone C, Bonavina L, Chella B, Peracchia A. Enoscopic iverticulotomy for the treatment of Zenker’s iverticulum: results in 10 patients with staple-assiste enoscopy. Ann Otol Rhinol Laryngol. 1999;108(8):810–815.
19. A. Cricopharyngeal ysfunction has multiple causes,
incluing such neurogenic an myogenic etiologies as stroke, multiple sclerosis, peripheral neuropathy, Parkinson isease, an ermatomyositis. The exact cause is unknown, but the primary theory is that the cricopharyngeus muscle, which is normally in a state of tonic contraction, fails to relax an allow the passage of foo into the cervical esophagus. This prouces a Zenker iverticulum, which is consiere a false iverticulum (only involves an outpouching of the mucosa an submucosa) an can be conrme with a barium swallow (C). Enoscopic evaluation of a suspecte Zenker iverticulum is iscourage as it can lea to an iatrogenic perforation. Patients escribe ifculty swallowing foo, which worsens throughout the ay as the iverticulum increasingly gets lle with foo. Another key element of the iagnosis is the classic history of an inability to hanle saliva secretion, such that the patient escribes expectoration of saliva. Patients also report hoarseness. Diverticulectomy is often performe uring surgery for a Zenker iverticulum. However, the most important aspect of management is cricopharyngeal myotomy, which is necessary to correct the unerlying pathology (B). Weight loss results from a ecrease caloric intake. Although one shoul always be suspicious of carcinoma in a patient with ifculty swallowing an weight loss, the long uration of symptoms makes carcinoma unlikely (D, E). References: Cameron JL, Cameron AM. The management of Barrett’s esophagus. In: Cameron JL, Cameron AM, es. Current surgical therapy. 11th e. Philaelphia: W.B. Sauners; 014. Cameron JL, Cameron AM. The management of pharyngeal esophageal (Zenker) iverticula. 11th e. Philaelphia, PA: W.B. Sauners; 014.
20. B. Patients with newly iagnose esophageal cancer
frequently present with poor nutritional status, which only worsens after starting neoajuvant therapy. As such, although nutritional optimization is an important component in the management of esophageal cancer, the optimal approach remains unene. Percutaneous gastrostomy, however, shoul be iscourage because it may compromise the gastric conuit neee uring esophageal reconstruction an will elay chemotherapy for an aitional to 4weeks. The role for parenteral nutrition is limite because of its high cost an high rate of complications (A). Nasogastric tube insertion can lea to migration of the tube an aspiration (D). Esophageal stents are frequently offere because they can signicantly improve the ysphagia associate with esophageal cancer. Unfortunately, its role in improving nutritional status has ha inconsistent results in the literature (C). Stent migration an chest iscomfort are common an lea to the frequent removal of the stents (E). Aitional stuies are neee to etermine the best approach for nutritional optimization in this patient population. References: Jones CM, Grifths EA. Shoul oesophageal stents be place before neo-ajuvant therapy to treat ysphagia in patients awaiting oesophagectomy? Best evience topic (BET). Int J Surg. 014;1(11):117–1180. Mão-e-Ferro S, Serrano M, Ferreira S, et al. Stents in patients with esophageal cancer before chemoraiotherapy: high risk of complications an no impact on the nutritional status. Eur J Clin Nutr. 016;70(3):409–410. Naharaja V, Cox MR, Eslick GD. Safety an efcacy of esophageal stents preceing or uring neoajuvant chemotherapy for esophageal cancer: a systemic review an meta-analysis. J Gastrointest Oncol. 014;5():119–16.
21. C. Surgical intervention in esophageal cancer is an
area of active research. The three stanar approaches inclue TTE, THE, an a combination of the two using a three-incision esophagectomy. TTH was initially escribe as a two-stage proceure by Dr. Ivor Lewis in which he performe mobilization of the stomach using an upper miline laparotomy incision followe by resection of the esophagus using a right thoracotomy incision several ays later (A). A large multicenter prospective stuy comparing THE an TTE faile to emonstrate any ifference in overall mortality an morbiity between the two approaches (D). However, THE has been shown in several stuies to be associate with a lower total hospital length of stay (B). THE is performe with a left cervical incision an miline laparotomy (E). It is often performe for patients with istal esophageal cancer. References: D’Amico TA. Outcomes after surgery for esophageal cancer. Gastrointest Cancer Res. 007;1(5):188–196. Hulscher JB, Tijssen JG, Obertop H, van Lanschot JJ. Transthoracic versus transhiatal resection for carcinoma of the esophagus: a meta-analysis. Ann Thorac Surg. 001;7(1):306–313. Litle VR, Buenaventura PO, Luketich JD. Minimally invasive resection for esophageal cancer. Surg Clin North Am. 00;8(4):711–78. Rentz J, Bull D, Harpole D, et al. Transthoracic versus transhiatal esophagectomy: a prospective stuy of 945 patients. J Thorac Cardiovasc Surg. 003;15(5):1114–110.
Alimentary Tract—Stomach NAVEEN BALAN, AMY KIM YETASOOK, AND KATHRYN T. CHEN
8
ABSITE 99th Percentile High-Yields I. Ulcers A. Peptic ulcer isease (PUD): imbalance of pepsin/aci an mucosal protection 1. Almost always cause by Helicobacter pylori (gram-negative spirochete) an NSAID overuse . Triple therapy: PPI, clarithromycin, amoxicillin, or metroniazole ×14 ays 3. Daintree Johnson classication for types of gastric ulcers a) Type I: along lesser curvature in the antrum, solitary, not aci associate b) Type II: prepyloric, solitary, aci associate c) Type III: prepyloric an uoenal, one in each location, aci associate ) Type IV: proximal stomach/caria, solitary, not aci associate e) Type V: anywhere in the stomach, usually multiple, NSAID associate 4. Biopsy all gastric ulcers: higher risk of cancer
Operation
Procedure
Result
Complication
Antrectomy
Resection of distal stomach (need reconstruction with Billroth I or II)
Removal of antral G cells
Incomplete antrectomy can lead to continued ulcer disease
Truncal vagotomy (TV)
Ligation of anterior and posterior vagal trunks 4 cm proximal to GEJ
Removal of vagal stimulation to gastric body parietal cells, decreasing acid secretion
Loss of parasympathetic innervation to pylorus which prevents gastric emptying (primarily to solid foods)
Selective vagotomy (SV)
Ligation of anterior and posterior vagal trunks distal to hepatic/ celiac branches
Lower rate of delayed gastric emptying but higher risk of recurrent ulcer disease
Highly selective vagotomy (HSV)
Similar to SV except preserves crow’s foot ębers innervating pylorus and antrum
Lowest rate of delayed gastric emptying but highest risk of recurrent ulcer disease
Pyloroplasty
Widen the pyloric channel, multiple approaches
Improved gastric emptying (usually with TV/SV)
Can lead to rapid gastric emptying
B. Surgical inications: surgery for bleeing PUD entails oversewing of the ulcer (to ligate the bleeing artery) with consieration for truncal vagotomy (to ecrease aci secretion); more time-consuming proceures (selective vagotomy) reserve for elective ulcer surgery, as are antrectomy an Billroth II (when obstruction complicates PUD)
87
88
PArt i Patient Care
1. Truncal vagotomy an selective vagotomy always nee rainage proceure (e.g., pyloroplasty) whereas highly selective vagotomy oes not
Duodenal Ulcer* Problem
Treatment
Bleeding
Oversew bleeder
Perforation
Graham patch
*For gastric ulcer, ęrst-line treatment is same; however, ęrst biopsy ulcer to rule out malignancy.
II. Upper Gastrointestinal Blee A. Meical management: hyration, PPI bi (noninferior to continuous PPI) ×7 hours B. Proceural interventions: enoscopic hemostasis attempts ×, angioembolization(usually GDA); n look EGD only for high-risk lesion (visible vessel or aherent clot) C. Surgery inications: multiple faile enoscopies, hemoynamic instability, associate perforation III. Bariatric Surgery A. Inications: 1. BMI >40 (morbi obesity) or BMI 35 to 40 with ≥1 weight-relate comorbiity (iabetes, hypertension, hyperlipiemia, GERD, obstructive sleep apnea, pseuotumor cerebri, severe lifelimiting arthritis) B. Sleeve gastrectomy: most commonly performe, lower excess boy weight (EBW) loss an incience of vitamin eciency; postoperative complications inclue staple line leak, bleeing, e novo reux, incisura stricture, twisting/kinking of sleeve, weight regain C. Roux-en-Y gastric bypass: higher EBW loss an incience of vitamin eciency; postoperative complications inclue anastomotic leak, internal hernia, marginal ulcer, perforate marginal ulcer, bile reux, jejunojejunostomy, intussusception, roux stasis synrome, gastrogastric stula, umping synrome, an weight regain D. Pulmonary embolus is leaing cause of eath, occurring within a few weeks after ischarge E. Common postoperative complications:
Complication
Symptoms
Etiology
Management
Alkaline reĚux (requires surgery more than any other postgastrectomy syndrome)
Bilious emesis does NOT relieve epigastric pain
Bile reĚux into the stomach usually with Billroth II anatomy
Workup with EGD (pathology: foveolar hyperplasia with mucosal congestion and edema); conęrm with HIDA demonstrating biliary secretion into stomach/esophagus; treatment: convert to Roux-en-Y
Small intestinal bacterial overgrowth (SIBO)
Bloating, chronic watery diarrhea
Bacterial overgrowth in blind loop due to immotility
Labs for B12 deęciency, macrocytic anemia, conęrm with lactulose breath test and treat with antibiotics (rifaximin)
Early dumping syndrome
Nausea, vomiting, diarrhea 15–30 min postprandially
Hyperosmolar load and release of serotonin, neurotensin, bradykinins, and enteroglucagon
Eat smaller, more frequent meals with low carbs, high protein, and fat throughout the day, separate solids from liquids, octreotide is most eěective
Late dumping syndrome
Dizziness, fatigue, diaphoresis 1–3 h postprandially
High carbohydrate load causing hyperinsulinemia leading to reactive hypoglycemia
Similar to above; if refractory, convert to Roux-en-Y
Marginal ulcer
Nausea, pain, and perforation/bleeding at anastomosis
NSAIDs, smoking, H. pylori infection, acid exposure to small bowel
Discontinue NSAIDs, treat H. pylori, give acid suppression therapy, bleeding or perforation may need surgery
CHAPtEr 8 Alimentary Tract—Stomach
89
IV. Gastric Cancer A. Aenocarcinoma: risk factors inclue H. pylori infection, iet (high salt, nitrosamine-containing foos), obesity, smoking, ETOH, hereitary iffuse gastric cancer (CDH1 mutations) 1. Staging: EGD for biopsy, EUS for T/N stage, CT chest/abomen/pelvis, +/− PET-CT, iagnostic laparoscopy to evaluate for peritoneal isease . Early gastric cancer: cTis or T1a-enoscopic or surgical resection; T1b-surgical resection with 5 cm gross margins to achieve R0 resection 3. Locoregional isease: T+ or noal involvement nee neoajuvant chemotherapy followe by surgical resection an ajuvant chemotherapy 4. Surgical options: istal versus subtotal versus total gastrectomy with R0 goal, nee 16 lymph noes; CDH1 nees total gastrectomy, also acceptable option for palliation in stage IV isease 5. Siewert-Stein classication: a) Type I: epicenter within 1 to 5 cm above GE junction, treat as esophageal cancer b) Type II: epicenter within 1 cm above an cm below GE junction, treat as esophageal cancer c) Type III: epicenter within to 5 cm below GE junction, treat as stomach cancer 6. Incience of PROXIMAL gastric cancer has been rising the fastest (ue to obesity an reux); proximal gastric cancer is two times more common in men an white race 7. H. pylori is PROTECTIVE against proximal gastric cancer but associate with increase risk of istal gastric cancer B. Gastrointestinal stromal tumor (GIST) 1. Enoscopy: submucosal mass with central umbilication an ulceration . Essential features: majority ue to mutations in protooncogene KIT (most common) or PDGFRA; spinle cell histology; can occur anywhere along GI tract, but stomach is most common (60%) followe by jejunum/ileum 3. High-risk features for recurrence: tumor size >5 cm or >5 mitoses/hpf 4. Imatinib: tyrosine kinase inhibitor, highest response rate with exon 11 KIT mutation a) Neoajuvant therapy: use when surgical resection will result in signicant morbiity b) Ajuvant therapy: high-risk features for recurrence an tumor rupture uring surgery C. MALT lymphoma: B-cell lymphoma, arises in the setting of chronic H. pylori infection 1. Histology: lymphoepithelial lesions are pathognomonic . Treatment: triple or quaruple antibiotic therapy will eraicate cancer in majority of early cases
90
PArt i Patient Care
Fig. 8.1
CHAPtEr 8 Alimentary Tract—Stomach
Fig. 8.2
91
92
PArt i Patient Care
Questions 1. A 56-year-ol male with fatigue unergoes upper enoscopy after initial workup shows a microcytic anemia. He is foun to have an ulcer with irregular borers with biopsy showing a ense lymphoi inltrate with prominent lymphoepithelial lesions. Which of the following is true for this malignancy? A. Tumors with chromosomal translocation t(11;18) respon poorly to antibiotics B. After successful treatment, yearly enoscopy is use for surveillance C. It is most commonly a result of a grampositive ro D. There is no role for surgical resection E. Early-stage isease requires chemoraiation as rst-line treatment 2. A 46-year-ol female was incientally foun to have a peunculate mass along the greater curvature of the stomach on CT imaging following a motor vehicle collision. Further workup with enoscopy shows a submucosal mass with central umbilication an ulceration that is foun to be CD117-positive after biopsy. Which of the following is true regaring the management of this lesion? A. It is consiere a raiosensitive tumor B. The highest response rate to therapy involves mutations in the KIT proto-oncogene at exon 11 C. It arises from an enoermal-erive component D. Most patients become symptomatic early in the course of their isease E. Early tumors can be treate with enoscopic mucosal resection
3. A 41-year-ol female presents to the emergency epartment with acute severe abominal pain an nausea but no vomiting. She reports a history of uncomplicate Roux-en-Y gastric bypass years ago but enies other abominal surgeries. Abominal exam reveals mil tenerness without guaring or reboun tenerness. A CT scan shows ilate proximal small bowel but no intraabominal free air or ui. What is the next best step in the management of this patient? A. Avise the patient to eat smaller, more frequent meals B. Exploratory laparotomy C. Nasogastric tube ecompression followe by 4-hour water-soluble contrast challenge D. Antibiotic therapy E. Upper enoscopy 4. A 40-year-ol male with severe epigastric pain is foun to have multiple uoenal ulcers on EGD. What is the normal location of the cells that secrete the majority of the hormone that is being overprouce in this patient? A. Stomach boy B. Stomach antrum C. Pancreas D. Duoenum E. Jejunum 5. Three years after a laparoscopic Roux-en-Y gastric bypass (LRYGB), a 45-year-ol male presents with symptoms an signs of a small bowel obstruction (SBO). He reports a 150-lb weight loss. Which of the following is the most likely etiology? A. An internal hernia B. Ahesions C. Roux compression ue to mesocolon scarring D. Kinking of the jejunojejunostomy E. Incarcerate abominal wall hernia
CHAPtEr 8 Alimentary Tract—Stomach
6. A 79-year-ol male with chronic back pain an chronic obstructive pulmonary isease (COPD) requiring supplemental oxygen presents to the emergency epartment (ED) with epigastric abominal pain that starte suenly ays ago. His abominal examination is signicant for epigastric tenerness but is otherwise unremarkable. A compute tomography (CT) scan emonstrates a small amount of free air uner the right hemiiaphragm but no contrast extravasation. An upper gastrointestinal (GI) water-soluble contrast stuy emonstrates a uoenal ulcer but no extravasation. Which of the following is the best management? A. Nasogastric tube ecompression, intravenous (IV) antibiotics, an proton pump inhibitor (PPI) B. Exploratory laparotomy C. Diagnostic laparoscopy D. Oral antibiotics, clear liqui iet for weeks, an follow-up in clinic E. Serial abominal exam in the ED for 6 to 8hours an, if improving, he may be ischarge with oral antibiotics 7. Which of the following is true regaring the management of obesity? A. Inications for bariatric surgery inclue a boy mass inex (BMI) greater than 30 with weight-relate comorbiities or BMI greater than 35 B. Sibutramine acts by inhibiting pancreatic lipase C. Roux-en-Y gastric bypass (RYGB) oes not have a restrictive component D. RYGB has a lower 30-ay mortality compare with biliopancreatic iversion (BPD) E. Patients with obesity-relate comorbiities o not nee to attempt nonoperative management before obesity surgery 8. Which of the following is the gol stanar for the iagnosis of gastroparesis? A. Upper enoscopy B. Plain abominal x-rays C. Nuclear meicine scan D. CT E. It is consiere a clinical iagnosis.
93
9. Which of the following is true regaring gallstone isease after weight loss surgery? A. The rate of postoperative cholecystectomy is the same regarless of the type of weight loss surgery B. Prophylactic cholecystectomy shoul be performe at the time of surgery in most patients C. Ursoiol is recommene for 6 months after gastric bypass surgery D. Decrease secretion of calcium an mucin contributes to gallstone formation after weight loss surgery E. Acute cholecystitis after weight loss surgery is uncommon 10. Which of the following is the rst manifestation of gastric leak following Roux-en-Y gastric bypass? A. Abominal pain B. Tachycaria C. Nausea D. Increase serum glucose E. Tachypnea 11. A 45-year-ol male with a history of laparoscopic gastric baning 5 years ago presents to the ED with complaints of pain at his port site. He rst notice it several ays ago after he got his gastric ban ajuste in clinic. On exam, the port site appears erythematous, warm, an is tener to palpation. He is afebrile an normotensive. Which of the following is the best next step? A. CT of the abomen B. Amit to the hospital, start IV antibiotics an ui resuscitation C. EGD D. Incision an rainage E. Discharge with oral antibiotics
94
PArt i Patient Care
12. A 60-year-ol man presents with a 1-hour history of worsening epigastric pain. He has a history of uoenal ulcer, an the results of a recent biopsy weeks earlier were negative for Helicobacter pylori. Upright chest raiograph emonstrates free air uner the iaphragm. The patient is hemoynamically stable. At surgery, a perforate uoenal ulcer is foun with mil peritoneal contamination. Which of the following is the best management option? A. Graham patch of uoenal ulcer B. Graham patch of uoenal ulcer with truncal vagotomy an pyloroplasty C. Truncal vagotomy an antrectomy with Billroth I reconstruction D. Truncal vagotomy an antrectomy with Billroth II reconstruction E. Graham patch of uoenal ulcer with a highly selective vagotomy 13. Which of the following is true regaring postvagotomy iarrhea? A. It is effectively treate with octreotie B. It oes not improve with oral cholestyramine C. Cariovascular manifestations are common D. Most patients require the creation of a reverse jejunal segment E. Diarrhea may improve with the aministration of coeine 14. A 45-year-ol woman is unergoing an exploratory laparotomy for Zollinger-Ellison synrome (ZES). Preoperative localization stuies faile to emonstrate the location of the tumor. At surgery, no obvious tumor is seen espite an extensive Kocher maneuver an careful inspection. An intraoperative ultrasoun scan is negative. The next step in the management woul be: A. Closing the abomen B. Distal pancreatectomy an splenectomy C. Proximal pancreaticouoenectomy D. Blin proximal uoenotomy E. Blin istal uoenotomy
15. Which of the following is true regaring TNM (tumor, noes, an metastases) staging for gastric aenocarcinoma? A. Compute tomography scan is the most accurate means of etermining T an N staging B. The accuracy of enoscopic ultrasoun (EUS) is higher for N stage than T stage C. Fifteen lymph noes are require for an oncologic resection to appropriately stage the patient D. Magnetic resonance imaging (MRI) with gaolinium shoul be routinely performe E. T3 invaes the subserosa 16. Which of the following is associate with hypergastrinemia? A. Diabetes B. Hypothyroiism C. Hyperparathyroiism D. Chronic gastritis E. D-cell hyperplasia 17. A 46-year-ol male unergoes a istal gastrectomy for a tumor in the gastric antrum that was biopsy proven to be aenocarcinoma. The specimen is sent for pathology. Pathology reveals microscopic evience of tumor at the margins. Which of the following most accurately escribes this resection? A. D1 resection B. D resection C. R0 resection D. R1 resection E. R resection 18. Which of the following is consiere to be a risk factor for gastric cancer? A. Pernicious anemia B. Bloo group O C. Carbonate aciic soa D. Female sex E. Diabetes 19. Which of the following is true regaring the types of gastric ulcers? A. Type II ulcers are the most common B. Type IV ulcers occur near the gastroesophageal junction C. Type I ulcers usually have increase aci secretion D. Type III ulcers are associate with ecrease aci secretion E. Type I gastric ulcers are prepyloric
CHAPtEr 8 Alimentary Tract—Stomach
95
20. Which of the following is true regaring gastrointestinal stromal tumor (GIST)? A. The extent of the tumor is best etermine preoperatively by enoscopy B. They arise from smooth muscle cells C. Malignant potential is reaily etermine by histologic features D. They can be manage by laparoscopic wege resection E. They rarely present with GI bleeing
26. Which of the following is true regaring ZES? A. Symptoms ecrease with fasting B. Ulcers are most often locate in the istal uoenum C. It is most commonly familial D. It is the most common functional neuroenocrine tumor E. Treatment with proton pump inhibitors (PPIs) can control symptoms in the majority of patients
21. Which of the following is true regaring postgastrectomy bile reux? A. It is more likely to occur after a Billroth I than a Billroth II reconstruction B. Symptoms usually correlate with the amount of bile entering the stomach C. In symptomatic patients, meical management is generally effective D. Creation of a Roux-en-Y gastrojejunostomy is an effective surgical option E. Most patients with bile reux into the stomach will evelop symptoms
27. A 70-year-ol man presents with an 8-hour history of acute abominal pain an a history of melena. On examination, the patient is febrile to 101°F, with a bloo pressure of 105/70 mmHg an a heart rate of 130 beats per minute an has iffuse abominal tenerness with reboun an guaring. The rectal examination is guaiac positive. Laboratory values are signicant for a white bloo cell count of 16,000 cells/μL an a hematocrit of 6%. CT emonstrates extravasation of oral contrast in the proximal uoenum. After resuscitation, management consists of: A. Closure of the perforation with omental patch plus an HSV B. Closure of the perforation an omental patch via the open approach C. Perform uoenotomy over perforation, oversew posterior ulcer, close uoenotomy, an place omental patch D. Vagotomy an antrectomy with oversewing of the posterior ulcer an omental patch E. Closure of the perforation an omental patch via laparoscopic approach
22. The best test for localization of a gastrinoma is: A. MRI B. CT C. Abominal ultrasoun D. Octreotie scan E. Selective angiography 23. The best test to conrm eraication of H. pylori after treatment is: A. H. pylori serology B. Urea breath test C. Rapi urease test D. Histologic biopsy E. Antral mucosal biopsy with culture 24. Which of the following is true regaring a highly selective vagotomy (HSV)? A. The anterior an posterior vagal trunks are ivie B. The nerve of Grassi is spare C. The anterior Latarjet nerve is ivie D. The crow’s feet to the antrum are spare E. The celiac branch is ivie 25. The most common metabolic isorer after gastric resection is a eciency of: A. Iron B. Vitamin B1 C. Folate D. Calcium E. Vitamin D
28. A 50-year-ol woman with a history of iabetes presents with symptoms of early satiety, nausea, vomiting, an epigastric pain. Upper enoscopy reveals a large mass of unigeste foo particles in the stomach that is partially obstructing the pylorus. Which of the following is true regaring this conition? A. Most patients require surgery B. It can be treate with oral aministration of cellulase C. Psychiatric treatment is critical in long-term management D. The patient likely has patchy areas of alopecia E. Peptic ulcer isease is a risk factor
96
PArt i Patient Care
29. A 70-year-ol man presents to the ED with suen onset of severe epigastric pain associate with retching but with little vomitus. His bloo pressure is 140/90 mmHg an his heart rate is 90beats per minute. Attempts by the ED physician to place a nasogastric tube are unsuccessful. An upright chest raiograph reveals a large gas bubble just above the left iaphragm. Which of the following is true regaring this conition? A. The stomach is likely twiste along the axis, transecting the lesser an greater curvature B. In chilren it is largely ue to a paraesophageal hernia C. It is associate with Bergman’s tria D. Percutaneous gastrostomy tube for enitive management is acceptable in select patients E. It is initially manage conservatively for the majority of patients 30. Which of the following escribes the association between Sister Mary Joseph noule an gastric cancer? A. A metastatic left axillary lymph noe B. A metastatic left supraclavicular lymph noe C. An ovarian mass from gastric metastasis D. Umbilical metastasis suggesting carcinomatosis E. An anterior noule palpable on rectal examination suggesting rop metastasis 31. A 68-year-ol woman presents with an upper GI hemorrhage. She has a history of ulcer isease an has recently complete a treatment for H. pylori. Upper enoscopy reveals brisk arterial bleeing from a uoenal ulcer locate on the posterior wall. Despite numerous attempts to control the bleeing enoscopically, the ulcer continues to blee. The patient has receive 4 units of bloo. Her hematocrit is 5%, her bloo pressure is 110/60 mmHg, an her heart rate is 10 beats per minute. Which of the following is the best management option? A. Duoenotomy, oversewing the ulcer, truncal vagotomy, an pyloroplasty B. Duoenotomy an oversewing the ulcer C. Truncal vagotomy an antrectomy with Billroth I reconstruction D. Truncal vagotomy an antrectomy with Billroth II reconstruction E. Highly selective vagotomy
32. A 4-year-ol alcoholic male with recurrent episoes of pancreatitis presents to the ED with one episoe of hematemesis in the morning. He oes not appear to have any active bleeing currently. CT scan emonstrates splenic artery thrombosis. Lipase an liver function tests are normal. EGD emonstrates isolate gastric varices that are not currently bleeing an one -cm ulcer at the angularis. Which of the following enoscopic features confers the lowest risk of rebleeing? A. Oozing ulcer B. Nonbleeing ulcer with overlying clot C. Nonbleeing visible vessel D. Visible ulcer base E. Flat pigmente spot 33. Which of the following is true regaring gastric polyps? A. Funic gastric polyps have the highest risk of harboring malignant cells B. Aenomatous gastric polyps are the most common type C. Hamartomatous polyps are associate with H.pylori infection D. Heterotopic polyps most commonly present with gastrointestinal bleeing E. Inammatory polyps o not have a risk of malignancy 34. Bleeing from a Dieulafoy gastric lesion is ue to: A. Antral vascular ectasia B. Abnormal gastric rugal fols C. Ingeste foreign material D. An abnormal submucosal vessel E. A premalignant lesion 35. The most sensitive an specic iagnostic test for gastrinoma is: A. Basal an stimulate gastric aci outputs B. Octreotie scan C. Fasting serum gastrin D. Calcium stimulation test E. Secretin stimulation test
CHAPtEr 8 Alimentary Tract—Stomach
97
Answers 1. A. Lymphoepithelial tissue on biopsy is virtually pathog-
nomonic for gastric MALT lymphoma, an inolent malignancy primarily thought to arise from chronic H. pylori infection (gram-negative spirochete) (C). Treatment with triple therapy or quaruple therapy antibiotics to eraicate H. pylori is the rst-line treatment for patients with early-stage 1 or isease. Most cases take 1 year to achieve remission; however, although rare, it can take up to 3 years. Refractory cases, as well as stage 3 an 4 isease, require chemoraiation using CHOP (cyclophosphamie, oxorubicin, vincristine, an prenisone) (E). Surgical resection is reserve for cases complicate by perforation, bleeing, or obstruction (D). Tumor biology affects the response to antibiotics, namely the t(11;18) chromosomal translocation that has a 5 cm or >5 mitoses/50 hpf or gastric tumors >10 cm or >5 mitoses/50 hpf, or patients with tumor rupture. The most common averse effect of imatinib is eema. While small Tis or T1a gastric aenocarcinomas can be treate with enoscopic resection, complete resection of gastric GISTs typically requires at least a wege resection (E).
3. B. One of the more feare postoperative complications
following Roux-en-Y gastric bypass is acute afferent loop synrome with small bowel obstruction (SBO) of the biliopancreatic limb. A high inex of suspicion is neee to iagnose this complication, usually base on patient symptoms an evience of proximal SBO on imaging. Unlike most
ahesive SBOs, which can be ecompresse with vomiting, obstruction of the afferent loop via an internal hernia causes a close loop obstruction with a high risk of perforation with urgent surgical exploration inicate to relieve the obstruction. Eating smaller, more frequent meals is the rst-line treatment of early umping synrome postgastrectomy (A). Nasogastric tube ecompression with water-soluble contrast challenge is the conservative management for ahesive SBO an is not appropriate in this patient (C). Antibiotic therapy is a treatment for small intestine bacterial overgrowth (SIBO) but not for bowel obstruction (D). While upper enoscopy woul be useful in the iagnosis of reux gastritis or marginal ulcer, it woul be of low utility in this patient (E).
4. B. This patient with multiple uoenal ulcers likely has
Zollinger-Ellison synrome seconary to hypersecretion of gastrin from a gastrinoma. Gastrin is usually prouce by antral G cells an acts on parietal cells to prouce hyrochloric aci an chief cells to prouce pepsinogen, both cell types of which are most preominant in the stomach boy (A). The pancreas is the site of the secretion of many hormones incluing somatostatin from D cells, insulin from beta cells, an glucagon from alpha cells (C). The uoenum an jejunum are the sites of the secretion of cholecystokinin (CCK) from I cells an secretin from K cells (D, E). The uoenum is the most common site for gastrinomas (50%–88%), followe by the pancreas (5%).
5. A. The most common etiology of small bowel obstruction
in the Unite States is ahesions from previous abominal surgery. However, this oes not hol true for patients that have previously ha an LRYGB. In this proceure, a potential hernia site (Petersen space hernia) is create, increasing the risk for the evelopment of an internal hernia, which is the most common cause of SBO in this patient population with an incience of 1% to 5%. This potential space results from herniation of intestinal loops through a efect in the mesentery an between small bowel limbs, transverse mesocolon, an the retroperitoneum. Aitionally, when compare to its open counterpart, the laparoscopic approach further facilitates a Petersen hernia because of the ecrease frequency of postoperative ahesions, which seemingly have a physiologic role of preventing bowel mobility, an thus, internal herniation. Risk of SBO is signicantly higher with a retrocolic versus an antecolic approach. Roux compression ue to mesocolon scarring is the secon most common etiology for SBO in patients with LRYGB followe by ahesions (B, C). Kinking of the jejunojejunostomy an incarcerate abominal wall hernia occur less frequently (D, E). Reference: Champion JK, Williams M, Husain S, Johnson AR. Small-bowel obstruction after laparoscopic Roux-en-Y gastric bypass: etiology, iagnosis, an management. Arch Surg. 003;13(4):988–993.
6. A. Nonoperative management for perforate peptic
ulcer isease is gaining popularity an is now accepte as an appropriate rst-line management for poor surgical caniates (e.g., COPD using home oxygen) who are stable, have
98
PArt i Patient Care
no evience of peritonitis, an have no contrast extravasation. Conservative management is also more favorable if the uration of symptoms has laste more than 4 hours. By this time the perforation has typically been seale. Self-sealing of the perforation is achieve by either ahesion formation to the cauate lobe, the greater omentum, the gallblaer, or the falciform ligament. In one stuy, only 3 out of 109 patients manage nonoperatively evelope an intraabominal abscess (which can be manage with antibiotics an percutaneous rainage). This may speak to the intrinsic immune function of the omentum an the fact that the upper GI tract has a low bacterial loa. Eighty percent of nonoperative cases respon favorably, an morbiity is not signicantly increase. Patients eeme appropriate caniates for nonoperative management shoul be amitte, place NPO (nothing by mouth), an given IV ui resuscitation, IV antibiotics covering gram-negative an anaerobic organisms, an PPIs. Nasogastric tube insertion is critical to help ecompress the stomach an allow the perforation to heal. CT scan may be consiere for patients who fail to improve or those who eteriorate clinically. Surgery is the next step for patients who fail conservative management (B, C). Outpatient follow-up is not appropriate because nonoperative management shoul be performe in a monitore setting with frequent abominal exams an follow-up esophagogastrouoenoscopy (EGD) to ensure that the perforation has seale (D, E). References: Nusree R. Conservative management of perforate peptic ulcer. Thai J Surg. 005;6:5–8. Hanumanthappa MB, Gopinathan S, Guruprasa R. A nonoperative treatment of perforate peptic ulcer: a prospective stuy with 50 cases. J Clin Diagnostic Res. 01;41:4161.
7. D. Obesity has been linke to multiple comorbiities,
incluing hypertension an iabetes, an is on the rise. As such, many clinicians have turne to meical management an/or bariatric surgery to help ght this epiemic in cases where iet an exercise fail. Two FDA-approve meications to help treat obesity inclue sibutramine an orlistat. Sibutramine blocks the presynaptic uptake of serotonin, thereby potentiating its anorexic effects in the CNS. Orlistat inhibits pancreatic lipase, which ecreases ietary fat absorption an results in weight loss (B). A signicant complication limiting its use for most patients is severe atulence. Inications for weight loss surgery inclue BMI >35 with associate obesityrelate comorbiities (e.g., hypertension, iabetes) or BMI >40 (A). Aitionally, all patients will nee to emonstrate that they have successfully attempte an faile nonoperative weight loss management such as iet an exercise programs (E). Patients will also nee to be evaluate by a physiatrist an eeme suitable for the proceure. The four stanar approaches in the Unite States inclue laparoscopic gastric baning, sleeve gastrectomy, BPD, an RYGB. Laparoscopic gastric baning an sleeve gastrectomy are consiere restrictive proceures as they physically limit the intake of foo. BPD is consiere a malabsorptive proceure as it involves constructing an alimentary channel istally to the GI tract an thereby preventing the absorption of caloric intake. RYGB is consiere a combine approach an involves creating a small restricte gastric remnant (restrictive component) an a roux-limb from the stomach to the istal jejunum (malabsorptive component) (C). These
proceures result in up to 50% resolution of weight-relate comorbiities an up to 50% excess weight loss. Compare to BPD, RYGB has a lower 30-ay mortality an is slightly favore by surgeons as it is technically easier to perform. RYGB has a slightly higher mean excess weight loss at years compare to sleeve gastrectomy, but sleeve gastrectomy has a higher perioperative leak rate. Both proceures are equally effective in eliminating type iabetes mellitus. References: Duarte MIX e T, Bassitt DP, Azeveo OC e, Waisberg J, Yamaguchi N, Pinto Junior PE. Impact on quality of life, weight loss an comorbiities: a stuy comparing the biliopancreatic iversion with uoenal switch an the bane Roux-en-Y gastric bypass. Arq Gastroenterol. 014;51(4):30–37. Santry HP, Gillen DL, Lauerale DS. Trens in bariatric surgical proceures. JAMA. 005;94(15):1909–1917. O’Brien P. Surgical treatment of obesity. Endotext. 016;19:9–46. Zingmon DS, McGory ML, Ko CY. Hospitalization before an after gastric bypass surgery. JAMA. 005;94(15):1918–194.
8. C. Gastroparesis is ene as elaye gastric emptying
without a mechanical cause for obstruction. Although iabetes is the most common known cause of gastroparesis (9%), iiopathic gastroparesis occurs more frequently (36%). The most common symptoms are nausea, early satiety, an abominal bloating. Most patients o not have abominal pain. Although symptoms alone can be suggestive of this conition, it nees to be conrme by imaging (E). Gastric emptying scintigraphy (elaye gastric emptying stuy) is the gol stanar in iagnosing gastroparesis. This involves asking the patient to eat a small meal along with a raioactive tracer. The rate of emptying is measure 1, , 3, an 4 hours after the meal is ingeste. If more than 10% of the meal remains in the stomach after 4 hours, the stuy is consiere consistent with gastroparesis (A, B, D).
9. E. Gallstone formation occurs in 30% to 5% of patients
unergoing weight loss surgery, but only 7% to 15% are symptomatic. Among those 7% to 15% who o become symptomatic, acute cholecystitis is uncommon. Rapi weight loss is a known risk factor for cholelithiasis. In fact, excess weight loss greater than 5% is consiere the strongest preictor of postoperative cholecystectomy an occurs more commonly in patients who have ha a gastric bypass versus laparoscopic baning or sleeve gastrectomy (A). Several mechanisms have been shown to contribute to gallstone formation uring weight loss incluing increase secretion of calcium an mucin into bile, increase concentrations of arachionic aci erivatives, an bile stasis seconary to stringent ietary restrictions postoperatively (D). Prophylactic cholecystectomy at the time of weight loss surgery has been a point of ebate in the surgical community. Proponents argue that it helps prevent the morbiity of symptomatic biliary isease an avois the nee for treatments such as enoscopic retrograe cholangiopancreatography (ERCP), which can be particularly challenging in this patient population (e.g., RYGB). However, it has been shown in several large stuies that the rate of postoperative cholecystectomy remains uner 15%; therefore, the routine removal of the gallblaer uring weight loss surgery is not currently supporte by the American Society of Metabolic an Bariatric Surgery (B). In contrast, symptomatic patients may unergo concomitant cholecystectomy safely. Ursoiol after gastric bypass can signicantly ecrease the rate of gallstone
CHAPtEr 8 Alimentary Tract—Stomach formation, but because it has not been shown to be cost effective an lea to improve outcomes, it is not routinely recommene (C). References: D’Hont M, Sergeant G, Deylgat B, Devrient D, Van Rooy F, Vansteenkiste F. Prophylactic cholecystectomy, a manatory step in morbily obese patients unergoing laparoscopic Roux-en-Y gastric bypass? J Gastrointest Surg. 011;15(9):153–1536. Shiffman ML, Shamburek RD, Schwartz CC, Sugerman HJ, Kellum JM, Moore EW. Gallblaer mucin, arachionic aci, an bile lipis in patients who evelop gallstones uring weight reuction. Gastroenterology. 1993;105(4):100–108. Sugerman HJ, Brewer WH, Shiffman ML, et al. A multicenter, placebo-controlle, ranomize, ouble-blin, prospective trial of prophylactic ursoiol for the prevention of gallstone formation following gastric-bypass-inuce rapi weight loss. Am J Surg. 1995;169(1):91–96. Tucker ON, Fajnwaks P, Szomstein S, Rosenthal RJ. Is concomitant cholecystectomy necessary in obese patients unergoing laparoscopic gastric bypass surgery? Surg Endosc. 008;(11):450–454. Villegas L, Schneier B, Provost D, et al. Is routine cholecystectomy require uring laparoscopic gastric bypass? Obes Surg. 004;14():06–11.
10. B. The rate of obesity is rising in the Unite States an
an increasing number of patients are unergoing weight loss surgery. Gastric leak in the early postoperative perio may be an inication to go back to the operating room, so early recognition of this complication is important. The rst manifestations of a gastric leak are tachycaria an fever (A, C–E). This may also be accompanie by tachypnea, abominal pain, chest pain, oliguria, an/or hypotension. Reference: Bekehit M, Katri K, Nabil W. Earliest signs an management of leakage after bariatric surgeries: single institute experience. Alexandria J Med. 013;49(1):9–33.
11. A. Laparoscopic gastric baning involves placing an
inatable balloon aroun the proximal stomach at the angle of His. A properly place lap ban will have an approximately 45° upwar angle from the horizontal plane on a plain lm of the abomen. The proceure was very popular when it rst appeare but lost traction after subsequent stuies emonstrate that it was far inferior to gastric bypass. Aitionally, patients with laparoscopic bans were more likely to require revisions for complications associate with the gastric ban. One such complication is ban erosion (BE) into the stomach an/or ajacent organs. This may present as port site erythema (inammation tracking own the tube), fooling the clinician into thinking the patient may only have an overlying skin infection. In fact, most patients with BE presenting with port site erythema o not have a subfascial port infection. BE can occur many years after surgery, an one propose mechanism involves overtightening of the ban (e.g., after clinic visit). CT of the abomen shoul be performe in patients suspecte of having BE an, if foun, the port site shoul be completely eate an the patient shoul be scheule for laparoscopic removal of the ban. EGD may emonstrate BE if it has completely eroe into the gastric lumen but may miss partial BE (C). Incision an rainage are not inicate because there is no abscess (D). The patient shoul be monitore for the evelopment of a subsequent port site infection, but the rst step is to get a CT scan (B–E). References: Dilorenzo N, Lorenzo M, Furbetta F. Intragastric gastric ban migration an erosion: an analysis of multicenter experience on 177 patients. Surg Endosc. 013;7(4):1151–1157.
99
Naef M, Naef U, Mouton WG, Wagner HE. Outcome an complications after laparoscopic Sweish ajustable gastric baning: 5-year results of a prospective clinical trial. Obes Surg. 007;17():195–01. Stroh C, Hohmann U, Will U, etal. (008).
12. E. In the majority of patients with a perforate uo-
enal ulcer, simple closure of the ulcer with an omental (Graham) patch is all that is necessary (A). This is then followe by treatment of H. pylori. In aition, a Graham patch alone shoul be use if the patient is unstable, if there is extensive exuative peritonitis, or if the perforation is long staning (>4 hours). However, in the setting of a patient with a known ulcer iathesis who has either alreay been treate for H. pylori or is H. pylori negative, an ulcer surgery shoul be ae to the operation, provie the patient is a goo operative risk, is hemoynamically stable, an oes not have extensive peritonitis. The options are either to perform a highly selective vagotomy (HSV) or a vagotomy an pyloroplasty (B). An HSV is the preferre approach in the stable goo-risk patient, provie the surgeon is comfortable with the proceure. Pyloroplasty is typically performe along with a vagotomy because the wiene outlet from the stomach to the uoenum helps circumvent any unwante effects of the ecrease gastric peristalsis an overall change in gastric emptying patterns that occur following vagotomy. The entire proceure can be performe laparoscopically in select patients. Truncal vagotomy an antrectomy (C, D) is generally not recommene in the setting of perforation because of the high associate morbiity an mortality rates. References: Caiere GB, Bruyns J, Himpens J, Van Alphen P, Verturyen M. Laparoscopic highly selective vagotomy. Hepatogastroenterology. 1999;46(7):1500–1506. Joran PH Jr, Thornby J. Perforate pylorouoenal ulcers: longterm results with omental patch closure an parietal cell vagotomy. Ann Surg. 1995;1(5):486–488. Siu WT, Leong HT, Law BKB, et al. Laparoscopic repair for perforate peptic ulcer: a ranomize controlle trial. Ann Surg. 00;35(3):313–319.
13. E. Postvagotomy synromes inclue iarrhea, gas-
tric atony, an incomplete vagotomy (leaing to recurrent ulceration). Diarrhea follows truncal vagotomy an may be confuse with umping synrome. The iarrhea associate with vagotomy occurs more frequently an is not associate with the other cariovascular manifestations seen with umping synrome (C). The initial treatment is similar to that for umping synrome, with ietary moications such as frequent small meals with ecrease ui intake an an increase in ber. A propose mechanism of the iarrhea is an increase in stool bile salts. Oral cholestyramine is often helpful because it bins bile salts (B). Loperamie an coeine have also been shown to elay intestinal transit time an improve symptoms. In the very rare patient who oes not respon to meical management, reversal of a segment of jejunum is effective in slowing transit time an improving iarrhea (D). Octreotie is not effective for postvagotomy iarrhea an may make the situation worse by ecreasing pancreatic secretions an thus increasing steatorrhea (A). References: Duncombe V, Bolin T, Davis A. Double-blin trial of cholestyramine in post-vagotomy iarrhea. Gut. 1977;18(7):531–535. O’Brien JG, Thompson DG, Mcintyre A. Effect of coeine an loperamie on upper intestinal transit an absorption in normal subjects an patients with postvagotomy iarrhea. Gut. 1988;9(3):31–318.
100
PArt i Patient Care
14. D. More than 80% of gastrinomas are localize preoper-
atively. For those that cannot be localize, surgical exploration is still inicate because excision of the primary tumor leas to a ecrease rate of liver metastasis. When exploring, it is important to be aware that 80% of gastrinomas are foun within the gastrinoma (Passaro) triangle, an area ene by the junction of the cystic uct an common bile uct, the secon an thir portions of the uoenum, an the neck an boy of the pancreas. As many as 60% of gastrinomas are within the wall of the uoenum, primarily in the rst an secon portions an can be very small. Thus, the next maneuver woul be to perform a blin proximal uoenotomy to manually palpate the uoenal wall for tumors. Closing the abomen (A) woul be inappropriate. Blin istal pancreatectomy an splenectomy (these share bloo supply) (B) or istal uoenotomy (E) woul have very low yiels. A pancreaticouoenectomy (Whipple proceure) (C) woul not be inicate in this setting. It is potentially inicate for multiple uoenal or proximal pancreatic hea tumors that coul not be enucleate.
15. C. Achieving an aequate lymphaenectomy with a
≥15 lymph noe harvest uring an oncologic resection of gastric cancer is important in accurately staging the patient an reucing the noal false negative rate. Staging of gastric cancer involves epth of invasion (T1 invaes lamina propria; T, muscularis propria or subserosa; T3, serosa; T4, ajacent structures), noes, an istant metastasis (E). EUS is the best moality for assessing tumor epth of invasion an noal status. It is approximately 80% accurate in etermining whether the tumor is transmural (invaing serosa, T3) but only 50% accurate in assessing whether pathologically enlarge lymph noes are present (B). EUS seems to be more accurate with avance isease than early isease. CT scanning is the preferre metho for etermining istant metastases, but it is not as useful for T an N staging (A). The routine use of MRI an positron emission tomography scanning for staging of gastric cancer has not as yet been establishe (D). N1 isease inclues 1 to 6 regional noes; N, 7 to 15 regional noes; an N3, more than 15 regional noes. References: Puli SR, Batapati Krishna Rey J, Bechtol ML, Antillon MR, Ibah JA. How goo is enoscopic ultrasoun for TNM staging of gastric cancers? A meta-analysis an systematic review. World J Gastroenterol. 008;14(5):4011–4019. Willis S, Truong S, Gribnitz S, Fass J, Schumpelick V. Enoscopic ultrasonography in the preoperative staging of gastric cancer: accuracy an impact on surgical therapy. Surg Endosc. 000;14(10):951–954. Xi W, Zhao C, Ren G. Enoscopic ultrasonography in preoperative staging of gastric cancer: etermination of tumor invasion epth, noal involvement an surgical respectability. World J Gastroenterol. 003;9():54–57.
16. D. When consiering gastrinoma, it is important to
be aware of the ifferential iagnosis of an elevate gastrin level. Causes of hypergastrinemia with increase aci prouction inclue gastrinoma, G-cell hyperplasia (not D-cell) (E), retaine antrum after istal gastrectomy, renal failure, an gastric outlet obstruction. Hypergastrinemia with normal or low aci prouction inclues pernicious anemia, postvagotomy states, use of aci-suppressive meication, an chronic gastritis. Hypothyroiism is associate with a low gastrin level, whereas hyperthyroiism increases gastrin levels (B). Diabetes (A) an hyperparathyroiism (C) o not affect gastrin levels.
References: Seino Y, Matsukura S, Inoue Y, Kaowaki S, Mori K, Imura H. Hypogastrinemia in hypothyroiism. Am J Dig Dis. 1978;3():189–191. Korman MG, Laver MC, Hansky J. Hypergastrinemia in chronic renal failure. BMJ. 197;1(5794):09–10.
17. D. R0 resection is resection of all gross an microscopic
tumors (C). R1 inicates removal of all macroscopic isease but microscopic margins are positive for isease. An R resection inicates that gross resiual isease is left behin (E). A D1 resection (A) refers to removal of perigastric lymph noes; D (B) refers to the aitional resection of lymph noes along the name vessels aroun the stomach. A D3 resection is a D resection plus removal of para-aortic lymph noes.
18. A. Risk factors for gastric cancer inclue ietary factors
such as a large consumption of smoke meats, pickle foos, high nitrates, an high salt, whereas a iet high in fruits an vegetables may be protective (D, E). Other risk factors inclue smoking, low socioeconomic status, Black race, H. pylori infection, chronic atrophic gastritis, bloo type A, previous partial gastrectomy, achlorhyria, pernicious anemia, polyps (aenomatous an hyperplastic), male sex, an certain familial synromes such as hereitary nonpolyposis colorectal cancer, Li-Fraumeni synrome, familial aenomatous polyposis, an Peutz-Jeghers synrome (B, D). Peutz-Jeghers synrome is associate with a markely increase risk of cancer in the esophagus, stomach, small bowel, colon, pancreas, breast, lung, uterus, an ovary, with a cumulative 93% risk of cancer. Carbonate aciic soa has not been shown to increase the risk for cancer (C). Gastric cancer has been categorize by Lauren into intestinal an iffuse types base on histology. The intestinal type is thought to be more relate to environmental factors, is associate with chronic gastritis, an is well ifferentiate. The iffuse type is usually poorly ifferentiate an associate with signet rings an occurs in younger patients an in association with familial isorers an with type A bloo. The iffuse type has a worse prognosis. References: Bernt H, Wilner GP, Klein K. Regional an social ifferences in cancer incience of the igestive tract in the German Democratic Republic. Neoplasma. 1968;15(5):501–515. Giariello FM, Brensinger JD, Tersmette AC, et al. Very high risk of cancer in familial Peutz-Jeghers synrome. Gastroenterology. 000;119(6):1447–1453. Wyner EL, Kmet J, Dungal N, Segi M. An epiemiological investigation of gastric cancer. Cancer. 1963;16(11):1461–1496.
19. B. Gastric ulcers have been categorize into ve types.
The most common is the type I lesion (≈60%) (A), which is locate near the angularis incisura at the borer between the antrum an the funus, usually along the lesser curve. These patients usually have normal or ecrease aci secretion. Type II gastric ulcers are locate in the funus an are associate with a concomitant uoenal ulcer. Type III gastric ulcers are prepyloric. Both types II an III gastric ulcers are usually associate with increase gastric aci secretion. Type III ulcers are thought to behave like uoenal ulcers. Type IV gastric ulcers are locate near the gastroesophageal junction. Like type I ulcers, type IV gastric ulcers have normal or low aci prouction an are associate with impaire mucosal efense. Type V gastric ulcers are consiere a iffuse process an are associate with NSAID use.
CHAPtEr 8 Alimentary Tract—Stomach
20. D. GISTs were previously calle leiomyomas or leiomyo-
sarcomas because they were thought to arise from smooth muscle cells, but they in fact originate from mesenchymal components (from Cajal cells) (B). They stain positive for CD117 (c-kit). They are most commonly foun in the stomach an, although rare, they are the most common mesenchymal tumors of the intestinal tract. Because they are not epithelial tumors an grow in the wall of the stomach, they ten to be large at the time of presentation. They cause mucosal ulceration an frequently present with GI bleeing (E). Large tumors may also prouce symptoms of weight loss, abominal pain, an fullness an early satiety. An abominal mass may be palpable. An enoscopic biopsy specimen may be negative in as many as one-half of cases ue to sampling error because most of the tumor is submucosal (A). A CT scan provies a better assessment of the extent of the tumor. Determining whether a GIST is malignant is not straightforwar because there are no iscriminating cellular features (C). The malignant potential is etermine by mitotic activity (>5 mitoses/50 high power el) with 1 cm. Lymph noe issection is not necessary because tumors sprea hematogenously an lymph noe metastasis is extremely rare. Wege resection with 1 cm margins is aequate treatment in most cases. This can be performe laparoscopically. However, microscopically positive margins have not been emonstrate to affect survival. References: Dempsey DT. Stomach. In: Brunicari FC, Anersen DK, Billiar TR, etal., es. Schwartz’s principles of surgery. 8th e. New York: McGraw-Hill; 005:933–996. Mercer DW, Robinson EK. Stomach. In: Townsen CM, Jr, Beauchamp RD, Evers BM, Mattox KL, es. Sabiston textbook of surgery: the biological basis of moern surgical practice. 17th e. Philaelphia: W.B. Sauners; 004:165–13. Novitsky YW, Kercher KW, Sing RF, Henifor BT. Long-term outcomes of laparoscopic resection of gastric gastrointestinal stromal tumors. Ann Surg. 006;43(6):738–745. Sexton JA, Pierce RA, Halpin VJ, et al. Laparoscopic gastric resection for gastrointestinal stromal tumors. Surg Endosc. 008;(1):583–587. Malangoni MA, etal. Stomach. In: Cameron JL, e. Current surgical therapy. 8th e. Philaelphia: Mosby; 004:67–100.
21. D. Bile reux into the stomach can occur without previ-
ous surgery, but in most instances it follows ablation of the pylorus, such as after gastric resection or pyloroplasty. After such proceures, most patients will have bile in the stomach on enoscopic examination, along with some egree of gross or microscopic gastric inammation. However, only a small fraction of patients will have a signicant egree of symptoms such as nausea, epigastric pain, an bilious vomiting consistent with alkaline (bile) reux gastritis (B, E). Symptoms often evelop months or years after the inex operation. The ifferential iagnosis inclues afferent or efferent loop obstruction, gastric stasis, an small bowel obstruction. These other iagnoses can be rule out using a combination of abominal raiographs, upper enoscopy, an abominal CT scan. A hepatoiminoiacetic aci (HIDA) scan is particularly helpful for emonstrating bile reux. Bile reux an gastritis are more likely to occur after Billroth II reconstruction (A) than after Billroth I an least likely after vagotomy an pyloroplasty. Meical management of symptomatic patients is not particularly effective (C). The surgical proceure of choice is to convert the Billroth II into a Roux-en-Y gastrojejunostomy with a lengthene jejunal limb (at least 45 cm).
101
22. D. More than 90% of gastrinomas have receptors for
somatostatin. Octreotie scanning (somatostatin receptor scintigraphy) has been shown to be the most sensitive test for localization of gastrinomas. However, successful localization epens on size an location. Somatostatin receptor scintigraphy is poor for very small tumors (cm). Even when bleeing recurs after having been controlle enoscopically, enoscopic treatment can again be attempte with a high rate of success, thus avoiing surgery. The bleeing is usually from a posterior ulcer that has eroe into the gastrouoenal artery (remember anterior ulcers cause a free perforation an peritonitis, posterior ulcers penetrate an blee). Surgical management ecisions shoul be base on the hemoynamic stability of the patient, the patient’s overall meical conition, an whether the patient has a history of ulcer isease that has been treate for H. pylori. In the patient who is actively bleeing, the uoenum shoul be opene across the pylorus as is use in a pyloroplasty. The ulcer be shoul be oversewn with multiple gure-of-eight sutures. If the patient has a history of ulcers that have been treate for H. pylori an is stable in the operating room, an ulcer operation shoul be performe. The best option in this type of emergent setting is to perform a truncal vagotomy an to close the longituinal uoenotomy in a transverse fashion as with a pyloroplasty. If the patient is a high surgical risk an unstable, another option woul be to simply perform a smaller uoenotomy, oversew the ulcer, simply close the uoenotomy, an treat postoperatively for H. pylori (B). Although vagotomy an antrectomy are another option, they woul selom be use in the emergent setting because of the higher associate morbiity rate (D, E). An HSV (C) woul not aress the actively bleeing ulcer. References: Brullet E, Calvet X, Campo R, Rue M, Catot L, Donoso L. Factors preicting failure of enoscopic injection therapy in bleeing uoenal ulcer. Gastrointest Endosc. 1996;43():111–116. Lau JY, Sung JJ, Lam YH, et al. Enoscopic retreatment compare with surgery in patients with recurrent bleeing after initial enoscopic control of bleeing ulcers. N Engl J Med. 1999;340(10):751–756.
32. D. Recurrent episoes of acute pancreatitis preispose
patients to eveloping splenic vein thrombosis, which can result in isolate gastric varices. Historically, patients were offere a splenectomy as a prophylactic measure to prevent severe upper GI bleeing. However, with improve imaging we are better able to ientify splenic vein thrombosis, an we now know that only 4% of patients will have clinically signicant gastric variceal bleeing, so routine splenectomy has fallen out of favor. This patient also has a concomitant ulcer, which coul have been contributing to hematemesis. The Forrest classication graes peptic ulcers base on enoscopic features an allows the clinician to etermine risk of rebleeing. The risk ecreases in the following orer: active spurting bleeing (17%–100%), active oozing bleeing (17%–100%), nonbleeing visible vessel (0%–81%), aherent clot (14%–36%), at pigment spot (0%–13%), an clean visible ulcer base (0%–10%) (A–C, E). Although patients with high-risk peptic ulcers (active bleeing/oozing, nonbleeing visible vessel) may benet from a secon-look enoscopy, current guielines recommen against routine secon-look enoscopy. References: Forrest JH, Finlayson NDC, Shearman DJC. Enoscopy in gastrointestinal bleeing. Lancet. 1974;304(7877):394–397. Heier TR, Azeem S, Galanko JA, Behrns KE. The natural history of pancreatitis-inuce splenic vein thrombosis. Ann Surg. 004;39(6):876–88. Laine L, Jensen DM. Management of patients with ulcer bleeing. Am J Gastroenterol. 01;107(3):345–360.
104
PArt i Patient Care
33. E. Hyperplastic polyps are by far the most common
gastric polyps (70%–90%) (B). Other types inclue aenomatous, hamartomatous, inammatory (pseuopolyps), funic glan, an heterotopic. Hyperplastic polyps are seen in association with chronic atrophic gastritis, which is ue to H. pylori infection (C). Hyperplastic polyps are further classie into polypoi foveolar hyperplasia an typical hyperplastic polyps. Polypoi foveolar hyperplasia oes not seem to have malignant potential, whereas the typical hyperplastic polyp has an approximately % chance of eveloping malignancy. Aenomatous polyps have the highest risk of malignancy (10%–0%), an the risk of malignancy seems to be relate to size an histology (greater risk for villous than tubular) (A). Funic gastric polyps are associate with long-term PPI use, an the risk of cancer is negligible. Aitionally, hamartomatous, inammatory, an heterotopic polyps o not seem to have a risk of malignancy. Heterotopic polyps are usually the result of ectopic pancreatic tissue an are typically benign lesions without clinical signicance (D). However, large heterotopic polyps can lea to obstruction an intussusception. Treatment for most polyps is simply enoscopic polypectomy. Aitional surgical resection is recommene for polyps that are sessile an larger than cm, those with areas of invasive tumor, an those that cause symptoms (bleeing or pain). References: Orlowska J, Jarosz D, Pachlewski J, Butruk E. Malignant transformation of benign epithelial gastric polyps. Am J Gastroenterol. 1995;90(1):15–159. Jalving M, Koornstra JJ, Wesseling J, Boezen HM, DE Jong S, Kleibeuker JH. Increase risk of funic glan polyps uring long-term proton pump inhibitor therapy. Aliment Pharmacol Ther. 006;4(9):1341–1348.
34. D. A Dieulafoy lesion is a congenital malformation in
the stomach (typically on the lesser curvature) characterize by a submucosal artery that is abnormally large an tortuous. As a result of its relatively supercial location, it may eroe through the mucosa an become expose to gastric secretions, leaing to massive upper GI hemorrhage. On enoscopy, the mucosa of the stomach appears normal, an the only ning is a pinpoint area of mucosal efect with brisk arterial bleeing. The lesion may easily be misse if the bleeing is not active. Dieulafoy lesion is not premalignant (E) an is not associate with the ingestion of foreign material (C). Treatment is enoscopic, via electrocautery, heater probe, or injection with a sclerosing agent. Surgery, which consists of a wege resection, is reserve for the rare patient
who is not controlle enoscopically. Antral vascular ectasia (A) is seen in a conition known as watermelon stomach an can lea to signicant acute or chronic GI bloo loss. Dilate mucosal bloo vessels containing thrombus, mucosal bromuscular ysplasia, an hyalinization are prominent features. It erives its name from the mucosal vessels that create parallel lines in the mucosal fols (B). The stomach is typically not enlarge. It is seen preominantly in elerly women with autoimmune isease or elerly males with cirrhosis. Reference: Selinger CP, Ang YS. Gastric antral vascular ectasia (GAVE): an upate on clinical presentation, pathophysiology, an treatment. Digestion. 008;77():131–137.
35. E. The most sensitive an specic test for gastrinoma
(ZES) is the secretin stimulation test. An IV bolus of secretin is aministere, an gastrin levels are checke before an after injection. An increase in serum gastrin of 10 pg/ mL or greater has the highest sensitivity an specicity for gastrinoma. There are numerous other causes of hypergastrinemia. They can be ivie into those associate with an increase aci prouction an those with a ecrease aci prouction (A). In the latter situation, the hypergastrinemia is reactive ue to hypo- or achlorhyria. In aition to ZES, G-cell hyperplasia, gastric outlet obstruction, an retaine antrum after Billroth II reconstruction are associate with increase aci prouction. Reactive hypergastrinemia is seen with atrophic gastritis, pernicious anemia, an gastric cancer; in patients receiving H-receptor antagonists an PPIs; an after vagotomy. Hypergastrinemia is also seen in chronic renal failure ue to ecrease catabolism. Given this broa ifferential, fasting serum gastrin levels (C) are not sufciently specic to establish the iagnosis of ZES in the majority of patients unless gastrin levels are extremely high (>1000pg/mL). The secretin stimulation test has higher sensitivity an specicity than the calcium stimulation test (D). The calcium stimulation test is use if the secretin test result is negative an there is a high suspicion for ZES in the presence of hypergastrinemia. Once the iagnosis of ZES is establishe, a nuclear octreotie scan (B) seems to be the most sensitive test to localize the tumor. Reference: Berna M, Hoffmann K, Long S. Serum gastrin in Zollinger-Ellison synrome: II. Prospective stuy of gastrin provocative testing in 93 patients from the National Institutes of Health an comparison with 537 cases from the literature: evaluation of iagnostic criteria, proposal of new criteria, an correlations with clinical an tumoral features. Medicine. 006;(6):331–364.
Alimentary Tract— SmallBowel ZACHARY N. WEITZNER, FORMOSA CHEN, AND BEVERLEY A. PETRIE
9
ABSITE 99th Percentile High-Yields I. Duoenum A. Releases alkaline mucus from Brunner glans to neutralize gastric aci B. S cells release secretin (inhibits gastric aci, stimulates pancreatic bicarb, increases bile prouction in liver) an I cells release cholecystokinin (CCK) (inhibits gastric emptying, stimulates pancreatic enzyme prouction, increases bile, gallblaer contraction, satiety) C. Leafy appearing villi, absorb iron, eroe in celiac isease D. Bulb: 1st segment; ulcerations ue to Helicobacter pylori, Zollinger–Ellison synrome E. Descening: n segment; pancreatic an common bile uct empty F. Transverse: 3r segment; anterior to inferior vena cava, aorta, vertebral column G. Ascening: 4th segment; joins jejunum; ens at ligament of Treitz H. Bloo supply: celiac axis via gastrouoenal artery an SMA axis via pancreaticouoenal arteries II. Jejunum A. Does not have Brunner glans; thus marginal ulcers are more likely with Billroth II than with Billroth I B. Begins at ligament of Treitz C. Ientiable by long vasa recta from SMA an plicae circularis D. Dense villi for absorption of water, lipis, NaCl, glucose, amino acis III. Ileum A. Three meters long, short vasa recta from SMA, atter mucosa with Peyer patches lymphoi tissue B. Terminal ileum (TI) is chief site of absorption of B1, folate, an bile salts (conjugate in TI, unconjugate elsewhere in ileum) IV. Absorptive Defects A. Diagnostic tests 1. Suan re stain: etects presence of fat in stool . Schilling test: etects B1 eciency; oral labele B1 given with unlabele IM B1 to force urinary excretion; if raiolabele B1 in urine, able to absorb B1; if none, repeate with PO intrinsic factor; if labele B1 in urine → pernicious anemia; if not, primary small bowel (SB) malabsorption 3. D-xylose test: PO D-xylose given, if not excrete in urine, then primary SB absorption ecit B. Short Gut Synrome (SGS) 1. Dene by malabsorption ue to loss of intestinal length; iagnosis base on symptoms . Due to both loss of absorption an ecrease transit time 3. Risk of SGS with less than 180 cm of small bowel; lower risk with competent ileocecal valve a) Having 50% to 100% of functional large intestine is equivalent to 50 cm small bowel b) Ileum can aapt after resection by ilating, elongating, increasing size of villi to increase absorption, more important than jejunum in SGS 105
106
PArt i Patient Care
4. Complications: total parenteral nutrition (TPN) associate liver isease, cholelithiasis, calcium oxalate nephrolithiasis, coagulopathy, bacterial overgrowth, sepsis 5. Tx: prevention of sepsis, slow transit (loperamie, etc.), reuce GI secretion with octreotie an PPI, restrict oxalate in iet 6. Teuglutie: GLP- agonist to promote intestinal absorption an health 7. Surgical tx: slow transit by interposing colon, reversing intestinal segments, lengthening proceures (STEP), intestinal transplantation V. Structural Disease A. Duoenal iverticula 1. Most common small bowel iverticula; most are asymptomatic . Operate only if symptomatic with small bowel obstruction (SBO), biliary obstruction, concern for malignancy VI. Mechanical Disease A. Small bowel obstruction 1. Most commonly ue to ahesions in rst worl; hernia elsewhere . Early small bowel follow-through (SBFT) to etermine likelihoo of resolution with conservative management 3. Close-loop obstruction may have normal lactate 4. Avoi nitrous oxie uring anesthesia as gas thir spaces 5. Law of LaPlace: P=T/r 6. Malignancy is suspecte for SBO in a virgin abomen; always check for a hernia B. Stricture 1. Heineke-Mikulicz: stricture 15 cm or multiple consecutive strictures, bowel ivie at mipoint of stricture, stenotic areas overlappe, longituinal enterotomies along stricture, then anastomose in sie-to-sie isoperistaltic fashion VII. Small Bowel Neoplasms A. Carcinoi: most common SB neoplasm 1. Slow-growing tumors of Kulchitsky enterochromafn cells, usually nonfunctional, carcinoi synrome when functional . Most commonly locate in ileum followe by rectum 3. Strong esmoplastic reaction an mesenteric brosis surrouning can obstruct 4. Carcinoi synrome: secretes serotonin, histamine, VIP, braykinin, prostaglanins which cause ushing, iarrhea, bronchoconstriction, an right-sie valvular heart isease (most commonly tricuspi insufciency); tx=octreotie 5. Elevate chromogranin-A, pancreatic polypeptie, NSE, 5-HIAA 6. Resect if possible; can ebulk liver mets 7. Gastric carcinoi (types I an II have hypergastrinemia; type III oes not) a) Type I: autoimmune etiology associate with atrophic gastritis; associate with hypergastrinemia an in setting of chronic atrophic gastritis or pernicious anemia, rarely malignant, usually women, small an multicentric; if less than 1 cm without concerning risk factors, can remove enoscopically b) Type II: associate with the Zollinger-Ellison synrome (hypergastrinemia) an MEN-1; grows slowly but more likely to metastasize to lymph noes an istant sites; somatostatin analogs may be initiate an result in tumor regression; nees gastric resection c) Type III: usually poorly ifferentiate neuroenocrine cell, normal gastrin level, high rate of cancer, an usually nees partial gastrectomy an lymph noe issection
CHAPtEr 9 Alimentary Tract—SmallBowel
107
8. Appeniceal carcinoi: a) cm or at base of appenix: right hemicolectomy 9. Bronchial carcinoi: a) Carcinoi tumors may be iagnose by bronchoscopy, appearing as pink or purple friable enobronchial masses covere by intact epithelium b) Tx: Complete surgical resection with meiastinal lymph noe sampling or issection, regarless of the presence of noal involvement 10. Diagnosis a) Octreotie scan: best for localizing; inium 111-labele somatostatin scintigraphy b) Ga-68 DOTATATE PET/CT: better than octreotie scan for localizing NETs c) Chromogranin A level most sensitive for etection B. Aenocarcinoma 1. Secon most common primary SB tumor . Risk factors: smoking, EtOH, peptic ulcer isease, celiac isease, Crohn isease, FAP, HNPCC, PJS 3. Goal is R0 resection with 10 mesenteric lymph noes, may nee Whipple C. Lymphoma 1. From lymphoi tissues such as Peyer patches, thus more common in jejunum an ileum . B-cell NHL most common, better prognosis than T-cell lymphomas 3. Enteropathy-associate T-cell lymphoma has poor prognosis, associate with celiac 4. Tx is surgical resection with ajuvant CHOP or R-CHOP chemotherapy D. Metastases 1. Melanoma most common; colon, breast, lung, kiney also seen VIII. Fistulas an Ileostomies A. High output stula >500 cc/ay, unlikely to spontaneously resolve B. Check for FRIENDS: foreign boies, raiation, inammation/inammatory bowel isease, neoplasm, istal obstruction, sterois/sepsis C. High ileostomy output consiere >100 cc/ay an manage with antimotility agents, maintain hyration D. Loss of bicarb leas to uric aci nephrolithiasis in high output stulae an ostomies IX. Crohn Disease A. Abominal pain, iarrhea +/– hematochezia, ileitis, obstruction, perianal isease, lateral ssures B. Aphthous ulcers, anemia, vitamin eciencies, malnutrition, cholesterol gallstones, oxalate kiney stones, blin loop synrome, stulae, megaloblastic anemia (ecrease B1) C. Extraintestinal sx that resolve with resection: uveitis, erythema noosum D. Extraintestinal sx that o not resolve with resection: ankylosing sponylitis, pyoerma gangrenosum E. Tx: inuction therapy with sterois (sometimes iniximab), maintenance therapy with biologics an mesalamine/sulfasalazine F. Skip lesions, rare rectal involvement but common perianal involvement, fat creeping, eep transmural ulcers that are serpiginous, common stenotic lesions, noncaseating granulomas on histology G. Surgery to treat obstruction, maintain intestinal length; o not perform stulotomies
108
PArt i Patient Care
Fig. 9.1 Heineke-Mikulicz Stricturoplasty an Finney Stricturoplasty.
CHAPtEr 9 Alimentary Tract—SmallBowel
109
Questions 1. A 55-year-ol woman with a history of total abominal colectomy with en ileostomy for refractory Crohn isease presents with a bulge ajacent to her ileostomy. The bulge has been present for months an has always been easily reucible. However, she frequently evelops bloating, obstipation, an low ostomy output requiring manual reuction of the bulge. What is the best treatment option for this patient? A. Primary parastomal hernia repair B. Relocation of her ileostomy with mesh closure of the previous efect C. Parastomal hernia repair with mesh D. Ileal pouch-anal anastomosis E. Observation 2. A 6-year-ol female presents to clinic for her -week follow-up appointment after unergoing a low anterior resection with iverting loop ileostomy for rectal cancer. Her incisions are healing well. She states her ostomy has put out 1.9 L per ay an she has ouble her aily ui intake ue to increase thirst. Her serum creatinine remains normal an she has no electrolyte abnormalities. What is the best treatment option for this patient? A. Amission to the hospital for IV hyration B. Oral loperamie an close outpatient monitoring C. Revision to a more istal ostomy D. Methylnaltrexone an close outpatient monitoring E. Observation 3. A 55-year-ol man with a history of heavy nonsteroial antiinammatory rugs (NSAID) use is amitte with a perforate antral ulcer. He unergoes a Billroth II reconstruction. On postoperative ay 4, he evelops acute abominal pain an hemoynamic instability. Which of the following complications is most likely causing this presentation? A. Anastomotic leak of gastrojejunostomy B. Efferent limb synrome C. Duoenal stump blowout D. Marginal ulcer E. Internal hernia
4. Two weeks after an open aortic aneurysm repair, the patient presents with marke abominal istention without nausea or vomiting. There is no tenerness on abominal examination. Plain lms are unremarkable. CT scan reveals a large amount of ascites but is otherwise unremarkable. Paracentesis reveals turbi ui that is culture negative. Flui analysis reveals a triglycerie level of 400 mg/L. The white bloo cell count is 600 cells/μL with a preominance of lymphocytes. Which of the following is true about this conition? A. The patient shoul be place on total parental nutrition (TPN) an NPO B. Octreotie is not useful C. The patient shoul immeiately be reexplore D. Interventional raiology (IR) embolization is rst-line treatment E. Most patients respon to a high-protein, lowfat iet with meium-chain triglyceries 5. Which of the following is true regaring short bowel synrome in aults? A. The presence of an intact ileocecal valve reuces malabsorption B. It is ene as less than 300 cm of the resiual small bowel C. Resection of the ileum is better tolerate than resection of the jejunum D. The presence of an intact colon oes not alter the severity E. It is most commonly cause by multiple operations requiring small bowel resection 6. Which of the following is true regaring the management of short bowel synrome? A. Glutamine shoul be avoie B. Octreotie is the cornerstone of management C. Coeine is contrainicate D. Early enteral feeing is inicate E. Patients who require TPN after 6 months will require permanent TPN
110
PArt i Patient Care
7. A 6-year-ol boy has short bowel synrome cause by migut volvulus that evelope uring infancy an has since been epenent on TPN, which he has tolerate well. He has approximately 8 cm of small bowel remaining with an intact colon. The small bowel is markely ilate without evience of small bowel obstruction. Which of the following is the best option? A. Serial transverse enteroplasty proceure (STEP) B. Continue with TPN C. Small bowel transplantation D. Small bowel tapering proceure E. Tapering an lengthening proceure (Bianchi) 8. Which of the following is true regaring small bowel neoplasms? A. Aenocarcinoma is the most common type B. Small bowel lymphoma most commonly occurs in the uoenum C. The incience of primary small intestinal cancers is increasing D. Five-year survival is higher for aenocarcinoma compare with carcinoi tumors E. Small bowel lymphoma is primarily treate by chemotherapy 9. A 68-year-ol woman presents with an exacerbation of congestive heart failure an acute abominal pain. Physical examination of the abomen is signicant for mil iffuse abominal tenerness but no reboun or guaring. CT arteriography of the abomen emonstrates iffuse narrowing of the superior mesenteric artery (SMA) an its branches but no vascular occlusion, pneumatosis, free air, or portal venous gas. Which of the following is an appropriate management option? A. IV heparin rip B. Exploratory laparotomy C. Aggressive ui resuscitation D. Intraarterial papaverine E. Increase cariac output 10. Which of the following is true regaring carcinoi? A. The majority of carcinoi synrome is from appeniceal tumors that have metastasize B. The most common symptom of carcinoi synrome is iarrhea C. Chromogranin A will not be elevate in nonfunctioning tumors D. Patients are at an increase risk for glossitis E. Urinary 5-hyroxyinoleacetic aci (5-HIAA) is not sensitive for etecting metastatic carcinoi
11. Which of the following is the best test for prognosis an monitoring treatment response in carcinoi tumors? A. Platelet serotonin levels B. 4-hour urinary 5-HIAA test C. Serum chromogranin A levels D. Serum serotonin levels E. Neuron-specic enolase 12. Which of the following is true regaring small bowel obstruction? A. The most common worlwie etiology is ahesions from prior surgery B. It is more frequent with upper intestinal than lower intestinal surgery C. In a complete close-loop obstruction, serum lactate can be normal D. Partial obstruction symptoms typically improve within 4 hours with conservative management E. Abominal pain isproportionate to exam nings occurs early in the setting of obstruction 13. Which of the following is true regaring uoenal iverticula? A. They ten to occur on the antimesenteric sie of the bowel B. Most are ientie in young patients C. Treatment with enoscopic interventions is contrainicate D. Malabsorption ue to bacterial overgrowth within the iverticula manates surgery E. When iscovere incientally at surgery, they shoul be left alone 14. Which of the following is the most common cause of obscure GI bleeing in aults? A. Small intestine angioysplasia B. Meckel iverticulum C. Crohn isease D. Infectious enteritis E. Vasculitis 15. Which of the following is true regaring GISTs of the small bowel? A. Most patients are symptomatic with GI bleeing B. They stain positive for CD134 C. Patients eeme caniates for chemotherapy shoul receive it for 1 year D. A patient with a 6-cm tumor shoul receive ajuvant chemotherapy E. Malignancy is primarily etermine by evience of local invasion
CHAPtEr 9 Alimentary Tract—SmallBowel
16. A hernia sac containing a Meckel iverticulum is known as: A. Petit hernia B. Littre hernia C. Spigelian hernia D. Richter hernia E. Grynfeltt hernia 17. Superior mesenteric artery (SMA) (Wilkie) synrome: A. Involves the secon portion of the uoenum B. Causes venous outow obstruction from the left kiney C. Is best iagnose with arteriography D. Shoul initially be manage with a high caloric intake iet E. Is best manage by gastrojejunostomy 18. A 45-year-ol woman with a history of laparotomy an 5000 cGy of abominal an pelvic irraiation for ovarian cancer 10 years ago presents with symptoms an signs of an acute bowel obstruction. CT scan shows a complete small bowel obstruction at the level of the mi jejunum with no evience of any masses. Which of the following is true about this conition? A. If a stricture is present, it is best manage by strictureplasty B. Sterois shoul be aministere C. Acute raiation enteritis is ue to an obliterative arteritis D. The risk of this complication increases in the setting of iabetes E. The egree of raiation amage is not affecte by whether the patient receive chemotherapy 19. A 75-year-ol male with a history of chronic obstructive pulmonary isease (COPD) presents to the ED with a 1-ay history of abominal istention an nausea. He enies abominal pain. Abominal examination is benign. Laboratory values are normal. CT scan emonstrates free air uner the iaphragm an thin-walle, airlle cysts within the bowel wall. Which of the following is true regaring this conition? A. Laparotomy is inicate B. The primary form occurs more commonly than the seconary form C. It is unlikely to be relate to the patient’s COPD D. It is most commonly seen in the ileum E. It is associate with steroi use
111
20. Which of the following is true regaring PeutzJeghers synrome? A. Patients shoul begin breast an cervical cancer screening at age 5 B. It is autosomal recessive C. Small bowel obstruction is uncommon D. Prophylactic colectomy is recommene to most patients starting at age 0 E. These patients are not at increase risk for small bowel cancer 21. Which of the following is correct with regars to Crohn isease? A. Mesenteric fat wrapping is consiere pathognomonic B. Symptoms of ankylosing sponylitis improve with resection of isease bowel C. The majority of patients with an initial presentation of terminal ileitis progress to Crohn isease on long-term follow-up D. Exaggerate skin injury after minor trauma (pathergy) is a commonly associate conition E. Pyoerma gangrenosum is commonly foun on the initial presentation of Crohn isease 22. Which intestinal cells have been implicate in the formation of gastrointestinal stromal tumors (GISTs)? A. Goblet cells B. Interstitial cells of Cajal C. Enteroenocrine cells D. Paneth cells E. Absorptive enterocytes 23. A 46-year-ol woman is about to unergo hepatic resection for a metastatic carcinoi tumor. During anesthesia inuction, her bloo pressure ecreases to 80 mmHg systolic an her heart rate increases to 110 beats per minute. Her entire boy appears ushe. Her temperature is normal, as is en-tial CO. Management consists of: A. Corticosterois B. Antihistamine C. Octreotie D. Abort operation E. Dantrolene
112
PArt i Patient Care
24. A 70-year-ol woman presents with vague abominal pain, iarrhea, steatorrhea, an anemia with an elevate mean corpuscular volume. Her meical an surgical history is unremarkable. A CT scan of the abomen an pelvis is negative. An upper GI series an small bowel follow-through are signicant only for a large jejunal iverticulum. Which of the following is true regaring this patient? A. It is typically cause by an autoimmune etiology B. A long-chain triglycerie iet may be helpful C. The iverticulum shoul be resecte D. Broa-spectrum antibiotics are inicate only if the patient presents with a fever an leukocytosis E. Vitamin B1 is inicate 25. A 57-year-ol male with no past surgical history presents with ays of abominal pain, nausea, an vomiting. On exam he is istene an tympanic an is milly tener to palpation without reboun or guaring. He has no groin hernias. Compute tomography (CT) scan emonstrates multiple ilate loops of small bowel with a transition point in the istal small bowel, with some ajacent mesenteric fat straning. He has a mil leukocytosis. His last bowel movement was 1 ay ago. He has not passe atus for over a ay. He has not ha any similar symptoms previously. A nasogastric (NG) tube is place, intravenous (IV) uis are aministere, an the patient is place NPO (nothing by mouth). Which of the following is the best next step in management? A. A 4-hour trial of NG tube suction, then exploratory laparotomy if high output continues B. Exploratory laparotomy C. Water-soluble oral contrast challenge D. Water-soluble oral an rectal contrast challenge E. Diagnostic laparoscopy 26. Which of the following has been shown to be the most efcacious means of reucing postoperative ileus in patients unergoing bowel resection? A. Early ambulation B. Gum chewing C. Alvimopan D. Ketorolac combine with reuction in opioi use E. Nasogastric intubation
27. Which of the following is true regaring Crohn isease? A. It is more common in iniviuals of high socioeconomic status B. The most common inication for surgery is perforation C. It has a unimoal istribution D. It is more prevalent in females E. The most common initial presentation is an acute onset of abominal pain an iarrhea 28. The earliest lesion characteristic of Crohn isease is: A. Aphthous ulcer B. Caseating granuloma C. Noncaseating granuloma D. Cobblestone mucosa E. Serosal thickening 29. Which of the following is the best therapeutic option for mil active Crohn isease? A. Sulfasalazine B. Prenisone C. Buesonie D. Metroniazole E. Iniximab 30. Which of the following is true regaring the principles of operative management of the small bowel in Crohn isease? A. The optimal margin is at least 4 cm beyon grossly visible isease B. Frozen section shoul be obtaine to conrm the absence of active isease in at least one margin C. A 3-cm stricture segment of uoenum is best manage by resection D. A 10-cm stricture segment of jejunum can be manage by a Heineke-Mikulicz strictureplasty rather than by resection E. Strictures longer than 10 cm are best manage by resection
CHAPtEr 9 Alimentary Tract—SmallBowel
113
Answers 1. C. This patient presents with a chronic, reucible para-
stomal hernia. The enitive treatment for parastomal hernia repair is ostomy reversal; however, given this patient’s Crohn isease, ostomy takeown with ileoanal anastomosis or ileal pouch-anal anastomosis is contrainicate (D). Given the recurrent iscomfort an intermittent obstruction from her hernia, simple observation woul not be appropriate (E). Once taught as the surgical treatment of choice for parastomal hernia repair, ostomy relocation is no longer avise as it creates the potential for a new parastomal hernia an for hernias from prior ostomy site an laparotomy incisions. SAGES now recommens against ostomy relocation as the treatment of parastomal hernias (B). Primary parastomal hernia repair results in high tension with a high rate of hernia recurrence (A). The most effective treatment for parastomal hernias ajacent to ostomies that are unable to be reverse is mesh repairs (C). Stanar approaches inclue the Sugarbaker repair, in which an unerlay mesh is place on the efect with the stoma exiting the peritoneum at the sie of the mesh, an the keyhole approach, when the stoma is brought through a hole create in the mesh. Reference: Gillern S, Bleier JIS. Parastomal hernia repair an reinforcement: the role of biologic an synthetic materials. Clin Colon Rectal Surg. 014;7(4):16–171.
2. B. This patient presents with high ostomy output, usu-
ally ene as over 1. L per ay. High ostomy output can result in ehyration, loss of bicarbonate resulting in uric aci nephrolithiasis, an skin breakown. In the absence of ehyration or electrolyte abnormalities requiring inpatient amission, high ostomy output can be manage as an outpatient with close follow-up an titration of oral loperamie or Lomotil. When high ostomy output results in symptomatic ehyration or electrolyte isturbances, amission for inpatient hyration an monitoring is recommene (A). Ostomy revision is not typically require for high ostomy output an iverting loop ileostomies are usually istal enough to aequately absorb ui (C). Methylnaltrexone is an opioi antagonist use to treat opioi-inuce ileus an is not absorbe enterally (D). Observation is not appropriate for high ileostomy output (E).
3. C. The most serious complication after a Billroth II recon-
struction is a uoenal stump blowout. In a Billroth II, the pylorus an antrum are resecte, an the rst portion of the uoenum is oversewn. The biliary system rains into the uoenum, which then rejoins path of foo at a surgically constructe gastrojejunostomy. The inammation cause by a perforate ulcer results in inammation of the uoenum, which may leak after oversewing. Aitional causes of uoenal stump blowout inclue afferent limb obstruction an pancreatitis. Efferent limb synrome is the result of obstruction of the efferent limb istal to the gastrojejunostomy, resulting in bile reux an bilious emesis (B). Anastomotic leak of the gastrojejunostomy is possible but less common an catastrophic than uoenal stump blowout. A marginal ulcer is more likely to occur after a Billroth II reconstruction,
as compare to after a Billroth I, because the alkaline environment secrete by Brunner glans in the uoenum is not present in the jejunum. However, marginal ulcers, which are present on the jejunal sie of the anastomosis, present with abominal pain an possible upper GI bleeing (D). Any proceure that can result in ahesions has the potential to cause an internal hernia, though the highest chance is after a gastric bypass. The presentation of internal hernia is typically inolent with vague abominal pain, nausea, an vomiting an can be confuse for gastroenteritis or peptic ulcer isease. A high inex of suspicion is require, an the iagnosis often requires iagnostic laparoscopy for conrmation (E).
4. E. The patient has chylous ascites. In Western countries,
chylous ascites is most often ue to malignancy an cirrhosis, whereas infectious etiologies such as tuberculosis an lariasis preominate in Eastern an eveloping countries. Other causes inclue postlaparotomy inammatory isorers, trauma, raiation therapy, congenital lymphatic abnormalities, an pancreatitis. The operations most associate with this complication inclue aortic aneurysm repair, retroperitoneal lymph noe issection, inferior vena cava surgery, an liver transplantation, because these are operations in which retroperitoneal lymphatics are most likely to be interrupte. The mechanisms thought to lea to the evelopment of chylous ascites inclue exuation of chyle ue to obstruction of the cisterna chyli, irect leakage of chyle through a lymphoperitoneal stula, an exuation through ilate retroperitoneal vessels. The iagnosis of chylous ascites is best establishe by analysis of the ui. Chyle typically has a turbi appearance; however, it may be clear in fasting patients. Elevate triglycerie levels in the ui are consiere iagnostic, usually above 00 mg/L, although some use a threshol above 110 mg/L. In aition, the white bloo cell count is greater than 500, with a preominance of lymphocytes. The total protein level is between .5 an 7.0g/L. Cultures are negative, except for cases of tuberculosis, in which aenosine eaminase is also positive in the ui. The initial treatment of chylous ascites is to aminister a high-protein, low-fat iet with meium-chain triglyceries. This iet minimizes chyle prouction an ow. Meium-chain triglyceries are absorbe by the intestinal epithelium an are transporte to the liver through the portal vein an o not contribute to chylomicron formation. Conversely, long-chain triglyceries are converte to monoglyceries an free fatty acis, which are then transporte to the intestinal lymph vessels as chylomicrons. If this iet regimen fails, placing the patient NPO an on TPN with octreotie has been shown to be useful in patients with postoperative chylous ascites (A, B). If these meical approaches fail, then lymphoscintigraphy is often useful to localize lymph leaks an the site of obstruction. In some instances, IR can percutaneously inject glue to stop leak. Surgical reexploration with localization an closure of the lymphatic leak shoul be performe if leak persists beyon weeks (C). Alternatively, in facilities with capabilities to perform
114
PArt i Patient Care
percutaneous lymphangiography, embolization of lymphatics may be attempte after faile ietary management (D). This latter complication may be ue to a high plasminogen concentration in the ascitic ui. Reference: Cárenas A, Chopra S. Chylous ascites. Am J Gastro-
Intestinal aaptation occurs over a perio of 1 to years in most aults. Thus, the nal etermination of whether permanent TPN will be necessary is not etermine until after this perio (E).
enterol. 00;97(8):1896–1900.
7. A. Many patients with short bowel synrome can even-
5. A. The total length of small bowel is approximately 0
feet (each foot is equal to ≈30 cm), or approximately 600 cm (6 m). Short bowel synrome is ene as the presence of less than 180 cm of resiual an functional small bowel in ault patients (B). Thus, resection of less than 50% of the small intestine is generally well tolerate. In approximately 75% of cases, short bowel synrome results from one massive small bowel resection, as oppose to multiple sequential resections (E). In aults, the most common etiologies inclue acute mesenteric ischemia, malignancy, an Crohn isease. In peiatric patients, the most common etiologies inclue intestinal atresia, migut volvulus, an necrotizing enterocolitis. Resection of the jejunum is better tolerate than resection of the ileum because the absorption of bile salts an vitamin B1 occurs in the ileum (C). An intact ileocecal valve is thought to reuce malabsorption because it increases the resience time of the chyme in the small intestine. Likewise, an intact colon is important because it has a tremenous water-reabsorbing capacity an electrolytes an can also absorb fatty acis (D). With an intact colon, a shorter small bowel remnant is tolerate. The key to preventing short bowel synrome is avoiance of excessive small bowel resection. In Crohn patients, the use of strictureplasty as oppose to resection is recommene when possible. Also, one shoul resect only obviously ea bowel in acute mesenteric ischemia, leaving marginal bowel in situ an performing a secon-look proceure.
6. D. In the early phase of short bowel synrome, treat-
ment is irecte at slowing intestinal transit; reucing GI secretions; an maintaining nutrition, ui, an electrolyte balance. Transit time is slowe by the aministration of narcotics such as coeine an iphenoxylate, as well as with the antimotility agents Lomotil (iphenoxylate an atropine) an loperamie (C). Massive small bowel resection is associate with hypergastrinemia an aci hypersecretion. The increase aciity in the small bowel results in the inhibition of igestive enzymes. This can be controlle with H-receptor antagonists or proton pump inhibitors such as omeprazole an thus shoul be starte in all patients with short gut synrome (B). Nutrition is achieve with the institution of TPN. In aition, enteral feeing shoul be institute as soon as postoperative ileus has resolve. Enteral feeing assists in the process of intestinal aaptation an prevents the evelopment of villous atrophy associate with being NPO for a prolonge perio of time. Glutamine is helpful because it serves as a trophic factor for the gut an is consiere the principal fuel of the small intestine (A). Cholestyramine is also useful in controlling iarrhea ue to unabsorbe bile salts. The role of octreotie is controversial. Short-term use leas to a reuction in iarrhea, but long-term use may lea to steatorrhea, gallstones, an an inhibition of intestinal aaptation. More recently, a high-carbohyrate, low-fat enteral iet rich in glutamine combine with growth hormone aministration has shown promise in improving intestinal absorptive capacity.
tually iscontinue TPN, particularly if the bowel length is more than 10 cm in aults or more than 60 cm in chilren. Treatment options for short bowel synrome epen on the length of small bowel remaining, whether the remnant small bowel is markely ilate, whether the patient remains TPN epenent, an whether multiple complications of TPN have evelope such as catheter-relate infections, vena cava thrombosis, an liver amage (B). A short remnant (5 cm or >5 mitoses per 50 high-power el (E). The ajuvant treatment of GISTs inclues chemotherapy with imatinib (Gleevec), a tyrosine kinase inhibitor. In one stuy, imatinib controlle tumor growth in as many as 85% of avance GISTs. Currently, imatinib is recommene for unresectable, metastatic, or recurrent lesions. Ajuvant therapy shoul continue for a total of 3 years (C). Patients that harbor an exon 9 KIT mutation will require a higher ose of imatinib (800 mg aily versus 400 mg). The most useful inicators of survival an the risk of metastasis inclue the size of the tumor at presentation, the mitotic inex, location within the GI tract, an the absence of tumor rupture. References: Blay JY, Bonvalot S, Casali P, et al. Consensus meeting for the management of gastrointestinal stromal tumors. Report of the GIST Consensus Conference of 0-1 March 004, uner the auspices of ESMO. Ann Oncol. 005;16(4):566–578. Crosby JA, Catton CN, Davis A, et al. Malignant gastrointestinal stromal tumors of the small intestine: a review of 50 cases from a prospective atabase. Ann Surg Oncol. 001;8(1):50–59. Dematteo RP, Ballman KV, Antonescu CR, et al. Ajuvant imatinib mesylate after resection of localise, primary gastrointestinal stromal tumour: a ranomise, ouble-blin, placebo-controlle trial. Lancet. 009;373(9669):1097–1104. Joensuu H, Eriksson M, Sunby Hall K, et al. One vs three years of ajuvant imatinib for operable gastrointestinal stromal tumor: a ranomize trial: A ranomize trial. JAMA. 01;307(1):165–17.
16. B. A hernia sac containing a Meckel iverticulum is
calle a Littre hernia. Lumbar hernias can be either congenital or acquire an occur in the lumbar region of the posterior abominal wall. Hernias through the superior lumbar
117
triangle (Grynfeltt triangle) (E) are more common than those through the inferior lumbar triangle (the Petit triangle) (A). The Petit triangle is boune by the external oblique muscle, latissimus orsi muscle, an iliac crest. The Grynfeltt triangle is boune by the quaratus lumborum muscle, the 1th rib, an the internal oblique muscle. A spigelian hernia occurs through the spigelian fascia, which is compose of the aponeurotic layer between the rectus muscle meially an the semilunar line laterally (C). Nearly all spigelian hernias occur in the spigelian belt locate below the umbilicus but above the epigastric vessels. The absence of posterior rectus fascia may contribute to an inherent weakness in this area. A Richter hernia occurs when only the antimesenteric borer of the bowel herniates through the fascial efect (D). It involves only a portion of the circumference of the bowel. As such, incarceration an strangulation may occur in the absence of any evience of bowel obstruction. Reference: Skanalakis PN, Zoras O, Skanalakis JE, Mirilas P. Spigelian hernia: surgical anatomy, embryology, an technique of repair. Am Surg. 006;7(1):4–48.
17. D. The SMA leaves the aorta at a ownwar an acute
angle. SMA synrome or Wilkie synrome is a rare conition characterize by compression of the thir portion of the uoenum by the SMA as it passes over this portion of the uoenum (A). It occurs most often in the setting of profoun weight loss. Factors that preispose to the conition inclue supine immobilization, scoliosis, placement of a boy cast, an eating isorers. Symptoms inclue profoun nausea an vomiting, abominal istention, weight loss, an postpranial epigastric pain, which varies from intermittent to constant, epening on the severity of the uoenal obstruction. Weight loss usually occurs before the onset of symptoms. It is believe to occur more commonly in women, likely seconary to the increase prevalence of anorexia. However, a recent stuy of SMA synrome among intellectually isable chilren showe that it preominantly affects males. The iagnosis can be mae by a CT scan, which emonstrates a ecrease aortomesenteric angle an a ecrease istance between the aorta an the SMA, as well as evience of obstruction of the uoenum (C). It can also be iagnose by a barium upper GI series or hypotonic uoenography, emonstrating abrupt or near-total cessation of ow of barium from the uoenum to the jejunum. Conservative measures that are trie initially are primarily focuse on weight gain to increase the mesenteric root fat pa. The operative treatment is uoenojejunostomy (E). Nutcracker synrome is characterize by compression of the left renal vein by the aorta, superior to the uoenum (B). References: Ason DE, Mitchell JE, Trenkner SW. The superior mesenteric artery synrome an acute gastric ilatation in eating isorers: a report of two cases an a review of the literature. Int J Eat Disord. 1997;1():103–114. Agrawal GA, Johnson PT, Fishman EK. Multietector row CT of superior mesenteric artery synrome. J Clin Gastroenterol. 007;41(1):6–65. Geskey JM, Erman HJ, Bramley HP, Williams RJ, Shaffer ML. Superior mesenteric artery synrome in intellectually isable chilren. Pediatr Emerg Care. 01;8(4):351–353.
18. D. The small-intestinal epithelium is acutely susceptible
to raiation injury because raiation has its greatest impact on rapily proliferating cells. Raiation-inuce injury to the
118
PArt i Patient Care
bowel can present with acute or chronic enteritis. Approximately 75% of patients unergoing raiation therapy for abominal an pelvic cancers evelop acute raiation enteritis transiently. Chronic raiation enteritis results from an obliterative arteritis in the submucosal vessels, while acute raiation enteritis is a transient perio of nausea, vomiting, iarrhea, an abominal pain that occurs aroun 3 weeks after treatment (C). This leas to progressive submucosal brosis an stricture formation. Not infrequently, patients with raiation-inuce injury may evelop a small bowel obstruction. The risk of raiation enteritis correlates with the amount of raiation receive. It is uncommon if the total raiation ose is less than 4000 cGy. The risk of raiation amage increases if the patient receive chemotherapy or has unerlying vascular isease or iabetes (E). Early symptoms of raiation amage inclue iarrhea, abominal pain, an malabsorption an are usually self-limite. The treatment of acute raiation enteritis inclues antispasmoic agents, analgesic agents, an antiiarrheal agents. Sterois are not use in the management of raiation enteritis (B). Only a small group of patients with chronic raiation enteritis will require surgery for either SBO from stricture formation or stulas. Unlike Crohn isease, for which strictureplasty is use, it is not recommene for raiation enteritis because there is a high risk of tissue breakown (A). The extent of macroscopic raiation injury is ifcult to etermine on gross inspection. Extensive lysis of ahesions shoul be avoie because this creates a risk of an enterotomy an subsequent stula formation as well. The two main surgical proceures are primary resection with reanastomosis or bypass. If the source of obstruction is a loop of bowel stuck in the pelvis, it is best treate with a bypass rather than an attempt to take own the ahesions an risk injury. References: Gallan RB, Spencer J. Natural history an surgical management of raiation enteritis. Br J Surg. 1987;74(8):74–747. Tavakkoli A, Ashley SW, Zinner MJ. Small Intestine. In: Brunicari F, Anersen DK, Billiar TR, Dunn DL, Hunter JG, Matthews JB, Pollock RE, es. Schwartz’s principles of surgery. 10th e. New York: McGraw-Hill Eucation; 015:1146–1151.
19. E. Pneumatosis intestinalis is a raiographic ning
an not a isease unto itself. Its iscovery on imaging is vexing because it can be a completely benign ning, or it can be associate with life-threatening bowel ischemia. It has been ivie into primary an seconary pneumatosis intestinalis. The primary form is less common an is terme pneumatosis cystoides intestinalis (B). It consists of thin-walle, air-lle cysts within the bowel wall, usually in the colon, but it can occur anywhere in the GI tract (D). It is an inciental ning, an the iagnosis is reaily mae on plain raiograph or CT scan. The gas can appear as linear, curvilinear, bubbly, or cystic. There is no specic treatment (A). Seconary pneumatosis intestinalis occurs when there is an unerlying isease process. The exact cause of pneumatosis intestinalis is unclear, but there seem to be several pathways that allow gas to enter the bowel wall. Immunoecient an inammatory bowel states lea to a loss of mucosal barrier function that may permit air to enter the bowel wall. Bowel obstruction leas to gas formation uner pressure. Alterations in bacteria ora, with invasion of the bowel wall, likewise lea to gas formation. In aults, seconary pneumatosis intestinalis is most often associate with COPD (C). It is also
seen with collagen vascular isease, celiac sprue, Crohn isease, use of sterois, an in immunoecient states. More ominously, it is also associate with ischemic bowel. Thus, it is important to recognize that not all cases of pneumatosis are benign. In neonates, it is most commonly associate with necrotizing enterocolitis. The ning of pneumatosis intestinalis in association with necrotizing enterocolitis oes not manate surgical exploration. It is also seen with pyloric stenosis, Hirschsprung isease, an other causes of bowel obstruction. Pneumoperitoneum can rarely be the result of a benign case of pneumatosis intestinalis because the air-lle cysts are thin-walle an can burst. References: Mularski RA, Ciccolo ML, Rappaport WD. Nonsurgical causes of pneumoperitoneum. West J Med. 1999;170(1):41–46. Peter S, Abbas M, Kelly K. The spectrum of pneumatosis intestinalis. Arch Surg. 003;138(1):68–75. Hsueh KC, Tsou SS, Tan KT. Pneumatosis intestinalis an pneumoperitoneum on compute tomography: beware of non-therapeutic laparotomy. World J Gastrointest Surg. 011;3(6):86–88.
20. A. Peutz-Jeghers synrome features mucocutaneous
melanotic pigmentation an hamartomatous polyps (not aenomatous) of the small intestine. It is an autosomal ominant inherite synrome (B). The skin lesions are foun in the circumoral region of the face, buccal mucosa, forearms, palms, soles, igits, an perianal area, whereas the hamartomas are usually in the jejunum an ileum. The most common symptom is recurrent colicky abominal pain (C). Symptoms of a bowel obstruction evelop in as many as 50% of patients, which is usually ue to intussusception or obstruction by the polyp itself. Hemorrhage or chronic anemia can also occur as a result of the polyps. The polyps can also unergo aenomatous change. Patients are at signicantly increase risk of eveloping cancer in the GI tract (esophagus, stomach, small intestine, colon, an pancreas) an extraintestinal cancer (testis, breast, uterus, ovary). Female patients shoul begin breast an cervical cancer screening starting at age 5. Over the long term, cancer evelops in as many as 90% of patients. Compare with the general population, they are at 500 times increase risk of the evelopment of small intestine cancer (E). Operative intervention is only inicate in the presence of symptoms (D). References: Boarman LA, Pittelkow MR, Couch FJ, et al. Association of Peutz-Jeghers-like mucocutaneous pigmentation with breast an gynecologic carcinomas in women. Medicine (Baltimore). 000;79(5):93–98. Giariello FM, Brensinger JD, Tersmette AC, et al. Very high risk of cancer in familial Peutz-Jeghers synrome. Gastroenterology. 000;119(6):1447–1453. Wu YK, Tsai CH, Yang JC. Gastrouoenal intussusception ue to Peutz-Jeghers synrome: a case report. Hepatogastroenterology. 1994;41():134–136. van Lier MGF, Wagner A, Mathus-Vliegen EMH, Kuipers EJ, Steyerberg EW, van Leeram ME. High cancer risk in Peutz-Jeghers synrome: a systematic review an surveillance recommenations. Am J Gastroenterol. 010;105(6):158–164.
21. A. The ning of “creeping fat” or mesenteric fat wrap-
ping is a gross feature of Crohn isease that is consiere pathognomonic. It inicates the encroachment of mesenteric fat onto the serosal surface of the bowel. The presence of fat wrapping correlates well with the presence of unerlying acute an chronic inammation. A recent stuy suggests that
CHAPtEr 9 Alimentary Tract—SmallBowel aiponectin, an aipocyte-specic protein with antiinammatory properties foun in mesenteric aipose tissue, may play an important role in the inammation seen in Crohn isease. Terminal ileitis refers to any acute inammation of the istal ileum ajacent to the ileocecal valve an is therefore not pathognomonic. Terminal ileitis is associate with numerous infectious causes incluing Yersinia enterocolitica an pseuotuberculosis, Mycobacterium, cytomegalovirus (in acquire immunoeciency synrome), Salmonella, Campylobacter, an Shigella, among others. The ning of terminal ileitis oes not warrant bowel resection. Overall, a minority of patients (10% in one stuy) who present with terminal ileitis progress to Crohn isease on long-term follow-up (C). The majority of extraintestinal manifestations in inammatory bowel isease improve with bowel resection but ankylosing sponylitis an primary sclerosing cholangitis o not (B). Pyoerma gangrenosum is rarely the initial presentation of Crohn isease (E). These patients present with small papules often on the lower extremities that resemble a “cat’s paw” appearance an can progress to larger ulcerations with necrotic centers. Rarely, patients evelop pathergy, a conition in which minor trauma leas to the evelopment of large an ifcult-to-heal ulcers (D). Debriement of these lesions shoul be avoie because this worsens the lesion. Iniximab or another tumor necrosis factor-alpha inhibitor shoul be use. References: Menachem Y, Gotsman I. Clinical manifestations of pyoerma gangrenosum associate with inammatory bowel isease. Isr Med Assoc J. 004;6():88–90. Yamamoto K, Kiyohara T, Murayama Y, et al. Prouction of aiponectin, an anti-inammatory protein, in mesenteric aipose tissue in Crohn’s isease. Gut. 005;54(6):789–796. Hatemi I, Hatemi G, Celik AF, et al. Frequency of pathergy phenomenon an other features of Behçet’s synrome among patients with inammatory bowel isease. Clin Exp Rheumatol. 008;6(4 Suppl 50):S91–S95.
22. B. There are four main cell types in the small intestine:
absorptive enterocytes (E), which make up 95% of intestinal cells; goblet cells (A); Paneth cells (D); an enteroenocrine cells (C). Goblet cells secrete mucus. Paneth cells secrete several substances incluing lysozyme, tumor necrosis factor, an cryptiins, which assist in host mucosal efense. There are more than 10 istinct types of enteroenocrine cells that secrete various gut hormones. The interstitial Cajal cell is a specialize cell of mesoermal origin that seems to regulate peristalsis. It is referre to as an intestinal pacemaker cell. The cells normally express KIT, a tyrosine kinase receptor. These cells have been implicate as the cells of origin of GISTs. References: Miettinen M, Majii M, Lasota J. Pathology an iagnostic criteria of gastrointestinal stromal tumors (GISTs): a review. Eur J Cancer. 00;38:S39–S51. Sircar K, Hewlett BR, Huizinga JD, Chorneyko K, Berezin I, Riell RH. Interstitial cells of Cajal as precursors of gastrointestinal stromal tumors. Am J Surg Pathol. 1999;3(4):377–389.
23. C. The patient has a carcinoi crisis. This has been
escribe after anesthetic inuction as well as after other stressful situations such as biopsies or invasive proceures. Carcinoi crisis is characterize by hypotension, bronchospasms, ushing, an tachycaria. The primary treatment is IV octreotie aministere as a bolus of 50 to 100 μg. Even
119
more rarely, a carcinoi crisis can manifest with hypertension. Octreotie is effective for a hypertensive crisis as well. Ajunctive treatment with antihistamines may also be of benet ue to frequent histamine release from carcinoi tumors (B). If the above measures o not resolve the crisis, then aborting the proceure may be necessary (D). Dantrolene is the preferre choice of management for malignant hyperthermia (E). This iagnosis is supporte by an increase in en-tial CO. Corticosterois are not use in the management of carcinoi crisis (A). References: Bax NDS, Woos HF, Batchelor A, Jennings M. Octreotie therapy in carcinoi isease. Anticancer Drugs. 1996;7(Suppl 1):17–. Warner RR, Mani S, Profeta J, Grunstein E. Octreotie treatment of carcinoi hypertensive crisis. Mt Sinai J Med. 1994;61(4):349–355.
24. E. The patient has a blin loop synrome, which is ue
to bacterial overgrowth (A). Symptoms inclue iarrhea, steatorrhea, megaloblastic anemia, weight loss, abominal pain, an eciencies of fat-soluble vitamins. The megaloblastic anemia is ue to the utilization of vitamin B1 by the bacteria. The unerlying cause may be an intestinal abnormality such as a iverticulum, stula, an intestinal stricture, or it may follow a Billroth II proceure. In the patient presente, the large jejunal iverticulum is likely the etiology. The iagnosis can be conrme by various means. A barium stuy is useful to ene the anatomic abnormality. The d-xylose test involves ingesting xylose, which is metabolize by the bacteria. Excessive CO in the breath conrms the iagnosis. Cultures of the small intestine can be obtaine; however, passing an intestinal tube istal enough to obtain an aequate culture can be challenging. Another useful stuy is the Schilling test. Oral raiolabele vitamin B1 is aministere along with parenteral unlabele vitamin B1. The unlabele vitamin B1 saturates liver receptors. Thus, if the oral raiolabele vitamin B1 is properly absorbe an liver receptors are saturate, the raiolabele vitamin B1 will be excrete in high concentrations in the urine. With pernicious anemia an blin loop synrome, oral absorption will be low, an thus urinary excretion of raiolabele vitamin B1 will be low. When the test is repeate after the aition of intrinsic factor, vitamin B1 excretion will increase, whereas with blin loop synrome, vitamin B1 excretion will remain low. The initial treatment of blin loop synrome consists of broa-spectrum antibiotics incluing metroniazole with tetracycline as well as vitamin B1 supplementation given parenterally. This shoul be given to all patients presenting with blin loop synrome (D). Prokinetic agents o not seem to help. In aition, ietary moications such as a lactose-free iet are useful because patients with blin loop synrome often become lactose intolerant. Meium-chain triglycerie iets are more reaily absorbe than long-chain triglyceries because they o not require igestive enzymes (B). Resection of the iverticulum is not recommene initially (C). Surgery shoul be reserve for patients who fail repeate meical management attempts. References: Ross CB, Richars WO, Sharp KW, Bertram PD, Schaper PW. Diverticular isease of the jejunum an its complications. Am Surg. 1990;56(5):319–34. Woos K, Williams E, Melvin W, Sharp K. Acquire jejunoileal iverticulosis an its complications: a review of the literature. Am Surg. 008;74(9):849–854.
120
PArt i Patient Care
25. C. Surgical ogma has state that a small bowel obstruc-
tion (SBO), in the absence of prior surgery or visible external hernia, requires surgical intervention, as the ifferential invariably are all surgical iseases, such as internal hernia, appenicitis, intussusception, inammatory bowel isease, malignancy, or obstructe Meckel iverticulum. However, in a recent stuy, as many as 40% of SBO in patients without prior history of surgery resolve nonoperatively. Of those that require surgical intervention, the majority were foun to have ahesions espite no prior operations (A). Most centers have transitione to a water-soluble oral contrast challenge to help ecie which patients with ahesive SBOs will require surgical intervention. This is being performe even in patients without prior abominal surgery. This entails performing nasogastric ecompression for two hours followe by aministration of a water-soluble contrast either by mouth or via NG tube. This is followe by plain lms 8 hours later. Patients with plain lms emonstrating contrast in the colon after 8 hours are unlikely to require surgical intervention while those without contrast in the colon after 8 hours are more likely to fail nonoperative management (C). Rectal contrast is not typically use in the workup nor in management of ahesive SBO (D). Reference: Ng YYR, Ngu JCY, Wong ASY. Small bowel obstruction in the virgin abomen: time to challenge surgical ogma with evience: Small bowel obstruction in the virgin abomen. ANZ J Surg. 018;88(1–):91–94.
26. C. Postoperative ileus remains a major source of pro-
longe hospitalization in patients unergoing abominal surgery. The use of early ambulation, early postoperative feeing protocols, an routine nasogastric intubation have not been shown to be associate with earlier resolution of postoperative ileus (A, E). Reucing opioi use in combination with the use of nonsteroial antiinammatory rugs such as ketorolac has been shown to reuce the uration of ileus in most stuies. The mechanism may be a combination of the reuction in opiois an the antiinammatory properties of ketorolac. However, ketorolac has been associate with an increase risk of operative site an gastrointestinal (GI) bleeing as well as ui retention (D). Recently, ketorolac has been also shown to increase the risk of reamission an reinterventions after GI surgery. Another rug that has been investigate is erythromycin, which is useful for gastroparesis because it works by its agonistic effect on the motilin receptor. However, it oes not seem to be useful for ileus an shoul be avoie in cases of obstruction, as woul all promotility agents (B). Metoclopramie is a opaminergic antagonist with antiemetic an prokinetic properties, but it has also not been shown to be useful for ileus. Gum chewing has ha conicting results in the literature, but a recent ranomize controlle trial from New Zealan emonstrate a signicant reuction in postoperative ileus in patients with colorectal cancer unergoing bowel resection (7% versus 48%). The most efcacious agent, however, is alvimopan (Entereg), which has been emonstrate in ranomize stuies to improve postoperative ileus in patients unergoing bowel resection. Alvimopan is an opioi receptor antagonist. It bins μ-opioi receptors in the GI tract an selectively inhibits the opioi effects on GI function an motility while not affecting opioi analgesia. It is the rst US Foo an Drug Aministration–approve rug for postoperative ileus. It is
approve for short-term (maximum 15 oses over 5 ays) in-hospital use only. Patients on long-term narcotics (e.g., for chronic pain) shoul not use Alvimopan because this population has an increase risk of myocarial infarction. References: Luwig K, Enker WE, Delaney CP, et al. Gastrointestinal tract recovery in patients unergoing bowel resection: results of a ranomize trial of alvimopan an placebo with a stanarize accelerate postoperative care pathway. Arch Surg. 008;143(11):1098–1105. Wolff BG, Weese JL, Luwig KA, et al. Postoperative ileus-relate morbiity prole in patients treate with alvimopan after bowel resection. J Am Coll Surg. 007;04(4):609–616. Su’a BU, Hill AG. Perioperative use of chewing gum affects the inammatory response an reuces postoperative ileus following major colorectal surgery. Evid Based Med. 015;0(5):185–186. Kotagal M, Hakkarainen TW, Simianu VV, Beck SJ, AlfonsoCristancho R, Flum DR. Ketorolac use an postoperative complications in gastrointestinal surgery. Ann Surg. 016;63(1):71–75.
27. A. Crohn isease is the most common primary surgi-
cal isease of the small bowel. Acute onset of abominal pain an iarrhea is not the most common presentation for Crohn isease; the majority of patients rst present with an insiious onset of vague abominal iscomfort (E). It has a bimoal istribution, with one large peak in the secon an thir ecaes of life an a secon smaller peak in the sixth ecae (C). Several risk factors for Crohn isease have been ientie, incluing living in northern latitues, Ashkenazi Jewish escent, smoking, an a familial inheritance. The relative risk among rst-egree relatives of patients with Crohn isease is as high as 14 to 15 times greater than in the general population. It is also more common in urban areas an in patients with a high socioeconomic status. Most stuies suggest that Crohn isease is approximately of equal prevalence in females an males (D). Breastfeeing may also be protective against the evelopment of Crohn isease. Although meical management is the rst-line treatment for Crohn isease, about 75% of patients will ultimately nee surgery. The most common reasons for surgery inclue stula, abscess, an obstruction; perforation is quite rare (B). References: Passier JLM, Srivastava N, van Puijenbroek EP. Isotretinoin-inuce inammatory bowel isease. Neth J Med. 006;64():5–54. Strong SA. Surgical management of Crohn’s isease. In: Holzheimer RG, Mannick JA, es. Surgical treatment: evidence-based and problem-oriented. Munich: Zuckschwert; 001:714–75.
28. A. In the early stages of Crohn isease, patients emon-
strate small supercial ulcers in the mucosa known as aphthous ulcers. These supercial ulcers are often surroune by a halo of erythema. The ulcers form as a result of submucosal lymphoi follicle expansion. As the isease progresses, the ulcers coalesce to form larger ulcers, which are stellate shape, as well as eep linear ulcers. Further coalescence of the ulcers leas to a cobblestone appearance (D), which is a hallmark of Crohn isease. Other hallmarks of Crohn isease inclue noncaseating granulomas (C), transmural inammation, serosal thickening (E), an “skip lesions,” meaning that the areas of intestinal inammation are iscontinuous. The noncaseating granulomas are foun in both areas of active isease, an grossly normal-appearing intestine is seen in all layers of the bowel wall an in mesenteric lymph noes (B). Because the inammation is transmural, iname loops
CHAPtEr 9 Alimentary Tract—SmallBowel of bowel become ahere to one another, thereby leaing to brosis, stricture formation, intraabominal abscess, stulas, an, rarely, free perforation. Reference: Levine MS. Crohn’s isease of the upper gastrointestinal tract. Radiol Clin North Am. 1987;5(1):79–91.
29. A. Numerous pharmacologic agents are use to treat
Crohn isease. Treatment options shoul be ivie into those use for maintenance therapy for mil active isease, those use to treat an acute exacerbation, an rugs for maintaining remission. In patients with mil active isease, the most commonly use rug is sulfasalazine, an aminosalicylate that acts as an antiinammatory agent. This is particularly useful in patients with colitis an ileocolitis. Mesalamine is another antiinammatory agent in the same family as sulfasalazine. It seems to have fewer sie effects owing to the fact that it is activate by colonic bacteria, thus limiting its action to the colon. For acute are-ups, the treatment of choice remains corticosterois, prenisone in particular. Prenisone is highly effective in inucing remission (in approximately three-fourths of patients); however, ue to the sie effects of long-term use, it is not recommene for longterm prevention of remission (B). Buesonie (C), a synthetic glucocorticoi, is another option. It has an avantage over prenisone in that it has a markely reuce systemic absorption an thus fewer long-term sie effects. Nevertheless, it can also suppress the arenal glan. If corticosterois are ineffective in inucing remission, the next step woul be to aminister iniximab (E), a monoclonal antiboy that targets tumor necrosis factor-alpha. Care must be use in aministering iniximab. Because it targets tumor necrosis factor-alpha, a cytokine that regulates inammatory reactions, patients who receive iniximab are at increase risk of acquiring opportunistic infections such as tuberculosis an aspergillosis. It is also associate with activation of latent multiple sclerosis, emyelinating central nervous system isorers, an worsening congestive heart failure. Iniximab has also been shown to be effective in healing complex stulas associate with Crohn isease. Rarely, it has been associate with T-cell lymphoma an almost exclusively in young teenage males. Antibiotics have an ajunctive role in the treatment of infectious complications associate with Crohn isease (D). They are use to treat patients with perianal isease, enterocutaneous stulas, an active colon isease an ai in situations in which bacterial overgrowth has occurre. Once remission has been achieve after an acute are-up, it is important to maintain remission. Although corticosterois woul theoretically be useful, the sie effects preclue longterm aministration. Iniximab is use to maintain remission, as are azathioprine an 6-mercaptopurine. These latter
121
rugs act by inhibiting DNA synthesis an thus suppressing the function of T cells an natural killer cells. A secon-line agent for maintenance of remission is methotrexate.
30. D. Approximately three-fourths of patients with Crohn
isease will eventually require surgery. Inications for surgery inclue failure of meical management, intestinal obstruction, stula, abscess, bleeing, an perforation. In chilren, growth retaration is another inication. Because patients with Crohn isease will often require repeat operations, it is important to avoi unnecessary resection of small bowel because this puts the patient at risk of short bowel synrome. As such, several principles of surgical management shoul be followe. Surgical resection shoul be limite to the segment of bowel that is causing the complication. Other areas of active isease shoul be left alone, provie they are not causing obvious complications. Resection margins of cm beyon grossly visible isease are recommene (A). Resection margins have not been shown to affect recurrence. The presence of microscopic isease in the resection margin also oes not aversely affect outcome or recurrence. Thus, frozen section is unnecessary (B). When the inication for surgery is SBO, strictureplasty has been shown to be equally effective as resection for jejunal an ileal isease while sparing bowel length. Two types of strictureplasty are recommene: the Heineke-Mikulicz pyloroplasty (for strictures 50 years ol who are unergoing screening colonoscopy C. The mean withrawal time is ≥6 minutes in colonoscopies with normal results that are performe in patients with intact anatomy D. Mucosally base peunculate polyps an sessile polyps 50% rop in PTH at 10 minutes post excision (check again 0 minutes post excision if criterion not met) . Location of ectopic superior glans: tracheoesophageal groove & retroesophageal > intrathyroial, caroti sheath, in cervical thymus 3. Ectopic inferior glans (more variable): thymus > tracheoesophageal groove, intrathyroial a) Perform cervical thymectomy if unable to locate inferior glan 4. If unable to locate parathyroi, complete operation an perform postoperative localization stuies 5. Correcting hyperparathyroiism oesn’t improve osteoporosis t-score but it ecreases the rate at which it rops III. Arenal A. Incientaloma 1. Majority are benign, nonfunctioning (0), BMP to check potassium c) Pheochromocytoma: 4-hour urine metanephrines 3. If nonfunctional, rule out malignancy a) Higher malignancy risk: size >4 cm, >30 Hounsel units, heterogeneity, >50% washout b) Role of FNA for arenal mass is very limite (1) If (+) cancer history, consier biopsy to rule out arenal metastasis () Primary arenal cortical carcinoma cannot be iagnose with FNA (3) FNA of unsuspecte pheochromocytoma can trigger catecholamine surge c) Consier arenalectomy for lesions >4 to 5 cm (except myelolipoma) or lesions with suspicious raiographic features regarless of size B. Cushing isease versus primary hypercortisolism 1. Twenty-four-hour urine cortisol, ACTH level, high-ose examethasone suppression test . Patients with subclinical hypercortisolism may have normal 4-hour cortisol levels; if there is high suspicion, procee with low-ose examethasone suppression testing C. Pheochromocytoma 1. Associate synromes: von Hippel-Linau, MEN , neurobromatosis 1 (von Recklinghausen isease) . Alpha-blockae prior to initiating beta-blockers D. Hyperalosteronism (Conn synrome) 1. Hypertension, hypokalemia, alkalosis . Alosterone to renin ratio ≥30 (90% sensitive), plasma alosterone concentration >10 3. Unilateral aenoma > bilateral arenal hyperplasia; aenomas are usually small E. Arenal cortical carcinoma consierations 1. High attenuation (>0 Hounsel units), >4 cm, heterogenous appearance on CT . Often are functional: hypercortisolism, hyperalosteronism, hyperanrogenism 3. Open arenalectomy; laparoscopic arenalectomy is currently contrainicate (higher rate of local recurrence, poorer isease-free survival) 4. Mitotane for positive margins, vascular or capsular invasion, rupture/spillage, unresectable/ recurrent/metastatic isease (most commonly liver & lung) 5. Associate synrome: Li-Fraumeni (p53) F. Arenal metastases 1. Lung, kiney, melanoma, breast most common; often bilateral . Bilateral arenalectomy may benet select patients a) Evaluate for an correct arenal insufciency prior to bilateral arenalectomy to prevent perioperative arenal crisis; 30% of patients have entire glan replace with tumor
CHAPtEr 13 Endocrine Surgery
177
IV. Thyroiitis
Thyroiditis
Hyper-/hypothyroid
Cause & associations
Treatment
Hashimoto (chronic or lymphocytic) thyroiditis
Hypothyroid May have thyrotoxicosis with transient hyperthyroidism due to gland destruction
• Autoimmune, (HLA) -DR3, -DR5, & -B8 • Antithyroid peroxidase (anti-TPO) antibodies (90%); antithyroglobulin antibodies (20%–50%) • Female > male, 30–60 years old, painless goiter • Risk of primary thyroid lymphoma
• Levothyroxine • Thyroidectomy reserved for large goiters with compressive symptoms
Postpartum thyroiditis
Hyperthyroid with decreased uptake on RAI scan due to follicular cell destruction → euthyroid period → transient hypothyroidism → euthyroid (recovery)
• Autoimmune, may have higher antithyroid titers • Painless
• No treatment unless symptoms are severe • Hyperthyroid—betablockers (propranolol if breastfeeding) • Hypothyroid— levothyroxine
Acute (suppurative) thyroiditis
Euthyroid
• Bacterial (Staphylococcus aureus and Streptococcus pyogenes) > fungal > parasitic • Painful
• Ultrasound-guided ęneneedle aspiration with Gram stain and culture to determine etiology, antibiotics
Subacute granulomatous (de Quervain) thyroiditis
Hyperthyroid with decreased uptake on RAI scan due to gland destruction → euthyroid period → transient hypothyroidism → euthyroid (recovery)
• Preceded by viral URI • Female > male, painful goiter, fatigue, weight loss
• Supportive care, NSAIDs, steroids
Riedel (ębrous) thyroiditis
Hypothyroid
• Firm, nontender, extensive ębrosis that may compress adjacent structures • May have elevated serum IgG4
• Glucocorticoids, tamoxifen, levothyroxine • Surgery reserved for severe compressive symptoms
V. Bethesa Classication
Category
Cytopathology
Malignancy rate
Management
I
Nondiagnostic/inadequate
0%–20%
Repeat FNA with ultrasound guidance
II
Benign
0%–3%
Clinical and sonographic followup
III
Atypia of undetermined signięcance/ follicular lesion of undetermined signięcance (FLUS)
5%–15%
Repeat FNA, molecular testing, or lobectomy
IV
Follicular neoplasm/suspicious for follicular neoplasm, e.g., Hürthle cell (oncocytic) type
15%–30%
Molecular testing and lobectomy
V
Suspicious for malignancy
60%–75%
Near-total thyroidectomy or lobectomy
VI
Malignant
97%–99%
Near-total thyroidectomy or lobectomy
178
PArt i Patient Care
IV. Hyperparathyroiism
Type
Cause
Primary
Parathyroid adenoma (85%) or hyperplasia Lithium-induced parathyroid hyperplasia
↑/↓
Secondary
Chronic renal failure, hyperphosphatemia Vitamin D deęciency
↓/↑
Tertiary
Persistent hyperfunctioning parathyroid glands after kidney transplantation
↑/↓
↑
3.5-gland parathyroidectomy
↑↑/↓
↑
Parathyroidectomy with en bloc ipsilateral thyroidectomy + isthmusectomy & ipsilateral central neck dissection (lateral neck dissection only if evidence of gross LN involvement). Repeat neck exploration for recurrent disease.
Parathyroid carcinoma
Serum Ca/Phos
Urine Ca
Treatment
↑ (calcium creatinine clearance ratio >0.02 suggestive)
Adenoma—parathyroidectomy of aěected gland Hyperplasia—3.5-gland excision or 4-gland excision with autotransplantation Phosphate binders & dietary modięcation, calcium supplementation, vitamin D, cinacalcet 3.5-gland parathyroidectomy for calciphylaxis, bone pain, intractable pruritus, persistent anemia, or pathologic fractures
Fig 13.1 Location of Lost Parathyroi Glans.
CHAPtEr 13 Endocrine Surgery
179
Questions 1. A 55-year-ol male has recalcitrant hypertension espite taking three antihypertensive meications. He is subsequently foun to have an alosterone to renin ratio greater than 30. A CT reveals a 1-cm left arenal lesion an a -cm right arenal lesion. What is the next best step in management? A. Right arenalectomy B. Left arenalectomy C. Bilateral arenalectomy D. Selective venous sampling E. 11C-metomiate scan 2. A 5-year-ol obese male who unerwent CT scan months ago following a car collision was incientally foun to have a 3-cm left arenal mass. Which of the following is the next appropriate step? A. Laparoscopic left arenalectomy B. Surveillance CT performe at 6 months an then annually for 1 to years C. No further testing necessary D. PET CT E. Overnight low-ose (1 mg) examethasone suppression test 3. A 40-year-ol female presents with incientally iscovere mil elevation in serum calcium. She is otherwise healthy. A PTH level is elevate as well. Both ultrasoun an sestamibi scan of the neck are negative. Which of the following is true about this conition? A. It may represent tertiary hyperparathyroiism B. A 4-hour urine calcium is inicate C. She shoul procee to neck exploration D. It shoul be treate with cinacalcet E. Selective venous sampling is inicate 4. A 40-year-ol female presents with incientally iscovere hypercalcemia to 11.7 mg/L. She is otherwise healthy. A PTH level is elevate as well. Both ultrasoun an sestamibi scan of the neck are negative. Urine calcium is elevate. What is the most appropriate next step? A. Procee to neck exploration B. Treatment with cinacalcet C. Selective venous sampling D. MRI of the neck E. Observation
5. Following total thyroiectomy for follicular cancer, a 65-year-ol female presents to the emergency epartment 4 ays later complaining of circumoral numbness an tingling of her ngers. Phosphate level is normal. Which of the following is true about this conition? A. It likely represents hungry bone synrome (HBS) B. It may lea to a shortene QT on ECG C. The risk can be reuce by routine postoperative calcium an vitamin D supplementation D. Most patients are symptomatic E. It is more common with thyroiectomy for benign lesions 6. A 45-year-ol man with episoic severe hypertension is foun to have an elevate plasma metanephrine level an a serum calcium level of 11.5 mg/L. Which of the following woul be inicate in the workup? A. CT scan of the sella turcica B. Calcitonin level C. Serum gastrin level D. Serum prolactin level E. A 4-hour urine cortisol 7. Which of the following laboratory nings is characteristically associate with primary hyperparathyroiism? A. Elevate serum phosphate B. Increase serum chlorie C. Decrease urinary calcium D. Metabolic alkalosis E. Elevate calcium with a ecrease PTH 8. A 60-year-ol woman presents with fatigue, weakness an confusion. She has history of kiney stones an pathologic fractures. On physical she has a palpable neck mass. Her serum calcium level is 14.8 mg/L. The most likely iagnosis is: A. Parathyroi aenoma B. Parathyroi hyperplasia C. Parathyroi cancer D. Breast cancer with bone metastasis E. Seconary hyperparathyroiism
180
PArt i Patient Care
9. During neck exploration for primary hyperparathyroiism, only three parathyroi glans are ientie, all of which appear normal in size. Which of the following woul be appropriate? A. Perform a transcervical thymectomy B. Remove all three glans an reimplant one in the forearm C. Remove two an a half glans an then close D. Perform meian sternotomy to look for ectopic parathyroi E. Obtain biopsy samples of all three parathyroi glans an then close 10. After total thyroiectomy an postoperative ioine ablation for a 5-cm follicular thyroi cancer, the best test to monitor for recurrent isease is: A. Serum thyroi-stimulating hormone (TSH) B. Serum calcitonin C. Serum thyroglobulin D. 131I scan E. Cross-sectional CT or MRI 11. Which of the following is true regaring arenal cortical carcinoma? A. Associate evience of hormonal excess is common B. The iagnosis is generally mae by CT-guie neele biopsy C. Staging is base on tumor histology D. Because of malignant potential, arenal masses larger than 3 cm shoul be excise E. Laparoscopic arenalectomy is the preferre approach for surgical resection 12. Malignancy within a thyroglossal uct cyst is typically: A. Follicular thyroi B. Papillary thyroi C. Squamous cell D. Anaplastic thyroi E. Hürthle cells 13. After a total thyroiectomy, the right vocal cor is note to be xe in a parameian position. This most likely represents: A. Injury to the recurrent laryngeal nerve (RLN) B. Injury to the internal branch of the superior laryngeal nerve C. Injury to the external branch of the superior laryngeal nerve D. Trauma from enotracheal intubation E. Compression from hematoma
14. The most common pituitary neoplasm associate with MEN 1 secretes: A. ACTH B. Prolactin C. Growth hormone D. Thyroi-stimulating hormone E. Follicle-stimulating hormone 15. Which of the following features of Graves isease oes not improve with antithyroi therapy? A. Tremor B. Anxiety C. Graves ermopathy D. Gastrointestinal isturbance E. Exophthalmos 16. A 56-year-ol male presents with refractory hypertension espite being starte on hyrochlorothiazie an lisinopril by his primary care physician. His bloo pressure is 18/9 mmHg. Laboratory stuies are remarkable for an alosterone-renin ratio of 5 an hypokalemia. Which of the following is the next best step? A. Triamterene B. Amilorie C. Spironolactone D. Phenoxybenzamine E. Eplerenone 17. A 40-year-ol female presents with a 4 cm thyroi noule that is biopsy-proven papillary thyroi carcinoma. The patient is taken to surgery, an nal pathologic evaluation reveals a 4-cm papillary thyroi carcinoma with microscopic invasion of the perithyroial tissue, but no vascular invasion. A 1-cm lymph noe in the lateral neck is positive. Which of the following answer choices correctly pairs this patient’s American Thyroi Association (ATA) risk stratication with her pathologic nings? A. Low risk; no vascular invasion B. Intermeiate risk; tumor 4 cm C. Intermeiate risk; positive lymph noe 1 cm an the contralateral arenal glan is normal on CT. Alternatively, some surgeons recommen routine arenal venous sampling in most patients, especially those oler than 40 years ol as they are more likely to have nonfunctioning arenal aenomas. In the setting of bilateral aenomas on CT, arenal venous sampling shoul be performe (A, B). It woul be inappropriate to perform bilateral
arenalectomy without further attempts at localizing the source (C). Functional nuclear meicine stuies can also ai with lateralization but is typically performe if venous sampling is unsuccessful (E). Further, if this patient’s hyperalosteronism is ue to hyperplasia, it woul be treate meically. Reference: Yeh MW, Livhits MJ, Duh Q. The arenal glans. In: Townsen CM Jr, Beauchamp RD, Evers BM, Mattox KL. Sabiston textbook of surgery: the biological basis of modern surgical practice. 0th e. Elsevier; 016:963–995.
2. E. Incientalomas are iscovere in 1% to 4% of imaging
stuies that are evaluating an unrelate issue. The majority of incientalomas are nonfunctioning aenomas (60%). The remaining tumors in a patient that o not have a history of
186
PArt i Patient Care
malignancy inclue pheochromocytoma, cortisol-proucing aenoma, alosteronoma, arenocortical carcinoma, an myelolipoma. There shoul be a high level of suspicion for arenal metastasis in a patient with a history of malignancy an/or bilateral lesions. All arenal incientalomas shoul unergo biochemical testing to evaluate for subclinical Cushing synrome, pheochromocytoma, an alosteronoma (E). A functional incientaloma is an inication for arenalectomy. For patients with negative biochemical testing an size 5 cm. For patients with 3 to 5 cm nonfunctioning incientalomas, arenalectomy can be consiere for patients with few surgical risk factors an those with concerning raiographic features (irregular borers, central necrosis, high vascularity, an internal calcications) (A). PET CT is not part of the initial workup (D). No further testing woul be incorrect (C). Reference: Yeh MW, Livhits MJ, Duh Q. The arenal glans. In: Townsen CM Jr, Beauchamp RD, Evers BM, Mattox KL. Sabiston textbook of surgery: the biological basis of modern surgical practice. 0th e. Elsevier; 016:963–995.
3. B. Surgery is inicate in asymptomatic patients uner
the age of 50 that are suspecte to have primary hyperparathyroiism. Familial hypocalciuric hypercalcemia (FHH) causes mil increase in serum calcium an can initially be misiagnose as primary hyperparathyroiism. It is a benign conition ue to mutations in CASR, which encoes a calcium receptor. The lack of calcium signal increases the PTH level, which increases renal calcium reabsorption. Thus, part of the workup of primary hyperparathyroiism is to obtain a 4-hour urine calcium. Hypercalciuria with a high PTH level an high serum calcium level conrms primary hyperparathyroiism. A low urine calcium level suggests FHH. Once FHH is rule out, four-glan neck exploration can be performe without the nee for further imaging (C). Tertiary hyperparathyroiism typically occurs in patients with renal failure, most of whom have unergone kiney transplantation (A). Cinacalcet is inicate for patients with seconary hyperparathyroiism (D). Selective venous sampling is an invasive proceure that is inicate in patients with recurrent hyperparathyroiism, when other forms of imaging fail to ientify the abnormal glan (E).
4. A. Noninvasive localization stuies shoul always be
employe before taking a patient to surgery for primary hyperparathyroiism. Inications for parathyroiectomy in the asymptomatic patient inclue serum calcium >1 mg/L above normal, age 33). PTH levels are increase (E). Hypercalcemia typically results in hypercalciuria, with the exception being in patients with familial hypocalciuric hypercalcemia (C).
8. C. Parathyroi carcinoma is extremely rare an accounts
for less than 1% of cases of primary hyperparathyroiism. It shoul be suspecte in the setting of severe symptoms of hypercalcemia, in association with very high serum calcium (usually 14.6–15.0 mg/L) an PTH, history of kiney stones an pathologic fractures, an a palpable neck mass (A, B). Benign causes of hyperparathyroiism very rarely result in a palpable neck mass an are less likely to cause a hypercalcemic crisis. Determination of malignancy is ifcult because, similar to other enocrine malignancies, there are not any classic histologic features that reliably istinguish parathyroi malignancy from benign isease. Thus, one must look for evience of local invasion at the time of surgery as well as enlarge lymph noes. Treatment is surgical an involves en bloc resection of the parathyroi tumor with the ipsilateral thyroi glan, as well as a moie raical lymph noe issection if noal metastasis is present. Recently, cinacalcet was approve by the US Foo an Drug Aministration an is effective in controlling the hypercalcemia associate with parathyroi carcinoma. Breast cancer with bone metastasis may be associate with a paraneoplastic synrome in which a high level of PTH-relate protein is foun. This is unlikely to present with a palpable neck mass (D). Seconary hyperparathyroiism is associate with a low level of serum calcium (E). Reference: Shane E. Parathyroi carcinoma. J Clin Endocrinol Metab. 001;86():485–493. Sharretts JM, Kebebew E, Simons WF. Parathyroi cancer. Semin Oncol. 010;37(6):580–590.
9. A. On occasion, espite careful neck exploration, only
three parathyroi glans will be encountere. A careful search for the ectopic glan shoul be conucte (B, C). The inferior glans are more likely to be ectopic than the superior ones. Most inferior glans are to be foun within cm of the inferior thyroi pole. If not foun, the next step is to perform a cervical thymectomy an sen the tissue for frozen section. If still glans are not foun, the caroti sheath shoul be opene. Intraoperative ultrasonography shoul then be use to etermine whether there is an intrathyroial parathyroi glan. If ultrasonography is not available, ipsilateral thyroi lobectomy shoul be consiere. Another useful moality in
10. C. Serum thyroglobulin levels are the most useful
preictive value of serum thyroglobulin levels, measure uring the rst year of follow-up after thyroi hormone withrawal, in thyroi cancer patients. J Clin Endocrinol Metab. 003;88(3):1107–1111. Duren M, Siperstein AE, Shen W, et al. Value of stimulate serum thyroglobulin levels for etecting persistent or recurrent ifferentiate thyroi cancer in high- an low-risk patients. Surgery. 1999;16(1):13–19. Lal G, Clark OH. Thyroi, parathyroi an arenal. In: Brunicari FC, Anersen DK, Billiar TR, etal., es. Schwartz’s principles of surgery. 8th e. New York: McGraw-Hill; 005:1395–1470.
11. A. Arenocortical carcinomas are rare. They shoul
be suspecte in the presence of large tumors (>5–6 cm) or if the CT scan shows evience of necrosis, hemorrhage, or local invasion. Approximately 60% of patients with arenocortical carcinoma present with hormonal excess, incluing Cushing synrome an virilization. There are no istinctive histologic or cytologic features that istinguish arenocortical carcinoma from an aenoma (C). Thus, one must rely on evience of local invasion, lymph noe metastasis, or istant metastasis. CT-guie neele biopsy is not recommene (B). The best chance for cure is surgical resection. Open arenalectomy is the stanar of care for surgical resection for arenal cortical carcinoma as the laparoscopic approach is associate with higher local recurrence rates an poorer isease-free survival (E). Arenal masses that are hormonally active shoul be excise. In the absence of hormonal activity an in the absence of CT scan features suggestive of malignancy, resection is recommene for asymptomatic masses if they are larger than 5 to 6 cm (D). Reference: Ng L, Libertino JM. Arenocortical carcinoma: iagnosis, evaluation an treatment. J Urol. Publishe online 003:5–11.
188
PArt i Patient Care
12. B. The frequency of thyroi carcinoma among patients
with a surgically remove thyroglossal uct cyst in one large series was 0.7%. The majority is papillary cancer that is foun incientally after a Sistrunk proceure (performe for the cyst) (A, C–E). If iscovere incientally, the patient shoul subsequently unergo a total thyroiectomy because aitional cancer is usually foun within the thyroi glan as well. Reference: Heshmati HM, Fatourechi V, van Heeren JA, Hay ID, Goellner JR. Thyroglossal uct carcinoma: report of 1 cases. Mayo Clin Proc. 1997;7(4):315–319.
13. A. The RLN innervates the intrinsic muscles of the lar-
ynx, except the cricothyroi muscles, which are innervate by the external branch of the superior laryngeal nerve (C). The internal branch of the superior laryngeal nerve provies sensory input for the pharynx (B). Injury to one RLN leas to paralysis of the ipsilateral vocal cor. The cor becomes xe in either the parameian position or the abucte position. If the cor becomes xe in the parameian position, the patient will have a weak voice, whereas if it becomes xe in the abucte position, the patient will have a hoarse voice an an ineffective cough. If both RLNs are injure, an airway obstruction may evelop acutely in the patient. Trauma from enotracheal intubation or compression from hematoma oes not typically cause vocal cor paralysis (D, E).
14. B. Pituitary tumors are the thir most common tumors
in MEN 1. The majority are prolactinomas (A, C–E). They may cause bitemporal hemianopsia ue to local compression of the optic chiasm resulting in loss of peripheral vision or may lea to amenorrhea an galactorrhea in women or hypogonaism in men. Women are more likely to present early in the course of the isease as they are more likely to have hormonal symptoms. Men typically present later with mass-effect of the tumor (visual changes, heaaches, etc.).
15. E. Graves isease is the most common cause of hyper-
thyroiism in the Unite States an is ue to antiboies targeting thyrotropin receptors, which increase prouction of thyroi hormone. Patients present with anxiety, rapi or irregular heart rate, heat intolerance, weight loss, thinning hair, ecrease libio, iarrhea, thick an shiny skin (Graves ermopathy), an exophthalmos. The preferre therapy is raioactive ioine ablation, but meical therapy with propylthiouracil (PTU) or methimazole is also available. Exophthalmos evelops in about 10% of patients an is the only symptom that is resistant to antithyroi therapy an even worsens after raioactive ioine ablation (A–D). Some stuies suggest that the use of prenisone before antithyroi therapy can help improve exophthalmos. References: Bartalena L, Marcocci C, Bogazzi F, et al. Relation between therapy for hyperthyroiism an the course of Graves’ ophthalmopathy. N Engl J Med. 1998;338():73–78. Shiber S, Stiebel-Kalish H, Shimon I, Grossman A, Robenshtok E. Glucocorticoi regimens for prevention of Graves’ ophthalmopathy progression following raioioine treatment: systematic review an meta-analysis. Thyroid. 014;4(10):1515–153. Stein JD, Chilers D, Gupta S. Risk factors for eveloping thyroi-associate ophthalmopathy among iniviuals with Graves’ isease. JAMA. 015;133(3):90–96.
16. C. Primary hyperalosteronism shoul be suspecte
in patients with hypertension an hypokalemia. Primary
hyperalosteronism results from autonomous alosterone secretion, which, in turn, leas to suppression of renin secretion. The iagnosis is mae by emonstrating a combination of inappropriate potassium excretion in the urine (kaliuresis), low plasma renin, an a high alosterone-to-renin ratio (>0). While it was previously believe that an arenal aenoma (Conn synrome) was the most common cause of primary hyperalosteronism, we now know that nearly 60% of cases are ue to iiopathic bilateral arenal hyperplasia (IBAH). It is important to clearly establish the etiology because the management is ifferent. An arenal aenoma shoul be remove with a unilateral arenalectomy but IBAH is manage with meical therapy alone using a mineralocorticoi replacement such as spironolactone or eplerenone. Amilorie an triamterene are also potassium-sparing iuretics but are less optimal (A, B). A ouble-blin ranomize controlle stuy emonstrate the superiority of spironolactone in controlling hypertension compare with eplerenone (E). Bilateral arenalectomy is consiere in cases of severe refractory hypertension. However, this has a high risk of complications an will subject the patient to lifelong epenence of mineralocorticois (urocortisone) an sterois. Phenoxybenzamine is an alpha-1 receptor antagonist use in the preoperative management of pheochromocytoma (D). References: Kaplan NM. The current epiemic of primary alosteronism: causes an consequences. J Hypertens. 004;(5):863–869. Stowasser M. Upate in primary alosteronism. J Clin Endocrinol Metab. 009;94(10):363–3630. Parthasarathy HK, Ménar J, White WB, et al. A ouble-blin, ranomize stuy comparing the antihypertensive effect of eplerenone an spironolactone in patients with hypertension an evience of primary alosteronism. J Hypertens. 011;9(5):980–990.
17. C. The AJCC/TNM staging system oes not aequately
preict the risk of recurrence in ifferentiate thyroi cancer. Thus, the ATA evelope a 3-tiere clinic-pathologic risk stratication for recurrence in 009 with moications in 015. For papillary thyroi carcinoma, low-risk patients inclue those having intrathyroial tumors without extrathyroial extension, vascular invasion, metastases, aggressive histology, an clinical N0 or ≤5 N1 micrometastases (5 pathologic N1 noes with all involve noes 3 cm in largest imension. This patient has intermeiate isease base on factors: microinvasion into perithyroial tissue an a metastatic lymph noe 90% of cases). In the complete form, the eciency leas to a ecrease in both cortisol an alosterone. This leas to ambiguous genitalia in females (ue to anrogen excess), salt wasting with hypernatremia, an hypokalemia. The remaining answer choices can also cause congenital arenal hyperplasia but are less commonly foun (A, B, D, E).
54. A. A patient with a history of primary hyperparathy-
roiism, newly enlarging thyroi noule, an elevate calcitonin level likely has multiple enocrine neoplasm-A. These patients are at risk for eveloping meullary thyroi carcinoma (MTC). The characteristics of MTC that affect surgical approach inclue the following: (1) MTC is more aggressive than other thyroi cancers with higher recurrence an mortality rates. () MTC oes not take up raioactive ioine, an raiation therapy an chemotherapy are ineffective (B, E). (3) MTC is multicentric in 90% of MEN patients. (4) In patients with palpable isease, more than 70% have noal metastases (D). (5) The ability to measure postoperative stimulate calcitonin levels has allowe assessment of the aequacy of surgical extraction. The two main factors affecting survival are stage an age at iagnosis (D). A key factor in survival is early etection via calcitonin screening in at-risk patients. In one large stuy, biochemical cure preicte a survival rate of 97.7% at 10 years. Management of MTC inclues total thyroiectomy with routine central noe issection (A). It shoul be note that MEN A is rare, an in fact, most MTCs
Clark OH. Meullary thyroi carcinoma: clinical characteristics, treatment, prognostic factors, an a comparison of staging systems. Cancer. 000;88(5):1139–1148. Moigliani E, Cohen R, Campos JM, et al. Prognostic factors for survival an for biochemical cure in meullary thyroi carcinoma: results in 899 patients. The GETC Stuy Group. Groupe ’étue es tumeurs à calcitonine. Clin Endocrinol (Oxf). 1998;48(3):65–73.
with signicant suppression in hyperthyroi states. In most states of hyperthyroiism, free T4, total T4, an total T3 are elevate (A–C). Thyroi scan is not use in the initial workup for hyperthyroiism (E).
56. E. The thyroi glan is supplie by paire superior
thyroi arteries from the external caroti arteries an the inferior thyroi arteries from the thyrocervical trunk. The superior thyroi artery is the rst branch of the external caroti artery (B). During thyroiectomy, care must be taken when ligating the superior thyroi arteries to avoi injury to the external branch of the superior laryngeal nerve (D). To avoi injury, ligating the artery an vein separately an close to the thyroi glan is recommene. In approximately 3% of iniviuals, a thyroiea ima artery also provies bloo to the thyroi glan an arises either from the aorta or the innominate artery. When ligating the inferior thyroi arteries, care must be taken to avoi injury to the RLNs (C). The inferior thyroi arteries usually supply the parathyroi glans (A). Ligation of the main trunk of the inferior thyroi arteries uring total thyroiectomy can lea to parathyroi glan ischemia. There are three main pairs of veins raining the thyroi glan: the superior, mile, an inferior thyroi veins. The mile veins are the least constant. The superior an mile veins rain into the internal jugular veins, whereas the inferior veins rain into the brachiocephalic veins.
57. C. The superior laryngeal nerve an RLN arise from
the vagus nerve. The superior laryngeal nerve ivies into two branches an is both motor an sensory to the larynx (D). The internal branch is sensory to the supraglottic larynx, an, although rare, injury uring thyroi surgery woul lea to aspiration (A). The external branch innervates the cricothyroi muscle. Injury to the external superior laryngeal nerve causes an inability to tense the ipsilateral vocal cor. This oes not cause hoarseness, but rather results in voice fatigue, an in singers creates ifculty in hitting high notes. It has been referre to as the nerve of Amelita Galli-Curci or “high note” nerve after the opera singer who unerwent thyroi goiter surgery in the 1930s an lost her ability to sing afterwar. The left RLN loops aroun the aorta at the ligamentum arteriosum. The right RLN loops aroun the right subclavian artery. The RLN innervates the intrinsic muscles of the larynx with the exception of the cricothyroi muscle, which is innervate by the external laryngeal nerve (E).
196
PArt i Patient Care
Injury to one RLN leas to paralysis of the ipsilateral vocal cor, which becomes xe in the parameian or abucte position. Bilateral RLN injury may lea to airway obstruction an complete loss of the voice (B).
58. C. A nonrecurrent laryngeal nerve is rare an occurs
much more commonly on the right (A, B). It branches off the vagus nerve in the neck an heas irectly to the larynx, as oppose to arising from the vagus after passing the subclavian artery (D). The anomalous location, as oppose to its normal position in the tracheoesophageal groove, makes it more prone to injury (E). On the right, a patient can have both a nonrecurrent nerve an a recurrent nerve. Nonrecurrent left laryngeal nerves have been reporte but are extremely rare. The recurrent laryngeal nerve is most vulnerable to injury uring the last to 3 cm of its course but also can be amage if the surgeon is not alert to the possibility of nerve branches an nonrecurrent nerves, particularly on the right sie.
59. E. PTH increases the bone resorption by stimulating
osteoclasts an inhibiting osteoblasts, leaing to the release of calcium an phosphate into the circulation. At the kiney, PTH limits calcium excretion at the istal convolute tubule via an active transport mechanism an inhibits phosphate an bicarbonate reabsorption, the latter leaing to a mil metabolic aciosis (B, C). PTH also enhances hyroxylation of 5-hyroxyvitamin D to 1,5-hyroxyvitamin D in the kiney, which in turn irectly increases intestinal calcium absorption (not a irect effect of PTH) (D). Cholecalciferol is hyroxylate to 5-hyroxyvitamin D in the liver. This is not regulate by PTH (A).
60. A. Lateral aberrant thyroi is a term use to enote
what appears to be ectopic thyroi tissue foun within the neck. In most instances, it actually represents metastatic thyroi cancer within a lymph noe, most often of the papillary type. It is not typically associate with the remaining answer choices (B–E). Reference: Jong D, Demeter S, Jarosz J. Primary papillary thyroi carcinoma presenting as cervical lymphaenopathy: the operative approach to the “lateral aberrant thyroi. Am Surg. 1993;59:17–176.
61. C. The accepte management of low-risk papillary thy-
roi cancer is either right hemithyroiectomy or total thyroiectomy with or without postoperative 131I. In patients with papillary carcinoma with a history of raiation exposure, there is a higher rate of multicentricity. As such, total thyroiectomy is the recommene proceure (A, B). Postoperative raioactive ioine following total thyroiectomy is inicate for tumors larger than 4 cm, gross extrathyroial extension of the tumor regarless of size, lymph noe metastases, an for high-risk features incluing tall-cell or columnar-cell variant (E). An ae avantage of postoperative raiation is that it allows for the continue monitoring for recurrence with thyroglobulin. Prophylactic central neck noe issection is gaining popularity as well. Moie raical neck issection woul not be inicate unless there were obvious lateral neck noes (D). References: Guerrero MA, Clark OH. Controversies in the management of papillary thyroi cancer revisite. ISRN Oncol. 011;011:30318. Hay ID, Thompson GB, Grant CS, et al. Papillary thyroi carcinoma manage at the Mayo Clinic uring six ecaes (1940–1999): temporal trens in initial therapy an long-term outcome in 444 consecutively treate patients. World J Surg. 00;6(8):879–885.
Skin and Soft Tissue ERIC O. YEATES, AREG GRIGORIAN, AND CHRISTIAN DE VIRGILIO
14
ABSITE 99th Percentile High-Yields I. Most Common Skin Cancers A. Basal cell carcinoma: most common skin cancer an overall cancer 1. Majority foun on hea an neck, more commonly on upper lip . Typically appears as a shiny, pearly skin noule with rolle borers 3. Treatment 1. Excision with 4 to 5 mm margins for low risk, an 1 to cm for high risk . Low risk is trunk an extremity lesions 5 mm miline shift C. Massive subarachnoi hemorrhage: CTA hea to evaluate for rupture aneurysm or arteriovenous malformation D Start VTE chemoprophylaxis 48 hours from most recent stable CT; low-molecular weight heparin preferre in TBI II. Spinal Cor Injury (SCI) A. Unstable spine injury: isruption of /3 of the longituinal spinal columns (requires surgery) B. High SCI (above T6), concern for neurogenic shock acutely (hypotension, vasoilation with warm skin, braycaria, or inappropriately normal heart rate for trauma setting); long term concern (months later) for autonomic ysreexia with braycaria, iaphoresis, an uncontrolle hypertension
Injury
Location
Mechanism
Management
Hangman’s fracture (bilateral pedicles)
Cervical
Hyperextension of the neck, caused by hanging, diving
Traction and external immobilization (Halo) vs spinal fusion
Dens fracture
C2 (odontoid process)
Hyperextension of the neck, falls in the elderly
Type I is above the base and considered a stable fracture. Type II (which extends to the base of dens) is unstable and need surgical ęxation. Type III fractures extend into the C2 vertebral body—these tend to have a beĴer healing rate than type II and rarely require surgery.
Chance fracture (horizontal disruption of all columns)
Thoracolumbar
Flexion-distraction injury from rapid deceleration during blunt trauma
Orthotic brace for low-grade injury, surgery if neurologic deęcits or ligamentous injury; also higher risk of hollow viscous injury
219
220
PArt i Patient Care
III. Neck Trauma A. Screening for blunt cerebrovascular injury (caroti or vertebral artery): use expane Denver criteria
Risk factors for BCVI
Signs/symptoms of BCVI
LeFort II/III or mandible fracture
Pulsatile bleeding from neck or nasal/oral cavity
Complex skull fracture or basilar skull fracture
Carotid bruit
Traumatic brain injury (GCS cm segmental loss: esophagectomy, wie rainage, an reconstruction if stable or iversion with esophagostomy (spit stula) if unstable 4. Surgical access: if cervical esophagus use left neck incision, if upper /3 of esophagus use right posterolateral thoracotomy, if istal 1/3 esophagus use left posterolateral thoracotomy C. Tracheal injury: ebrie, repair with primary closure in one layer (two layers coul lea to stenosis) using absorbable suture an buttress with strap muscles D. Surgical airway: cricothyroiotomy preferre over tracheostomy; chilren (age 50% circumference: segmental resection with primary anastomosis for all segments except D C. Colorectal injuries 1. Colon an intraperitoneal rectal: a) Nonestructive (50% circumference): segmental resection with 1° anastomosis if stable, segmental resection left in iscontinuity with planne n look operation if unstable . Extraperitoneal rectum: proximal iversion alone (no presacral rainage or washout) VI. Retroperitoneal hematomas A. Zone I: explore all hematomas B. Zone II: selective exploration for expaning hematoma (blunt or penetrating) C. Zone III: explore penetrating, o not explore blunt VII. Orthopeic an Neurovascular Injury Patterns
Associated neurovascular injury
Complication/deęcit
Anterior (more common) shoulder dislocation
Axillary nerve injury
Weak shoulder abduction
Posterior shoulder dislocation (e.g., seizures)
Axillary artery injury
Humeral shaft fracture
Radial nerve palsy
Wrist-drop
Supracondylar humerus fracture
Brachial artery injury
Forearm compartment syndrome, Volkmann ischemic contracture
Colles fracture (distal radius)
Median nerve compression
Pain, paresthesias in digits 1–3 ½
Fracture/Injury
Scaphoid fracture Posterior (more common) hip dislocation (adducted and internally rotated)
Snuġox tenderness, avascular necrosis; often normal initial XR Sciatic nerve injury (peroneal branch)
Posterior knee dislocation
Popliteal artery injury
Fibula head fracture (or prolonged lithotomy)
Peroneal nerve injury
Foot drop
Exposure/maneuver
Location
Right posterolateral thoracotomy
Mid esophagus
Left posterolateral thoracotomy
Distal esophagus, descending aorta (distal to left subclavian takeoě)
Left anterolateral thoracotomy
Left distal subclavian artery
Median sternotomy
Ascending and arch of aorta, innominate artery, bilateral common carotid artery, superior vena cava, proximal right and left subclavian artery
Left infraclavicular incision
Left mid-subclavian artery
Kocher maneuver
Head of pancreas, SMV, SMA
Left medial visceral rotation (MaĴox maneuver)
Aorta, celiac trunk, SMA, left renal artery, common iliac arteries
Right medial visceral rotation (CaĴell-Braasch maneuver)
Inferior vena cava, right renal vessels, common iliac veins
Pringle maneuver
Control intrahepatic liver hemorrhage, max clamping 30–60 min
222
PArt i Patient Care
VIII. Acute Compartment Synrome (ACS) A. Risk factors: open fractures > close fractures, crush injuries, young, male, long bone fracture (e.g., tibia, raius), high voltage burns, >6 hours ischemia with reperfusion, combine arterial/venous injury B. Diagnosis: high clinical suspicion, compartment pressure >30 mmHg, elta P (compartment pressure/ iastolic pressure) units of packe re bloo cells [PRBCs]), presence of a pseuoaneurysm or suspecte arteriovenous stula provie that they are hemoynamically stable. Serial abominal exams an trening the hematocrit woul be inappropriate in the presence of active extravasation of contrast (D). If angioembolization is not available, laparoscopic an open splenectomy are both reasonable options in hemoynamically stable patients that meet the above inications for surgery (A, E), whereas in the unstable patient or with iffuse peritonitis, open splenectomy is recommene. Once in the operating room, attempts at splenic preservation via splenorrhaphy are reasonable in hemoynamically stable patients (B). Reference: Stassen NA, Bhullar I, Cheng JD, et al. Selective nonoperative management of blunt splenic injury: an Eastern Association for the Surgery of Trauma practice management guieline. J Trauma Acute Care Surg. 01;73(5 Suppl 4):S94–S300.
30. C. The physical exam nings are concerning for the
presence of a urethral injury. The most common location is at the prostatic urethra. Genitourinary injuries may occur in up to 15% of patients with pelvic fractures. Hea injury is the most common associate injury seen in patients with pelvic fractures. Clinical suspicion of a urethral injury warrants the performance of a RUG to ientify the presence an location of a urethral injury. Blin insertion of a Foley catheter is contrainicate in this patient (A). CT abomen with IV contrast is helpful for ientifying injuries to the kineys an elaye acquisition images may also ai in the ientication of ureteral or blaer injuries (B). A CT cystogram accurately iagnoses both extraperitoneal an intraperitoneal blaer injuries (D). Intravenous pyelogram is use to ientify renal injuries an is rarely performe (E). Management of urethral injuries epens on the location an severity of injury, as well as presence of associate injuries, an surgical expertise. Reference: Johnsen NV, Dmochowski RR, Mock S, Reynols WS, Milam DF, Kaufman MR. Primary enoscopic realignment of urethral isruption injuries—A ouble-ege swor? J Urol. 015;194(4):10–106.
31. C. Although A, B, C (Airway, with cervical spine pre-
cautions; Breathing; Circulation with hemorrhage control) has always been the recommene sequence in trauma patients, recent recommenations are shifting to C, A, B in those with penetrating injuries who are severely hypotensive, as the combination of rapi-sequence intubation an positive pressure ventilation can worsen hypotension an lea to cariac arrest (A). Thus, bloo proucts woul be the preferre rst step, followe by immeiate transport to the
CHAPtEr 16 Trauma operating room. Some meical centers are now proviing initial resuscitation with whole bloo for the trauma patient in shock. Given the location of the injury (zone I of the neck), one shoul have a high suspicion for a right subclavian or innominate artery injury. Once in the operating room (if possible), the patient is preppe an rape prior to intubation. REBOA (E) is utilize for control of vascular injuries below the iaphragm. Proximal control of such an injury on the right via an open approach is best achieve by a meian sternotomy. If the same injury were present on the left, proximal control of the left subclavian artery is best achieve via a left anterolateral thoracotomy. Enovascular balloon occlusion is another option. If bloo is exsanguinating through the bullet hole, manual compression in this area is ineffective. Temporary tamponae can be achieve via insertion an ination of a Foley balloon irectly into the woun, permitting rapi transportation to the operating room. Thoracostomy is inicate for pneumothorax or hemothorax seen on raiograph imaging or after primary survey suggestive of these conitions (B). The above patient has not ha a cariopulmonary arrest, nor oes he meet any inication for ED thoracotomy (D). References: American College of Surgeons Committee on Trauma. Advanced trauma life support program for doctors. 9th e. American College of Surgeons; 01. Demetriaes D, Chahwan S, Gomez H, et al. Penetrating injuries to the subclavian an axillary vessels. J Am Coll Surg. 1999;188(3):90–95.
32. D. With penetrating neck trauma, there is concern that
bleeing may rapily compress the trachea. As such, the rst step in the management algorithm is to establish an airway, particularly in the presence of an expaning hematoma or epresse level of consciousness. If the patient has a “har sign” of a vascular injury, such as a rapily expaning or pulsatile hematoma, visible exsanguination, palpable thrill or auible bruit, or ense neurologic ecit (such as this patient with GCS 8), the patient shoul then be transporte irectly to the OR. If the patient is hemoynamic unstable, without har signs, the presumption shoul be that the patient exsanguinate in the el. Thus, shock is another inication for immeiate surgical exploration (this patient has a low BP as well). Conversely, in the absence of har signs, the next step woul be to obtain CT arteriography of the neck vessels. This historically has been achieve with formal arteriography, because of the ease an rapiity of its use (B, C). In aition, an assessment for injuries to the aeroigestive tract (triple enoscopy an/or esophagography) an cervical spine nees to be performe (E). As a general guie, repairing a caroti artery injury in a patient with a neurologic ecit is recommene as it may result in improve neurologic function, whereas caroti ligation typically oes not. Repair can be achieve by primary suturing, resection with a primary reanastomosis, or interposition graft placement (saphenous vein or polytetrauoroethylene) (A).
33. D. Resuscitative thoracotomy is a potentially lifesaving
proceure. Inications an guielines continue to evolve. There are many articles in the literature on the topic, with variable nings an recommenations. However, several overarching themes consistently permeate these stuies. Outcomes are better for those with SOL than those without, penetrating trauma than blunt, chest trauma than abominal, isolate injury than multiple injuries, without hea injury than with, short uration
239
of CPR than long uration, an stab wouns than GSW (A–C, E). Thus, the best scenario for resuscitative thoracotomy woul be an isolate stab woun to the chest, with SOL (survival from poole ata is 1%). Such a patient is much more likely to have arreste ue to cariac tamponae an therefore has not suffere exsanguinating hemorrhage. Conversely, at the other extreme, for blunt trauma without SOL, survival was only 0.7%. The following are consiere SOL: agonal respirations, cariac electrical activity, palpable pulse, measurable bloo pressure, spontaneous movement, or pupillary reactivity. Thus, the benet of resuscitative thoracotomy for SOL an penetrating chest trauma is clear. Less compelling but still potentially benecial inications woul be penetrating chest trauma without SOL, penetrating extrathoracic injury with or without SOL, an blunt trauma with SOL. There is no benet for blunt trauma with no SOL. For those that survive, a surprising majority survive with favorable neurologic outcomes. References: Burlew CC, Moore EE, Moore FA, et al. Western Trauma Association critical ecisions in trauma: resuscitative thoracotomy: resuscitative thoracotomy. J Trauma Acute Care Surg. 01;73(6):1359–1363. Seamon MJ, Haut ER, Van Arenonk K, et al. An evience-base approach to patient selection for emergency epartment thoracotomy: a practice management guieline from the Eastern Association for the Surgery of Trauma. J Trauma Acute Care Surg. 015;79(1):159–173.
34. E. The preferre access for young chilren an infants
following trauma is via the peripheral percutaneous route (antecubital fossa or saphenous vein at the ankle). After two unsuccessful attempts, consieration shoul be given to IO infusion via a bone marrow neele (18 gauge in infants, 15 gauge in young chilren). IO cannulation of the proximal tibia provies goo short-term access for resuscitation because it targets the noncollapsible veins of the meullary sinus. The optimal site of insertion is the anteromeial tibia to 3 cm below the tibial tuberosity, ensuring to angle away from the growth plates. This can be performe using a bone marrow neele or an IO vascular access system such as the EZ-IO®. Once the patient has been resuscitate, follow-up attempts at peripheral access shoul be mae. If a patient has obvious eformities in the tibiae (as in this patient), the next location for IO cannulation woul be the istal femur just above the femoral conyles (D). In aults, there has been a shift in recent years, an sternal IO access is now consiere the preferre initial site for cannulation (thinner cortex an abunant re bone marrow) followe by the tibia. The proximal humerus is an aitional option in aults. It is also important to note that serum electrolytes, bloo gases, an type an cross can all be performe using bloo from interosseous access. A istal saphenous vein cutown is another option in chilren ages 1 to 6 years, but in a chil younger than 1 year of age, it woul be challenging an not appropriate in the setting of obvious leg eformity (B). In hypovolemic peiatric patients younger than 6 years of age, percutaneous femoral vein cannulation is another alternative but is associate with an increase risk of venous thrombosis an woul be much more challenging in a chil younger than 1 year (C). Subclavian an internal jugular central lines woul be too ifcult to perform in the trauma setting in such a small chil an woul be associate with an increase risk of iatrogenic injury (A). The interosseous cannula shoul
240
PArt i Patient Care
be remove expeitiously (within 4 hours) because of the potential risk of infectious complications incluing osteomyelitis. Extremity compartment synrome is another potential complication of IO infusion. References: Cullen PM. Intraosseous cannulation in chilren. Anaesth Intensive Care Me, 01; 13:8–30. Pasley J, Miller CHT, DuBose JJ, et al. Intraosseous infusion rates uner high pressure: a caaveric comparison of anatomic sites. J Trauma Acute Care Surg. 015;78():95–99.
35. D. Both bloo volume an re cell volume increase in
the pregnant patient, but bloo volume increases more than re cell volume. Bloo volume increases by approximately 50% as term approaches, whereas re cell volume increases by approximately 30%, resulting in a functional hemoilution an resultant physiologic anemia of pregnancy (A). Thus, pregnant patients are less likely to manifest signs of bloo loss such as tachycaria an hypotension, an if such signs are present, they are inicative of an even more severe bloo loss than in the nonpregnant patient (on the orer of 1500–000 mL of bloo loss). The pregnant patient has an increase tial volume an minute ventilation, esigne to increase oxygen release to the fetus. This results in a mil respiratory alkalosis, with a PCO in the 7 to 3 range (B). Oxygen consumption is increase, an functional resiual capacity is ecrease. In aition, the ,3-iphosphoglycerate level is increase to enhance the release of oxygen to the fetus. However, these physiologic changes result in less pulmonary reserve in an acutely ill pregnant patient. The use of raiographs is thought to be safe for the fetus after the 0th week of gestation (C). The glomerular ltration rate increases, resulting in a ecrease in serum creatinine (E). Other important aspects to be aware of are that the gravi uterus can compress the IVC, resulting in ecrease venous return. Therefore, the pregnant patient shoul be place in the left lateral position at approximately 15 egrees. Pregnant patients are more prone to aspiration, so early NG tube ecompression is important. Finally, the progressive stretching of the peritoneum leas to esensitization so that a pregnant patient is less likely to emonstrate peritoneal signs. Reference: Shah AJ, Kilcline BA. Trauma in pregnancy. Emerg Med Clin North Am. 003;1(3):615–69.
36. D. In the past, the injury escribe woul have been
ealt with by performing a formal lobectomy (A). However, pulmonary tractotomy is now use as a less aggressive alternative. The technique involves using a linear stapling evice to insert irectly into the injure bullet tract. Two hemostatic staple lines are create, an the lung is ivie in between. This allows irect access to the bleeing vessels within the parenchyma as well as any leaking bronchi. Bleeing vessels can then be oversewn with a polypropylene monolament (C). Lobectomy is a better choice for a completely evascularize or estroye lobe. A pneumonectomy is rarely inicate an, in the trauma setting, is associate with an 80% mortality rate (B). Similarly, ligation of a lobar pulmonary artery has a high rate of morbiity (E). References: Cothren C, Moore EE, Bif WL, et al. Lungsparing techniques are associate with improve outcome compare with anatomic resection for severe lung injuries. J Trauma. 00;53(3):483–487.
Kim DY, Coimbra R. Thoracic amage control. In: Di Saverio S, Tugnoli G, Catena F, Ansaloni L, Naioo N, es. Trauma surgery: volume 2: thoracic and abdominal Trauma. Springer Milan; 014:35–46.
37. C. Flail chest occurs when two or more ribs are fracture in at least two locations. Paraoxical movement of this free-oating segment of chest wall is typically not sufcient alone to compromise ventilation (B). Rather, pain an splinting, in conjunction with unerlying pulmonary contusions, may result in hypoxemia an hypercarbia ue to shunting an ineffective ventilation, respectively. Most patients can be manage without intubation (E). Respiratory failure often oes not occur immeiately, an frequent reevaluation is warrante. The initial chest raiograph usually unerestimates the egree of pulmonary contusion, an the lesion tens to evolve with time an with ui resuscitation (A). Intravenous ui aministration shoul be limite as overzealous resuscitation may result in blossoming of pulmonary contusions (D). The most important aspect of treatment of ail chest is pain control. Stanar approaches inclue the use of patient-controlle analgesia an oral pain meications an the placement of continuous epiural catheters. Although the treatment of ail chest has historically been nonoperative, recent literature inicates that internal xation of the chest wall in select patients without pulmonary contusion ecreases intubation time, ecreases mortality, shortens uration of mechanical ventilation as well as hospital stay, ecreases complications, an improves cosmetic an functional results. In the presence of a pulmonary contusion, however, internal xation may not be as benecial. Eastern Association for the Surgery of Trauma (EAST) guielines recommen ORIF in aults with ail chest after blunt trauma. Situations in which internal xation shoul be consiere inclue ail chest in patients who are alreay unergoing thoracotomy for an intrathoracic injury, ail chest without pulmonary contusion, noticeable paraoxical movement of a chest wall segment while a patient is being weane from the respirator, an severe eformity of the chest wall. References: Kasotakis G, Hasenboehler EA, Streib EW, et al. Operative xation of rib fractures after blunt trauma: a practice management guieline from the Eastern Association for the Surgery of Trauma. J Trauma Acute Care Surg. 017;8(3):618–66. Leinicke JA, Elmore L, Freeman BD, Colitz GA. Operative management of rib fractures in the setting of ail chest: a systematic review an meta-analysis. Ann Surg. 013;58(6):914–91. Voggenreiter G, Neueck F, Aufmkolk M, Obertacke U, Schmit-Neuerburg KP. Operative chest wall stabilization in ail chest–outcomes of patients with or without pulmonary contusion. J Am Coll Surg. 1998;187():130–138.
38. A. BCI shoul be suspecte in anyone with severe blunt
chest trauma. Attempts to ientify a BCI an stratify severity on the basis of CK-MB, nuclear scans, an echocariography have not been successful because these moalities lack sensitivity. ECG is the most commonly recommene tool for the initial iagnosis of BCI. The presence of a sternal fracture is not a marker for BCI (D). A normal screening ECG has a negative preictive value of 95% (E). Aition of a normal cariac troponin increases the negative preictive value to 100%. If a stable patient has an abnormal cariac troponin level or ECG, he/she shoul be amitte for observation to a monitore be. However, troponin level oes not correlate with risk of cariac complications in BCI. If the patient is
CHAPtEr 16 Trauma unstable, an emergent echocariogram shoul be performe. If a tamponae is seen, emergent sternotomy shoul be performe for suspecte cariac rupture. Very rarely, BCI can lea to coronary artery thrombosis, valvular isruption, or septal isruption (C). In an unstable patient with BCI without an anatomic abnormality on echocariography, invasive bloo pressure monitoring with pressor support shoul be institute. Most patients with a iagnosis of myocarial contusion have a benign course, with very few eveloping arrhythmias or heart failure (B). References: Clancy K, Velopulos C, Bilaniuk JW, et al. Screening for blunt cariac injury: an Eastern Association for the Surgery of Trauma practice management guieline. J Trauma Acute Care Surg. 01;73(5 Suppl 4):S301–S306. Velmahos GC, Karaiskakis M, Salim A, et al. Normal electrocariography an serum troponin I levels preclue the presence of clinically signicant blunt cariac injury. J Trauma. 003;54(1):45–50.
39. E. The Cattell maneuver involves a right meial visceral
rotation of the cecum an ascening colon. It is achieve by incising the peritoneal reection at the white line of Tolt. It is useful for exposing right retroperitoneal structures, such as the IVC an the right ureter (C). Further cephala, mobilization an meial rotation of the uoenum (Kocher maneuver) aitionally assists in exposing the suprarenal IVC below the liver. The Kocher maneuver is not useful for exposing the celiac axis (D). This is best one by combining a Mattox maneuver with a ivision of the left crus of the iaphragm an iviing the celiac plexus (A). The Mattox maneuver consists of a left meial rotation of the escening colon (again at the line of Tolt), spleen, an/or kiney towar the miline. Exposure of injuries to the istal IVC an iliac vein bifurcations can be exceeingly ifcult. On occasion, ivision of the right common iliac artery is neee to expose an repair an injury of this area. A primary repair of the iliac artery can then be performe. On rare occasions, with massive bleeing, the junction of the superior mesenteric vein (not artery), splenic, an portal veins may nee to be expose by ivision of the neck of the pancreas (B). References: Asensio JA, Chahwan S, Hanpeter D, et al. Operative management an outcome of 30 abominal vascular injuries. Am J Surg. 000;180(6):58–533. Hoyt DB, Coimbra R, Potenza BM, Rappol JF. Anatomic exposures for vascular injuries. Surg Clin North Am. 001;81(6):199–1330.
40. B. Most veins can be safely ligate in the setting of trau-
matic injury. However, certain veins are less likely to tolerate ligation well. These inclue the superior vena cava (because it may result in an acute superior vena cava synrome), the renal veins close to the renal parenchyma (because there is then inaequate outow for the kiney), the IVC above the renal veins (because it will impair outow to both kineys), or just at the iaphragm (because this will cause an acute Bu-Chiari synrome), an the portal vein (because it supplies 75% of the bloo to the liver) (A, E). An exception to the aforementione is ligation of the left renal vein close to the IVC is well tolerate because rainage can occur via the arenal, gonaal, an iliolumbar veins. This is sometimes performe uring open abominal aortic aneurysm repair. The portal vein has been ligate successfully, provie aequate ui is aministere to compensate for the ramatic but transient eema that occurs in the bowel, but ligation seems to be associate with a higher mortality rate than repair.
241
Ligation of the IVC below the renal veins is better tolerate than the suprarenal IVC; however, marke leg swelling may evelop an may require fasciotomies. Ligation of the superior mesenteric vein is also fairly well tolerate an better tolerate than portal vein ligation, although again it is preferable to repair the superior mesenteric vein if the patient is stable an it is technically feasible because there is similarly marke bowel eema an risk of bowel infarction as with portal vein repair. Arteries for which repair shoul always be attempte inclue the innominate, brachial, superior mesenteric, proper hepatic, iliac, femoral, an popliteal arteries an the aorta (C, D). If enitive repair is preclue ue to hemoynamic instability or if a amage control approach is eeme appropriate, perfusion or ow may be maintaine via a temporary intravascular shunt. In the forearm, either the raial or ulnar artery can be ligate, provie the other vessel is palpable. Similarly, in the lower leg, at least one of the two palpable vessels (anterior or posterior tibial artery) shoul be salvage. Because of the excellent collateralization aroun the shouler, ligation of the subclavian artery is well tolerate. In fact, the artery is often occlue uring stent-grafting of thoracic aneurysms or aortic transection. Reference: Rich NM, Mattox KL, Hirshberg A. Vascular trauma. n e. Elsevier Science; 004.
41. A. Extremity compartment synrome can occur any-
where in the extremities, incluing the buttocks, shoulers, an hans (E). The mechanisms of compartment synrome are numerous an can be ivie into extrinsic an intrinsic causes. Extrinsic causes inclue constriction by a cast, tight circumferential ressings, or eschar from a burn. Intrinsic causes are ivie into bleeing, eema, an exogenous ui. Bleeing is usually ue to trauma but can also be seen after relatively minor injuries in patients with an unerlying coagulopathy or those receiving anticoagulants. Eema of the compartment is the largest an broaest category. It is most often seen after reperfusion of an ischemic limb, from either an arterial embolus or thrombosis or trauma. Ischemia/reperfusion is also seen in a person with a rug overose or an alcoholic who falls asleep on the limb, in patients with profoun shock in whom iffuse muscle ischemia with subsequent reperfusion evelops, an after massive iliofemoral eep venous thrombosis. Finally, inavertent infusion of IV ui into the subcutaneous tissue can lea to compartment synrome. Diagnosis of compartment synrome begins by having a high clinical inex of suspicion an knowing the clinical scenarios in which it occurs. The most common features are severe pain in the limb typically out of proportion to the physical exam, pain on passive motion of the limb, an tense eema with tenerness on palpation of the compartment. Distal arterial pulses typically remain palpable with compartment synrome. The anterior compartment of the leg is usually the rst compartment to be involve in the lower extremity (C). The eep peroneal nerve runs within it so numbness in the rst web space of the toe is one of the early nings (D). Once the iagnosis is suspecte, conrmation is sought by oing irect pressure measurements of the iniviual compartments. If the pressures are increase more than 30 mmHg in any of the compartments, then strong consieration shoul be given to performing a four-compartment fasciotomy. The use of an absolute value has been questione because the perfusion pressure necessary for oxygenation
242
PArt i Patient Care
is partly epenent on the patients’ bloo pressure an, therefore, coul lea to unnecessary fasciotomies (B). The use of ifferential pressure (Δp = iastolic bloo pressure— intracompartmental pressure), with a propose threshol of 30 mmHg, has been propose to be of greater iagnostic value. It is also important to remember that there is no absolute pressure level that rules compartment synrome in or out. The measurements shoul be use in conjunction with the patient’s clinical examination. The eep posterior compartment is the one that is most commonly inaequately ecompresse. Because this compartment contains the tibial nerve, missing this compartment can have evastating consequences. The soleus muscle must be etache from the tibia to ecompress the eep posterior compartment. Buttock compartment synrome has been escribe in patients with obesity after prolonge anesthesia as well. Reference: von Keuell AG, Weaver MJ, Appleton PT, et al. Diagnosis an treatment of acute extremity compartment synrome. Lancet. 015;386(10000):199–1310.
42. C. The management of liver injuries has unergone a
major evolution in the past 5 years, from routine laparotomy in the past to the current application of selective nonoperative management in hemoynamically stable patients, liberal use of angiographic embolization, an operative management with selective packing an amage control when the patient is col an coagulopathic. In a patient who has sustaine blunt trauma an is hemoynamically stable, a CT scan with IV contrast shoul be performe. If a contrast blush is seen in the liver, the patient shoul be taken to angiography for embolization, provie there are no other injuries that require operative intervention. Conversely, if the patient is hemoynamically unstable (as in this patient), the patient shoul be taken to the operating room an unergo packing of all four quarants to obtain temporary hemostasis while anesthesia attempts to “catch up” or aequately resuscitate the patient. Strong consieration shoul be given to activating the institutional massive transfusion protocol in aition to aministering tranexamic aci. Given that this patient ha continue bleeing espite application of a Pringle maneuver, he has likely sustaine an injury to the retrohepatic IVC or hepatic veins. If the bleeing is controlle with packing an, in aition, the patient is col (temperature 1.1, but ui creatinine woul be much higher than serum creatinine (B). Patients with abominal compartment synrome have abominal istention, oliguria, ecrease lung compliance, an hypotension. This patient with normal vital signs oes not have abominal compartment synrome (C). Reference: Runyon BA, Montano AA, Akriviais EA, et al. The serum-ascites albumin graient is superior to the exuate-transuate concept in the ifferential iagnosis of ascites. Ann Intern Med. 199;117(3):15–0.
Vascular—Arterial AMANDA C. PURDY AND NINA M. BOWENS
17
ABSITE 99th Percentile High-Yields I. Peripheral Arterial Disease (PAD) A. Normal ankle-brachial inex (ABI) ranges from 1 to 1.; ABI 5.5 cm in males, growth of >0.5 cm in 6 months or >1 cm in 1 year
CHAPtEr 17 Vascular—Arterial
247
E. Surgical options inclue open AAA repair an enovascular repair (EVAR) F. Compare to open AAA repair, EVAR has a lower perioperative (30-ay) mortality rate but similar longterm mortality G. Complications of AAA repair: 1. MI is the most common cause of in-hospital eath after AAA repair . Kiney injury: increase incience if intraoperative hypotension or suprarenal aortic cross-clamp 3. Ischemic colitis: risk factors (coverage of the IMA or internal iliac, rupture AAA, intraoperative hypotension; can present with abominal pain an iarrhea (sometimes blooy); iagnose with exible sigmoioscopy; treat with antibiotics, NPO, an resuscitation, if patient eteriorates or evelops peritonitis will require surgery 4. Aortoenteric stula (after open or enovascular repair): upper GI blee usually >6 months after surgery; ue to infecte graft that eroes into the uoenum; upper enoscopy (rst step in workup) usually negative, CT shows ui/air aroun aortic graft/sac; management is graft excision, close the uoenum, an revascularization either with in situ human aortic homograft, neoaortoiliac proceure (NAIS) or extra anatomic axillobifemoral bypass V. Mesenteric Ischemia A. Acute mesenteric ischemia (AMI) etiologies: embolism (to SMA, most common), thrombosis, low ow state (non occlusive), venous thrombosis B. AMI presentation: writhing aroun complaining of severe abominal pain, but not signicantly tener on exam (pain out of proportion to physical exam); x with CTA an have preop iscussion regaring bowel viability, quality of life, an possible bowel resection C. Surgery for AMI ue to embolism: resect frankly necrotic bowel, open SMA embolectomy 1. Transverse arteriotomy proximal to the mile colic artery, then embolectomy . If any bowel with questionable viability: o not resect, leave abomen open, plan for n look within 4 to 48 hours D. Chronic mesenteric ischemia (CMI) from atherosclerosis of the celiac, superior mesenteric, an/or inferior mesenteric arteries. E. CMI presentation: severe abominal pain about 30 to 60 minutes after eating, often leaing to “foo fear” (intestinal angina) an weight loss F. First-line treatment for CMI is angioplasty an stenting; if bypass is require in high-risk or sick patient, perform retrograe mesenteric bypass from common iliac artery or infrarenal aorta to avoi supraceliac clamping G. Aggressive ui resuscitation shoul be use with caution in non occlusive mesenteric ischemia in the setting of ecompensate congestive heart failure, use irect intraarterial papaverine instea H. Venous thrombosis usually treate with anticoagulation (unless peritonitis); nee hypercoagulable workup
248
PArt i Patient Care
Fig. 17.1 Types of Endoleaks
CHAPtEr 17 Vascular—Arterial
249
QUESTIONS 1. A 5-year-ol woman presents to the trauma bay after a motor vehicle accient. A pan-CT scan is negative for any acute injuries but oes show an inciental focal ilation of the mi-splenic artery to a iameter of .5 cm. The best management of this ning is: A. No further management is necessary B. Elective splenectomy C. Elective coil embolization D. Open repair with vein interposition graft E. Surveillance with repeat imaging in 6 months 2. A 40-year-ol woman with refractory hypertension unergoes further workup. Her plasma an urine metanephrines are normal, alosterone to renin ratio is 3 cm in patients with low surgical risk, an any size in women of chilbearing age. Coil embolization is appropriate for proximal an mi-portion aneurysms as the spleen continues to be perfuse by the short-gastric arteries avoiing splenic infarction. For istal-thir aneurysms, resection with splenectomy is usually performe (B). Open repair with vein interposition graft has largely fallen out of favor (D). Because this patient is a woman of chilbearing age, it woul be inappropriate to procee with conservative management an surveillance (A, E). Patients foun to have a splenic artery aneurysm without inication for repair shoul be followe with annual CT angiogram or ultrasoun. Women are at highest risk for splenic artery aneurysm rupture uring the 3r trimester of pregnancy. Reference: Chaer RA, Abularrage CJ, Coleman DM, et al. The Society for Vascular Surgery clinical practice guielines on the management of visceral aneurysms. J Vasc Surg. 00;7(1 S):3 S–39 S.
2. C. This patient has bromuscular ysplasia (FMD). Renal
artery stenosis is a cause of seconary hypertension an can be ue to atherosclerosis or FMD. FMD is an iiopathic isease of the musculature of the arterial walls leaing to stenosis of small an meium-size arteries an is most common in women from 30 to 60 years ol. The most commonly involve arteries are the renal, caroti, an vertebral arteries. The “string-of-beas” appearance is a classic imaging ning seen in FMD. FMD is noninammatory an there is no role for sterois (B). Patients with renal artery stenosis ue to atherosclerosis shoul receive aspirin an statin; most are manage meically. A renal stent may be consiere in the case of refractory hypertension or ash pulmonary eema (A, D). On the other han, patients with renal artery stenosis ue to FMD are most appropriately treate with percutaneous angioplasty, as stents have a high rate of fracture when use for FMD renal isease. Open bypass is more invasive an has similar success as angioplasty (E). Reference: Gornik HL, Persu A, Alam D, et al. First international consensus on the iagnosis an management of bromuscular ysplasia. J Hypertens. 019;37():9–5.
3. D. Early failure (within 30 ays) after surgery generally
inicates a technical error. Technical errors inclue anastomotic stenosis, a kink or twist within the graft, poor choice of proximal or istal target, an inaequate-caliber saphenous vein. Intermeiate failures, from 30 ays to years after
bypass, are generally cause by myointimal hyperplasia (A). Late graft failures (beyon years) are cause by progression of atherosclerotic occlusive isease, either within the inow or outow vessels (B). A persistent valve woul be a potential problem with an in situ vein bypass (not with a reverse vein), in which case valves are intentionally cut with a valvulotome (E). Young patients may have a more aggressive form of atherosclerotic isease (virulent isease), an some have postulate that this may be seconary to an unerlying hypercoagulable state (C). Reference: McCreay RA, Vincent AE, Schwartz RW, Hye GL, Mattingly SS, Griffen WO Jr. Atherosclerosis in the young: a virulent isease. Surgery. 1984;96(5):863–869.
4. C. In an acutely ischemic limb, in aition to the neu-
rovascular exam of the ischemic limb, the most important aspects of the physical examination are the cariac exam an the neurovascular examination of the nonischemic limb. If the nonischemic limb has normal pulses an no other evience of chronic ischemia (e.g., hair loss, thin ry skin), then the ischemia is most likely embolic in nature. Fining an irregularly irregular rhythm woul further conrm that the heart is the most likely source of the clot ue to atrial brillation. With an absent femoral pulse, the embolus has likely loge in the common femoral artery. Because the patient escribe has class b ischemia (immeiately threatene), heparin shoul be starte, an revascularization shoul be performe without elay (E). In class 1 ischemia (not threatene; no sensory or motor loss), there is no immeiate urgency to going to the operating room. Heparin shoul be starte. It is then useful to obtain imaging to conrm the iagnosis. This can be achieve via an arterial uplex scan or CT angiogram, which has replace iagnostic arteriography as the gol stanar (A). An avantage of CT over angiogram is that it may etect etiologies of acute ischemia that woul otherwise be unsuspecte, such as an aortic issection or aneurysm, an one can image the chest an abomen for possible pathology. Following iagnosis, if the patient is not immeiately threatene, they may unergo enitive treatment via thrombolytic therapy or open embolectomy (B, C). Native arterial occlusions ue to cariac embolization ten to respon less favorably to thrombolytic therapy. Thus, open embolectomy is preferre by some. For the patient in the vignette, a transfemoral approach is optimal because it can be one with the patient uner local anesthesia an allows selective embolectomy own the supercial femoral an profuna femoral arteries. The below-knee popliteal artery approach to embolectomy is reserve for situations in which the patient has normal femoral an popliteal pulses an the embolus is loge in the tibial vessels (E). However, such an approach is technically more ifcult. If the limb is not immeiately threatene, istal clots are better manage by lytic therapy as the tPA can be irecte via catheter irectly into the involve vessel. Echocariogram woul eventually be useful to look for a cariac source of thrombus, but it woul not be of immeiate help in the management (D). With the avent of hybri operating rooms, patients
CHAPtEr 17 Vascular—Arterial with more avance ischemia (class ) can be taken irectly to the operating room where a iagnostic angiography followe by immeiate intervention can be achieve. Reference: Results of a prospective ranomize trial evaluating surgery versus thrombolysis for ischemia of the lower extremity. The STILE trial: The STILE investigators (appenix A). Ann Surg. 1994;0(3):51–68.
5. C. This patient’s history an CT scan nings are most
consistent with acute mesenteric ischemia. Acute mesenteric ischemia can be ivie into four major causes. Embolization from a cariac source is the most common cause (30%–50% of cases), is seen most often in the setting of atrial brillation an is the likely etiology in the patient presente. The ning of an irregularly irregular heart rhythm suggests an arterial embolism from atrial brillation. The most common site of mesenteric embolization is the superior mesenteric artery (SMA) (ue to its angle from the aorta). The embolus typically occlues the SMA just istal to the mile colic artery. These patients often have sparing of the proximal jejunum an transverse colon because the mile colic artery remains patent. Celiac artery embolization is rare, given its take-off at a right angle to the aorta. The inferior mesenteric artery orice is so small that a cariac thrombus rarely loges insie. Mesenteric arterial thrombosis is usually ue to unerlying mesenteric artery atherosclerosis. In this situation, the patient will typically have a long-staning history of pain after eating, fear of eating, an weight loss, an the physical examination will reveal evience of iffuse atherosclerosis an bruits. Mesenteric venous thrombosis is a thir etiology an is most often seen in patients with hypercoagulable states. The acute venous occlusion leas to massive bowel eema with seconary arterial insufciency from bowel wall istention. Patients with mesenteric venous thrombosis ten to present in a less ramatic fashion, often with ays or weeks of abominal pain. Finally, nonocclusive mesenteric ischemia results from shock that creates hypoperfusion of the bowel, such as with cariac failure or severe hypovolemia. The classic nings in acute mesenteric ischemia are the suen onset of severe pain out of proportion to the physical examination nings. Elevate serum lactate levels shoul raise the suspicion of ischemic bowel, but they are not sensitive enough to etect early bowel ischemia. A plain abominal raiograph is often unremarkable, although it may emonstrate evience of eema in the small bowel wall. If the patient has peritoneal signs on abominal examination, this will inicate that the bowel has alreay been infarcte. In the absence of peritonitis an because the ifferential iagnosis is extensive, CT provies the greatest iagnostic yiel initially (E). However, CT scan may not be iagnostic because it may not necessarily emonstrate opacication in the mesenteric veins or arteries (epening on the timing of contrast). The rst step in the management is the aministration of IV heparin. Following heparin, for an embolus, immeiate surgery offers the best chance of treatment an woul involve an SMA embolectomy (D). If the history were suggestive of unerlying mesenteric atherosclerosis (longstaning postpranial abominal pain an weight loss) with thrombosis, arteriography woul be helpful because the management woul involve an arterial bypass or stenting (B). If the CT scan reveale a thrombus in a mesenteric vein, enitive treatment woul be heparin alone, provie there is no peritonitis. For nonocclusive
255
ischemia, correcting the unerlying shock is the initial management. Catheter irecte papaverine may also be useful. There are some case reports in which mesenteric emboli have been successfully manage with lytic therapy, but this is not the stanar approach an is not the best option for elevate lactate suggesting a compromise bowel (A).
6. C. Mesenteric venous thrombosis accounts for approxi-
mately 10% to 15% of cases of mesenteric ischemia. It tens to have a slow, insiious onset, as in this case. Risk factors for mesenteric venous occlusion inclue hypercoagulable states such as factor V Leien, antithrombin III eciency, an protein C an S eciency, as well as liver isease with portal hypertension, pancreatitis, an any intraperitoneal inammatory conitions. Venous thrombosis is less ramatic than arterial occlusion. Abominal pain is vague, an tenerness is mil or equivocal. CT may emonstrate a thickene bowel wall with elaye passage of IV contrast agent into the portal system an a lack of opacication of the portal or superior mesenteric vein. If the iagnosis is establishe from the CT scan, further iagnostic tests are unnecessary. Another useful iagnostic moality is uplex ultrasoun scanning. Arteriography may emonstrate venous congestion an a lack of prompt lling of the portal system (D). If the patient is manifesting peritoneal signs, operative exploration is inicate (E). However, in the absence of peritonitis, therapy shoul consist of ui hyration, hemoynamic support, anticoagulation with heparin, an serial examination. If peritonitis subsequently evelops, exploratory laparotomy is appropriate to assess bowel viability with segmental bowel resection. Surgical thrombectomy of the venous system is not likely to be successful. Fibrinolytic therapy has been use increasingly, but is not yet the stanar treatment of choice, an is ieal when symptoms are of short uration (A). Following heparin, warfarin or a novel oral anticoagulant, such as apixaban or rivaroxaban, is recommene for 3 to 6 months if the hypercoagulable state is provoke or temporary (B). Lifelong warfarin or NOAC is recommene if the venous thrombosis is unprovoke or associate with a permanent thrombophilic state. The family history of venous thrombosis in this patient is highly suggestive of an inherite hypercoagulability an woul warrant lifelong anticoagulation. Aitionally, any mesenteric arterial embolism requires lifelong anticoagulation. Reference: Kumar S, Sarr MG, Kamath PS. Mesenteric venous thrombosis. N Engl J Med. 001;345(3):1683–1688.
7. C. Cilostazol has a number of functions incluing inhib-
iting platelet aggregation an smooth muscle proliferation, increasing vasoilation, an lowering high-ensity lipoprotein an triglycerie levels. Cilostazol has been shown to signicantly increase walking istance by 50% to 67% in patients with clauication in several ranomize trials an results in improvement in physical functioning an quality of life. This rug is contrainicate in patients with congestive heart failure. This rug is more effective than pentoxifylline in the treatment of clauication (A). Pentoxifylline is a methylxanthine erivative that has hemorrheologic properties. Two meta analyses showe that it improves walking istance, but in some more recent ranomize stuies, it prove to be no better than placebo. Pentoxifylline improves symptoms of clauication by increasing re bloo cell exibility
256
PArt i Patient Care
an reucing bloo viscosity. Antiplatelet meications such as aspirin are use in the treatment of peripheral vascular isease an for cariac an stroke prevention but o not appear to improve walking istance (B). Aspirin has been foun to reuce the vascular eath rate by approximately 5% in patients with any manifestation of atherosclerotic isease (e.g., coronary, peripheral). Clopiogrel is effective in reucing overall acute cariovascular events, especially in patients with lower extremity occlusive isease, but is much more expensive (D). It oes not seem to irectly improve walking istance. Pure vasoilators have not been efcacious in the treatment of peripheral vascular isease because most patients with such occlusive isease alreay exhibit marke vasoilation. Anticoagulants also have not been shown to alter the course of peripheral atherosclerosis (E). Reference: Money SR, Her JA, Isaacsohn JL, et al. Effect of cilostazol on walking istances in patients with intermittent clauication cause by peripheral vascular isease. J Vasc Surg. 1998;7():67–74.
8. E. Leg eema after femoral-to-popliteal arterial bypass is
common. In most instances, it is ue to lymphatic isruption. This isruption occurs at both the groin an popliteal incisions as well as from harvesting of the saphenous vein. Deep venous thrombosis can occur after this proceure but is relatively uncommon (A). Reperfusion eema may be associate with compartment synrome an can present with the Ps (pain, pallor, paralysis, paresthesia, an poikilothermia) (B). It is more likely to present after revascularization ue to acute limb ischemia. The saphenous veins are part of the supercial venous system, which contributes a minority of the venous rainage in the leg, so swelling seconary to venous congestion is not expecte after a saphenous vein harvest (C). Cellulitis woul present with erythema, pain, warmth, an possible systemic signs such as fever or leukocytosis (D). Reference: AbuRahma AF, Wooruff BA, Lucente FC. Eema after femoropopliteal bypass surgery: lymphatic an venous theories of causation. J Vasc Surg. 1990;11(3):461–467.
9. D. The patient’s history an examination are most consis-
tent with symptoms of coronary-subclavian steal synrome. Most patients with a coronary artery bypass graft have unergone a left internal mammary artery-to-left anterior escening graft. In the setting of subclavian artery stenosis or occlusion proximal to the take-off of the internal mammary artery, arm exercise leas to vasoilation of the arm vessels an lower resistance. Bloo will travel through the path of least resistance an ow in a reverse fashion from the left anterior escening artery into the left internal mammary artery an towar the arm, leaing to the evelopment of angina. The ifferential bloo pressure in the arms is the clue, as is the left arm clauication. Treatment involves relieving the subclavian artery obstruction. This can be one by subclavian artery stenting but on occasion requires a caroti-to-subclavian artery bypass (A). Since the problem is not relate to unerlying cariac isease, caroti stenting, increasing betablocker ose, or increasing ose of nitrates will not resolve the patient’s chest pain with exercise (B, C, E). Reference: Bryan F, Allen R, Lumsen A. Coronary subclavian steal synrome: report of 5 cases. Ann Vasc Surg. 1995;9(1):115–1.
10. B. Initial management of patients with acute mesenteric
ischemia inclues ui resuscitation an systemic anticoagulation with heparin sulfate to prevent further thrombus propagation. Signicant metabolic aciosis shoul be correcte with soium bicarbonate. A central venous catheter, peripheral arterial catheter, an Foley catheter shoul be place for ui resuscitation an hemoynamic status monitoring. Appropriate antibiotics are given before surgical exploration. The operative management of acute mesenteric ischemia is ictate by the cause of the occlusion. For an SMA embolus, exposure of the SMA is obtaine via rotation of the small bowel to the right an by sharply issecting the ligament of Treitz. The SMA will be foun at the root of the mesentery. The primary goal in the surgical treatment of embolic mesenteric ischemia is to restore arterial perfusion with removal of the embolus from the vessel. This is one by performing a Fogarty embolectomy using a transverse arteriotomy (longituinal arteriotomy will cause stenosis upon closure) (D). It is important to avoi resecting bowel until perfusion has been restore; that way, bowel viability can be better establishe. After restoration of SMA ow, an assessment of the intestinal viability is mae, an nonviable bowel is resecte. Because the amount of bowel resecte can be extensive an this places the patient at risk of short bowel synrome, bowel that is of borerline viability shoul be left in place with a planne secon-look proceure performe 4 to 48 hours later to reassess whether aitional bowel resection is neee (E). Low-ose opamine leas to vasoilatation of mesenteric arteries; however, its benets are unclear (C). Intraoperative angiography will not provie any aitional information that woul assist in the surgical management of SMA embolus (A).
11. B. The most common cause of mesenteric ischemia is
a cariac embolus to the SMA. The SMA provies bloo to the bowel from the ligament of Treitz to the mi transverse colon. Cariac embolus tens to loge just past the SMA origin at a point where the artery begins to narrow, which is just beyon the rst jejunal branches. These patients often have sparing of the proximal jejunum an transverse colon because the mile colic artery remains patent. Thrombosis of the SMA, conversely, is usually cause by unerlying atherosclerotic isease that occurs at the SMA origin an woul thus not spare the proximal jejunum (A). Mesenteric venous thrombosis an nonocclusive mesenteric ischemia woul more likely cause patchy areas of ischemia (C–E). Reference: Elrup-Jorgensen J, Hawkins RE, Breenberg CE. Abominal vascular catastrophes. Surg Clin North Am. 1997;77(6):1305–130.
12. E. The timing of CEA after a stroke is controversial. A elay in surgery increases the risk of recurrent stroke. The risk is highest within the rst month. Conversely, operating too early (within 4 hours) creates a potential risk of a reperfusion injury, particularly if a large infarction is present on compute tomography (CT) an if hypertension cannot be controlle postoperatively. Intracranial bleeing is thought to occur because of altere autoregulation an hyperperfusion of ischemic tissue. In the North American Symptomatic Caroti Enarterectomy Trial (NASCET), however, postoperative intracranial hemorrhage occurre in only 0.% of patients. Until recently, CEA was routinely elaye for 4 to
CHAPtEr 17 Vascular—Arterial 6 weeks after a stroke. Subsequent analysis of the NASCET showe that patients with a stable, nonisabling acute stroke, a normal CT scan, an a normal level of consciousness can safely unergo CEA shortly after the iagnosis is mae, the symptoms have stabilize, an preoperative risk assessment is complete. Thus, the operation is not urgent (D). Delaying the surgery for 6 weeks or more eliminates much of the benet of CEA because the risk of recurrent stroke is greatest early on (B, C). Current treatment guielines from the American Acaemy of Neurology an from the American Stroke Association/American Heart Association recommen that CEA for patients with nonisabling strokes shoul preferably be performe within weeks of the primary stroke. Patients with a large stroke on CT scan or those with a miline shift may be at higher risk of reperfusion injury, particularly if they have a epresse level of consciousness. Operation shoul be elaye until these patients improve an plateau in their clinical recovery, which is usually in the range of 4 to 6 weeks. If the stroke is completely isabling (A), there remains little if any motor cortex to protect from future stroke, so CEA is not inicate. Thus, patients with severe neurologic ecits, without meaningful recovery or with marke alteration of consciousness, are not caniates for CEA because the goal of CEA is to prevent further amage to the ipsilateral motor cortex. References: Henerson RD, Eliasziw M, Fox AJ, Rothwell PM, Barnett HJ. Angiographically ene collateral circulation an risk of stroke in patients with severe caroti artery stenosis. North American Symptomatic Caroti Enarterectomy Trial (NASCET) Group. Stroke. 000;31(1):18–13. North American Symptomatic Caroti Enarterectomy Trial Collaborators, Barnett HJM, Taylor DW, et al. Benecial effect of caroti enarterectomy in symptomatic patients with high-grae caroti stenosis. N Engl J Med. 1991;35(7):445–453. Sacco RL, Aams R, Albers G, et al. Guielines for prevention of stroke in patients with ischemic stroke or transient ischemic attack: a statement for healthcare professionals from the American Heart Association/American Stroke Association Council on Stroke: co-sponsore by the Council on Cariovascular Raiology an Intervention: the American Acaemy of Neurology afrms the value of this guieline. Circ. 006;113(10):e409–e449.
13. E. The ACAS ranomize patients with asymptomatic
caroti artery stenosis of 60% to 99% to either CEA an aspirin or aspirin alone (C). The stuy was interrupte because of a signicant benet ientie in patients unergoing CEA. A relative reuction in stroke rate by 50%, from 11% to 5% at 5 years, was observe in patients unergoing CEA (A). The Asymptomatic Caroti Surgery Trial conrme the ACAS nings that in patients with 60% to 99% stenosis, the net 5-year risk was 6.4% for all strokes or eath in patients unergoing CEA, versus 11.8% in those not unergoing surgery. This was a net absolute gain of 5.4% (relative risk reuction, 46%). The trial also showe that patients who unerwent CEA were much less likely to have a fatal or isabling stroke (3.5% in the surgery group versus 6.1% in the no-surgery group). The stuies have foun that there is less or no benet in women (E). The greatest benet was in men younger than 75 years of age. CEA for asymptomatic stenosis will only benet the group as a whole if the combine stroke an eath rate is less than 3% (B). Keeping this combine enpoint low is epenent on both patient risk an surgeon skill (C). There is no benet to CEA once the ICA
257
is completely occlue (100%) (D). There is no further ow in the artery, thus the embolic risk is eliminate. The benet of aggressive meical management (incluing antiplatelet agents) is that it can also be protective from coronary events. The biggest limitation of ACAS is that it i not inclue the use of a statin, which, in aition to its lipi-lowering response, also has pleiotropic effects such as plaque stability, which may prove to be a more important contributor in preventing the progression to stroke in caroti isease. The Aggressive Meical Treatment Evaluation for Asymptomatic Caroti Artery Stenosis (AMTEC) trial attempte to compare moern meical management with CEA, but the stuy was prematurely terminate an the results are not yet available. Newer stuies are neee to etermine if moern meical therapy continues to be inferior to surgical intervention in patients with caroti isease. Some authors have suggeste that we shift away from using ecrease luminal caliber as our primary eterminant of choosing which asymptomatic patients to offer surgery. Newer methos of ientifying high-risk patients such as those with plaque ulceration an instability shoul be stuie to either replace or supplement existing societal guielines. References: Enarterectomy for asymptomatic caroti artery stenosis. Executive Committee for the Asymptomatic Caroti Atherosclerosis Stuy. JAMA. 1995;73(18):141–148. Halliay A, Mansel A, Marro J, et al. Prevention of isabling an fatal strokes by successful caroti enarterectomy in patients without recent neurological symptoms: ranomise controlle trial. Lancet. 004;363(940):1491–150. Kolos I, Loukianov M, Dupik N, Boytsov S, Deev A. Optimal meical treatment versus caroti enarterectomy: the rationale an esign of the Aggressive Meical Treatment Evaluation for Asymptomatic Caroti Artery Stenosis (AMTEC) stuy. Int J Stroke. 015;10():69–74. Weyer GW, Davis AM. Screening for asymptomatic caroti artery stenosis. JAMA. 015;313():19–193.
14. D. The rst NASCET stuy foun that CEA was of
benet for symptomatic severe ICA stenosis (70%–99%). A symptomatic caroti artery stenosis was ene as a nonisabling stroke, a hemispheric transient ischemic attack, or a retinal symptom (amaurosis fugax). Life-table estimates of the cumulative risk of any ipsilateral stroke at years were 6% in the aspirin group an 9% in the aspirin an CEA group. In the secon NASCET stuy, there was no benet for symptomatic patients with less than 50% stenosis (E). For symptomatic patients with stenosis from 50% to 69%, there was a very moest benet: 5-year risk of ipsilateral stroke was 15.7% in the CEA group an .% in the meical group (P = 0.04). The benet was greatest in men, in those with hemispheric symptoms (as oppose to retinal ones), an with recent stroke. Women appeare to have less risk of stroke an also ha higher perioperative mortality than men. ACAS emonstrate the benet of CEA compare with aspirin for asymptomatic ICA stenosis of 60% to 99%. However, the benet is much less than for symptomatic high-grae stenosis. Thus in this question, choice A woul be benecial but of less benet than choice D (symptomatic). Choice B woul be of no benet because the stenosis is moerate, an the symptoms are on the wrong sie (retinal is ipsilateral). In choice C, the symptoms are also on the wrong sie with respect to the stenosis.
258
PArt i Patient Care
15. A. New neurologic ecits that present within the
rst 1 hours of operation are almost always the result of thromboembolic phenomena stemming from the CEA site. Possibilities inclue the evelopment of thrombus on the enarterectomize arterial surface, a resiual intimal ap in the ICA leaing to occlusion, or a resiual ap in the external caroti artery (ECA) leaing to ECA thrombosis an retrograe embolization of the clot into the ICA. Immeiate heparinization an exploration are inicate without the nee for conrmatory arteriography or noninvasive tests. On reexploring the woun, the ECA an ICA shoul be palpate for the presence of a pulse. If there is no pulse, this inicates thrombosis, an initial on-table arteriography is not necessary. The artery shoul be reopene an inspecte to look for a cause of the thrombosis. Before closing the arteriotomy, care shoul be taken to ensure that there is goo back-bleeing from the ICA. Fogarty balloon embolectomy of the cephala ICA shoul be avoie because this can lea to a caroti-cavernous sinus stula. The arteriotomy shoul then be reclose with a patch. On-table arteriography shoul then be performe to ensure that the istal ICA is patent an to etermine whether there is an embolus in the mile cerebral artery. If an embolus is present in the intracranial caroti or mile cerebral artery, local infusion of a lytic agent shoul be consiere (B). If on reopening the woun, an excellent pulse is present in the ICA an ECA, with normal signals on han-hel Doppler ultrasonography, on-table arteriography is performe (C, D). If arteriography reveals an intimal ap or irregular mural thrombus at the enarterectomy site, then reopening of the vessel is inicate. Neurologic ecits that evelop 1 to 4 hours after the operation are usually ue to thromboembolic phenomena stemming from the CEA site but may also be cause by a postoperative hyperperfusion synrome. These latter conitions may be worsene by immeiate heparinization an reexploration. Therefore, ecits occurring 1 to 4 hours after the operation shoul be promptly investigate with hea CT an CT arteriography (E).
16. C. The incience of hyperperfusion synrome after a
CEA is reportely 0.3% to 1%. It is thought to occur as a result of impaire autoregulation of cerebral bloo ow an oes not nee to be taken back to the OR (E). The thought is that longstaning, severe caroti stenosis leas to hypoperfusion, leaing to a compensatory ilation of cerebral vessels istal to the stenosis as part of the normal autoregulatory response to maintain aequate cerebral bloo ow. After CEA restores normal pressure, however, autoregulation is impaire an oes not immeiately ajust to the suen increase in bloo ow. Risk factors associate with cerebral hyperperfusion inclue recent stroke, surgery for very tight ICA stenosis, concomitant contralateral ICA occlusion, evience of chronic ipsilateral hypoperfusion, stage bilateral CEA performe within months of each other, an poorly controlle pre- an postoperative hypertension. Pathologic changes range from mil cerebral eema an petechial hemorrhage to severe intracerebral hemorrhage an eath, particularly if not promptly treate (B). The synrome is herale by an ipsilateral frontal heaache, most commonly occurring at a meian of the fth postoperative ay (A). By that time, the patient is alreay at home. Thus, it is imperative to warn patients of this rare synrome an ieally have the patient check his or
her bloo pressure aily for the rst week postoperatively. The heaache may be followe by focal motor seizures that are often ifcult to control. Management consists of controlling bloo pressure, ieally with a beta-blocker, with the avoiance of vasoilators (as these may increase cerebral bloo ow), an use of antiseizure meications (D). Reference: Schroeer T, Sillesen H, Sørensen O, Engell HC. Cerebral hyperperfusion following caroti enarterectomy. J Neurosurg. 1987;66(6):84–89.
17. D. This patient has Takayasu arteritis, an inammatory
isease of the aorta an its branches, as well as the coronary an pulmonary arteries (A–C, E). It occurs most commonly in young women, with a meian age of 5 years. The clinical course has been escribe as beginning with constitutional symptoms such as fever an malaise. However, a National Institutes of Health stuy showe that only one-thir of patients recall such symptoms. Characteristic clinical features inclue hypertension, retinopathy, aortic regurgitation, cerebrovascular symptoms, angina, congestive heart failure, abominal pain or gastrointestinal bleeing, pulmonary hypertension, an extremity clauication. The gol stanar for iagnosis is arterial imaging, with the emonstration of occlusive isease in the subclavian arteries. Unlike atherosclerosis, which tens to affect the origin of these vessels, Takayasu arteritis affects the miportions of these arteries. Characteristic signs an symptoms inclue pulselessness or bloo pressure ifferential in the arms, upper or lower extremity clauication, syncope, amaurosis fugax, blurre vision, an palpitations. Treatment initially consists of steroi therapy with the aition of cytotoxic agents use in patients who o not achieve remission. Carotiynia, which is pain along iname arteries, is pathognomonic for Takayasu arteritis. Surgical treatment with arterial bypass is only performe in avance states an in situations in which the patient oes not respon to meical therapy. It shoul ieally be performe when the isease is not active. Because the isease causes transmural arterial inammation with concentric brosis, there is no role for enarterectomy, an angioplasty has not been met with goo results.
18. A. The most common mechanisms of blunt caroti
injury inclue motor vehicle accients, st ghts, hanging, an intraoral trauma. However, it has also been reporte with relatively minor trauma, such as after chiropractic manipulation of the neck an forceful sneezing. Bif et al. have grae blunt caroti injury as follows: grae I: luminal irregularity or issection with less than 5% luminal narrowing; grae II: issection or intramural hematoma with greater than or equal 5% luminal narrowing; grae III: pseuoaneurysm; grae IV: occlusion; grae V: transection with free extravasation. Horner synrome (oculosympathetic paresis) is common with this injury an is thought to be relate to the involvement of the internal part of the pericaroti sympathetic plexus (B). The ecision to perform surgery is base on (1) injury severity, () presence or absence of symptoms, an (3) surgical accessibility of the lesion (C). In general, there is little role for surgical intervention in patients with grae I or II blunt caroti injury as in this patient (E). Antiplatelet therapy with aspirin is the best treatment option. However, some trauma centers chose to use subtherapeutic heparin initially in case patients may require a surgery. Minor (intimal)
CHAPtEr 17 Vascular—Arterial injuries ten to heal themselves (D). Pseuoaneurysms typically o not an are a relative inication for surgery if accessible in the neck. References: Bif WL, Moore EE, Offner PJ, Brega KE, Franciose RJ, Burch JM. Blunt caroti arterial injuries: implications of a new graing scale. J Trauma. 1999;47(5):845–853. Bromberg WJ, Collier BC, Diebel LN, et al. Blunt cerebrovascular injury practice management guielines: the Eastern Association for the Surgery of Trauma. J Trauma. 010;68():471–477.
19. E. The patient has a symptomatic high-grae caroti
stenosis, an, as such, an intervention is inicate. With the history of raiation therapy an neck issection, the patient has what is terme a “hostile neck.” This increases the risk of caroti enarterectomy, in terms of cranial nerve injury an woun healing. The previous neck issection results in a paucity of tissue coverage between the skin an the caroti artery. This can lea to the catastrophic complication of caroti blow out. The best alternative in this patient woul be to perform caroti stenting with a cerebral protection evice (A–D). Patients with asymptomatic ICA stenosis in the 50% to 69% range shoul be starte on meical therapy with an antiplatelet agent (for all patients), antihypertensive agent (if they have hypertension), an the use of a high-intensity statin irrespective of lipi levels (ue to the pleiotropic effect of plaque stabalization). Reference: Harro-Kim P, Kakhoayan Y, Dereyn CP, Cross DT 3r, Moran CJ. Outcomes of caroti angioplasty an stenting for raiation-associate stenosis. AJNR Am J Neuroradiol. 005;6(7):1781–1788.
20. C. Recurrent caroti stenosis can occur after CEA. The
risk of more than 50% restenosis is 5.8%, 9.9%, 13.9%, an 3.4% at 1, 3, 5, an 10 years, respectively. However, severe (>80%) stenosis evelops in only .1% of patients. Early (within 4 weeks) restenosis is usually ue to a technical error. Recurrent caroti stenosis occurring beyon 1 month but within the rst years after CEA is usually seconary to myointimal hyperplasia. This type of stenosis tens to have a benign course (the lesion is smooth an less prone to embolization), with a low risk of recurrent stroke. In aition, reoperative CEA carries a higher risk of cranial nerve injury (7.3% rate of permanent injury in one series) (A). The patient is asymptomatic. If the patient ha a symptomatic recurrence, the best option woul be caroti stenting (B, D–E). When the recurrent stenosis evelops or more years after CEA, recurrent atherosclerosis is the usual cause.
21. D. Suen occlusion of the ICA in a young patient is
highly suggestive of a spontaneous issection. This is further supporte by the tapere occlusion seen on imaging (escribe as “ame-shape”). On the other han, occlusion ue to atherosclerosis typically occurs ush with the common caroti, an in oler patients ICA issection may occur either spontaneously or after trauma. Cervical artery issection is a signicant cause of stroke in patients younger than 40 years. Common presenting symptoms of ICA issection are heaache, transient ischemic attack an/or stroke, an Horner synrome (ptosis, miosis, anhyrosis). Risk factors for issection inclue history of infection (syphilis), smoking, Ehlers-Danlos synrome type IV, cystic meial necrosis, Marfan synrome, family history, oral contraceptives, an atherosclerosis. In a young female, bromuscular ysplasia
259
woul be high on the ifferential. The iagnosis is mae by uplex scan an/or CT angiography. Duplex scan may be iagnostic, if it emonstrates a membrane within the lumen, consistent with a issection. The most likely mechanism of acute issection is an intimal tear followe by an acute intimal issection, which prouces luminal occlusion ue to seconary thrombosis. The occlusion angiographically is typically to 3 cm beyon the bifurcation. Autopsy stuies have shown a sharply emarcate transition between the normal caroti artery an the issecte segment. Treatment is with anticoagulation an, in most cases, results in complete resolution within a few months. Stenting may be an option in symptomatic patients in the absence of occlusion (C). CEA, Fogarty embolectomy, or lytic therapy is not appropriate for a spontaneous issection (A, B, E).
22. E. Thromboangiitis obliterans (Buerger isease) is a
progressive nonatherosclerotic segmental inammatory isease that most often affects small- to meium-size arteries, veins, an nerves of the upper an lower extremities (C). The typical age at onset is 0 to 50 years, an the isorer is more common in men who smoke. The isease also affects the veins, an specically the upper extremities may be affecte by a migratory supercial thrombophlebitis. Patients initially present with foot, leg, arm, or han clauication. Progression of the isease leas to ischemic rest pain an ulcerations of the toes, feet, an ngers. Characteristic angiographic nings may show isease connement to the istal circulation, usually infrapopliteal an istal to the brachial artery. The occlusions are segmental an show skip lesions with extensive collateralization, the so-calle corkscrew collaterals. The iagnosis is ifcult to establish an is a iagnosis of exclusion because there are no pathognomonic features. As such, the isease can be confuse with chronic embolization an other iseases. Several criteria have been establishe to conrm the iagnosis: age younger than 45 years; current (or recent) smoker; istal extremity ischemia (clauication, pain at rest, ischemic ulcers, gangrene); exclusion of autoimmune iseases, hypercoagulable states, an iabetes mellitus; exclusion of a proximal source of emboli by echocariography an arteriography; an characteristic arteriographic nings in the involve limbs. The aortoiliac segments are typically spare, as are the coronary arteries (A, B). The mainstay of treatment revolves aroun smoking cessation. In patients who are able to abstain, isease remission is impressive an amputation avoiance is increase. The role of surgical intervention is minimal because there is usually no acceptable target vessel for bypass (D). Sympathectomy may result in mil improvement of symptoms. Reference: Olin JW. Thromboangiitis obliterans (Buerger’s isease). N Engl J Med. 000;343(1):864–869.
23. C. The ABI normally varies between 1 an 1. because
the ankle pressure in the supine position can be as much as 0% higher than in the arm (A). Peripheral arterial isease has been ene as a value less than 0.9 an inicates some egree of stenosis. Patients with clauication typically have an ABI between 0.5 an 0.7, an those with rest pain have an ABI less than 0.4 (B). Patients with iabetes an enstage renal isease are at risk of eveloping calcication of the arterial meial layer, known as meial calcinosis, or
260
PArt i Patient Care
Mönckeberg arteriosclerosis. This process makes bloo vessels rigi an ifcult to compress, causing falsely increase pressure reaings. The process tens to affect tibial vessels primarily an spares igital vessels in the toes. As such, toe pressures are more reliable, as are other measures of istal perfusion such as transmetatarsal pulse volume recorings an transcutaneous oximetry (D, E). Reference: Belkin M, Whittemore A, Donalson M, et al. Peripheral arterial occlusive isease. In: Townsen CM, Jr, Beauchamp RD, Evers BM, Mattox KL, es. Sabiston textbook of surgery: the biological basis of modern surgical practice. 17th e. Philaelphia: W.B. Sauners; 004:199.
24. B. Absolute contrainications to thrombolytic therapy
inclue recent stroke or transient ischemic attack, active or recent bleeing, an signicant coagulopathy. Relative contrainications inclue patients with recent major surgery (within weeks, an greatest with recent neurosurgery or eye surgery), recent trauma, uncontrolle hypertension, intracranial tumors, an pregnancy (A). Thrombolytic therapy is most effective in patients with ischemia of less than weeks’ uration (D). The risk of bleeing with thrombolytic therapy is increase with the longer uration of therapy an with ecreasing brinogen levels. In most series, thrombolytic therapy is use for as long as 48 hours, at which point the bleeing risk increases signicantly (E). The causes of acute limb ischemia can be ivie into embolic an thrombotic. The heart is the most common source of emboli leaing to acute ischemia, most often in the setting of atrial brillation. Other cariac sources inclue mural thrombus after an acute myocarial infarction, valvular isease, an atrial myxoma. Other sources of emboli inclue arterial aneurysms an atherosclerotic plaques. Thrombosis is most often cause by unerlying atherosclerosis in the peripheral arteries, an these patients typically have a history of clauication. The severity of acute limb ischemia is base primarily on the motor an sensory examination. Patients shoul be place in four categories: class 1 (nonthreatene) has normal motor an sensory function; class (threatene) inclues a—sensory ecit only an b—(immeiately threatene) both motor an sensory ecit; an class 3 inicates irreversible complete motor an sensory loss. In aition, consieration shoul be given to the uration of ischemia. As a general rule, patients with class 1 ischemia can be treate with multiple options, a trial of heparin alone, thrombolytic therapy, or operative embolectomy/ bypass. Patients with class ischemia nee prompt restoration of bloo ow, so heparin alone is not acceptable. With class b ischemia, the threat of limb loss is more immeiate. Since thrombolytic therapy may require more than 4 to 48 hours to restore ow, class b ischemia (motor an sensory ecit) is a relative contrainication to thrombolysis (C). Such a patient shoul be taken to the operating room. Category 3 ischemia is consiere irreversible an requires amputation. Irreversible ischemia is conrme by an absence of arterial or venous Doppler signals, uration of ischemia of more than 6 to 8 hours, presence of mottling of the skin, absence of capillary rell, an complete anesthesia an paralysis. References: Norgren L, Hiatt WR, Dormany JA, et al. Inter-society consensus for the management of peripheral arterial isease (TASC II). J Vasc Surg. 007;45 Suppl S:S5–S67. Semba CP, Murphy TP, Bakal CW, Calis KA, Matalon TA. Thrombolytic therapy with use of alteplase (rt-PA) in peripheral arterial
occlusive isease: review of the clinical literature. The Avisory Panel. J Vasc Interv Radiol. 000;11():149–161. Results of a prospective ranomize trial evaluating surgery versus thrombolysis for ischemia of the lower extremity. The STILE trial: The STILE investigators (appenix A). Ann Surg. 1994;0(3):51–68.
25. C. The ipsilateral greater saphenous vein is the conuit
of choice for lower extremity istal bypass for peripheral arterial isease (contralateral vein for trauma). An ieal conuit shoul be a minimum of 3 mm (but ieally 4 mm). When the greater saphenous vein is not available, options inclue the lesser saphenous an cephalic veins. Ectopic veins (i.e., lesser saphenous, arm veins) are generally inferior to a single-segment saphenous vein, although they are still superior to the performance of synthetic grafts. A composite graft, which is a vein graft sewn to a polytetrauoroethylene graft, has a patency rate similar to that of a prosthetic graft an tens to evelop neointimal hyperplasia. The bypass shoul be as short as possible (proximal inow from the most istal normal artery (in this case, popliteal), an istal outow to where the artery reconstitutes most proximally (in this case, above the ankle). Options A an B are suboptimal because it involves a longer bypass than is necessary given the patent femoral artery an normal popliteal pulse an harvesting contralateral vein. Enovascular approaches (such as angioplasty) are options but are less urable, particularly in the presence of a long segment of occlusion. However, in a relatively healthy patient, with a goo saphenous vein an goo runoff into the foot, a bypass is likely the better option (D). Amputation of the toe is unlikely to heal in the absence of a palpable peal pulse an such a low ABI (E). Reference: Gentile AT, Lee RW, Moneta GL, Taylor LM, Ewars JM, Porter JM. Results of bypass to the popliteal an tibial arteries with alternative sources of autogenous vein. J Vasc Surg. 1996;3():7–79.
26. E. This is Raynau isease. First escribe in 186 by
Maurice Raynau, the term Raynau isease applies to a heterogeneous symptom array associate with peripheral vasospasm, more commonly occurring in the upper extremities. The characteristically intermittent vasospasm classically follows exposure to various stimuli, incluing col temperatures, tobacco, or emotional stress. Formerly, a istinction was mae between Raynau isease an the Raynau phenomenon for escribing a benign isease occurring in isolation or a more severe isease seconary to another unerlying isorer, respectively. However, collagen vascular isorers evelop in many patients at some point after the onset of vasospastic symptoms; the rate of progression to a connective tissue isorer ranges from 11% to 65% in reporte series. Characteristic color changes occur in response to the arteriolar vasospasm, ranging from intense pallor to cyanosis to reness as the vasospasm occurs. The igital vessels then relax, eventually leaing to reactive hyperemia. The majority of patients are women younger than 40 years of age. As many as 70% to 90% of reporte patients are women, although many patients with only mil symptoms may never present for treatment. Geographic regions locate in cooler, amp climates such as the Pacic Northwest an Scaninavian countries have a higher reporte prevalence of the isease. Certain occupational groups, such as those that use vibrating tools, may be more preispose to Raynau isease or
CHAPtEr 17 Vascular—Arterial igital ischemia. The exact pathophysiologic mechanism behin the evelopment of such severe vasospasm remains elusive, an much attention has focuse on increase levels of α-arenergic receptors an their hypersensitivity in patients with Raynau isease, as well as abnormalities in the thermoregulatory response, which is governe by the sympathetic nervous system. There is no cure for Raynau isease; thus, all treatments mainly palliate symptoms an ecrease the severity an perhaps frequency of attacks. Conservative measures preominate, incluing the wearing of gloves, use of electric or chemically activate han warmers, avoiing occupational exposure to vibratory tools, abstinence from tobacco, an relocating to a warmer, rier climate. The majority (90%) of patients will respon to avoiance of col an other stimuli. The remaining 10% of patients with more persistent or severe synromes can be treate with a variety of vasoilatory rugs, albeit with only a 30% to 60% response rate. Calcium channel blocking agents such as iltiazem an nifeipine are the rugs of choice. The selective serotonin reuptake inhibitor uoxetine has been shown to reuce the frequency an uration of vasospastic episoes but is not the rst-line treatment (C). Intravenous infusions of prostaglanins have been reserve for nonresponers with severe symptoms (B). Upper extremity sympathectomy may provie relief in 60% to 70% of patients; however, the results are short-live, with a graual recurrence of symptoms in 60% within 10 years (A). Cervical sympathectomy has fallen out of favor an has been replace by localize igital sympathectomy using microsurgery. This involves stripping the aventitia of igital arteries an thus removing sympathetic bers. A col stimulation test or nail fol capillaroscopy may be use to conrm the iagnosis of Raynau isease, but there is no role for arteriography (D).
27. D. Pseuoaneurysms can manifest with pain, a pulsa-
tile mass, an/or compression of ajacent structures. Large, expaning, painful pseuoaneurysms are at signicant risk of rupture an shoul be repaire urgently. Smaller, stable pseuoaneurysms may be observe (E). Duplex ultrasonography has been the iagnostic proceure of choice because it helps ene size, morphology, an location. Pseuoaneurysms less than cm in iameter have a higher likelihoo of spontaneous thrombosis with compression therapy, whereas larger ones an those in patients receiving anticoagulation therapy are likely to persist. However, given the reporte high failure rates with ultrasoun compression (A, B), ultrasonography-guie thrombin injection is the best treatment option an is the treatment of choice. Surgery is reserve for infecte or rapily expaning pseuoaneurysms (C). Reference: Wixon CL, Philpott JM, Bogey WM Jr, Powell CS. Duplex-irecte thrombin injection as a metho to treat femoral artery pseuoaneurysms. J Am Coll Surg. 1998;187(4):464–466.
28. A. Colonic ischemia is a recognize complication after
AAA repair, whether open or enovascular. It occurs in approximately 1% to 3% of cases. It is thought to be ue to either ligation of the inferior mesenteric artery (IMA) or ligation or exclusion of internal iliac arteries. The most common presentations inclue an unexpectely early return of bowel function manifeste by iarrhea, left lower quarant pain, abominal istention, persistent leukocytosis, elevate white
261
bloo cell count, an lactic aciosis. Diagnosis is conrme by exible proctosigmoioscopy, which reveals a friable mucosa. Proctosigmoioscopy may not be able to accurately istinguish partial ischemia from full-thickness necrosis. Initial management is meical an consists of nasogastric tube ecompression, IV hyration, placing the patient on NPO, an broa-spectrum antibiotics. Full-thickness necrosis of the colon shoul be suspecte in patients with evience of peritonitis or unremitting aciosis. In such cases, laparotomy with colonic resection an colostomy is inicate (C, D). The mortality rate after emergent colectomy approaches 50%. Arteriography woul not typically be helpful because the usual cause is an intene ligation or exclusion of an internal iliac artery or IMA (B, E). Reference: Becquemin JP, Majewski M, Fermani N, et al. Colon ischemia following abominal aortic aneurysm repair in the era of enovascular abominal aortic repair. J Vasc Surg. 008;47():58–63.
29. B. The presentation is consistent with a rupture AAA.
If the patient was hemoynamically unstable, he shoul be taken irectly to the operating room (D). If the patient is relatively stable (as in this case), a CT scan is preferre to conrm the presence of a rupture AAA an etermine feasibility of enovascular repair, provie there is a coorinate multiisciplinary rupture aneurysm team that has immeiate enovascular capabilities. Although most surgeons woul approach a rupture AAA via the enovascular approach, recent Cochrane analysis emonstrate no ifference in 30-ay mortality for patients with rupture AAA that were treate with an enovascular approach compare to an open approach. Although ultrasonography is useful for etermining the presence of an AAA, it is not accurate for etermining the presence of a retroperitoneal rupture (C). Ultrasonography woul be reasonable to perform in this patient was unstable, an no pulsatile mass coul be felt on physical examination, so as to conrm that an aneurysm was present. Once in the operating room, the patient shoul be preppe an rape before anesthesia inuction because the anesthesia may inuce a precipitous ecrease in bloo pressure (E). Because of the large retroperitoneal hematoma that is typically foun, proximal control is best achieve by clamping the aorta at the iaphragm. Most surgeons woul recommen a policy of “permissive hypotension” en route to the operating room. Excessive ui aministration an elevation of the bloo pressure may further exacerbate bleeing (A). References: Bager SA, Harkin DW, Blair PH, Ellis PK, Kee F, Forster R. Enovascular repair or open repair for rupture abominal aortic aneurysm: a Cochrane systematic review. BMJ Open. 016;6():e008391. Lee WA, Hirneise CM, Tayyarah M, Huber TS, Seeger JM. Impact of enovascular repair on early outcomes of rupture abominal aortic aneurysms. J Vasc Surg. 004;40():11–15. Van Der Vliet JA, Van Aalst DL, Schultze Kool LJ. Hypotensive hemostasis (permissive hypotension) for rupture abominal aortic aneurysm: are we really in control? Vascular. 007;15(4):197–00.
30. C. Popliteal aneurysms are the most common periph-
eral artery aneurysms (overall, aortic an iliac aneurysms are more common). They can be suspecte on physical examination. They are bilateral in 50% of patients. Patients who are foun to have a popliteal aneurysm shoul unergo
262
PArt i Patient Care
screening for an AAA because 30% will have a concomitant AAA. The most frequent complication of popliteal aneurysms is leg ischemia ue to thrombosis an embolization from the aneurysm. Guielines for repair are controversial. Some authors recommen repair for all popliteal aneurysms. Most woul agree that inications for repair are (1) all aneurysms larger than cm, () aneurysms with intraluminal thrombus, regarless of size, or (3) those that are symptomatic or have evience of previous embolization (A, B). Diagnosis is mae by uplex ultrasonography, which can measure the aneurysm size an etect the presence of thrombus. Arteriography assists in operative planning but shoul not be use for iagnosis because it oes not etect the thrombus nor accurately measure the size. The surgical approach to the popliteal artery is either via the meial approach or the posterior approach (E). The posterior approach is ieal if the aneurysm is just behin the knee joint. Magnetic resonance imaging an CT angiography can be use as alternatives for operative planning. The stanar operative approach involves bypassing the aneurysm with saphenous vein an interval ligation of the popliteal artery. With this approach, the aneurysm sac is not opene, an as such, there is a small risk of continue aneurysm expansion an compression of ajacent structures. Formal enoaneurysmorrhaphy, as is one with an open AAA repair, is another alternative. In the setting of acute thrombosis, lytic therapy is the initial treatment of choice. Enovascular stent grafting is another option, especially if no suitable vein is available in a high risk patient, but this may have a lower primary patency (D). References: Ascher E, Markevich N, Schutzer RW, Kallakuri S, Jacob T, Hingorani AP. Small popliteal artery aneurysms: are they clinically signicant? J Vasc Surg. 003;37(4):755–760. Lowell RC, Gloviczki P, Hallett JW Jr, et al. Popliteal artery aneurysms: the risk of nonoperative management. Ann Vasc Surg. 1994;8(1):14–3.
31. E. A patient with an upper gastrointestinal blee an a
history of aortic surgery shoul be presume to have an aortoenteric stula until proven otherwise. The treatment algorithm epens on the hemoynamic stability of the patient. If the patient is unable to be stabilize ue to massive hemorrhage, the patient shoul be taken emergently to the operating room, even if a iagnosis has not yet been establishe. Oftentimes, the patient will have a so-calle heral blee, after which the bleeing may temporarily stop, allowing a workup for an aortoenteric stula. The iagnosis can be ifcult to establish. Upper enoscopy is negative surprisingly often an has a low sensitivity but shoul be the rst step in the workup. Duoenal graft erosion typically occurs at the fourth portion of the uoenum, an nings may be subtle, such as mil mucosal erosion. CT scan is highly useful, as in the presence of an aortoenteric stula will likely emonstrate perigraft ui, air, or inammation, inicative of a graft infection (though less likely contrast extravasation). Flui an inammatory changes aroun a graft woul be abnormal nings beyon 6 weeks after surgery (A). If the CT scan nings are negative, a nuclear-tagge white bloo cell scan may be useful for establishing a graft infection (C). Arteriography is of limite benet for the iagnosis of vascular graft infections but can be useful in preoperative planning (B). In some instances, no source of an upper gastrointestinal
blee is foun, an thus one must empirically procee to graft excision (D). The classic operative management consiste of obtaining proximal aortic control of the aorta at the iaphragm, graft excision, closure of the aortic stump in two layers, closure of the uoenum, placing omentum in the area of the aortic stump closure, followe by an extra anatomic axillobifemoral bypass. Recently, the more accepte treatment is excision of the aortic graft an in situ placement of a human aortic homograft. Reference: Berger P, Moll FL. Aortic graft infections: is there still a role for axillobifemoral reconstruction? Semin Vasc Surg. 011;4(4):05–10.
32. C. Common iliac aneurysms are usually iagnose inci-
entally. In most cases, they are foun in association with an aortic aneurysm. Rare presentation inclues the evelopment of a stula with the ajacent iliac vein or compression of the iliac vein. The natural history of common iliac aneurysms is less well ene. In a recent stuy, the expansion rate of common iliac aneurysms was 0.9 cm per year, an hypertension preicte faster expansion. Because no rupture of a common iliac aneurysm smaller than 3.8 cm was observe, the recommene threshol for elective repair of asymptomatic patients was larger than 3.5 cm (A, B, D, E). Treatment options inclue open surgical replacement with prosthetic graft or enovascular stent grafting. In patients with suitable anatomy, namely, the presence of proximal an istal laning zones, stent grafting has become the treatment of choice. Enovascular repair is associate with fewer complications overall but poses a higher risk of creating buttock clauication ue to occlusion of the internal iliac artery. Reference: Huang Y, Gloviczki P, Duncan AA, et al. Common iliac artery aneurysm: expansion rate an results of open surgical an enovascular repair. J Vasc Surg. 008;47(6):103–110.
33. B. Popliteal aneurysms rarely rupture (A). Most com-
monly, they cause acute or chronic ischemia. In most series, the most common symptom is thrombosis, in as many as 49%, followe by istal embolization. As the aneurysm continues to grow, less commonly, it can compress ajacent structures, such as the popliteal vein (D, E). Chronic embolization can lea to occlusions of the infrapopliteal vessels an can complicate revascularization (C). If they present with acute ischemia, thrombolysis is the intervention of choice, followe by operative repair. Recently, enovascular stent grafting has been use, although long-term patency ata are still lacking. References: Dorigo W, Pulli R, Turini F. Acute leg ischemia from thrombose popliteal artery aneurysms: role of preoperative thrombolysis. Eur J Vasc Endovasc Surg. 00;3(3):51–54. Shortell CK, DeWeese JA, Ouriel K, Green RM. Popliteal artery aneurysms: a 5-year surgical experience. J Vasc Surg. 1991;14(6):771–776.
34. C. Recent stuies have shown that AAAs as large as
5.5 cm in iameter can be safely observe (D). Another recent ranomize stuy inicate that although the perioperative mortality rate of EVAR is lower than that of open repair, long-term mortality is the same (C). Women have been shown to have higher perioperative mortality rates than men with either EVAR or open repair. EVAR shoul not lower the size threshol for repair in a high-cariac risk patient if the AAA has not yet reache the 5.5-cm threshol (B). Following are the guielines for treatment of AAAs as reporte by a
CHAPtEr 17 Vascular—Arterial subcommittee of the Joint Council of the American Association for Vascular Surgery an Society for Vascular Surgery: 1. The arbitrary setting of a single-threshol iameter for elective AAA repair that is applicable to all patients is not appropriate because the ecision for repair must be iniviualize in each case. 2. Ranomize trials have shown that the risk of rupture of small AAAs is quite low an that a policy of careful surveillance of those with a iameter of as large as 5.5 cm is safe, unless there is rapi expansion (>1 cm/yr) or symptoms evelop. However, early surgery is comparable to surveillance with later surgery, so patient preference is important, especially for AAAs 4.5 to 5.5 cm in iameter. 3. Base on the best available current evience, a iameter of 5.5 cm appears to be an appropriate threshol for repair in an average patient. However, subsets of younger, low-risk patients with a long projecte life expectancy may prefer early repair. If the surgeon’s personal ocumente operative mortality rate is low, repair may be inicate at smaller sizes if that is the patient’s preference. 4. For women or for AAAs with a greater than average rupture risk, 4.5 to 5 cm is an appropriate threshol for elective repair (A). 5. For high-risk patients, elay in repair until the iameter is larger is warrante, especially if enovascular aortic repair is not possible (E). 6. In view of its uncertain long-term urability an effectiveness as well as the increase surveillance buren, EVAR is most appropriate for patients at increase risk of conventional open aneurysm repair. EVAR may be the preferre treatment metho if anatomy is appropriate for oler high-risk patients, those with a hostile abomen, or other clinical circumstances likely to increase the risk of conventional open repair. 7. Use of EVAR in patients with unsuitable anatomy markely increases the risk of averse outcomes, the nee for conversion to open repair, or AAA rupture. 8. At present, there oes not seem to be any justication that EVAR shoul change the accepte size threshol for intervention in most patients. 9. In choosing between open repair an EVAR, patient preference is of great importance. It is essential that the patients be well informe to make such choices. References: Brewster DC, Cronenwett JL, Hallett JW Jr, et al. Guielines for the treatment of abominal aortic aneurysms. Report of a subcommittee of the Joint Council of the American Association for Vascular Surgery an Society for Vascular Surgery. J Vasc Surg. 003;37(5):1106–1117. Mureebe L, Egorova N, McKinsey JF, Kent KC. Gener trens in the repair of rupture abominal aortic aneurysms an outcomes. J Vasc Surg. 010;51(4 Suppl):9 S–13 S.
35. B. Enoleak is a common complication after EVAR
that can lea to aneurysm enlargement an even rupture. Enoleaks occur in as many as 40% of patients after EVAR. Most enoleaks are foun in the immeiate postoperative perio, but late enoleaks also evelop. For this reason, routine lifelong postoperative surveillance with CT scanning is recommene. New enoleaks have been ientie as
263
late as 7 years after EVAR. Enoleaks are classie into ve major types (types I–V) base on the source of communication between the circulation an the aneurysm sac. The most common type of leak after enovascular repair is a type II leak, which results from retrograe lling of the aneurysm sac from the lumbar arteries or the IMA. Management of type II leaks is controversial an is base on whether the aneurysm is enlarging or stable. Options inclue coil embolization of the vessel, laparoscopic ligation, or observation. Type I leaks occur at the stent–graft attachment sites (either at the aorta or at the iliac arteries) (A); type III leaks occur at a stent–stent interface an are also known as moular isassociations (C); type IV leaks are irectly through the graft an are ue to graft material porosity (D). They usually heal spontaneously. The most angerous type of leak is a proximal type I leak because there is a failure to achieve a proximal seal, leaing to continue lling of the aneurysm sac at systemic pressures. Type I leaks require immeiate treatment when iscovere, typically by eploying another stent or, if unsuccessful, by open surgical conversion. Type III enoleaks represent a true mechanical failure of the enograft an require repair with an aitional enograft to eliminate systemic ow an pressure in the aneurysm. Type V leak is also referre to as enotension. This can be consiere iiopathic because the aneurysmal sac may appear to be enlarging without any evience of a leak site on imaging (E). Reference: Corriere MA, Feurer ID, Becker SY, et al. Enoleak following enovascular abominal aortic aneurysm repair: implications for uration of screening. Ann Surg. 004;39(6):800–807.
36. E. This patient most likely has an arterial thrombus seconary to heparin-inuce thrombocytopenia thrombosis (HITT). The classic laboratory ning is a ecrease in the platelet count of more than 50%. Although thrombocytopenia usually increases the risk of bleeing, HITT is paraoxically known to cause a hypercoagulable state; it is the secon most common acquire hypercoagulable state (smoking is the most common). There are two types of HITT with type II being more common an responsible for the clinical synrome. HITT type II is cause by antiboies to platelet-factor 4 an heparin sulfate resulting in a prothrombotic state (will appear as a white clot). It typically occurs 3 to 5 ays after starting heparin. If this is suspecte, heparin shoul be iscontinue, an the patient shoul be starte on a irect thrombin inhibitor. Argatroban is the recommene agent for patients with HITT an renal impairment. Lepiruin an bivaliruin both unergo renal excretion an shoul be avoie in patients with ESRD (A, D). The patient initially ha acute limb ischemia seconary to cariac emboli from atrial brillation. His symptoms resolve with initiation of heparin so it is unlikely that he has unerlying antithrombin III eciency (C). More stuies are neee to evaluate the role of tPA in HITT (B). References: Guzzi LM, McCollum DA, Hursting MJ. Effect of renal function on argatroban therapy in heparin-inuce thrombocytopenia. J Thromb Thrombolysis. 006;(3):169–176. Visentin GP, For SE, Scott JP, Aster RH. Antiboies from patients with heparin-inuce thrombocytopenia/thrombosis are specic for platelet factor 4 complexe with heparin or boun to enothelial cells. J Clin Invest. 1994;93(1):81–88.
Vascular—Venous AMANDA C. PURDY AND JOHN McCALLUM
18
ABSITE 99th Percentile High-Yields I. Deep Vein Thrombosis (DVT) an Pulmonary Embolism (PE) A. Can be provoke (known inciting event, such as recent surgery, malignancy) or unprovoke B. Most common inherite prothrombotic isorer: Factor-V Leien mutation (unable to breakown Factor-V); higher incience on left sie (May-Thurner synrome) C. Catheter-associate upper extremity DVTs—etermine if catheter is require, if it is, can keep catheter an start therapeutic anticoagulation; if not require, start anticoagulation an remove catheter in 3 to 5 ays; in both cases, continue anticoagulation for 3 to 6 months D. Malignancy-associate DVT/PE best treate with low-molecular-weight heparin (not warfarin)
Situation
Duration of anticoagulation
Provoked DVT/PE without associated malignancy
3 months
Provoked DVT/PE in association with malignancy
Indeęnite
Unprovoked DVT/PE in patient with low-moderate bleeding risk
Indeęnite
Unprovoked DVT/PE in patient with high bleeding risk
3 months
DVT/PE pregnant woman
3 months OR 6 weeks after delivery (whichever is longer)
Superęcial thrombophlebitis of legs
4–6 weeks of fondaparinux
E. Phlegmasia alba olens 1. Cause by massive DVT, eep venous channels are affecte while sparing collateral veins an therefore maintaining some egree of venous return; patients have pale blanching of extremity (milky color), eema, an iscomfort . Treatment is immeiate anticoagulation an leg elevation; may progress to cerulea olens F. Phlegmasia cerulea olens 1. Venous outow of the entire lower extremity is affecte; persistent venous obstruction eventually inhibits arterial inow . Patients present with pain, swelling, an a blue iscoloration (cyanosis) of the extremity; peal pulses may be iminishe 3. In aition to anticoagulation, catheter-irecte thrombolysis is also recommene (unless > weeks uration) G. Paget-Schroetter synrome (effort thrombosis of axillary or subclavian vein) 1. Thrombosis of the axillary an/or subclavian veins . Often seen in athletes who overuse the affecte extremity, leaing to recurrent vein compression
265
266
PArt i Patient Care
3. Often relate to thoracic outlet synrome, with compression of the subclavian vein in the costoclavicular space (between the 1st rib an the clavicle) 4. Diagnose with uplex ultrasoun or venogram 5. Treat with anticoagulation; if moerate-severe symptoms of 1 secon in the femoral or popliteal veins, or >0.5 secon in the saphenous, tibial, eep femoral, or perforator veins; also, uplex ultrasoun scan to rule out DVT E. Mainstay of treatment of venous insufciency is compression therapy F. If compression fails, can consier surgical intervention if incompetence/reux is in supercial or perforator veins: 1. For supercial veins (greater or small saphenous veins) or perforator veins—treat with enovenous ablation . If have concomitant symptomatic varicose veins: treat unerlying venous incompetence AND can also perform sclerotherapy or stab phlebectomy of supercial varicose veins 3. Complication of saphenous vein ablation: thrombus at saphenofemoral junction (potential risk for DVT), start ASA 4. Telangiectasias (spier veins): not always associate with reux—if no reux compression an ablation will not help; treatment is cosmetic: inclues injection sclerotherapy or transermal laser therapy
CHAPtEr 18 Vascular—Venous
Fig. 18.1 May-Thurner Synrome
267
268
PArt i Patient Care
Questions 1. Which of the following is the most common risk factor for spontaneous venous thromboembolism? A. Antithrombin III eciency B. Factor V Leien C. Protein C eciency D. Protein S eciency E. Antiphospholipi synrome 2. A 60-year-ol male presents with pain over his left mi-meial thigh. He ha a similar event in his other thigh a month earlier. He has note a ecrease appetite. On exam the skin over the meial thigh is re, warm, an tener. He has no varicose veins, nor evience of skin hyperpigmentation or leg swelling. Duplex scan shows an 8-cm segment of thrombosis of the mi saphenous vein, but no DVT. Which of the following is recommene? A. IV heparin followe by warfarin B. Warm compresses, nonsteroial antiinammatory rugs (NSAIDs), an a CT of the abomen C. Ligation of the sapheno-femoral junction D. Fonaparinux an a CT of the abomen E. Warm compresses an NSAIDs 3. A 5-year-ol male college swimmer presents with suen onset of right arm swelling an pain. A uplex ultrasoun scan emonstrates thrombosis of the axillary-subclavian vein. The patient is starte on IV heparin. The most important aitional ajunctive therapy for this patient is: A. First rib resection B. Catheter-guie thrombolysis C. Lifelong anticoagulation D. Venous stenting E. Physical therapy 4. Which of the following is true regaring venous circulation? A. The perforating veins in the leg irect bloo ow from eep to the supercial system B. The common iliac veins have valves C. In a healthy person, venous pressure increases with walking D. The greater saphenous vein joins the femoral vein to become the common femoral vein E. Muscle contraction plays no role in venous return
5. Which of the following is true regaring the initiation of heparin in a 100-kg patient with a newly iagnose DVT? A. A bolus of 10,000 units of heparin shoul be given before starting the rip B. Following a bolus a rip shoul be starte at 18 units/kg per hour C. Dosing shoul be ajuste using the international normalize ratio (INR) D. Activate partial thromboplastin time (aPTT) shoul be titrate to 100 to 10 secons after starting the rip E. Heparin shoul be stoppe if the platelet count ecreases below 00,000 6. A 35-year-ol female presents with left leg swelling. There are no precipitating factors. Ultrasoun conrms a left iliofemoral DVT, an the patient is starte on heparin. Workup reveals no evience of risk factors for DVT, such as recent surgery, prolonge immobilization, nor any evience of malignancy. Which of the following is most likely to be of long-term benet? A. Low-molecular-weight heparin (LMWH) B. Long-term (>1 months) anticoagulation C. Lifelong compression stocking D. Right-to-left femoral vein bypass E. Venous thrombectomy 7. Which of the following is true regar ing the management of DVT? A. For patients with proximal DVT of the leg an no cancer history, irect Xa inhibitor is recommene over warfarin B. For a leg DVT in association with malignancy, warfarin is preferre over LMWH C. For incientally iscovere DVT, anticoagulation is unnecessary D. In patients with isolate istal (calf) DVT of the leg, anticoagulation therapy is superior to serial imaging E. In patients with a secon episoe of DVT, three months of anticoagulation is recommene
CHAPtEr 18 Vascular—Venous
8. A 58-year-ol male with newly iagnose metastatic colon cancer presents to the ED with a swollen right leg an severe pain that starte 1 ay earlier. On exam, he has massive eema of the right leg that is tener to palpation. His foot appears blue. Duplex scan conrms a DVT. Which of the following is true about this conition? A. The risk of limb loss is low B. This occurs more commonly on the right sie C. A pale, white foot carries a worse prognosis than a blue foot D. Associate hypotension is usually the result of sepsis E. Catheter-irecte thrombolysis shoul be performe 9. Which of the following is the best inication for placement of an inferior vena cava (IVC) lter? A. A pregnant patient in the thir trimester iagnose with a new DVT B. A patient with severe pelvic fractures C. A patient with a large free-oating vena cava thrombus D. A recurrent DVT in a patient who is alreay therapeutic on warfarin E. Before planne thrombolysis of a new DVT 10. The most common electrocariographic change after pulmonary embolism (PE) is: A. Atrial brillation B. Right bunle branch block C. Nonspecic ST an T wave changes D. S1, Q3, T3 pattern E. Sinus tachycaria 11. Trauma patients sustaining what type of injury are at highest risk of venous thromboembolism? A. Hea trauma B. Femur fracture C. Pelvic fracture D. Splenectomy E. Spinal cor injury 12. A 45-year-ol woman presents with a nonhealing ulcer at the meial malleolus associate with leg eema an hyperpigmentation but no signs of infection. First-line management consists of: A. Wet-to-ry ressings B. Split-thickness skin grafting C. Subfascial perforator ligation D. Local woun ebriement followe by intravenous antibiotics E. Compression ressings
269
13. A 50-year-ol male presents with a meial malleolar ulcer that has faile to heal with 4 weeks of compression ressings. He has large varicose veins in the lower leg, eema, an hyperpigmentation. There is no eep vein thrombosis (DVT) ientie on uplex ultrasoun. However, there is incompetence of the supercial, eep, an perforator systems. Which of the following is the best next step? A. Vein stripping of the greater saphenous vein B. Raiofrequency ablation (RFA) of the greater saphenous vein an ultrasoun-guie perforator sclerotherapy C. RFA of the greater saphenous vein an compression stockings D. Continue with a 3-month course of compression ressing treatment E. Ultrasoun-guie perforator vein sclerotherapy 14. A 44-year-ol male presents to the emergency epartment (ED) with a temperature of 103°F. He is hypotensive espite a -L ui bolus. He is preppe for a right internal jugular 9-French central venous line to start pressors while being worke up for an unerlying cause. Following placement of the catheter, pulsatile bleeing is note from the catheter. What is the best next step? A. Downsize to a smaller catheter in the ED, transfer the patient to the intensive care unit (ICU), an remove it in several hours B. Immeiately remove the catheter an hol pressure for 10 to 15 minutes C. Immeiately remove the catheter an get a uplex ultrasoun stuy of the neck D. Remove the catheter uner irect surgical exposure E. Transfer patient to the ICU, then remove, hol pressure, an place a suture in the skin 15. A 40-year-ol woman presents with pain an tenerness at the site of a longstaning varicose vein in her calf. There is a palpable cor with surrouning erythema. Duplex scan shows localize thrombus within the varicose vein, an no DVT. Management consists of: A. Intravenous (IV) heparin soium B. Subcutaneous low-molecular-weight heparin C. Warm compresses an nonsteroial antiinammatory rugs D. Ligation of saphenous vein at saphenofemoral junction E. IV antibiotics
270
PArt i Patient Care
Answers 1. B. The primary risk factors for spontaneous venous
thromboembolism (VTE) as escribe by Virchow inclue stasis of bloo ow, enothelial injury, an hypercoagulability. In cases of spontaneous VTE, hypercoagulability is the most important factor. Factors that contribute to hypercoagulability inclue factor V Leien, prothrombin gene mutation, protein C an S eciency, antithrombin III eciency, elevate homocysteine levels, an antiphospholipi synrome. In aition, nonacquire causes of VTE inclue smoking (most common), obesity, pregnancy, malignancy, an use of oral contraceptives. In surgical patients, the cause of VTE is multifactorial because postoperative stasis from prolonge be rest an enothelial injury from trauma or recent surgery are signicant factors. In trauma patients, spinal cor injury has the highest risk of VTE. Other risk factors for VTE inclue history of VTE, avance age, an varicose veins. Factor V Leien is the most common genetic efect associate with thrombophilia (A, C–E). Factor V Leien is a single-point mutation in the gene that coes for coagulation factor V. It makes factor V resistant to inactivation by activate protein C (which is a natural anticoagulant protein). The mutation is transmitte in an autosomal ominant fashion an accounts for 9% of cases of anticoagulant protein resistance. The mutation is present in 4% to 6% of the general population an is associate with a sixfol increase risk of VTE in heterozygotes. In homozygotes, the risk is 80-fol. In patients with their rst VTE, factor V Leien was present in 15% to 0%. There is no stanar guieline for the uration of anticoagulation therapy in patients with an acquire hypercoagulable state. It is believe an iniviualize approach shoul be taken to access each person's risk of a recurrent VTE an compare this to their relative risk of a bleeing event. Interestingly, in one stuy, the risk of recurrent VTE was similar among carriers of the factor V Leien gene compare with those without this mutation, suggesting that they o not nee longer anticoagulation than the stanar recommenation for a rst-time event. References: Bauer KA. Duration of anticoagulation: applying the guielines an beyon. Hematology Am Soc Hematol Educ Program. 010;010(1):10–15. Mazza JJ. Hypercoagulability an venous thromboembolism: a review. WMJ. 004;103():41–49.
2. D. Unprovoke SVT, an in particular, recurrent unpro-
voke SVT, an more specically recurrent, unprovoke SVT in ifferent limbs (supercial migratory thrombophlebitis) shoul prompt concern for hypercoagulability an, in particular, malignancy. Supercial migratory thrombophlebitis is particularly associate with pancreatic cancer (Trousseau sign) an, to a lesser egree, stomach an lung cancer. Thus, treatment shoul inclue a targete workup for malignancy (that shoul be tailore to nings on history, review of systems, an physical exam) (B, C–E). Given the ecrease appetite, suspicion for GI cancer shoul be high an a CT scan appropriate. SVT within the saphenous vein in the upper thigh, within 3 cm of the saphenofemoral junction, an those with long segments (>5 cm) have an increase risk
of propagating into the eep system an thus benet from anticoagulation (A). A recent stuy comparing fonaparinux with placebo emonstrate a ecrease in DVT, recurrent thrombophlebitis, an clot progression with fonaparinux. References: Chengelis DL, Benick PJ, Glover JL, Brown OW, Ranval TJ. Progression of supercial venous thrombosis to eep vein thrombosis. J Vasc Surg. 1996;4(5):745–749. Decousus H, Pranoni P, Mismetti P, et al. Fonaparinux for the treatment of supercial-vein thrombosis in the legs. N Engl J Med. 010;363(13):1–13.
3. A. Paget-Schroetter synrome, also known as effort-in-
uce thrombosis, is a spontaneous thrombosis of the axillary-subclavian vein. It is thought to be, in most instances, a manifestation of thoracic outlet synrome, whereby a hypertrophie or aberrant muscle compresses the axillary-subclavian vein as it passes between the rst rib an the clavicle. It tens to evelop in young, active patients after vigorous activity (swimming, pitching, weightlifting), although it can also occur spontaneously. It usually presents in men more often than women. Seconary axillary/subclavian vein thrombosis can also present in those with meiastinal tumors, congestive heart failure (CHF), an nephrotic synrome. Diagnosis is best establishe via uplex ultrasonography. The patient shoul be promptly starte on IV heparin. The most important ajunctive measure to prevent recurrence an long-term swelling is thoracic outlet ecompression via rst rib resection. The timing is controversial but is not time sensitive. Systemic thrombolysis is not inicate. However, the journal CHEST recommens catheter-irecte thrombolysis for this conition if the patient has moerate-severe symptoms an presents with less than weeks of symptoms. The benet of catheter-irecte thrombolysis in this situation is a ecrease risk of postthrombotic synrome (B). A follow-up venogram is frequently obtaine to ientify any correctable anatomic abnormalities. Stenting a resiual stenosis in this area without ecompressing the thoracic outlet is contrainicate because the ongoing compression will invariably crush the stent an cause further venous amage, making any further intervention even more ifcult. Resiual venous stenoses can be treate with angioplasty, although some authors recommen oing this after the rst rib resection. A recent metaanalysis emonstrate a signicant improvement in symptoms in those that receive a rst rib resection compare to those that i not. More than 40% of patients in the control group neee to have a rib resection ue to recurrent symptoms. In an active athlete, an in particular one who performs repetitive movements with the arm overhea (which by itself can compress the vein), rst rib resection is the best option (C–E). References: Angle N, Gelabert HA, Farooq MM, et al. Safety an efcacy of early surgical ecompression of the thoracic outlet for Paget-Schroetter synrome. Ann Vasc Surg. 001;15(1):37–4. Lee WA, Hill BB, Harris EJ Jr, Semba CP, Olcott C IV. Surgical intervention is not require for all patients with subclavian vein thrombosis. J Vasc Surg. 000;3(1):57–67. Machleer HI. Evaluation of a new treatment strategy for Paget-Schroetter synrome: spontaneous thrombosis of the axillary-subclavian vein. J Vasc Surg. 1993;17():305–315.
CHAPtEr 18 Vascular—Venous Urschel HC Jr, Razzuk MA. Paget-Schroetter synrome: what is the best management? Ann Thorac Surg. 000;69(6):1663–1668. Lugo J, Tanious A, Armstrong P, et al. Acute Paget-Schroetter synrome: oes the rst rib routinely nee to be remove after thrombolysis? Ann Vasc Surg. 015;9(6):1073–1077.
4. D. The lower extremity veins are ivie into supercial,
perforating, an eep veins. The supercial venous system consists of the greater saphenous an lesser saphenous veins. The eep veins follow the course of major arteries. Paire veins parallel the anterior an posterior tibial an peroneal arteries an join to form the popliteal vein. The popliteal vein becomes the femoral vein as it passes through the auctor hiatus. In the proximal thigh, the greater saphenous vein joins with the femoral vein to become the common femoral vein. Multiple perforating veins traverse the eep fascia to connect the supercial an eep venous systems. The most important perforators are the Cockett an Boy perforators. The Cockett perforators rain the lower part of the leg meially, whereas the Boy perforators connect the greater saphenous vein to the eep vein higher up in the meial lower leg, approximately 10 cm below the knee. Bloo ows from the supercial to the eep venous system (A). Incompetence of these perforators is a major contributor to the evelopment of venous stasis an ulceration. There are no valves in the portal vein, superior vena cava, inferior vena cava (IVC), or common iliac vein (B). The calf muscles serve an important function in augmenting venous return by acting as a pump to return bloo to the heart (E). For this reason, patients who are berien are prone to venous stasis. Venous pressure rops ramatically with walking because of the action of the calf muscles but increases in patients with venous obstruction because this leas to persistent stasis that muscle contraction cannot overcome (C). This is why compression stockings are recommene for these patients as an ajunct to normal venous return.
5. B. If a heparin rip is starte, a bolus of 80 units/kg (8000
units for the above patient) shoul rst be given followe by the continue infusion of heparin at 18 units/kg per hour (A). In patients with DVT, the aPTT nees to be rawn every 6 to 1 hours with a goal rate of 60 to 90 secons (D). INR is checke in patients on warfarin (C). Heparin can potentially lea to heparin-inuce thrombocytopenia (HIT). This usually happens 5 ays or more after the initiation of heparin an will present as a 50% rop in platelet count (E). Reference: Hirsh J, Bauer KA, Donati MB, Goul M, Samama MM, Weitz JI. Parenteral anticoagulants: American college of chest physicians evience-base clinical practice guielines (8th eition). Chest. 008;133(6 Suppl):141S–159S.
6. B. DVT an pulmonary embolism (PE) affect up to
900,000 people per year in the Unite States, an their incience increases with age. When a patient presents with a DVT, always try to etermine which part of the Virchow tria (stasis, vascular injury, an hypercoagulability) can explain the event. This will serve as a reminer to perform a careful history an physical examination to assess risk factors for DVT. In most cases, the causes are multifactorial. The uration an type of anticoagulation epen on whether the DVT is provoke (i.e., malignancy, recent surgery, prolonge immobilization) or unprovoke, what the provoking factor is, an on the location (proximal or istal leg) of the
271
DVT. Proximal (iliofemoral) DVTs are more likely to lea to massive swelling an long-term sequelae of postphlebitic synrome. As such, more consieration shoul be given to the type of anticoagulant, the uration, use of thrombolytics, an mechanical thromboembolectomy as compare with istal DVT. LMWH is not as efcacious for proximal DVT (A). For a proximal (iliofemoral) DVT, that is unprovoke (no clear contributing factors), the recommenation is for long-term (>1 months) anticoagulation. The benet of compression stockings to prevent postphlebitic synrome is controversial (C). Most authors recommen years of compression; lifelong compression has no benet, has poor patient compliance, an is associate with signicant costs of renewing expensive stockings every 6 months. A rightto-left femoral vein bypass (with right leg saphenous vein) is rarely performe an woul be a last resort for chronic venous stasis that is unresponsive to enovascular options (D). Thrombolytic therapy is an option for select patients with severe iliofemoral DVT, particularly if they present with phlegmasia. Venous thrombectomy is reserve for patients with phlegmasia who have faile thrombolytic therapy (E). Reference: Heffner JE. Upate of antithrombotic guielines: meical professionalism an the funnel of knowlege. Chest. 016;149():93–94.
7. A. The American College of Chest Physicians release
upate guielines in 016 for the management of DVTs. One major change is that patients with proximal DVT of the leg an no cancer history shoul now be treate with a irect Xa inhibitor (abigatran, rivaroxaban, apixaban, or eoxaban) over warfarin. Aitionally, initial parenteral anticoagulation with a heparin rip is not require when using rivaroxaban an apixaban. However, a heparin rip shoul be starte before aministering abigatran or eoxaban an overlappe with warfarin therapy. Several reports have shown the superiority of these novel oral anticoagulants (NOACS). Aitionally, iarucizumab is now available as a reversal agent for abigatran allowing NOACs to be more commonly prescribe. In patients with a cancer history an proximal DVT of the leg, LMWH is recommene over warfarin an irect Xa inhibitors (B). This is unchange from the prior guielines. In patients with a proximal DVT of the leg provoke by surgery, 3 months of anticoagulation therapy is recommene over a longer time-limite perio (6, 9, 1, or 4 months). This recommenation applies to patients with both low an high bleeing risks. The management of isolate calf DVT remains controversial. Anticoagulation is recommene for those with severe leg symptoms or those with risk factors for propagation. In patients with an isolate istal DVT of the leg an without severe symptoms or risk factors for extension, serial imaging of the eep veins for weeks is recommene over anticoagulation therapy (D). There is no consensus on the uration of therapy for patients with a secon episoe of DVT because this epens on the presence of reversible risk factors, unerlying cause, malignancy, life expectancy, an the buren of therapy. However, most surgeons woul recommen at least 1 year of anticoagulation therapy for patients with a secon episoe of DVT an lifelong anticoagulation for patients with more than two episoes of DVT (E). Incientally iscovere DVTs shoul be treate with anticoagulation (C).
272
PArt i Patient Care
References: Connors JM. Antiote for factor Xa anticoagulants. N Engl J Me. 015;373(5):471–47. Heffner JE. Upate of antithrombotic guielines: meical professionalism an the funnel of knowlege. Chest. 016;149():93–94.
8. E. Massive iliofemoral DVT can lea to impaire arterial
bloo ow ue to massive swelling. Early on, the limb turns pale an is referre to as phlegmasia alba (white) olens. In a subgroup of patients, this may progress to impening gangrene phlegmasia cerulea (blue) olens as in the patient escribe. When the majority of the eep venous channels are burene with clots, the relatively smaller supercial venous channels are taske with raining the entire leg. Patients evelop a tener, pale, an eematous extremity. This is known as “milk-leg” since the pale extremity appears whitish (alba). As the isease progresses an the supercial venous channels are also affecte, the entire venous rainage of the leg is compromise, causing massive eema in the leg. As the swelling continues, arterial malperfusion ensues, leaing to severe ischemia (blue extremity), risking limb loss (C). DVT an as an extension, phlegmasia, both occur more commonly on the left. This is a result of the left iliac vein frequently being compresse by the right iliac artery (known as May-Thurner synrome) (B). Unerlying malignancy is the most common risk factor ientie for phlegmasia. The fastest an safest metho of conrming the iagnosis is with uplex ultrasoun. CT angiography is not require unless history, exam, an ultrasoun nings are equivocal. Initial treatment is similar to that for an acute DVT, with some qualiers. More emphasis shoul be place on leg elevation. Due to ui sequestration, patients may present with hypovolemic shock an thus may nee massive volume resuscitation (D). The risks of limb loss, pulmonary embolism, postphlebitic synrome, an mortality are all high (A). As such, thrombolytic therapy has emerge as the treatment of choice. Reference: Chinsakchai K, Ten Duis K, Moll FL, e Borst GJ. Trens in management of phlegmasia cerulea olens. Vasc Endovascular Surg. 011;45(1):5–14.
9. D. Enthusiasm for the aggressive use of IVC lters is
iminishing. Filters left in place for long perios of time can lea to complications, incluing migration of the lter, fracturing of the legs of the lter, vena cava perforation, an the increase risk of a recurrent DVT. In a prospective ranomize stuy of patients with DVT, the routine aition of an IVC lter i not improve mortality compare with heparin an warfarin alone. Aitionally, PREPIC trial has also emonstrate an increase number of recurrent PEs in the lter group compare to the anticoagulation only group (3% versus 1.5%). Thus, the majority of IVC lters are now retrievable an shoul optimally be remove within 9 to 1 weeks. Consensus opinion from most societies is that the strongest inication for an IVC lter placement is a patient who evelops a venous thromboembolic event (VTE [DVT or PE]) who has a contrainication to anticoagulation (such as active gastrointestinal bleeing). Other inications are a new VTE that evelops in a patient who is alreay receiving therapeutic anticoagulation, or a patient with a VTE who is alreay receiving anticoagulation an in whom a major hemorrhage evelops (B, C–E). Relative inications inclue prophylaxis in high-risk populations (severe hea, pelvic, or spinal cor trauma), massive PE treate with thrombolysis or
thrombectomy (to prevent further ecompensation), an the presence of a large free-oating thrombus in the IVC. Pregnant patients iagnose with a new DVT shoul be starte on anticoagulation with low-molecular-weight heparin for the remainer of the pregnancy an up to 6 weeks postpartum (A). Warfarin shoul be avoie since it is teratogenic. References: Decousus H, Leizorovicz A, Parent F, etal. A clinical trial of vena caval lters in the prevention of pulmonary embolism in patients with proximal eep-vein thrombosis. N Engl J Med. 1998;338(7):409-416. Millwar SF, Oliva VL, Bell SD, et al. Günther tulip retrievable vena cava lter: Results from the Registry of the Canaian Interventional Raiology Association. J Vasc Interv Radiol. 001;1(9):1053–1058. Rajasekhar A. Inferior vena cava lters: current best practices. J Thromb Thrombolysis. 015;39(3):315–37.
10. E. The most common ning on electrocariogra-
phy after a PE is sinus tachycaria (present in almost half of patients) (A–D). A heart rate greater than 100 beats per minute with associate tachypnea in the setting of suspecte PE shoul further raise concern. The classic ning on an electrocariogram is the S1, Q3, T3 pattern, which consists of a prominent S wave in lea I an a Q wave an inverte T wave in lea III. This electrocariographic ning inicates right ventricular strain from a large PE, but it is not commonly present. A large PE will lea to an enlargement of the right ventricle causing the interventricular septum to eviate to the left. The right bunle branch stretches, leaing to a right bunle branch block.
11. E. The increase risk of the evelopment of VTE in sur-
gical patients is multifactorial. Patients will have a perio of activate coagulation, transient epression of brinolysis, an temporary immobilization. In aition, many patients may have a central venous catheter in place an have concomitant cariac isease, malignancy, or intrinsic hypercoagulable states, all of which increase a patient's chance of a VTE. Trauma patients, in particular, have a high risk of VTE. In trauma patients, spinal cor injury (os ratio, 8.33) an fracture of the femur or tibia (os ratio, 4.8) were the injuries with the greatest risk of VTE (A–D). In one large prospective stuy, other risk factors in trauma patients on multivariate analysis inclue oler age, bloo transfusion, an nee for surgery. Reference: Geerts WH, Coe KI, Jay RM, Chen E, Szalai JP. A prospective stuy of venous thromboembolism after major trauma. N Engl J Med. 1994;331(4):1601–1606.
12. E. This patient has classic signs of chronic venous insuf-
ciency. Venous stasis ulcers are classically locate at the meial malleolus. The precise cause of venous stasis ulcers is unclear but seems to be multifactorial. The increase venous pressure from incompetent valves results in an impeance of capillary ow, which leas to leukocyte trapping. These leukocytes release oxygen free raicals an proteolytic enzymes that lea to local inammation. The increase venous pressure also leas to the leakage of proteins such as brinogen, which act as a barricae to oxygen an growth factors necessary for woun healing. First-line therapy for the treatment of venous stasis ulcers is compression therapy (A–D). The workup for this patient shoul inclue a uplex ultrasoun scan of the venous system, specically looking for valvular incompetence of the eep, supercial, an perforating veins.
CHAPtEr 18 Vascular—Venous A popular an effective compression banage is the Unna boot, which contains zinc oxie, glycerin, gelatin, an calamine lotion. The boot shoul be wrappe starting at the foot, up to just below the knee. It can remain in place for as long as a week. It shoul not be use in the setting of an active infection of the ulcer. In this situation, ebriement an antibiotics will be neee rst.
13. C. A spectrum of chronic venous isorers, from vari-
cose veins to venous stasis ulcers, aficts 0% to 5% of the population. The unerlying etiology is incompetence of the venous valves in either the eep, supercial (saphenous), or perforator veins. Patients with chronic venous isease are classie an treate base on the severity of their isease. The CEAP (clinical, etiologic, anatomic, an pathophysiologic) classication is use worlwie to stanarize this evaluation. It is important when iscussing treatment with a patient that he or she unerstans that this is an incurable isease an that the goal of intervention is to minimize symptoms an prevent recurrence. In general, supercial incompetence is ealt with rst. In a patient with a nonhealing woun that has incompetent valves in all three venous systems, (supercial, perforator, an eep) an is unresponsive to compression therapy, the supercial venous incompetence is aresse rst by obliterating the saphenous vein along with compression therapy. This can be one via saphenous vein stripping, foam sclerotherapy, or RFA. A recent ranomize stuy emonstrate equal results with all three approaches. That being sai, RFA is generally preferre an is the current recommenation of the American Venous Forum, ue to its less invasive nature as compare with stripping (A–E). Freeom of reux has been seen in 93% of patients at years after ablation therapy. Primary venous insufciency is a recognize risk factor for the evelopment of DVT, an it is important to rule this out before intervention to minimize treatment failure. If treating the supercial system is not successful, the next step is to treat the perforator incompetence. This is one via ultrasoun-guie sclerotherapy. There is no surgical treatment that is reliably effective for eep system incompetence. A recent ranomize stuy conrme the benet of early ablation of the saphenous vein to promote woun healing, as oppose to a longer (6 month) trial of compression therapy. References: Gohel MS, Heatley F, Liu X, et al. A ranomize trial of early enovenous ablation in venous ulceration. N Engl J Med. 018;378():105–114.
273
Meissner MH. What is effective care for varicose veins? Phlebology. 016;31(1 Suppl):80–87. O’Donnell TF Jr, Passman MA, Marston WA, et al. Management of venous leg ulcers: clinical practice guielines of the Society for Vascular Surgery an the American Venous Forum. J Vasc Surg. 014;60( Suppl):3S–59S.
14. D. The right internal jugular vein is the preferre option
for central line placement because it is easily accessible an has a lower risk of pneumothorax compare to a subclavian line. It also has a straight course into the right atrium. In 70% of iniviuals, the internal jugular vein lies anterolateral to the caroti artery. However, in some cases, it may lie irectly anterior or posterior to the caroti artery, increasing the risk of a caroti artery cannulation. If the caroti artery is entere with the probe neele (as evience by pulsatile bleeing), the neele shoul be immeiately remove an pressure shoul be hel for 10 minutes. If the artery is cannulate with a ilator or catheter, then the catheter shoul not be remove blinly. This coul lea to a potential airway-threatening hemorrhage. It is safer to remove the catheter in the operating room via irect surgical exposure, followe by suture repair of the artery (A–C, E). Reference: Kron I, Ailawai G. Cariovascular monitoring an support. In: Fischer JE, e. Fischer's mastery of surgery. 6th e. Lippincott Williams & Wilkins; 011:45–66.
15. C. The patient has supercial venous thrombosis (SVT)
or thrombophlebitis. This entity is essentially a clotte surface vein. A palpable cor is suggestive of the iagnosis, as are accompanying pain an erythema. There are a few pitfalls in the iagnosis an management of SVT. Patients with SVT may have a concomitant DVT (5%–40%); thus, a uplex ultrasoun scan of the venous system is essential. Secon, SVT can easily be misiagnose as cellulitis, in which case antibiotics may be inappropriately prescribe an a uplex ultrasoun scan not obtaine. SVT is generally best manage with warm compresses an NSAIDs. IV antibiotics are reserve for septic thrombophlebitis, which is typically associate with an intravenous line (E). Systemic anticoagulation is reserve for a SVT that is near the eep system (A, B). If anticoagulation is contrainicate, ligation of the saphenous vein at the saphenofemoral junction is inicate for a saphenous vein SVT (D). Varicose veins cause stasis an thus preispose to SVT.
Vascular—Access LUIS FELIPE CABRERA VARGAS, MARK ARCHIE, AND CHRISTIAN DE VIRGILIO
19
ABSITE 99th Percentile High-Yields I. Vascular access for hemoialysis (HD) in Patients with En-Stage Renal Disease (ESRD) A. Catheters 1. Temporary noncuffe, nontunnele ialysis catheter . Permanent (cuffe) or tunnele-ialysis catheter (TDC) B. Autogenous surgical access (arteriovenous stula [AVF]) 1. Preferre locations a) Upper > lower extremities; istal > proximal b) Nonominant > ominant arm c) Cephalic > basilic > brachial veins > femoral vein of the thigh . En-to-sie preferre to sie-to-sie anastomosis C. Nonautogenous surgical access (arteriovenous graft [AVG]) 1. Material: prosthetic (PTFE), biologic (bovine, human cryopreserve veins) . Locations: upper arm, forearm, thigh II. Comparing Vascular Access for HD
Catheters
AVF
AVG
1. Ideal for short-term hemodialysis, or as a bridging tool (AVF, graft or kidney transplantation) 2. Best avoided for long-term use 3. To reduce infection, often coated with antibiotics, silver, or heparin (reduces bacterial trapping) 4. Advantages: immediate use, easily removed if infected 5. Disadvantages: highest incidence of 1 year mortality, thrombotic complications, central vein stenosis, and bacterial infections
1. Best overall option for long-term hemodialysis 2. Advantages: lower risk of infection and thrombosis, highest long-term patency (70%–80% at 1 year), greatest blood Ěow volume during dialysis, lowest need for reinterventions 3. Disadvantages: require at least 6 weeks to mature, some never work
1. Ideal for patients who are not candidates for AVF 2. Advantages: widely available, short wait time to access (does not need to mature), if thrombose relatively easy to declot. 3. Disadvantages: lower long-term patency (about 50% at 1 year), higher thrombosis rates (due to intimal hyperplasia), higher infection rate (vs AVF)
III. Surgical Planning A. Age (elerly may be less suitable for AVF), han-ominance (prefer nonominant arm), comorbi conitions (iabetics may evelop severe raial/ulnar artery atherosclerosis), presence of pacemaker (leas to stenoses of proximal veins) B. Duplex ultrasoun vein an artery mapping 1. Veins (with soft tourniquet in place) a) Cephalic, basilic, brachial; assess patency an size (ieally > 3 mm) . Arteries a) Brachial, raial, ulnar; assess size (ieally > mm); severe meical calcication may be unclampable, look for ow limiting plaque 275
276
PArt i Patient Care
C. Fistula rst initiative (orer of preference): raio-cephalic (Cimino) > brachio-cephalic > brachio-basilic (may require stages, rst construction, secon supercialization) > AVG D. For istal stulas from raial or ulnar arteries, assess arterial ominance of the han with Allen test IV. Complications A. Excessive bleeing from ialysis puncture site 1. Nee to rule out stenosis of outow vein or central venous system (high venous pressure) . Rule out use of anticoagulants B. Aneurysm of AVF (may be ue to repeat neele trauma or central stenosis) 1. Most are benign an can be observe . Repair if rapily expaning, overlying skin thinning, skin ulceration, or excessive bleeing 3. Prior to repair, rule out stenosis of outow vein or central venous system as cause C. Pseuoaneurysm of AVG (ue to repeat neele trauma or infection) 1. Small or uninfecte ones can be observe a) Repair if large . Resect AVG if infecte D. Thrombosis E. Steal 1. Stage I: asymptomatic retrograe iastolic ow (US ning alone) . Stage II: pain on exertion an/or uring HD 3. Stage III: pain at rest 4. Stage IV: ulceration/necrosis/gangrene F. Ischemic monomelic neuropathy (IMN) G. High-output cariac failure
Fig. 19.1
Banding Procedure
Reduction of the access Ěow for high Ěowassociated steal syndrome Best results if intraoperative Ěow measurements are used Con: banding a low-Ěow vascular access will lead to ineĜcient dialysis and thrombosis
Access Ligation
Complete resolution of steal syndrome Loss of the vascular access Con: need new AVF with new risk of developing steal syndrome
V.Surgical Treatments of Steal Syndrome
Banding of the vascular access with a nonresorbable suture guided with a 4–5 mm dilatation balloon Controlled reduction in the vessel diameter Only for high Ěowassociated steal syndrome
MILLER Procedure (Minimally Invasive Limited Ligation EndoluminalAssisted Revision) Enhances access Ěow Ideal for low Ěowassociated steal syndrome
PAI Procedure (Proximalization of the Arterial InĚow) Cons: complex surgery Longer operative time Need to harvest suitable vein Hand perfusion reliant on a bypass graft
Ideal for brachial AV access patients with high Ěow induced steal or cardiac failure Creates bypass from distal artery to ęstula (while ligating original anastomosis)
DRIL Procedure (Distal Revascularization RUDI Procedure with Interval (Revision Using Ligation) Distal InĚow)
Uses smaller caliber graft to increase resistance
RUPI Procedure (Revision Using Proximal InĚow)
CHAPtEr 19 Vascular—Access
277
278
PArt i Patient Care
Questions 1. A 63-year-ol man with en-stage renal isease on ialysis via a left upper arm arteriovenous graft (AVG) presents to the emergency epartment with what he escribes as “pulsatile bleeing” from the area of where ialysis was performe the ay before. He states the bleeing stoppe after wrapping his arm. On physical exam, his temperature is 100.5°C an his heart rate is 100. The AVG has a thrill. Overlying the AVG, at the site of the blee, there is a small black eschar with a small aneurysmal swelling. His white bloo cell (WBC) count is 1,000 × 103/mm3 an a uplex ultrasoun emonstrates a pseuoaneurysm at the site of eschar. What is the next best step in management? A. Discharge the patient with oral antibiotics B. Amit the patient for intravenous (IV) antibiotics C. Obtain a stulogram D. Take patient to the operating room for graft excision E. Place a covere stent across the pseuoaneurysm 2. A 45-year-ol female returns to the emergency epartment 8 hours after having unergone a left upper arm arteriovenous graft (AVG) complaining of severe forearm pain. On examination, the forearm is soft an supple. The han appears to be pink, with a normal temperature, an a 1+ raial pulse. There is a goo thrill in the graft. She has both motor an sensory ecits in the han in both the meian an ulnar nerve istributions. The next step in the management is: A. Distal revascularization with interval ligation B. Observation C. Ligation of the graft D. Obtain nerve conuction stuies E. Forearm fasciotomy 3. A 65-year-ol female with en-stage renal isease presents with recurrent episoes of congestive heart failure. She is currently ialyze via a left arm brachiocephalic arteriovenous stula (AVF). Upon compression of the stula, her heart rate ecreases from 80 to 60 beats per minute, an bloo pressure increases from 10/70 to 140/80 mm-Hg. Which of the following is true about this conition? A. The stula shoul be ligate B. It is unlikely that the stula is contributing to the patient’s heart failure
C. Plicating the stula may help prevent another episoe of heart failure D. She shoul unergo a istal revascularization an interval ligation E. The stula shoul be converte to a graft 4. A 65-year-ol woman unergoes creation of an upper arm arteriovenous (AV) graft for hemoialysis in the left arm using a 6-mm polytetrauoroethylene graft. Three weeks later, the patient reports marke coolness, pallor, an numbness in the han as well as pain in the han at rest. Motor exam is normal. On examination, there is no palpable pulse at the raial artery an only a monophasic Doppler signal. Upon graft compression, a pulse becomes palpable an the signal becomes biphasic. Which of the following is the best management option? A. Distal revascularization an interval ligation of the brachial artery B. Ligation of the AV graft an placing the upper arm graft in same arm C. Ligation of the AV graft an placing the stula in the ominant arm D. Baning of the AV graft ajacent to the arterial anastomosis E. Baning of the AV graft ajacent to the venous anastomosis 5. A 45-year-ol male evelops progressive en-stage renal isease seconary to severe hypertension. His glomerular ltration rate is 19 mL/min, an ialysis is anticipate within the next 6 months. He is right-hane an appears to have goo veins in both arms an normal pulses. Which of the following is the best management plan for ialysis access? A. Delay access until about a month before anticipate ialysis B. Procee with left raiocephalic AVF (Cimino stula) C. Procee with left brachiocephalic AVF D. Procee with left brachiobasilic AVF E. Procee with right raiocephalic AVF
CHAPtEr 19 Vascular—Access
6. A 45-year-ol female with en-stage renal isease presents with recent onset of heaaches, hoarseness of her voice, an bilateral arm swelling for ays. She has a history of multiple proceures in both arms an legs for hemoialysis access. Most recently, she unerwent an arteriovenous graft (AVG) in her right upper arm weeks earlier. On examination her neck appears to be engorge an her face swollen. There are numerous visible veins on her chest wall. Which of the following is the best management option? A. Ligation of the AVG B. Plication of the AVG C. Attempt venoplasty of superior vena cava (SVC) D. Place stent in SVC E. Move AVG to right arm 7. A 45-year-ol male with long-staning iabetes an progressive en-stage renal failure presents to the emergency epartment (ED) with progressive shortness of breath, vague abominal pain, an marke leg eema. Laboratory values are remarkable for metabolic aciosis an azotemia but a normal white bloo cell count. Dialysis is urgently neee. Dialysis access woul be best institute via: A. Right internal jugular vein tunnele, cuffe catheter B. Right internal jugular vein nontunnele, uncuffe catheter C. Left internal jugular vein, tunnele, cuffe catheter D. Right subclavian vein uncuffe, nontunnele catheter E. Right femoral vein cuffe tunnele catheter 8. A left internal jugular vein central line is place. Fifteen minutes later, the patient is hypotensive. Distene neck veins are note. Breath souns are clear bilaterally. What is the most likely cause of the patient’s hypotension? A. Perforate right atrium B. Perforate subclavian vein C. Perforate subclavian artery D. Tension pneumothorax E. Perforate right ventricle
279
9. A 50-year-ol male with longstaning history of hemoialysis via a left brachiocephalic arteriovenous stula (AVF) presents with an aneurysm within the miportion of the AVF. He reports that there has recently been excessive bleeing when the neeles have been pulle out. On physical examination, the aneurysm is about 3 cm in size. The overlying skin appears supple, without ulceration. The next step in the management consists of: A. Fistulogram B. Resection/plication of the aneurysm C. Replacement of stula with an AV graft D. Ligation of the stula E. Observation 10. In comparing the three moalities use for hemoialysis (central venous catheter [permacath], arteriovenous [AV] graft, an AV stula), which of the following is true? A. They are equal in terms of 1-year patient mortality B. The primary patency for AV stula an AV graft is similar C. The seconary patency for AV stula an AV graft is similar D. Time to maturation for AV stulas an grafts is similar E. A permacath is the best ialysis option in the elerly 11. An intubate patient in the OR evelops an air embolism after central venous catheter insertion. Which of the following murmurs are associate with this conition? A. Austin-Flint murmur B. Carey Coombs murmur C. Means-Lerman scratch murmur D. Still murmur E. Millwheel murmur
280
PArt i Patient Care
Answers 1. D. This patient has a pseuoaneurysm of his AVG that
appears to be infecte, given his fever an elevate WBC. This requires excision of his graft. Though the bleeing has stoppe, an infecte pseuoaneurysm is inherently unstable an will likely blee again, which coul be catastrophic. Thus, antibiotics alone woul not be aequate (A, B). A stulogram is also not inicate as the graft is infecte (C). For a large, noninfecte pseuoaneurysm, a covere stent is a potential treatment option (E). Reference: Muoni A, Cornacchiari M, Gallieni M, et al. Aneurysms an pseuoaneurysms in ialysis access. Clin Kidney J. 015;8(4):363–367.
2. C. This patient has nings of ischemic monomelic neu-
ropathy (IMN), a rare complication after vascular access surgery. The incience of this complication is less than 1% an is more common in female an iabetic patients. IMN results in pain, numbness (in ngers), paresthesia, an motor weakness (intrinsic han muscles) usually shortly after surgery. It can be istinguishe from steal synrome by its faster onset an mil or absent signs of clinical ischemia. The pathophysiology of IMN is poorly unerstoo but is thought to be cause by a loss of bloo ow from istal nerve tissue leaing to istal neuropathies. IMN is a clinical iagnosis an electromyography an nerve conuction stuies are only neee when the clinical neurologic exam is equivocal (D). The recommene treatment for IMN is ligation of the newly create access, which may lea to resolution of neuropathy in some patients (B,C). Distal revascularization an interval ligation(DRIL) can be utilize in steal synrome, but not in IMN (A). This patient shoul not unergo forearm fasciotomy, as she has a soft an supple forearm, making compartment synrome very unlikely (E). References: Datta S, Mahal S, Govinarajan R. Ischemic monomelic neuropathy after arteriovenous stula surgery: clinical features, electroiagnostic nings, an treatment. Cureus. 019;11(7):e5191. Thimmisetty RK, Peavally S, Rossi NF, Fernanes JAM, Fixley J. Ischemic monomelic neuropathy: iagnosis, pathophysiology, an management. Kidney Int Rep. 017;(1):76–79.
3. C. Bloo ow through an AVF is essentially a left-to-right
shunt, an a portion of the cariac output is stolen by the stula (B). Although there is no change in peripheral oxygen consumption after stula placement, there is a rop in peripheral vascular resistance (PVR). Consequently, a compensatory increase in cariac output occurs. The increase in venous return increases cariac preloa an causes rises in atrial natriuretic peptie (ANP) an brain natriuretic peptie (BNP). The ecrease in afterloa results in a ecrease in alosterone an renin levels. This subsequently leas to a ecrease in afterloa as well as suppression of the renin-alosterone-angiotensin system, which promotes natriuresis. Compressing the stula increases PVR an afterloa, leaing to a ecrease in pulse rate an an increase in bloo pressure (Nicolaoni-Branham sign). Patients with higher stula ow will exhibit greater hemoynamic changes with stula occlusion. Objectively, the minimum stula ow rate require
to support hemoialysis is greater than 400 to 500 cc/min. However, when the ow rate excees 000 cc/min or 30% of the cariac output, there is a risk of high-output cariac failure. These patients, an those with clinically evient episoes of cariac failure, shoul unergo intervention aime at reucing ow rates. Surgical plication (narrowing the vein just beyon the anastomosis to the artery by suturing or baning) reuces the ow rate an can partially reverse the hemoynamic changes an prevent future episoes of heart failure. If heart failure continues to occur after an appropriate reuction in ow rates, eventual ligation of the stula is inicate (A). Distal revascularization an interval ligation (DRIL) is use to treat steal synrome, causing ischemic steal synrome istal to the stula. The proceure increases resistance to the stula an ecreases resistance to the istal extremity but may not effectively reuce stula ow in the setting of cariac failure (D). Converting a native stula to a graft woul not help because the large iameter of a graft woul maintain high ow rates (E). References: MacRae JM, Levin A, Belenkie I. The cariovascular effects of arteriovenous stulas in chronic kiney isease: a cause for concern?: cariovascular effects of arteriovenous stulas. Semin Dial. 006;19(5):349–35. High arteriovenous (AV) access ow an cariac complications. NKF Task Force on Cariovascular Disease, America Journal of Kidney Disease, 3(5).
4. A. A patient presenting with marke coolness, pallor,
pain at rest, an han numbness following an AV graft shoul be suspecte of having steal synrome. Ischemic steal synrome occurs in approximately % to 4% of patients unergoing AV access for hemoialysis. Risk factors for steal synrome inclue females, iabetes, age >60, an use of the brachial artery. Proximal stulas have a higher risk of eveloping steal synrome, while istal wrist stulas (Cimino stulas) have a very low risk. AV grafts also have a greater risk of steal compare with native AV stulas (B). This is likely ue to the fact that the large iameter of the graft creates a low-resistance be. In aition, steal seconary to grafts tens to occur early after the access placement, whereas steal after native AV stulas has a bimoal istribution, with some presenting early an others late after the native vein has unergone ilation with lowere resistance. Some egree of physiologic steal occurs in every patient with an AV stula, but only a small minority manifests severe symptoms. The steal synrome is cause by a iversion of bloo ow from the anastomose artery to the low-resistance vein. In aition, the low-resistance venous anastomosis leas to bloo owing in a retrograe fashion from the istal circulation into the stula. Mil steal can be manage conservatively with exercise. More severe symptoms require intervention. Although ligation of the AV graft woul have a great chance of resolving the steal synrome, the patient will require a new access an will again be at risk of eveloping steal (C). Several options exist for the management. The most effective treatment that maintains stula function is istal revascularization an interval ligation. The isavantage of this
CHAPtEr 19 Vascular—Access proceure is that it requires creating a new bypass, usually with a saphenous vein, from the native artery proximal to the AV graft to the artery istal to it, with interval ligation of the native artery just proximal to the istal anastomosis. Baning or plicating of the AV graft, ajacent to the arterial anastomosis, serves to increase the resistance in the graft an reuce steal. The primary isavantage of this approach (for grafts) is that inaequate baning leas to persistent steal, an excessive baning causes graft thrombosis (stulas less likely to thrombose) (D, E). Baning or plication is a more attractive option for steal in an autologous AV stula, such as a brachial artery cephalic vein stula, because the vein is more resistant to thrombosis. This is not yet the stanar approach, however. References: Walz P, Laowski JS, Hines A. Distal revascularization an interval ligation (DRIL) proceure for the treatment of ischemic steal synrome after arm arteriovenous stula. Ann Vasc Surg. 007;1(4):468–473. Yaghoubian A, e Virgilio C. Plication as primary treatment of steal synrome in arteriovenous stulas. Ann Vasc Surg. 009;3(1):103–107. Yu SH, Cook PR, Canty TG, McGinn RF, Taft PM, Hye RJ. Hemoialysis-relate steal synrome: preictive factors an response to treatment with the istal revascularization-interval ligation proceure. Ann Vasc Surg. 008;():10–14.
5. B. When permanent hemoialysis access is neee, the
nonominant arm (E) shoul be consiere rst in orer to mitigate the effects of potentially evastating complications, incluing severe steal synrome, limb ischemia, ischemic monomelic neuropathy, an nerve injury. Once the sie is etermine, the type of AVF must be consiere. Raiocephalic stulas shoul generally be place rst (assuming aequate artery an vein) because subsequent thrombosis will not preclue the placement of a brachiocephalic or brachiobasilic stula more proximally in the arm. Aitionally, raiocephalic stulas may cause ilation of the proximal arm veins, allowing higher success rates of more proximal stulas in the future. Raiocephalic stulas also rarely require a secon-stage supercialization or transposition proceure because the forearm cephalic vein is close enough to the skin to be use upon maturation. If raiocephalic is not possible or has faile, a brachiocephalic shoul be consiere next (C). Brachiocephalic stulas allow stulas to form on the orsal surface of the upper arm an allow easier cannulation an use uring hemoialysis. Further, epening on boy habitus, brachiocephalic stulas may also not require a secon stage to supercialize the stula close to the skin. The thir choice for autogenous stula is the brachiobasilic stula. Since the basilic vein is eep, it requires supercialization of the vein. Many surgeons perform this in two stages so as to allow the vein to mature before supercialization (D). Maturation of a stula typically requires at least 6 weeks an may require aitional interventions. Waiting until 1 month before ialysis will result in placement of a temporary ialysis catheter, which carries high mortality risks (A). Despite the avantages, the raiocephalic stula has a higher early failure or nonmaturation rate an may not be a goo option in iabetics ue to meial calcinosis within the raial artery. Further, when a patient is alreay hemoialysis epenent via tunnele catheter, there is ongoing ebate about whether the ability to rapily cannulate a graft (∼ weeks) shifts the preferences towar initial graft placement rather than stula rst. A forearm loop graft also has the avantage of ilating the basilic an upper cephalic veins for future stula creation.
281
References: Disbrow DE, Cull DL, Carsten CG 3r, Yang SK, Johnson BL, Keahey GP. Comparison of arteriovenous stulas an arteriovenous grafts in patients with favorable vascular anatomy an equivalent access to health care: is a reappraisal of the Fistula First Initiative inicate? J Am Coll Surg. 013;16(4):679–685; iscussion 685–686. Hakaim AG, Nalbanian M, Scott T. Superior maturation an patency of primary brachiocephalic an transpose basilic vein arteriovenous stulae in patients with iabetes. J Vasc Surg. 1998;7(1):154–157. [No authors liste]. NKF-K/DOQI clinical practice guielines for vascular access. Am J Kidney Dis. 006;48(Suppl. 1):S7–S409.
6. D. The patient is presenting with superior vena cava
(SVC) synrome with bilateral arm, neck, an face swelling an hoarseness of the voice. The patient likely has a preexisting central vein stenosis (in the SVC). A high proportion of patients with en-stage renal isease have central vein stenosis (5%–40%) ue to prior central venous access. These stenoses are often asymptomatic, an if SVC synrome oes evelop, it is usually insiious in onset. However, placement of an upper arm AVG access creates a suen, massive increase in venous return that cannot be accommoate by the stenosis, leaing to abrupt venous congestion (E). Central venous stenosis complicates hemoialysis access because it impairs venous stula outow an can reuce ow rates an reuce the likelihoo of maturation in stulas. Further, when access is place ipsilateral to a stenotic lesion, there is a high likelihoo of symptoms ue to the increase venous congestion combine with high venous resistance. Arteriovenous grafts are more likely to cause symptoms than stulas, an upper arm access is more likely to cause symptoms than forearm access. When central stenosis is suspecte, either from history or symptoms, a central venogram shoul be performe to iagnose the lesion. Concomitant enovascular venoplasty is a reasonable option an has a high rate of success. However, rst-line treatment is now enovascular stenting of the SVC (C). This is appropriate for both benign an malignant cases of SVC synrome. Ligation or plication of the graft is not inicate because this estroys the access an oes not aress the unerlying pathology (A, B). Open SVC repair via sternotomy for a benign lesion is overly invasive an unnecessary given the high initial success rates of enovascular treatment. References: Jones RG, Willis AP, Jones C, McCafferty IJ, Riley PL. Long-term results of stent-graft placement to treat central venous stenosis an occlusion in hemoialysis patients with arteriovenous stulas. J Vasc Interv Radiol. 011;(9):140–145. Rizvi AZ, Kalra M, Bjarnason H, Bower TC, Schleck C, Gloviczki P. Benign superior vena cava synrome: stenting is now the rst line of treatment. J Vasc Surg. 008;47():37–380. Trerotola SO, Kothari S, Sammarco TE, Chittams JL. Central venous stenosis is more often symptomatic in hemoialysis patients with grafts compare with stulas. J Vasc Interv Radiol. 015;6():40–46.
7. A. When hemoialysis is urgently neee, temporary
rapi vascular access must be establishe with a catheter that will support high ow (generally >400 cc/min) via lumens. If long-term ialysis is anticipate, as in this patient, a tunnele, cuffe hemoialysis catheter, or permacath, is preferre (B) an place into a central vein an exits the skin at least 10 cm away via a subcutaneous tract. Tunnele catheters are reay to use immeiately an are less prone to infection than a nontunnele, noncuffe catheter (Quinton catheter). Quinton catheters are preferre in patients neeing urgent ialysis for a short term, or for those with sepsis
282
PArt i Patient Care
(as they are remove rapily). The right internal jugular vein is the rst choice because it is the most irect route to the right atrium. Left-sie placement is less preferable because it jeoparizes venous patency for future permanent access in the left arm (as most patients are right-han ominant). Left-sie catheters also result in lower catheter bloo ow rates an increase the risk of stenosis/thrombosis ue to the longer an more tortuous length of contact with central vein sie-wall (C). The subclavian position is associate with higher rates of complications (D), namely central vein stenosis an pneumothorax, an in some stuies has a higher risk of infection when compare with internal jugular catheters. The femoral position carries the highest risk of infection, which is a signicant cause of mortality in patients with temporary access catheters (E). Femoral lines may compromise a future kiney transplant because it may lea to proximal iliac vein stenosis/thrombosis. Reference: [No authors liste]. NKF-K/DOQI clinical practice guielines for vascular access. Am J Kidney Dis. 006;48(Suppl. 1): S7–S409.
8. A. Clinical signs of cariac tamponae inclue hypo-
tension, istene neck veins, an mufe or istant heart souns (Beck tria). This patient exhibits two of these signs after an invasive proceure of the chest an likely evelope cariac tamponae as a result of perforation of the right atrium. Tamponae cause by central venous catheter placement is a known complication resulting from puncture by the wire, introucer, or the catheter itself. Perforation of the right atrium more often occurs because it has a thinner wall compare to the right ventricle (E). Placing the catheter tip at the right tracheobronchial angle helps avoi placing the catheter tip in the right atrium. A perforate subclavian artery or vein woul likely lea to hemothorax rather than pericarial tamponae (B, C). A tension pneumothorax is a known complication of line placement an may result in hypotension an istene neck veins, but breath souns woul not be clear bilaterally (D). References: Barton JJ, Vanecko R, Gross M. Perforation of right atrium an resultant cariac tamponae: a complication of catheterization to measure central venous pressure. Obstet Gynecol. 1968;3(4):556–560. Darling JC, Newell SJ, Mohamee O, Uzun O, Cullinane CJ, Dear PR. Central venous catheter tip in the right atrium: a risk factor for neonatal cariac tamponae. J Perinatol. 001;1(7):461–464. Hunt R, Hunter TB. Cariac tamponae an eath from perforation of the right atrium by a central venous catheter. AJR Am J Roentgenol. 1988;151(6):150.
9. A. AVF can eventually unergo aneurysmal egeneration
over time, an intervention is require to prevent rupture an exsanguination (E). High outow resistance is a common cause of aneurysm formation an must be rule out by a stulogram. Repeate neele cannulation can cause stenosis, resulting in higher pressures istal to the lesion an subsequent aneurysm formation. Alternatively, repeate neele cannulation can also lea to aneurysmal egeneration of the vein at the stick site. Therefore, cannulation must be avoie in areas unergoing aneurysmal change. A stulogram is iagnostic of the stenotic lesion an potentially therapeutic via venoplasty with or without stent placement. Further, the stulogram will also help istinguish between a true an
pseuo aneurysm. If no lesion is seen on the stulogram, a central venogram shoul be performe to rule out a central stenosis as a cause of high outow pressures. After treatment of the venous stenoses, bleeing may resolve because the abnormally high pressures within the stula return to normal. Thinne/atrophic skin, translucent skin, ulceration, suspecte infection, intraluminal thrombus, high-output cariac failure, steal synrome, or spontaneous bleeing from the stula prompts consieration for revision by resection an plication or reanastomosis with a healthy vein (B). The size of the aneurysm is not an inicator for revision. If no healthy vein is available, graft implantation is an option (C). If outow cannot be salvage, the access may require ligation (D). Reference: Cronenwett JL, Wayne Johnston K. Rutherford's vascular surgery. 7th e. New York, NY: Sauners/Elsevier; 010.
10. C. Fistulas are superior to grafts, which are superior to
catheters in terms of patient survival, mainly because of the infection risks of prosthetic material (A–E). Diabetics have an exaggerate increase in mortality ue to their epresse immune systems. Interestingly, espite the risk of high-output cariac failure associate with stula an graft, patients with tunnele catheters also have the highest risk of cariac-relate mortality. When comparing patency, stulas are known to have higher primary patency (intervention-free patency of 85% at 1 year, 50% at 5 years) compare to grafts (60% at 1 year, 10% at 5 years) (B). However, stulas have a higher rate of primary failure (nonmaturation or early thrombosis) of up to 40%. Furthermore, when comparing seconary patency (patency with interventions to maintain or reestablish ow), stulas an grafts are similar. Grafts o not require maturation because their lumen iameter oes not change (D). However, healing time of at least 10 ays must be observe after graft placement before cannulation to avoi massive pseuoaneurysm formation. Fistulas require at least 6 weeks for maturation, uring which time the outow vein unergoes remoeling seconary to increase ow resulting in an increase in iameter an further increase in ow. Fistulas are eeme mature if they meet the rule of sixes: at 6 weeks, they must be 6 mm in iameter, less than 6 mm from skin surface, support 600 mL/min ow (although a minimum of 400 mL/ min is aequate), an have a 6-inch straight segment for use. References: [No authors liste]. NKF-K/DOQI clinical practice guielines for vascular access. Am J Kidney Dis. 006;48(Suppl. 1): S7–S409. Lok CE, Sontrop JM, Tomlinson G, et al. Cumulative patency of contemporary stulas versus grafts (000–010). Clin J Am Soc Nephrol. 013;8(5):810–818.
11. E. Intubate patients with an air embolus may have
an abrupt increase in en-tial CO followe by a ecrease in en-tial CO an hypotension, an auscultation may reveal a “millwheel” murmur. This is often escribe as a lou churning soun. An Austin-Flint murmur is associate with aortic insufciency an is a mi-iastolic rumble hear best at the apex (A). Carey Coombs murmur is also a mi-iastolic rumble that is associate with rheumatic fever (B). Means-Lerman scratch murmur souns similar to a pericarial rub an may be hear in patients with hyperthyroiism (C). Still murmur is associate with a small ventral septal efect an is escribe as a vibratory systolic ejection rumble (D).
Transplant JOSEPH HADAYA, AREG GRIGORIAN, AND CHRISTIAN DE VIRGILIO
20
ABSITE 99th Percentile High-Yields I. Type of Transplant Rejection A. Hyperacute: occurs in minutes to hours after transplantation (type- hypersensitivity) 1. Due to the presence of preforme or natural antiboies against major bloo group (ABO) or HLA antigen (sensitization from prior transplants, pregnancy, transfusions) . Complement an coagulation cascae is activate causing graft thrombosis an ischemia 3. Requires prompt removal of transplante organ 4. Kiney, heart, pancreas, an lung allografts all are susceptible to hyperacute rejection; however, liver grafts resist this process, so ABO compatibility is not essential for liver transplantation B. Acute: occurs in ays to months 1. Cause by cellular (macrophages an T-lymphocytes) or humoral (antiboy-meiate) response an typically requires biopsy for iagnosis . Treate with immunosuppressants, sterois, antithymocyte globulin C. Chronic: occurs in months to years (major cause of graft failure an mortality) 1. Cause by cellular (cytotoxic T-lymphocyte, helper T cell) an antiboy-meiate reactions . Graual process resulting in brosis an progressive graft ysfunction 3. Treate by increasing immunosuppression, though usually requires retransplantation II. Liver Transplantation A. Inications: en-stage liver isease seconary to alcohol (most common, replace hepatitis C), nonalcoholic steatohepatitis, primary hepatic malignancy, cholangiocarcinoma, fulminant hepatic failure; most common inication in peiatrics: biliary atresia 1. Milan criteria for hepatocellular carcinoma: single tumor less than 5 cm OR 3 tumors less than 3 cm each B. Moel for en-stage liver isease (MELD): utilizes creatinine, INR, bilirubin, an soium 1. Originally evelope for risk assessment for TIPS, now accepte as the score to prioritize organ allocation for liver transplantation . Score is 0–40; MELD > 15 is inication for liver transplant 3. Consiere a superior scoring system to Chils-Pugh because all parameters are objective C. Operative consierations: for living onors, right lobe is utilize in aults, left lateral lobe utilize in chilren (size comparable) D. Postoperative complications 1. Infection is the most common cause of eath, accounting for over 50% of mortality . Bile leak: most common complication, 10% an 30% incience, may present with abominal pain, bilious rainage, fever; early an large-volume leaks manage surgically, late or small volume leaks, or strictures, can be manage by biliary stenting or rainage 3. Hepatic artery thrombosis: most common vascular complication, high mortality an graft loss rate (>50%) 283
284
PArt i Patient Care
a) Early thrombosis presents with transaminitis, hepatic failure, bile leak (ue to breakown of biliary anastomosis), or primary nonfunction b) Late thrombosis presents as biliary stricture an/or abscesses c) Doppler ultrasoun rst-line, may be conrme with angiography, CT scan, or surgical exploration ) If ientie early (15 mm long) E. Management: 1. Start with resuscitation! Normal saline bolus, maintenance IV uis at 1.5× maintenance rate, a KCl to maintenance uis once patient urinates . Goal is for patient to be resuscitate with correction of electrolyte abnormalities prior to surgery; preop lab goals: pH 100, glucose >7; increase risk of postoperative apnea if uncorrecte alkalosis prior to surgery 3. Surgery is Freet-Ramstet pyloromyotomy: partial-thickness longituinal incision in the pylorus 1 to mm proximal to the uoenum, exten proximally to normal antrum; recurrence most commonly a result of not extening myotomy far enough proximally to antrum
305
306
PArt i Patient Care
III. Malrotation an Migut Volvulus A. Presentation is typically bilious emesis ± abominal istention B. Diagnosis is via UGI series (uoenum oes not cross miline) C. If peritonitis is present, then avoi UGI an immeiately go to OR D. La proceure to minimize chance of future volvulus (create complete nonrotation anatomy) 1. De-torse the bowel in a counterclockwise fashion (“turn back the clock”) . Divie La bans (peritoneal attachments of right colon to paracolic gutter) 3. Straighten the uoenum an x the uoenum in the right upper quarant 4. Mobilize colon to patient’s left an x the cecum in the left lower quarant 5. Wien the base of the mesentery (key component) 6. ± Appenectomy IV. Esophageal Atresia an Tracheoesophageal Fistula A. Echocariogram to rule out associate cariac anomalies (right arch in 1%–%) B. Types:
Type
Esophageal atresia
Tracheoesophageal ęstula
Type A
Present
Absent
Type B
Present
Present; ęstula with proximal esophagus
Type C
Present
Present; ęstula with distal esophagus
Type D
Present
Present; ęstula with both proximal and distal esophagus
Type E
Absent
Present, also called H-type
C. Type C most common; treat with Immeiate repair D. Stage repair for Type A 1. Initial g-tube placement an allow patient to grow until two ens are close enough for repair V. Abominal Wall Defects
Factor
Omphalocele
Gastroschisis
Sac
Present
Absent
Location of defect
Central (through umbilicus)
To the right of the umbilicus
Umbilical cord
Inserts into sac
Normal
Defect Size
Large
Small
Contents
Bowel, liver
Bowel, gonads
Bowel
Normal
MaĴed
Malrotation
Present
Present
Small abdomen
Present
Present
GI function
Normal
Prolonged ileus
Associated anomalies
Common (30%–70%)
Unusual (atresia 15%)
Associated syndromes
Beckwith Wiedemann, Trisomy, Cantrell
Not observed
CHAPtEr 22 Pediatric Surgery
Fig. 22.1 Gastroschisis vs. Omphalocele.
Fig. 22.2 The La Proceure.
307
308
PArt i Patient Care
Questions 1. A 9-year-ol boy is seen in the emergency room with a 1-ay history of right lower quarant abominal pain an low-grae fever. On exam, he is focally tener in the right lower quarant. WBC count is 15,000/mcL an US shows a 9-mm noncompressible appenix an an appenicolith. Which of the following is true about this conition? A. If nonoperative management with antibiotics is to be consiere, a CT scan shoul be rst obtaine B. Success of nonoperative management in this patient is anticipate to be very high C. Nonoperative management tens to result in shorter hospital stay as compare to appenectomy D. Appenectomy is preferre in this patient E. Failure of nonoperative management is likely to manifest as peritonitis 2. A 10-year-ol boy is a restraine passenger in a high-spee motor vehicle collision. On arrival to the emergency epartment, his heart rate is 140 beats per minute an his systolic bloo pressure is 80 mmHg. There is an obvious eformity of his left thigh. GCS is 13. Pupils are equal an reactive. Abomen is milly tener to palpation. Focuse assessment with sonography in trauma (FAST) is positive for peritoneal ui. He is aministere 0 mL/kg of crystalloi, an BP remains 80 mmHg. Which of the following is the most appropriate next step? A. CT scan of hea/abomen/pelvis B. Start bloo prouct transfusion an transport to the OR for exploratory laparotomy C. Infuse aitional bolus of isotonic crystalloi D. Infuse lactate ringers E. Infuse 3% hypertonic saline 3. A 6-month-ol girl is brought to the trauma center for evaluation of a hea injury. Parents report that the patient rolle off a be. Which of the following injuries suggest abusive hea trauma? A. Isolate skull fracture B. Hea an neck bruising C. Subural hematoma D. Epiural hematoma E. Cortical contusion
4. A 1-ay-ol ex-7-week premature boy was previously avancing well on enteral fees. He becomes acutely istene. Initial abominal raiographs reveal moerate pneumatosis intestinalis an enteral feeings were hel. Three hours later, a repeat abominal raiograph reveals pneumoperitoneum. The patient is brought emergently to the operating room for laparotomy where three areas of necrotic bowel are encountere along with numerous other areas of patchy ischemia. What is the next best step in management? A. Resection of necrotic bowel only, with primary anastomoses B. Resection of both necrotic an patchy ischemic bowel with primary anastomosis C. Place rains without bowel resection D. Resection of all necrotic an ischemic bowel with primary anastomosis an proximal iverting stoma E. Resection of necrotic bowel only, leave in iscontinuity, secon look in about 48 hours 5. A previously healthy -month-ol girl is brought to the emergency epartment ue to a -hour history of intermittent inconsolable crying, vomiting, an apparent pain. She is not eating. Her parents brought her to the hospital after she passe a loose, maroon-colore stool. There are no signs of peritonitis on exam. WBC count is normal. Which of the following is recommene? A. CT scan of the abomen B. Laparoscopy C. Colonoscopy D. Nuclear scan E. Abominal ultrasoun 6. A 13-year-ol female presents with severe right lower quarant pain an emesis. At laparoscopy an ovarian torsion is foun. The ovary appears swollen with a blueish-black iscoloration. It remains unchange after etorsion. The next step in management is: A. Biopsy B. Oophoropexy C. Oophorectomy D. Salpingo-oophorectomy E. Close an obtain serial ultrasoun
CHAPtEr 22 Pediatric Surgery
7. A full-term baby girl has a iagnosis of a rightsie congenital lung malformation ientie on prenatal imaging. Chest raiograph in the newborn nursery shows a cystic lesion in the right lower lobe with no meiastinal shift. She is asymptomatic an on room air. What is the next step in management? A. CT scan of the chest prior to ischarge B. Discharge with CT angiogram of the chest within 6 months C. Right lower lobectomy D. Right tube thoracostomy E. Inpatient MRI of the chest 8. During laparoscopy for early acute appenicitis in a 5-year-ol boy, you n a large, right-sie renal mass. You perform an appenectomy an: A. Close, then obtain further workup B. Biopsy the mass C. Right nephroureterectomy D. Right nephroureterectomy with ipsilateral lymph noe sampling E. Right nephroureterectomy with ipsilateral lymph noe sampling an contralateral renal biopsy 9. A -week-ol boy presents with constipation an abominal bloating. He faile to pass meconium on the rst ays of life. Contrast enema emonstrates a slightly ilate sigmoi colon with a constricte rectum. What is the next most appropriate step in management? A. Rectal irrigations an IV antibiotics B. Creation of a leveling ostomy C. Suction rectal biopsy D. Change to an elemental formula E. Obtain a UGI contrast series with small bowel follow-through 10. A newborn is in severe respiratory istress an has a markely scaphoi abomen. Which of the following is true regaring this conition? A. A chest tube shoul be promptly place B. The patient shoul be ventilate with bag-mask ventilation C. Severe cases may benet from extracorporeal membrane oxygenation D. Ventilation with high-frequency oscillation is contrainicate E. Urgent thoracotomy is require
309
11. A full-term, healthy newborn boy is note to have imperforate anus. After 4 hours, no meconium is visualize in the perineal area. The most appropriate management shoul be: A. Observation for another 4 hours B. Diverting ileostomy C. Sigmoi colostomy D. Primary repair through the perineum E. Laparoscopic primary repair 12. A -month-ol infant has persistent jaunice. Ultrasonography fails to emonstrate a gallblaer. Technetium-99m hepatobiliary iminoiacetic aci (HIDA) scanning with phenobarbital pretreatment reveals uptake in the liver but not in the intestine. α1-Antitrypsin an cystic brosis etermination is normal. The most appropriate surgical management woul be: A. Kasai operation (hepatoportoenterostomy) B. Liver transplantation C. Percutaneous transhepatic liver rainage D. Enoscopic biliary stent placement E. Choleochojejunostomy 13. A 1-ay-ol full-term infant presents with bilious emesis. Abominal x-rays show multiple loops of ilate bowel. A contrast enema shows a microcolon. What is the pathophysiology behin this obstruction? A. A fetal mesenteric vascular accient B. Failure of recanalization of the bowel C. Lack of proper rotation of the bowel D. Lack of ganglion cells in the bowel E. A uplication of a segment of bowel 14. A newborn baby is born with an abominal wall efect. The efect involves the umbilicus an has a membrane associate with it. Which of the following is true regaring this type of efect? A. This patient requires immeiate surgical closure B. Mortality is most often the result of persistent sepsis C. The etiology is ue to an umbilical vein vascular accient D. The efect is usually associate with intestinal atresia E. These patients commonly have associate cariac an genetic abnormalities 15. The most common inication for extracorporeal membranous oxygenation (ECMO) in neonates is: A. Congenital iaphragmatic hernia B. Respiratory istress synrome C. Meconium aspiration D. Persistent pulmonary hypertension E. Congenital cariac abnormalities
310
PArt i Patient Care
16. Which of the following is true regaring Bochalek type of congenital iaphragmatic hernia (CDH)? A. Urgent surgical repair is inicate upon iagnosis B. Associate pulmonary hypoplasia leas to hypocarbia C. Most efects are on the right D. Pulmonary hypertension is a prominent feature E. The iaphragmatic efect is anteromeial
21. Operative management for a patient with malrotation an migut volvulus typically inclues reuction of the volvulus, ivision of La bans, an which of the following? A. Placement of the small intestine in the left lower quarant B. Cecopexy an gastropexy C. Broaen base of the small bowel mesentery D. Placement of the cecum in the right upper quarant E. Reconstruction of the ligament of Treitz
17. A full-term baby is born with rooling, coughing, an cyanosis after the rst feeing, but these resolve quickly an spontaneously. The next step in management shoul be: A. Immeiate intubation B. Placement of orogastric tube C. Two-view abominal x-ray D. Two-view chest x-ray E. Upper gastrointestinal (UGI) contrast series
22. A full-term baby boy is note to have facial features of trisomy 1 an bilious emesis. The rest of his exam is normal. Abominal x-rays show a ouble-bubble sign with no istal gas. Which of the following is the best next step in management? A. Serial abominal x-rays B. UGI contrast stuy C. Contrast enema D. Operative exploration E. Echocariogram
18. A patient is iagnose with pyloric stenosis after 3 ays of nonbilious emesis. This patient’s electrolyte an aci/base balance will result in: A. Respiratory alkalosis B. Hyperkalemia C. Aciuria D. Hyperchloremia E. Hyponatremia 19. A 900-g premature infant evelops formula intolerance with vomiting, abominal istention, an blooy stools. Labs show an elevate white bloo cell (WBC) count an platelets of 100,000/ mcL. Abominal x-rays show ilate loops of bowel with pneumatosis intestinalis. The most appropriate treatment woul be: A. Bloo an platelet transfusions B. Antibiotics an bowel rest/ecompression C. Ultrasoun an paracentesis D. Placement of a besie peritoneal rain E. Exploratory laparotomy 20. A healthy -week-ol girl evelops bilious emesis. On exam, her abomen is nontener an nonistene. What is the most appropriate stuy to make the iagnosis? A. -view abominal x-ray B. Ultrasoun C. UGI series D. Contrast enema E. Compute tomography (CT) scan of abomen/pelvis
23. A -year-ol chil presents with an abominal mass, “raccoon eyes,” an “blueberry mufn” skin lesions. These most likely represent: A. Rhabomyosarcoma B. Neuroblastoma C. Wilms tumor D. Hepatoblastoma E. Teratoma 24. The most common anomaly associate with gastroschisis is: A. Cariac B. Renal C. Limb D. Malrotation E. Down synrome 25. A newborn baby is born with a istene abomen an bilious emesis. Both parents are carriers for cystic brosis. On examination, the patient has a istene but soft abomen. Abominal x-rays show ilate loops of bowel with a groun-glass appearance. The most appropriate initial management is: A. Water-soluble contrast enemas B. Resection of terminal ileum with stoma C. Resection of terminal ileum with primary anastomosis D. UGI with small bowel follow-through E. Small bowel enterotomy with evacuation of meconium
CHAPtEr 22 Pediatric Surgery
311
26. A 6-month-ol boy presents to the ED crying in pain an has bilious emesis. On exam, he has a istene abomen, an there is a tener mass in the right groin. Appropriate management woul be: A. Ultrasoun of right groin B. Besie incision an rainage (I&D) of right groin C. IV antibiotics D. Attempt reuction E. Operative exploration
30. A newborn baby with a prenatal iagnosis of gastroschisis is born with the entire small intestine outsie of the abomen. The bowel appears ischemic an the abominal wall efect is small an tight. The most appropriate next step in management is: A. Place a besie silo B. Primary reuction an closure C. Open the abominal wall efect D. Resect the ischemic bowel E. Create a iverting ileostomy
27. A 4-year-ol girl presents with recurrent jaunice. Ultrasoun shows a 5-cm fusiform ilation of the common bile uct. During surgery, the posterior aspect of the cystic mass is rmly aherent to the portal vein. The most appropriate management is: A. Abort surgery, IV antibiotics, an reoperate in 3 months B. Place a rain into the cyst, IV antibiotics, an reoperate in 3 months C. Resect the anterior cyst, mucosectomy of the posterior cyst with reconstruction D. Internal rainage of the cyst with a Roux-en-Y cystojejunostomy E. Resect the cyst an portal vein with reconstruction of the portal vein an common bile uct (CBD)
31. A 1-week-ol full-term baby with abominal istention, fever, tachycaria, an low urine output is transferre to the NICU. The patient has not passe meconium. He ha a suction rectal biopsy showing aganglionosis. Digital rectal examination shows explosive, foul-smelling liqui stools. Despite broa-spectrum IV antibiotics an rectal irrigation, he is clinically eteriorating. The next step in management is to: A. Perform contrast enema B. Perform loop colostomy C. Perform subtotal colectomy an ileostomy D. Perform abominal ecompression for abominal compartment synrome E. A aitional antifungal coverage
28. A -week-ol, ex-5-week premature boy is in the neonatal ICU (NICU) an is iagnose with a left inguinal hernia. His current weight is 1 kg an he requires supplemental oxygen. The hernia is easily reucible. The next appropriate step in management is: A. Ultrasoun evaluation B. Immeiate open operative repair C. Immeiate laparoscopic repair D. Repair just prior to ischarge E. Delay repair until 1 year of age 29. The pathogenesis of necrotizing enterocolitis (NEC) is thought to be relate to: A. A genetic preisposition B. An enzyme eciency C. A perio of intestinal hypoperfusion D. Preexisting intestinal atresia E. An antibiotic reaction
32. A 4-week-ol infant presents with bilious vomiting, irritability, abominal wall eema, an erythema. Plain lms reveal proximal ilate bowel, with a paucity of istal bowel gas. Which is true regaring this patient? A. An urgent UGI series is inicate B. A trial of nasogastric tube ecompression is often helpful C. Enoscopic ecompression is often benecial D. A CT scan of the abomen an pelvis shoul be obtaine E. Delay in management may lea to a nee for intestinal transplantation 33. A neonate is foun to have bilateral unescene testes that are not palpable in the inguinal canal. Which of the following is true regaring this conition? A. A bilateral orchiopexy shoul be performe by 1 year of age B. Orchiopexy oes not improve fertility potential C. It is not associate with prune belly synrome D. Chorionic gonaotropin oes not ai in testicular escent E. The testicular arteries must be preserve uring operation
312
PArt i Patient Care
Answers 1. D. Appenectomy is the preferre treatment strategy
for this patient because of the presence of a fecolith. While appenectomy has been the gol stanar for the treatment of uncomplicate appenicitis, multiple stuies have emonstrate that nonoperative management of uncomplicate appenicitis is safe an effective. Nonoperative management consists of initial broa-spectrum IV antibiotics an IV uis. Patients can be transitione to oral antibiotics an ischarge when their pain improves, fever resolves, an they are able to tolerate a iet. The total antibiotic course shoul be 7 ays. Nonoperative management is initially successful in 85% to 9% of patients. Patients manage nonoperatively have a higher reamission rate within 1 year, primarily ue to recurrent appenicitis. Due to recurrence of appenicitis, the 1-year success rate of avoiing appenectomy is 67%. Most woul recommen only offering nonoperative management in chilren age 5 to 17 years meeting the following criteria: uncomplicate appenicitis conrme on imaging (US, CT, or MRI) (A), WBC between 5000 an 18,000, pain for 40-years, systolic bloo pressure < 90 mmHg, abnormal motor posturing . External ventricular rain (EVD) is preferre as it is iagnostic an therapeutic D. Approach to management of elevate ICP 1. In orer of intervention to be attempte: hea of be to 30 egrees, seation, hypertonic saline or mannitol (contrainicate if systemic hypotension), short-term mil hyperventilation (PaCO2 30–35 mmHg), ventricular rainage, barbiturates, paralysis, an ecompressive craniectomy . Hypertonic saline an/or mannitol shoul both be given as boluses an not continuous infusions as they will equilibrate an thus become ineffective; the goal of these interventions is to create an acute osmotic isequilibrium, which can only be achieve with a bolus E. Nutrition: start enteric feeing within 4 to 48 hours, postpyloric preferre F. Venous thromboembolism (VTE) prophylaxis: very high risk of VTE in TBI 1. Brain Trauma Founation guielines (016) leave the timing an choice of agent to the clinician’s jugment; however, most start low-molecular weight heparin (LMWH) 48 hours after the last stable CT G. Anticoagulation reversal agents
Drug
Reversal
Half-life (hrs)
Warfarin
Prothrombin complex concentrate (preferred), vitamin-K
20–60
Dabigatran
Idarucizumab
10–20
Edoxaban
Andexanet alpfa
10–15
Rivaroxaban
Andexanet alpfa
5–10
Apixaban
Andexanet alpfa
10–12
357
358
PArt i Patient Care
II. Spinal Cor Injuries
Syndrome
Epidemiology
Aěected spinal tracts
Clinical presentation
Prognosis
Central cord
Most common incomplete spinal cord injury syndrome, commonly in elderly with cervical spondylosis and spinal stenosis, hyperextension injury
Bilateral central corticospinal and lateral spinothalamic tracts
Motor deęcits in upper extremities more than lower extremities
Good prognosis although full functional recovery is rare
Anterior cord
Infarction of anterior spinal artery or trauma (e.g., penetrating trauma, burst fracture of vertebra, Ěexion injury)
Corticospinal and spinothalamic tracts
Motor loss, pain, and temperature loss (proprioception and vibratory sense preserved)
Worst prognosis of incomplete syndrome; low chance (10%–20%) of motor recovery
Posterior cord
Very rare, caused by infarction of posterior spinal artery, trauma (e.g., penetrating trauma), multiple sclerosis
Posterior columns
Loss of proprioception, light touch, vibratory sense (motor, pain, and temperature sensation preserved)
Recovery variable and related to completeness of lesion
BrownSéquard
Most commonly due to penetrating injury
Hemisection of the cord
Ipsilateral motor and proprioception loss, contralateral pain and temperature loss
Best prognosis for functional motor activity recovery (99% ambulate)
Questions 1. Which of the following is true regaring the management of severe traumatic brain injury (TBI) in aults? A. A CT scan is require prior to placement of an intracranial monitoring evice B. External ventricular rains (EVD) are preferre over intraparenchymal intracranial pressure monitors if both are available C. The goal cerebral perfusion pressure (CPP) is greater than 40 mmHg D. Decompressive craniectomy oes not lower mortality in cases of refractory intracranial hypertension as compare to meical management E. Heparin is the preferre agent for VTE chemoprophylaxis
2. Which of the following is true regaring gunshot wouns to the hea? A. Suicie attempts have the same mortality rate as assaults or accients B. The incience of vascular injury is low C. Extene antibiotic prophylaxis is recommene D. Bihemispheric injuries are a signicant risk factor for mortality E. GCS on arrival is not a signicant preictor of mortality 3. Which of the following is true regaring primary brain tumors? A. Meulloblastomas are the most common malignant tumors in aults B. Aults with glioblastoma have a 5-year survival rate of aroun 30% C. Corticosterois are use for symptomatic peritumoral vasogenic eema D. Brain tumors in infants typically present with focal neurologic ecits E. In chilren over the age of 10, infratentorial tumors are more common than supratentorial
CHAPtEr 27 Nervous System
4. A 6-year-ol intubate male is opening his eyes to voice an attempts to open his mouth. His only consistent motor movement is to occasionally withraw from painful stimuli. What is his current GCS score? A. GCS 4T B. GCS 8T C. GCS 9T D. GCS 11T E. GCS 13T 5. An 88-year-ol female is brought by ambulance to the ED after being struck by a vehicle while crossing the street. She is only responsive to painful stimuli an is promptly intubate for airway protection. Her seconary exam reveals only a small abrasion to the left forehea. Her systolic bloo pressure suenly increases to the 00s, an her left pupil becomes ilate an unresponsive to light. What is the next best course of action? A. Hypertension control with nicaripine continuous infusion B. Placement of intraparenchymal intracranial pressure monitor C. Immeiate mannitol bolus D. Rectal lorazepam an initiation of levetiracetam E. Raise hea of be 6. A 17-year-ol boy presents to the ED via ambulance after new-onset seizure activity that starte 30 minutes ago. He is unable to provie a goo history because of wor ning issues but is able to convey that his hea hurts. His parents state that he felt completely normal until about 4 weeks ago when he began to complain of left ear pain. Vital signs reveal a mil tachycaria an high fever. Physical exam shows absent light reex in the left eye an papilleema. Which of the following is contrainicate in the workup an subsequent treatment of his conition? A. Lumbar puncture B. Compute tomography with intravenous contrast C. Stereotactic neele aspiration D. Surgical ebriement E. Corticosterois
359
7. Which of the following is true regaring Cushing tria? A. The pulse pressure narrows B. The heart rate increases C. It oes not lea to changes on electrocariogram D. It is associate with hypocarbia E. It is a late manifestation of increase intracranial pressure 8. Which of the following is true regaring rupture intracranial aneurysms? A. Following repair, ui restriction is recommene B. Most arise from the posterior circulation C. The initial stuy of choice is a contrastenhance hea CT D. Following repair, the risk of cerebral vasospasm causing stroke persists for 3 weeks E. Outcomes are overall quite favorable 9. An 85-year-ol female presents to the ED after falling an striking her chin on the kitchen counter. She is unable to lift her arms or hans off the be an oes not respon to painful stimuli. However, she is able to wiggle her toes an seems to feel pain at her feet. She has a history of cervical raiculopathy. A igital rectal exam reveals goo sphincter tone an squeeze pressure. What is the most likely incomplete spinal cor injury that she has sustaine? A. Posterior cor synrome B. Anterior cor synrome C. Caua equina synrome D. Brown-Séquar synrome E. Central cor synrome 10. Which of the following is true regaring hea trauma an/or intracranial hemorrhage? A. The most common cause of subarachnoi hemorrhage is rupture of a berry aneurysm B. Epiural hematoma is typically associate with acceleration-eceleration injuries C. A single episoe of systolic bloo pressure (BP) less than 90 mmHg oubles the mortality rate in patients with hea trauma D. Xanthochromia is virtually pathognomonic for acute subural hemorrhage E. In the absence of other nings, reimaging for cerebral contusion is generally unnecessary
360
PArt i Patient Care
11. A 5-year-ol male is being evaluate in the emergency epartment (ED) after sustaining a blow to the hea with an unknown object uring an assault. He has a 6 cm, stellate laceration with an unerlying scalp hematoma. Compute tomography (CT) scan shows evience of a skull fracture. In which of the following situations can this patient be manage nonoperatively? A. Fracture penetrates ura but not brain B. 0.5 cm of skull epression C. Involvement of the frontal sinus only D. Pneumocephalus E. Gross woun contamination 12. A 45-year-ol female arrives at the ED after iving hea-rst into a half-empty swimming pool. She is combative an appears intoxicate. She is not able to move her lower extremities or trunk. You observe her lifting her arms an bening at the elbows but are unable to assess any movement in her hans. It has been 30minutes since she rst sustaine her injury. Which of the following is true regaring this patient? A. The likely site of her injury is C3-C4 B. In the absence of other injuries, methylprenisolone shoul be aministere immeiately C. This is a rare spinal cor injury after a iving accient D. Anticoagulation shoul be starte within to 3 ays an continue for to 3 months E. Mean arterial pressure shoul be maintaine between 65 an 75 mmHg for the rst 7 ays
13. Neurogenic thoracic outlet synrome most commonly affects which nerve? A. Raial B. Ulnar C. Meian D. Musculocutaneous E. Axillary 14. A 4-ay-ol female infant weighing 1400 g born at 8 weeks’ gestation is being monitore in the neonatal critical care unit because of multiple episoes of apnea an ifculty with feeing. Supplemental oxygen has been sufcient to maintain saturations. Over the last several hours, she has ha waxing an waning alertness an ecrease spontaneous eye movements. Her fontanelle appears to be full. Which of the following is the most appropriate next step? A. Immeiate aministration of furosemie an acetazolamie B. Besie intracranial ultrasoun C. Lumbar puncture D. Noncontrast CT of hea E. Aminister IV steroi bolus
Answers 1. B. One of the rst ecision points in managing a
patient with severe TBI is the placement of an intracranial pressure (ICP) monitor. ICP monitors are inicate in patients with a CT scan showing intracranial hemorrhage an who have a GCS of less than 8 (or higher than 8 but with a high risk of progression). Aitionally, ICP monitors are also inicate in patients with a low GCS who are having emergent extracranial surgery (A). A CT scan is not neee in this scenario. Though EVDs an intraparenchymal pressure monitors can both be use to measure ICP, EVDs are preferre as they are both iagnostic an therapeutic (B). Once an ICP monitor is place, CPP can be calculate with CPP = mean arterial pressure (MAP)−ICP. The goal CPP is greater than 60 mmHg in aults (C). However, the goal CPP is >40 mmHg for peiatrics patients.
All efforts shoul be mae to maintain an aequate CPP with techniques incluing seation, ventricular rainage, mannitol, hypertonic saline, an paralytics. If intracranial hypertension persists espite these measures, ecompressive craniectomy is often utilize, though there is still some controversy regaring its outcomes. In a ranomize controlle trial in 016, ecompressive craniectomy for refractory intracranial hypertension resulte in lower mortality compare to meical treatment alone (D). Although Brain Trauma Founation guielines leave the choice of VTE chemoprophylaxis to the clinician’s jugement, a national atabase stuy incluing over 10,000 patients emonstrate LMWH to be associate with reuce mortality an thromboembolic complications, regarless of timing of prophylaxis initiation in severe TBI patients (E).
CHAPtEr 27 Nervous System References: ACS Trauma Quality Improvement Program. Best Practices in the Management of Traumatic Brain Injury. American College of Surgeons, Committee on Trauma; January 015. https:// www.facs.org/-/meia/files/quality-programs/trauma/tqip/ tbi_guielines.ashx. Kolias PJ, Timofeev AG, IS, et al. Trial of ecompressive craniectomy for traumatic intracranial hypertension. N Engl J Med. 016;375(1):1119–1130. Benjamin E, Recinos G, Aiol A, Inaba K, Demetriaes D. Pharmacological thromboembolic prophylaxis in traumatic brain injuries: low molecular weight heparin is superior to unfractionate heparin. Ann Surg. 017;66(3):463–469.
2. D. Gunshot wouns to the hea have a high morbiity
an mortality. In a large meta-analysis, factors preictive of mortality inclue age greater than 40 years, GCS less than 9 on arrival, xe an ilate pupils, ural penetration, bihemispheric injuries, multilobar injuries, tranventricular injuries, an suicie attempts (A, D, E). In fact, suicies ha a six times higher rate of mortality compare to assaults or accients. Another interesting ning in this stuy was that vascular injuries were very common (38%–50%) with intracranial aneurysm, arterial issection, arterial occlusion, an arteriovenous stulas being the most common types in escening orer of incience (B). There is a lack of high-quality evience regaring the management of this type of injury. Though surgery is associate with lower mortality, it is unclear whether this is a result of surgery itself or ue to patient selection. The rate of CNS infection after penetrating TBI is less than 10% an there is no reuction in the risk of infection with prophylactic antibiotics (C). However, surgical intervention an ICP monitoring appear to be risk factors for infection, regarless of prophylactic use. References: Maragkos GA, Papavassiliou E, Stippler M, Filippiis AS. Civilian gunshot wouns to the hea: prognostic factors affecting mortality: meta-analysis of 1774 patients. J Neurotrauma. 018;35():605–614. Harmon LA, Haase DJ, Kufera JA, et al. Infection after penetrating brain injury-An Eastern Association for the Surgery of Trauma multicenter stuy oral presentation at the 3n annual meeting of the Eastern Association for the Surgery of Trauma, January 15–19, 019, in Austin, Texas. J Trauma Acute Care Surg. 019;87(1):61–67.
3. C. The types an presentations of brain tumors are signi-
cantly ifferent in chilren an aults. In aults, the majority of tumors are benign, with meningiomas being the most common. The most common malignant tumor is glioblastoma, which carries a 5-year survival rate of 5% (A, B). The management is typically focuse on maximal resection an is sometimes followe by raiation. Other consierations are seizure management an corticosteroi use for symptomatic peritumoral vasogenic eema (C). In chilren, brain tumors are relatively more common an are the most common cause of eath among chilhoo cancers. In chilren up to 14 years ol the most common brain tumor is a glioma, but pituitary tumors are the most common in chilren 15 years an oler. The most common malignant brain tumor in chilren is a meulloblastoma. The location of brain tumors in chilren also varies by age, with chilren age 4 to 10 years ol being more likely to have infratentorial tumors. All other ages are more likely to have supratentorial tumors (E). Supratentorial tumors ten to present with focal neurologic ecits epening on the exact location, an infratentorial tumors ten to
361
have cranial nerve palsies or cerebellar ysfunction. The caveat to this rule is in infants (who will not noticeably isplay these ecits) who more commonly present with macrocephaly, irritability, failure to thrive, loss of evelopmental milestones, an vomiting (D). References: Lapointe S, Perry A, Butowski NA. Primary brain tumours in aults. Lancet. 018;39(10145):43–446. Uaka YT, Packer RJ. Peiatric brain tumors. Neurol Clin. 018;36(3):533–556.
4. B. The Glasgow Coma Scale uses the combine scores
from the motor, verbal, an speech sections to give an estimate of a patient’s level of functional status. The scoring is as follows. For eye opening: 4: Spontaneously, 3: To verbal comman, : To pain, 1: No response. Best motor response scores: 6: Obeys comman, 5: Localizes pain, 4: Flexion withrawal, 3: Flexion abnormal (ecorticate), : Extension (ecerebrate), 1: No response, an for Best verbal response: 5: Oriente an converses, 4: Disoriente an converses, 3: Inappropriate wors; cries, : Incomprehensible souns, 1: No response. If the patient is intubate, the maximum score that he or she can get in the verbal category is 1T (the letter T inicating intubate) an maximum overall score of 11T. This patient opens his eyes to voice commans but not spontaneously, which correlates with an eye score of 3. The best calculate motor score is a 4 for withrawing from pain. This places his total GCS at 1T (verbal) + 3 (eye opening) + 4 (motor) = 8T.
5. C. Without a CT scan, one cannot be sure of the exact eti-
ology of these neurologic nings, but, base on the history an physical exam nings, this likely represents a close hea injury with an elevate intracranial pressure (ICP). A “blown” pupil in the setting of hea trauma is consistent with uncal herniation, which is often fatal an will cause permanent neurologic ecits if not treate promptly. Systolic bloo pressure greater than 180 mmHg can aggravate vasogenic brain eema an intracranial hypertension. However, systemic hypertension may be a physiologic response to reuce cerebral perfusion. Thus, early an aggressive treatment of hypertension shoul be avoie until ICP monitoring has been establishe (A). While this patient likely nees an ICP monitor, a iagnosis still nees to be mae before surgical treatment or invasive monitoring (B). Aitionally, an external ventricular rain is a better choice in this patient because it allows therapeutic rainage of cerebrospinal ui. Current inications for a mannitol bolus are for situations just like the above—a quick bailout maneuver to be use as a brige to more enitive therapies. Mannitol immeiately improves cerebral perfusion ue to the fact that it ecreases bloo viscosity an therefore increases cerebral bloo ow an cerebral oxygen elivery. Its osmotic properties take 15 to 30 minutes to work. There is some evience that prolonge or scheule use will rener it ineffective at best an potentially harmful. Immeiately following mannitol, the patient nees a CT scan an shoul be evaluate for possible surgical rainage of an intracranial hematoma. Lorazepam an levetiracetam (Keppra) are both meications use for the treatment of seizures, which is not consistent with her exam at this time (D). Raising the hea of the be can lower ICP, but with a blown pupil, the patient nees more aggressive treatment (E).
362
PArt i Patient Care
Reference: Brain Trauma Founation, American Association of Neurological Surgeons, Congress of Neurological Surgeons. Guielines for the management of severe traumatic brain injury. J Neurotrauma. 007;4 Suppl 1:S91–S95.
6. A. The tria of heaache, focal neurologic ecits, an
fevers shoul raise concern for brain abscess; however, this classic presentation is present in less than half of all patients. The most common presenting symptom is a heaache, which is present in approximately 70% of patients. They arise primarily by two forms of sprea: hematogenously from istant sites an irect sprea from contiguous sites of infection (otitis meia being most common). This leas to a wie array of potential pathogens, though the most common are Streptococcus spp. an Staphylococcus spp. Initial iagnosis shoul be obtaine by CT scan with contrast, which will show a rim-enhancing collection (B). Lumbar puncture is generally not iagnostic an contrainicate in the setting of elevate ICP. Changes in cerebrospinal ui volume in this setting can precipitate herniation. All patients shoul be starte on broa-spectrum antibiotics, which can be tailore once cultures are obtaine. Total uration of treatment is typically 4 to 6 weeks. Traitional management inclue surgical rainage an excision of the abscess cavity (D). However, serial neele aspiration has now become the treatment of choice unless the abscess is traumatic in origin (potentially has foreign ebris), fungal, multiloculate, or oes not improve with neele aspiration (C). Corticosterois are controversial in this setting but may be consiere when there is substantial mass effect from the abscess (E). References: Brouwer MC, Coutinho JM, van e Beek D. Clinical characteristics an outcome of brain abscess: systematic review an meta-analysis. Neurology. 014;8(9):806–813. Muzumar D, Jhawar S, Goel A. Brain abscess: an overview. Int J Surg. 011;9():136–144.
7. E. Cushing tria is a vasomotor an respiratory response
to an elevate ICP that inclues braycaria, irregular breathing, an elevation in systolic bloo pressure with a wiene pulse pressure (A). The increase ICP leas to impaire respiration, which worsens hypercarbia (D). Typically, Cushing tria is a late sign of elevate ICP an suggests imminent herniation. In aition to braycaria on ECG, Mayer waves can be seen with elevate ICP (B). The waves are cyclic changes in arterial bloo pressure brought about by oscillations in baroreceptor an chemoreceptor reex control systems an are note on ECG (C).
8. D. Intracranial aneurysms affect 4% of the population
but are asymptomatic in the majority of cases, an most patients are unaware of the iagnosis. Risk factors inclue female gener, polycystic kiney isease, an Marfan synrome. The majority of the aneurysms occur in the circle of Willis with the anterior communicating artery being the most frequent site (B). When the aneurysm ruptures, it can result in intraparenchymal an subarachnoi hemorrhage, which is a catastrophic event with a mortality rate up to 50% (E). Noncontrast CT hea is the stuy of choice to conrm the iagnosis (C). Bleeing on brain parenchyma elicits a vasospasm response, which can result in stroke an patients are at increase risk for 1 ays; thus, most neurosurgeons will start calcium channel blockers. Because
cerebral autoregulation is compromise, these patients shoul be given volume to maintain aequate cerebral perfusion pressure (A). Reference: Keey A. An overview of intracranial aneurysms. McGill J Med. 006;9():141–146.
9. E. Central cor synrome is the most common type of
incomplete spinal cor injury an is primarily foun in patients that suffere a hyperextension injury in the setting of previous cervical spine abnormalities. Symptoms inclue muscle weakness of the upper extremities with relative sparing of the lower extremities. Sensory function is variable. Posterior cor synrome is a relatively rare entity typically cause by infarction of the posterior spinal artery. Classic presentation inclues sparing of muscles with the loss of proprioception an vibration sensation below the level of the lesion with preservation of most motor function (A). Anterior cor synrome can be cause by either infarction of the anterior spinal artery or, less frequently, by fracture or islocation of vertebrae. It is characterize by loss of motor function, pain sensation, an temperature sensation but preservation of touch an proprioception (B). Caua equina synrome can be cause by trauma, mass lesions, or lumbar spinal stenosis an occurs at the level that the spinal cor has split into nerve roots. Symptoms can be variable but generally inclue paresthesia of the perineum, anus, an external genitalia (“sale anesthesia”), bilateral or unilateral paralysis, an incontinence of bowel an blaer (C). Brown-Séquar synrome is hemisection of the spinal cor from a mass lesion or more commonly trauma. It causes an ipsilateral loss of motor, proprioception, an vibration sensation with contralateral loss of pain an temperature sensation (D).
10. C. Traumatic brain injuries are among the most com-
mon presenting symptoms in emergency epartments in the Unite States, with over 1.7 million amissions each year. The early recognition an management of brain injury is critical in this patient population because it is consiere the most common cause of trauma-relate eath in patients reaching the hospital alive. Preventing seconary injury is an important part of management, an this involves maintaining cerebral perfusion pressure greater than 60 mmHg. One prospective trial foun that a single episoe of hypotension with a systolic bloo pressure of less than 90 mmHg ouble mortality in patients with brain injury. Trauma is consiere the most common etiology of subarachnoi hemorrhage, followe by rupture of berry aneurysms (A). In nontraumatic cases, patients may report mil “sentinel” heaaches in the prior weeks leaing up to a severe, unrelenting, “thunerclap” heaache. Noncontrast compute tomography (CT) scan is the iagnostic tool of choice to look for hyperensities suggestive of acute bleeing. Aitionally, xanthochromia of cerebrospinal ui is consiere pathognomonic for subarachnoi hemorrhage (D). Epiural hematoma is generally the result of irect trauma to the skull causing isruption of arterial vessels, particularly the mile meningeal artery. It initially presents with unconsciousness from the concussive effects of the injury, followe by a “luci” interval that progresses to somnolence, lethargy, an eventually a coma as the hematoma grows. Noncontrast CT scan will emonstrate a lentiform (biconvex), hyperense clot that oes not
CHAPtEr 27 Nervous System cross suture lines. Acute subural hematoma is generally the result of acceleration-eceleration injuries that tear the briging veins as the brain shifts in relation to the ura (B). Patients are often unconscious from the moment of impact. Noncontrast CT scan will emonstrate a hyperense, lunar (crescent-shape) lesion that oes not cross the miline. Cerebral contusion is ue to the brain irectly striking the skull in either a coup or countercoup mechanism after a close hea injury. Lesions on noncontrast CT scans are typically scattere, hyperense, an intraparenchymal, though they can also present as hypoense lesions. There is a signicant propensity for these lesions to worsen, an repeat imaging is typically recommene in the rst 4 hours (E). References: Chesnut RM, Marshall LF, Klauber MR, et al. The role of seconary brain injury in etermining outcome from severe hea injury. J Trauma. 1993;34():16–. Faul M, Xu L, Wal MM, Coronao VG. Traumatic brain injury in the United States: emergency department visits, hospitalizations and deaths 2002–2006. Centers for Disease Control an Prevention, National Center for Injury Prevention an Control; 010. https://www.cc. gov/traumaticbraininjury/pf/blue_book.pf
11. B. Any skull fracture with an overlying laceration is
consiere an open fracture. Traitional teaching is that all of these patients shoul be taken to the operating room to prevent infection. However, there seems to be a subset of patients that can be treate expectantly without signicant increases in morbiity. Nonoperative management of open skull fracture can be consiere in patients without evience of ural penetration, signicant intracranial hematoma, frontal sinus involvement, woun infection, pneumocephalus, or gross woun contamination (A, C–E). Aitionally, patients with less than 1 cm of skull epression can be manage nonoperatively. Reference: Bullock MR, Chesnut R, Ghajar J, et al. Surgical management of epresse cranial fractures. Neurosurgery. 006;58 (3 Suppl):S56–60.
12. D. Although it is ifcult to ascertain the exact level
of spine injury in a noncooperative patient, complete paralysis of the lower extremities an the trunk with preservation of her shoulers an elbows most likely inicates an injury at C5 or below (A). The most common spinal injury after a iving accient is C5 followe by C6 (C). The use of sterois in spinal cor injury has been controversial. However, recent level 1 evience recommens against the use of sterois in the management of acute spinal cor injury (B). Among trauma victims, patients with spinal cor injury an hea injury have the highest risk of venous thromboembolic events (VTEs). Without prophylaxis, the risk of VTE is about 40% after complete spinal cor injury. Mechanical prophylaxis with compression evices shoul be starte immeiately. Anticoagulation shoul be starte within 7 hours an continue for to 3 months. Low-molecular-weight heparin is preferre over heparin. Mean arterial pressure shoul be maintaine between 85 an 90 mmHg for the rst 7 ays (E).
363
References: Bailes JE, Herman JM, Quigley MR, et al. Diving injuries of the cervical spine. Surg Neurol. 1990;34(3):155–158. Theoore N, et al. Guielines for the management of acute cervical spine an spinal cor injuries: 013 upate. Neurosurgery. 013;7():1–59.
13. B. Neurologic symptoms occur in 95% of cases of tho-
racic outlet synrome. The lower nerve roots of the brachial plexus, C8 an T1, are most commonly (90%) involve, proucing pain an paresthesias in the ulnar nerve istribution (A, C–E). The secon most common anatomic pattern involves the upper three nerve roots of the brachial plexus, C5, C6, an C7, with symptoms referre to the neck, ear, upper chest, upper back, an outer arm in the raial nerve istribution.
14. B. Intraventricular hemorrhage (IVH) occurs in approx-
imately 15% to 0% of infants born with a birth weight of less than 1500 g. Because of the frequency of this conition, serial ultrasoun screening is recommene for all premature infants an any infants that show signs of IVH. In premature infants, the relative fragility of the germinal matrix makes them sensitive to changes in cerebral bloo ow with subsequent hemorrhage into the ventricles. Preisposing factors in aition to prematurity inclue maternal chorioamnionitis or preeclampsia, an neonatal respiratory istress, hypotension, or anemia. While 5% to 50% of infants can have clinically silent IVH, symptoms range from nonspecic changes in alertness to stupor or coma. Once it has been iagnose, management is largely supportive to prevent long-term complications such as posthemorrhagic hyrocephalus (PHH). Prior to the avent of intracranial ultrasoun, CT scan was utilize to make the iagnosis, but has now been largely abanone (D). Once the iagnosis is establishe, treatment is supportive, incluing correction of anemia (patients can suffer major bleeing), hypotension, aciosis, an ventilatory support. Treatments to try to prevent hyrocephalus have been largely ineffective. Though furosemie an acetazolamie have been use in oler chilren with PHH, they o not seem to alter the course in premature infants an coul potentially be eleterious (A). Serial lumbar puncture has been trie with no signicant change in eterioration or progression to permanent ventricular rainage proceures (C, E). Temporary ventricular rainage with transition to permanent rainage proceures if necessary is currently the treatment of choice for PHH with elevate intracranial pressures. Ultimately, if signicant hyrocephalus persists, the infant may nee a ventriculoperitoneal shunt. References: Mazzola CA, Chouhri AF, Auguste KI, et al. Peiatric hyrocephalus: systematic literature review an evience-base guielines. Part : management of posthemorrhagic hyrocephalus in premature infants. J Neurosurg Pediatr. 014;14 Suppl 1:8–3. Robinson S. Neonatal posthemorrhagic hyrocephalus from prematurity: pathophysiology an current treatment concepts: a review. J Neurosurg Pediatr. 01;9(3):4–58.
PART II
MEDICAL KNOWLEDGE
Anesthesia ERIC O. YEATES AND CATHERINE M. KUZA
28
ABSITE 99th Percentile High-Yields I. American Society of Anesthesiologists Physical Status (ASA PS)
ASA PS
Deęnition
Examples
I
Normal healthy patient
Young with no comorbidities
II
Mild systemic disease
Well-controlled hypertension, current smoker
III
Severe systemic disease
Poorly controlled hypertension, morbid obesity, stable angina, prior myocardial infarction, controlled congestive heart failure with no symptoms, end-stage renal disease on scheduled dialysis
IV
Severe systemic disease that is a constant threat to life
Unstable angina, congestive heart failure with symptoms, sepsis, end-stage renal disease not on scheduled dialysis
V
Moribund patient not expected to survive without the operation
Ischemic bowel, intracranial hemorrhage with midline shift, ruptured abdominal aortic aneurysm
VI
Brain-dead patient undergoing organ donation
Traumatic brain-injured patient with no brain stem reĚexes
II. Opioi Equivalents
Opioid
Oral
Parenteral
Morphine
30 mg
10 mg
Hydromorphone
7.5 mg
1.5 mg
Hydrocodone
30 mg
N/A
Fentanyl
N/A
0.1 mg
Codeine
200 mg
N/A
Oxycodone
20 mg
N/A
Tramadol
120 mg
N/A
365
366
PArt ii Medical Knowledge
III. Local Anesthetic Maximum Dose
Medication
Without epinephrine
With epinephrine
Lidocaine
5 mg/kg
7 mg/kg
Bupivacaine/Ropivacaine
2.5 mg/kg
3 mg/kg
0.5% = 5 mg/ml, 1% = 10 mg/ml, 2% = 20 mg/mg
IV. Common Intravenous Inuction Agents
Medication
Mechanism of action
Side eěects
Propofol
GABA agonist
Hypotension, propofol infusion syndrome
Etomidate
Mechanism unclear, modulates or activates GABA
Adrenal suppression, myoclonus
Ketamine
NMDA receptor antagonist
Emergence delirium, hypertension, arrhythmias, increased intracranial pressure
Midazolam
GABA agonist
Nausea, vomiting, delirium
V. Neuromuscular Blocking Drugs for Rapi Sequence Intubation
Medication
Mechanism of action
Onset (min)
Histamine release
Succinylcholine
Depolarizing
1–1.5
Rocuronium
Nondepolarizing
Vecuronium Cisatracurium/ atracurium
Reversal agent
Metabolism
Side eěects
Yes
Plasma cholinesterase
Hyperkalemia, malignant hyperthermia, bradycardia
N/A
1.5–3
Yes
Hepatic, metabolites excreted renally
Allergy
Sugammadex
Nondepolarizing
3–4
No
Hepatic
Allergy
Sugammadex
Nondepolarizing
5–7
No
Hofmann elimination
Bronchospasm
Neostigmine
VI. Steroi Potency: hyrocortisone < prenisone < methylprenisolone < examethasone VII. Malignant Hyperthermia: rare, severe reaction to meications use uring general anesthesia A. Genetics: rare, autosomal ominant isorer cause by a mutation in the ryanoine receptor, locate on the sarcoplasmic reticulum (in skeletal muscle) B. Triggering meications: volatile anesthetics (halothane, sevourane, esurane, isourane, enurane) or epolarizing muscle relaxants (succinylcholine, ecamethonium) C. Signs/symptoms: can occur immeiately an as late as 4 hours postoperatively 1. Hyperthermia, tachycaria, increase en-tial CO, muscle rigiity, rhabomyolysis, lactic aciosis D. Diagnosis: acutely, the iagnosis is clinical 1. Conrmatory testing or testing of close relatives who have suffere from malignant hyperthermia; this inclues a skeletal muscle biopsy followe by a caffeine-halothane contracture test (CHCT); the muscle is expose to halothane an caffeine with a positive test causing signicant muscle contraction; testing must take place in centers specialize in iagnosing malignant hyperthermia E. Treatment: stop all anesthetics, aminister .5 mg/kg of IV antrolene which inhibits calcium ion release from the sarcoplasmic reticulum (can aminister aitional 1–.5 mg/kg boluses, max cumulative ose of 10 mg/kg), cooling, correction of hyperkalemia, an ui resuscitation. F. Outcomes: mortality approximately 5%
CHAPtEr 28 Anesthesia
VIII. Propofol Infusion Synrome: rare synrome triggere by high ose (>4 mg/kg/hr) infusion >48 hours A. Mechanism: unknown, but possibly ue to the impairment of fatty aci metabolism B. Risk factors: chilren, concomitant catecholamine or steroi infusion, severe critical illness C. Signs/symptoms: metabolic aciosis, arrhythmias (most often braycaria), rhabomyolysis, hyperlipiemia, hepatomegaly (not splenomegaly), renal failure, cariovascular collapse D. Treatment: immeiate cessation of propofol, early hemoialysis, supportive care E. Screening tool: aily CPK an lactate levels
IX. Differential Diagnosis of Intraoperative Changes in En-Tial CO
Rising end-tidal CO 2
Dropping end-tidal CO 2
Hypoventilation
Hyperventilation
Rebreathing
Circuit disconnected
Malignant hyperthermia
Inadvertent extubation
Sepsis
Endotracheal tube obstruction
Hyperthyroidism (thyroid storm)
Hypothermia
Pheochromocytoma
Pulmonary embolism
Rhabdomyolysis
Cardiac arrest Reduced cardiac output Tension pneumothorax
367
368
PArt ii Medical Knowledge
QUESTIONS 1. A 35-year-ol man involve in a motorcycle cycle collision sustains a large laceration to his right thigh. The ecision is mae to washout an close the woun at besie with the assistance of proceural seation. The patient has an oral airway in place, is breathing spontaneously, an is maintaining aequate oxygen saturation with a simple face mask. With painful stimulation, he awakens briey an is able to follow simple commans. What level of seation is this patient currently uner? A. Minimal seation B. Moerate seation C. Conscious seation D. Deep seation E. General anesthesia
4. Which of the following is most likely associate with opioi abuse an postsurgical prescribing patterns? A. The majority of opiois abuse in the US originate from international rug cartels B. Heroin users rarely report previously abusing prescription opiois C. 30 pills of 5 mg oxycoone are the recommene amount to be prescribe after laparoscopic cholecystectomy D. New persistent opioi use after surgery is more common after major proceures compare to minor proceures E. Preoperative tobacco use is a signicant risk factor for new persistent opioi use after surgery
2. Which of the following is associate with opioi tolerance? A. Characterize by pronounce cravings an compulsive rug taking B. Decrease analgesic effect of opiois evelops before ecrease effects on respiratory epression C. Increase sleeping an eating, epression, an pupillary constriction D. Constipation resolves over time with longterm opioi use E. Genetic components associate with opioi use have not been ientie
5. A 75-year-ol woman is brought to the operating room for laparoscopic cholecystectomy. She has a history of progressive ementia an is unable to provie a meical history. Fifteen minutes into the operation performe uner general anesthesia, the anesthesiologist reports ifculty ventilating the patient, an she evelops a iffuse maculopapular rash with urticaria. Which of the following is the most likely offening agent? A. Rocuronium B. Latex C. Cefazolin D. Sevourane E. Propofol
3. A 68-year-ol woman is unergoing a laparoscopic liver resection. An arterial line an central line are place prior to surgical incision. As the hepatic parenchyma is being ivie, the anesthesiologist reports suen hypotension an a rop in en-tial CO. There is no break in the ventilatory circuit. There is only minimal bleeing at this time. There are ST changes note on the EKG. Which if the following is the next best step in management of this conition? A. Transthoracic echocariography (TTE) examination of the heart B. Aminister epinephrine C. Aminister ui bolus D. Emergently place a pulmonary artery catheter line E. Release (esufate) pneumoperitoneum
6. A 9-year-ol man unergoes a laparoscopic cholecystectomy for symptomatic cholelithiasis. Shortly after inuction, the anesthesiologist notes an increase in core boy temperature an ential CO. After aministration of antrolene an aborting the operation, his status improves. Which of the following is most likely associate with this iagnosis? A. It is an autosomal recessive isorer B. Genetic analysis is require for iagnostic conrmation C. It is more common in elerly patients D. It may present as late as 4 hours after anesthesia E. Mortality rate is less than 1%
CHAPtEr 28 Anesthesia
7. After excision of multiple subcutaneous lipomas uner local anesthesia, a 4-year-ol woman seizes violently. What is the maximum safe ose of a local anesthetic agent in a 70-kg woman? A. 10 to 0 mL 1% liocaine B. 40 to 50 mL % liocaine with epinephrine C. 40 to 50 mL 1% liocaine with epinephrine D. 40 to 50 mL 0.5% liocaine E. 40 to 50 mL 1% liocaine without epinephrine 8. A 0-year-ol man is about to unergo arthroscopic surgery on his left shouler. During anesthetic inuction with succinylcholine, the anesthesiologist note trismus that persiste for > minutes, an the mouth coul not be opene to perform irect laryngoscopy or place an enotracheal tube. The anesthesiologist was able to bag mask ventilate the patient. The ential CO, heart rate, an temperature remaine normal. Which of the following is the next best step in management? A. Aminister an aitional ose of succinylcholine B. Procee with surgery if the patient can be intubate C. Cancel surgery an sen the patient home D. Cancel surgery, aminister antrolene, an amit for 4-hour observation E. Cancel surgery, amit for 4-hour observation, an refer for muscle biopsy 9. Which of the following is the best immeiate way to conrm placement of an enotracheal tube in the airway after intubation? A. Direct visualization of tube passing through the vocal cors B. Auscultation of lungs C. Observation of conensation within tube D. Pulse oximetry E. Capnography 10. A 65-year-ol man is unergoing urgent surgery for gangrenous cholecystitis. The patient has a history of moerate aortic valve stenosis that was recently iagnose on echocariography but he enies any symptoms. Which of the following woul be most important goal in the anesthetic management? A. Preloa reuction B. Afterloa reuction C. Avoiance of hypotension D. Heart rate goal of >90 beats per minute E. Use of epherine for hypotension
369
11. A 9-year-ol boy has been in the peiatric intensive care unit for the last 7 ays after presenting to the hospital with inuenza infection leaing to respiratory failure requiring mechanical ventilation. He is receiving continuous fentanyl an propofol infusions for pain control an seation, respectively. This morning he evelope braycaria, an his urinary output ecrease. He is note to have hepatomegaly on physical examination. Laboratory values show an elevate creatinine, hyperlipiemia, hyperkalemia, an lactic aciosis. Which of the following is the best next step in management? A. Start bicarbonate infusion B. Perform liver biopsy C. Initiate hemoialysis D. Initiate treatment with low-ose epinephrine E. Discontinue propofol an start exmeetomiine infusion 12. A 55-year-ol iabetic man unerwent a rightsie vieo-assiste thoracoscopic surgery (VATS) for an empyema yesteray. This morning he is complaining of pain along his meial left forearm an has paresthesia of his fourth an fth igits. Which of the following risk factors are most likely associate with this complication? A. Male sex B. Emergency surgery C. Supine positioning uring surgery D. Hyperthermia uring surgery E. Diabetic neuropathy 13. Which of the following is true regaring invasive lines use for the monitoring of surgical patients? A. Trauma patients show improve mortality with placement of a pulmonary artery catheter (PAC) B. A normal Allen Test before raial artery cannulation will reuce incience of han ischemia C. PAC will provie irect measurement of systemic vascular resistance D. Systolic bloo pressure measure on a raial artery catheter will typically be higher than the aortic pressure E. A right bunle branch block seen on electrocariogram is consiere a contrainication for PAC placement
370
PArt ii Medical Knowledge
14. A 47-year-ol woman is recovering from pneumonia complicate by multiorgan system ysfunction. She is currently receiving hemoialysis after eveloping renal failure seconary to sepsis. This morning, a rapi response was calle for respiratory epression an confusion, which improve after the aministration of naloxone. Which of the following meications most likely contribute to her respiratory compromise? A. Fentanyl B. Hyromorphone C. Morphine D. Methaone E. Oxycoone 15. A 37-year-ol woman unerwent a percutaneous besie tracheostomy tube placement. On postoperative ay 1, she evelope signicant subcutaneous emphysema of the neck over the course of an hour, an her current oxygen saturation is 80%. A respiratory therapist attempte irectional suctioning, but they were unable to pass the catheter. What is the most appropriate next step in management? A. Remove the tracheostomy tube an attempt recannulation with a smaller caliber cannula B. Remove the tracheostomy tube an recannulate over a suction catheter C. Remove the tracheostomy tube an recannulate over a beroptic bronchoscope D. Replace the tracheostomy tube using a percutaneous tracheostomy kit E. Bag mask ventilation an prepare for orotracheal intubation 16. Which of the following correctly pairs the invasive mechanical ventilation moe with its mechanism of action? A. Synchronize intermittent mechanical ventilation (SIMV): every breath has a manate volume B. Airway pressure release ventilation (APRV): maintains continuous positive airway pressure (CPAP) with an intermittent release phase C. Assist-control (AC) ventilation: patient etermines the rate an volume of breaths D. CPAP: two ifferent pressure settings for inhalation an exhalation E. High-frequency oscillatory ventilation (HFOV): high respiratory rate with large tial volumes
17. Which of the following parameters is most likely to preict successful iscontinuation of mechanical ventilation? A. Rapi shallow breathing inex (RSBI) (f/VT) less than 105 B. Negative inspiratory force (NIF) −0 to −30 cm HO C. Successful spontaneous breathing trial (SBT) D. Respiratory rate less than 30 breaths per minute E. Tial volume greater than 5 mL/kg 18. A 66-year-ol woman presents in septic shock ue to a perforate uoenal ulcer. She is taken urgently to the operating room for an exploratory laparotomy. Due to persistent hypotension, opamine is infuse by the anesthesiologist an is eventually titrate to a rate of 15 mcg/kg per minute. At that rate, which of the following receptors is exerting the preominant effect? A. α1-Arenergic B. α-Arenergic C. β1-Arenergic D. β-Arenergic E. Dopaminergic 19. A 55-year-ol man with a history of chronic obstructive pulmonary isease (COPD) unergoes an interscalene regional block with bupivacaine for surgery of a left humerus fracture. Soon after placement of the block, the patient evelops signicant yspnea. Breath souns are equal to auscultation an clear. Which of the following factors is the most likely cause of his shortness of breath? A. Pneumothorax B. COPD exacerbation C. Inavertent intravascular injection of bupivacaine D. Air embolism E. An elevate left hemiiaphragm 20. At the en of a surgery, an anticholinesterase is aministere to a patient to reverse the neuromuscular blockae. Which of the following muscles woul be expecte to recover rst? A. Diaphragm B. Auctor pollicis C. Ocular muscles D. Pharyngeal E. Quariceps femoris
CHAPtEr 28 Anesthesia
21. A 40-year-ol man with obesity, hypertension, cirrhosis, iabetes mellitus, an chronic kiney isease (CKD) stage 1 unergoes general anesthesia for repair of an incarcerate inguinal hernia. He takes insulin, echothiopate, amloipine, an simvastatin at home. Propofol an pancuronium are use for inuction. At the en of the proceure, a peripheral nerve stimulator emonstrates no recovery of muscle twitches espite 60 minutes of time elapsing. Which of the following unerlying factors is most likely responsible for this conition? A. Diabetes mellitus B. Obesity C. Stage 1 CKD D. Pancuronium E. Simvastatin
371
23. A patient is given benzocaine spray in anticipation of a besie exible laryngoscopy. After several minutes, he evelops a heaache an shortness of breath. Pulse oximetry shows an SpO of 85%, while an arterial bloo gas shows an SaO of 80% with a PaO of 150 mmHg. Which of the following is the most appropriate treatment? A. Intubation B. Intravenous methylene blue C. Discontinue benzocaine an aminister prilocaine D. Metoclopramie E. Thiosulfate
22. Which of the following is associate with barbiturate toxicity? A. Myocarial epression B. Acute tubular necrosis C. Hepatotoxicity D. Peripheral neuropathy E. Seizures
ANSWERS 1. D. Level of seation is a continuum ene by the
patient's response to the meications aministere. During minimal seation, patients have a normal response to verbal stimulation (A). During moerate seation, patients have purposeful responses to verbal or tactile stimulation (B). While uner eep seation, repeate verbal or painful stimulation is neee to achieve purposeful movements. Intervention on the airway may be require at this level of seation. Uner general anesthesia, the patient is unarousable even with painful stimulus (E). Moerate seation an conscious seation are terms that are often use interchangeably (C). Reference: Practice guielines for moerate proceural seation an analgesia 018: a report by the American Society of Anesthesiologists Task Force on Moerate Proceural Seation an Analgesia, the American Association of Oral an Maxillofacial Surgeons, American College of Raiology, American Dental Association, American Society of Dentist Anesthesiologists, an Society of Interventional Raiology. Anesthesiology. 018;18(3):437–479.
2. B. Long-term opioi use commonly results in tolerance
an physical epenence. Tolerance escribes a ecrease in opioi potency with repeate aministration. Tolerance to analgesic effects of opiois evelops more quickly than
tolerance to respiratory epression, which partially explains the high overose rates. Tolerance within the colon typically oes not evelop an results in chronic constipation (D). Depenance is characterize by the unpleasant response to stopping or reucing intake of the rug, also referre to as withrawal symptoms. Opioi withrawal symptoms inclue lacrimation, piloerection, muscle aches, nausea, vomiting, iarrhea, pupillary ilation, insomnia, tachycaria, hyperreexia, an hypertension (C). Aiction is much less preictable an less common than both tolerance an epenance an is characterize by pronounce cravings, obsessive thinking, compulsive rug taking, an an inability to refrain from use (A). It is also now believe that opioi aiction has a fairly strong genetic component with heritability rates similar to iabetes an hypertension (E). References: Volkow ND, McLellan AT. Opioi abuse in chronic pain–misconceptions an mitigation strategies. N Engl J Med. 016;374(13):153–163. Akbarali HI, Inkisar A, Dewey WL. Site an mechanism of morphine tolerance in the gastrointestinal tract. Neurogastroenterol Mot. 014;6(10):1361–1367.
3. E. Given the unexplaine hypotension an ecrease in en-tial CO, this patient most likely has a CO embolism.
372
PArt ii Medical Knowledge
Clinically signicant CO embolism is very rare uring laparoscopic surgery but has a mortality rate of approximately 8%. CO embolism is thought to be cause by either intravascular injection of CO into a vessel with either a Veress neele or trocar uring initial insufation, or by gas entering an injure vessel later uring the operation. Signs of a CO embolism are unexplaine hypotension, hypoxia, or a suen ecrease in en-tial CO. Transesophageal echocariography (TEE) is the most sensitive metho for etecting CO embolism, though often not necessary when clinical suspicion is high (A). Precorial oppler is the most sensitive noninvasive test. If CO embolism is suspecte, insufation shoul be stoppe an the abomen esufate immeiately. Though historically it has been recommene to place the patient in the left lateral an Trenelenburg position to move the air bubble out of the pulmonary artery, new evience suggests that neither of the above positions results in signicant hemoynamic improvements. Rather, for proceures below the level of the heart, the patient shoul be place in the reverse Trenelenburg position to reuce further air entrainment. Vasopressor aministration an a ui bolus are reasonable interventions for persistent hypotension, but shoul be one after reucing the risk of further air entrapment (B, C). Pulmonary artery catheters have been shown to be ineffective at aspirating air with a success rate between 6% an 16% an shoul not be the next step in management (D). A "mill-wheel" murmur is present in less than half of patients. References: Cottin V, Delafosse B, Viale JP. Gas embolism uring laparoscopy: a report of seven cases in patients with previous abominal surgical history. Surg Endosc. 1996;10():166–169. Mirski M, Lele AV, Fitzsimmons L, et al. Diagnosis an treatment of vascular air embolism. Anesthesiology. 007;106:164–177.
4. E. Opioi abuse has risen substantially in the US in
recent years, prompting research investigating the causes of this new epiemic. Though the majority of opiois abuse in the US originate from legitimate prescriptions, only 0% of opioi users were the intene recipients of the initial prescription (A). The majority of opioi abusers receive pills for free from family members or friens with excessive pills or from other methos of iversion. Opioi abuse can also lea to further illicit rug use, as 50% to 85% of heroin users report having previously abuse prescription opiois (B). As excessive opioi prescriptions appear to be one of the inciting factors in opioi abuse, aitional attention has been place on prescribing patterns after surgery. A large retrospective stuy showe that new persistent opioi use was fairly common after both major an minor surgical proceures, with an incience of aroun 6%. The incience was not signicantly ifferent between major an minor surgeries inicating that pain is not the riving factor for this postsurgical complication (D). Risk factors inepenently associate with new persistent opioi use inclue preoperative tobacco use, alcohol an substance abuse isorers, moo isorers, anxiety, an preoperative pain isorers (E). To aress the overprescribing of opiois after surgery, one stuy ientie the number of pills (equivalent to 5 mg oxycoone) that woul fully supply the nees of 80% of patients unergoing a number of ifferent operations. Examples of these nees inclue 5 pills after a partial mastectomy, 15 pills after a laparoscopic cholecystectomy, an 15 pills after an open inguinal hernia repair (C).
References: Brummett CM, Waljee JF, Goesling J, et al. New persistent opioi use after minor an major surgical proceures in US aults. JAMA Sur. 017;15(6):e170504. Hill MV, McMahon ML, Stucke RS, et al. Wie variation an excessive osage of opioi prescriptions for common general surgical proceures. Ann Surg. 017;65(4):709–714.
5. A. A stuy one in France from 1997 to 004 looke at all patients who ha immeiate hypersensitivity reaction presume to be from allergic reaction. Of the 1816 patients that met criteria for the stuy, the top three offening agents for immeiate hypersensitivity reaction were neuromuscular blocking agents (58%), latex (0%), an antibiotics (13%) (B, C). Allergy to inhale anesthetics an hypnotics was much less common (D, E). In chilren, latex was more common than neuromuscular blocking agents, but the sample size for this population was much lower. References: Butterworth J, Mackey D, Wasnick J, etal., es. Inhalation anesthetics. In: Morgan & Mikhail's clinical anesthesiology. 5th e. McGraw-Hill; 013;44–88. Butterworth J, Mackey D, Wasnick J, etal., es. Intravenous anesthetics. In: Morgan & Mikhail's clinical anesthesiology. 5th e. McGrawHill; 013;141–156. Di Leo E, Delle Donne P, Calogiuri GF, Macchia L, Nettis E. Focus on the agents most frequently responsible for perioperative anaphylaxis. Clin Mol Allergy. 018;16:16. Mertes PM, Alla F, Tréchot P, Auroy Y, Jougla E, Groupe ’Etues es Réactions Anaphylactoïes Peranesthésiques. Anaphylaxis uring anesthesia in France: an 8-year national survey. J Allergy Clin Immunol. 011;18():366–373.
6. D. This patient likely has malignant hyperthermia, a rare
autosomal ominant isorer of skeletal muscle (A). The conition is characterize by a hypermetabolic state triggere by exposure to inhalation anesthetics (sevourane, esurane, isourane) an/or succinylcholine. The oler anesthetic agents associate with this reaction inclue halothane an enurane. It is not cause by nitrous oxie, intravenous anesthetic agents, or other neuromuscular blockers (except for succinylcholine). Malignant hyperthermia occurs when uncontrolle amounts of intracellular calcium accumulate in skeletal muscle. Symptoms may evelop as early as 30 minutes after anesthetic aministration an as late as 4 hours postoperatively. Even after treatment with antrolene, patients nee to be monitore because they can have a refractory response an go back into a malignant hyperthermic crisis. The initial clues occur in the operating room after inuction. Rather than achieving complete paralysis, the anesthesiologist may notice rigiity in the masseter muscle. Other nings inclue an increase in ential CO, tachycaria, an an increase in temperature. It is imperative that all anesthetics are immeiately stoppe an antrolene given (.5 mg/kg every 5 minutes) until resolution of symptoms. Dantrolene stabilizes muscle channels in the sarcoplasmic reticulum. The mortality rate was previously 30%, but recent evience suggests the mortality rate is now approximately 5% (E). A functional test on skeletal muscle biopsy (caffeine halothane contracture test) is use for iagnosis (B). More than 50% of the families show linkage of the invitro contracture test phenotype to the gene encoing the skeletal muscle ryanoine receptor. The test requires a muscle biopsy with exposure of the muscle to halothane an caffeine. A positive test will cause signicant muscle contraction. The majority of cases occur in chilren or young aults (C).
CHAPtEr 28 Anesthesia References: Jurkat-Rott K, McCarthy T, Lehmann-Horn F. Genetics an pathogenesis of malignant hyperthermia. Muscle Nerve. 000;23(1):4–17. Ellinas H, Albrecht MA. Malignant hyperthermia upate. Anesthesiol Clin. 00;38(1):165–181.
7. C. There are relatively few sie effects of local anes-
thetic agents such as liocaine, unless they are inavertently injecte intravenously or aministere in oses higher than recommene. Toxicity begins with neurologic signs an symptoms such as light-heaeness, facial paresthesias, blurre vision, an tinnitus. It can progress to lethargy, tremors, an tonic-clonic seizures. Neurologic symptoms precee the more severe cariovascular symptoms, which inclue hypertension an tachycaria (early symptoms) an later hypotension, cariovascular collapse, braycaria or conuction abnormalities, an even cariac arrest. The maximum oses for local injection of liocaine are 5 mg/kg without epinephrine an 7 mg/kg with epinephrine because the vasoconstriction elays the systemic release of liocaine. Because a 1% solution of liocaine contains 10 mg/mL, an easy way to remember this is to multiply the patient's weight by either 5 (no epinephrine) or 7 (with epinephrine) an then ivie by 10. Therefore, for this patient: 70 kg × 5 mg/kg = 350 mg an ivie by 10 mg/mL = 35 mL of 1% liocaine. For liocaine with epinephrine, 70 kg × 7 mg/kg = 490 mg an ivie by 10 = 49 mL of 1% liocaine. For a % liocaine solution, one woul ivie by 0 (4.5 mL an 17.5 mL, respectively, with an without epinephrine), an for a 0.5% solution, one woul ivie by 5 (70 mL an 98 mL, respectively, with an without epinephrine). Patients who experience local anesthetic systemic toxicity (LAST) shoul be treate by iscontinuing the local anesthetic, aministering uis, support with 100% FiO, hyperventilation, an aministering 0% intralipi with a bolus of 1 to 1.5 mL/kg over one minute. The bolus can be repeate every 3 minutes up to a total ose of 3 mL/kg, followe by an infusion of 0.5 mL/kg/min which is continue until the patient is hemoynamically stable for at least 10 minutes. CPR an epinephrine shoul be use in cariac arrest, an bicarbonate shoul be use in aciosis. Benzoiazepines are preferre over propofol to manage seizures. References: Warren JA, Thoma RB, Georgescu A, Shah SJ. Intravenous lipi infusion in the successful resuscitation of local anesthetic-inuce cariovascular collapse after supraclavicular brachial plexus block. Anesth Analg. 008;106(5):1578–1580. Neal JM, Mulroy MF, Weinberg GL, American Society of Regional Anesthesia an Pain Meicine. American Society of Regional Anesthesia an Pain Meicine checklist for managing local anesthetic systemic toxicity: 01 version. Reg Anesth Pain Med. 01;37(1):16–18. Cao D, Hear K, Foran M, Koyfman A. Intravenous lipi emulsion in the emergency epartment: a systematic review of recent literature. J Emerg Med. 015;48(3):387–97.
8. E. Masseter muscle rigiity, or trismus, is consiere a
normal reaction to the aministration of neuromuscular blocking agents. However, if this conition persists for more than 0 to 30 secons, it is consiere an abnormal response, an the clinician nees to have a high level of concern for malignant hyperthermia, an nonemergent surgeries shoul be cancele (B). Persistent trismus is not a sign of inaequate neuromuscular blockae, an thus aitional neuromuscular blocker aministration is not inicate (A). Masseter spasm is an early inicator of susceptibility to malignant hyperthermia. Other markers for malignant hyperthermia
373
inclue fevers, increase en-tial CO, generalize muscle rigiity, autonomic instability, an rhabomyolysis. The incience of patients who evelop masseter spasms an go on to evelop malignant hyperthermia is unknown. It shoul be note that isolate masseter spasm is not pathognomonic for malignant hyperthermia. The surgery shoul be cancele an the patient amitte for at least 4 hours of observation to watch for the evelopment of rhabomyolysis or malignant hyperthermia; the patient shoul not be sent home prior to 4 hours of observation an monitoring in the hospital (C). In the absence of hemoynamic instability, elevate CO, or fever, it is unnecessary to aminister antrolene (D). However, these patients shoul be referre to a center that can perform the necessary testing, incluing genetic testing an a caffeine halothane contracture test (muscle biopsy test). After muscle biopsy, the tissue is only viable for several hours, so testing must take place in centers specialize in iagnosing malignant hyperthermia (E). References: Schneierbanger D, Johannsen S, Roewer N, Schuster F. Management of malignant hyperthermia: iagnosis an treatment. Ther Clin Risk Manag. 014;10:355–36. Bauer SJ, Orio K, Aams BD. Succinylcholine inuce masseter spasm uring rapi sequence intubation may require a surgical airway: case report. Emerg Med J. 005;:456–458. Sheikh MM, Riaz A, Umair HM, Waqar M, Muneeb A. Succinylcholine-inuce masseter muscle rigiity successfully manage with propofol an laryngeal mask airway: a case report an brief review. Cureus. 00;1(7):e9376.
9. E. Although irect visualization of the tube passing
through the vocal cors, auscultation of the lungs, visualization of conensation within the tube, an pulse oximetry are goo ajuncts to conrm initial placement of the enotracheal tube, interpretation is subjective an not as accurate as more objective methos for conrming the position of the enotracheal tube within the trachea (A–C). Both the American College of Emergency Physicians an the American Society of Anesthesiologists recommen capnography or en-tial CO etection evices as the preferre conrmatory test for tracheal intubation (E). Patients shoul have a continuous uniform waveform of en-tial CO with similar amplitues to conrm tracheal intubation; however, this oes not ifferentiate a tracheal from a bronchial intubation. A capnographic waveform that shows en-tial CO etection but oes not have a continuous waveform or the amplitues get smaller an smaller until no aitional en-tial CO can be etecte is inicative of an esophageal intubation. Direct visualization of the tube passing through the cors is not always reliable, as the cors can be misientie or the tube can be isloge from the trachea before it is secure (A). Auscultation of the lungs is not always reliable because it is possible to get referre souns from the stomach (B). Conensation within the tube can occur even with esophageal intubation (C). Pulse oximetry is also not reliable, as hypoxia with esophageal intubation can be very elaye if the patient is preoxygenate well (D). References: American Society of Anesthesiologists Task Force on Management of the Difcult Airway. Practice guielines for management of the ifcult airway: an upate report by the American Society of Anesthesiologists Task Force on Management of the Difcult Airway. Anesthesiology. 003;98(5):169–77. Grmec S. Comparison of three ifferent methos to conrm tracheal tube placement in emergency intubation. Intensive Care Med. 00;8(6):701–704.
374
PArt ii Medical Knowledge
10. C. While asymptomatic aortic stenosis is not a contra-
inication to surgery, it requires careful intraoperative monitoring. The increase pressures require to overcome the stenosis cause concentric hypertrophy of the left ventricle, which in turn reuces the compliance of the ventricle. This makes these patients heavily preloa epenent for ventricular lling, an careful attention shoul be pai to maintaining aequate intravascular volume (A). In aition, up to 40% of the left ventricular en-iastolic volume (LVEDV) is provie by the atrial kick. Atrial arrhythmias can quickly lea to heart failure an shoul be aggressively treate, preferably with ebrillation. Braycaria (8 French) are require for aequate rainage D. Treatment of the organizing phase requires open rainage (e.g., Eloesser ap) E. If it progresses to an empyema, vieo-assiste thoracoscopic surgery shoul be performe if it oes not respon to chest tube rainage 13. Which of the following is true regaring antibiotic mechanisms? A. Penicillin-erivative antibiotics bin to the bacterial cell membrane an increase its permeability B. Piperacillin-tazobactam works partly by bining β-lactamases C. Metroniazole, though limite, has some effect against aerobic bacteria D. Linezoli competitively inhibits the 30S ribosome E. Clinamycin, like the macrolies, reversibly bins the 50S ribosome 14. Which of the following shoul be use as part of a screening tool to ientify non-ICU patients that are at increase risk of organ failure from infection? A. Temperature higher than 38°C B. Heart rate greater than 90/min C. Altere mentation D. White bloo cell (WBC) count greater than 1,000/mm3 E. PaCO less than 3 mmHg
CHAPtEr 31 Infection and Antimicrobial Therapy
15. A 56-year-ol HIV-positive (with a low CD4 count) patient presente to the ED with a spontaneous pneumothorax an unerwent a tube thoracostomy proceure. While trying to re-cap the 0-gauge neele use for anesthetizing the skin, the resient who performe the proceure was inavertently stuck resulting in visible bleeing from the skin. Which of the following is true regaring this exposure? A. Postexposure prophylaxis with a -rug regimen shoul be aministere for 8 weeks B. Postexposure prophylaxis with a 3-rug regimen shoul be aministere for 4 weeks C. Potential HIV infection shoul be isclose to future patients D. At least 6 months of postexposure treatment is recommene E. The hollow bore neele use for this proceure lowers the risk of HIV transmission 16. A 45-year-ol HIV-positive male presents to the ED with perianal pain for the past two ays. Physical exam reveals a small area of tenerness in the right posterolateral position istal to the external sphincter that is extremely tener. His CD4 count is 550 cells/mL, an he is currently on highly active antiretroviral therapy (HAART). Which of the following is the most correct management of this patient? A. Intravenous (IV) antibiotics B. Incision an rainage uner local anesthesia in the ED C. Oral antibiotics an incision an rainage uner local anesthesia in the ED D. IV antibiotics, exam uner anesthesia (EUA), an if an area of uctuance is ientie, then incision an rainage an biopsy E. IV antibiotics, EUA, incision an rainage, an biopsy of the area of tenerness even if no uctuance is ientie
401
17. A 6-year-ol man is postoperative ay 6 from an elective laparoscopic sigmoi colectomy for recurrent iverticulitis. He ha return of bowel function ays ago an was getting reay to be ischarge home. Throughout his hospital course, he has been having low-grae fevers. He is now complaining of tenesmus an urinary retention. Which of the following represents the most appropriate next step in management? A. Transition to nonnarcotic pain meications B. Blaer scan an in-an-out catheterization as neee C. Abominal raiography D. Compute tomography (CT) E. Diagnostic laparoscopy 18. A 60-year-ol man presents with gas gangrene of his left leg requiring below-knee amputation. Woun cultures were positive for Clostridium septicum. Aitional workup shoul inclue: A. Hea CT scan B. Bronchoscopy C. Colonoscopy D. HIV serology E. Chest CT scan 19. Which of the following is true regaring tetanus? A. It is highly contagious B. Trismus is usually the rst sign C. It is cause by a gram-negative anaerobic ro D. A prior history of surviving tetanus provies immunity E. The iagnosis is establishe by emonstrating the organisms in a woun 20. Which of the following is associate with an enotoxin? A. Streptococcus pyogenes B. Bacteroides fragilis C. Clostridium tetani D. S. aureus E. C. perfringens
402
PArt ii Medical Knowledge
Answers 1. B. The Stuy to Optimize Peritoneal Infection Therapy
infections. Their most recent recommenations aress timing of antibiotics, choice of antibiotics, hair removal techniques, normothermia, euglycemia, an inwelling urinary catheter use. Prophylactic antibiotics shoul be given within 1 hour prior to the incision (A). Aitionally, the chosen antibiotic shoul cover the most likely pathogen to be encountere uring the operation. Cefazolin oes not have appropriate anaerobic coverage for a colectomy (B). Surgical site hair shoul be remove with electrical clippers, rather than shaving, on the ay of or ay prior to surgery. Shaving has been shown to have a higher rate of meiastinitis in patients unergoing open-heart surgery (C). Inwelling urinary catheter use longer than ays after an operation is associate with higher rates of urinary tract infection an it is recommene to remove catheters prior to this point when possible (D). Maintaining euglycemia for the rst 48 hours postoperatively has been shown to ecrease surgical site infection. Aitionally, intravenous insulin infusion postoperatively was associate with ecrease eep sternal woun infections in open cariac surgery compare to sliing-scale insulin injections. References: Furnary AP, Zerr KJ, Grunkemeier GL, Starr A. Con-
short-course antimicrobial therapy for intraabominal infection. N Engl J Med. 015;37(1):1996–005.
tinuous intravenous insulin infusion reuces the incience of eep sternal woun infection in iabetic patients after cariac surgical proceures. Ann Thorac Surg. 1999;67():35–360. Rosenberger LH, Politano AD, Sawyer RG. The surgical care improvement project an prevention of post-operative infection, incluing surgical site infection. Surg Infect (Larchmt). 011;1(3):163–168.
(STOP-IT) trial was a ranomize controlle trial esigne to etermine the optimal length of antibiotic treatment after source control in patients with intraabominal infections. Patients were ranomize to receive antibiotics for 4 ays postoperatively versus ays after the resolution of fever, leukocytosis, an ileus. The meian uration of antibiotics was 4 ays versus 8 ays postoperatively, an there was no ifference in surgical-site infections, recurrent intraabominal infections, or mortality. Therefore, antibiotics shoul continue only for 4 ays postoperatively in most cases (A, C–E). Procalcitonin may be useful in helping etermine the uration of antibiotic therapy. A single absolute value is less useful than the tren over several ays. It is best use in cases where systemic inammation is present without an obvious infectious etiology. If the procalcitonin level eclines with antibiotic therapy, it is reasonable to complete a 5 to 10-ay course epening on the suspecte source(s). However, if the level is normal an/or oes not change with antibiotics, it woul be reasonable to stop antibiotic therapy. Reference: Sawyer RG, Clarige JA, Nathens AB, et al. Trial of
2. E. This patient is showing multiple signs of sepsis with
the most likely source being a catheter-associate urinary tract infection. The Surviving Sepsis Campaign is a joint collaboration committe to reucing morbiity an mortality relate to sepsis an septic shock. Upate recommenations etailing the ieal management of sepsis an septic shock are provie perioically. Flui resuscitation shoul begin as soon as possible with 30 cc/kg of crystalloi (C). Albumin (colloi) bolus has not been consistently emonstrate to be associate with improve outcomes in patients with septic shock an is signicantly more costly than crystalloi. As such, crystallois are preferre over collois (B). Empiric broa-spectrum intravenous antibiotics shoul also be aministere within one hour. However, appropriate cultures shoul be obtaine prior to starting antibiotics if this incurs no substantial elay. If hypotension persists espite ui resuscitation, vasopressors shoul be initiate with a MAP goal of >65 mmHg (A). Aitionally, intravenous hyrocortisone at 00 mg/ay shoul be consiere if shock is refractory to both ui resuscitation an vasopressors (C). This patient has an elevate lactate which shoul be measure at regular intervals an utilize as an enpoint for aequate resuscitation. Though he oes have a signicant aciemia, aministration of soium bicarbonate is not recommene for correction as long as his pH is greater than 7.15 an certainly shoul not occur prior to aequate ui resuscitation (D). Reference: Rhoes A, Evans LE, Alhazzani W. Surviving Sepsis Campaign: international guielines for management of sepsis an septic shock. Crit Care Med. 017;45(3):486–55.
3. E. The Surgical Care Improvement Project (SCIP) is a pro-
gram esigne to reuce the rates of postoperative surgical
4. E. Aspergillus species are wiely isperse in the envi-
ronment an, when implicate as a pathogen, primarily affect the lungs. It typically presents as one of four synromes: aspergilloma, allergic bronchopulmonary aspergillosis (ABPA), chronic necrotizing Aspergillus pneumonia, an invasive aspergillosis. Aspergilloma typically presents as an asymptomatic raiographic ning in patients with a preexisting cavitary lung isease such as sarcoiosis. A soft-tissue mass within a cavity that is surroune by a crescent of air (Mona sign) is iagnostic, an because the aspergilloma is not aherent to the cavity walls, the air will remain in a nonepenent position. Biopsy or bronchoscopy is not inicate or necessary for iagnosis (A, B). As long as the patient is asymptomatic, no further workup or treatment is necessary. The most common symptom associate with aspergilloma is hemoptysis, which can occasionally be life threatening. In this setting, an emergency bronchial artery embolization shoul be performe followe by surgical resection (D). ABPA is a noninvasive hypersensitivity isease that, if left untreate, can lea to brotic lung isease. Therapy is aime at the treatment of acute exacerbations either with inhale bronchoilators/sterois (mil isease) or systemic corticosterois (severe isease) to prevent long-term sequelae. Serial chest raiographs, pulmonary function tests, an IgE levels shoul be monitore because permanent pulmonary amage can take place even in asymptomatic patients. Invasive aspergillosis an chronic necrotizing Aspergillus pneumonia are both treate with intravenous antifungals (C). Invasive
CHAPtEr 31 Infection and Antimicrobial Therapy isease can be rapily fatal an is typically only foun in immunocompromise hosts. High-risk transplant patients, such as bone marrow recipients, receive prophylactic agents to prevent invasive infection. Reference: Limper AH, Knox KS, Sarosi GA, et al. An ofcial American Thoracic Society statement: treatment of fungal infections in ault pulmonary an critical care patients. Am J Respir Crit Care Med. 011;183(1):96–18.
5. E. Hepatitis B surface antigen is foun on the surface
of the hepatitis B virus an is foun in high quantities in the serum of iniviuals with acute or chronic infection. Antiboies against this antigen (anti-HBs) are consiere to represent an immunity to the virus either from previous infection or vaccination. All patients with chronic hepatitis B infection will be anti-HBs negative. Antiboies against hepatitis core antigen (anti-HBc) appear at the onset of symptoms an persist for life, though they o not confer immunity to the isease. Vaccination will not prouce antiboies to hepatitis B core antigen. Presence of these antiboies inicates either active or previous infection with hepatitis B but oes not confer a timeline associate with that infection. However, IgM against hepatitis B core antigen is only present for the rst 6 months of infection, so its presence inicates a recent exposure to the virus. The aforementione serologic proles represent: A, susceptible to infection; B, immunity from previous infection; C, immunity from vaccination; D, acute infection; E, chronic infection.
6. E. The risk of eveloping hepatitis B from a neelestick
injury is far greater than that of hepatitis C, particularly when the patient is hepatitis Be surface antigen (HBesAg) positive (A). If the patient's bloo is both HBeAg an HBsAg positive, the risk of eveloping clinical hepatitis is very high (%–31%). If the bloo is HBsAg positive but HBeAg negative, the risk rops to 1% to 6% (although seroconversion is still high at 3%–37%). Hepatitis B is highly infectious, an the virus can survive on rie bloo an on environmental surfaces for at least a week. The majority of health-care workers infecte with hepatitis B o not recall a neelestick exposure, though they were in contact with a hepatitis B–positive patient (D). For health-care workers who have never been vaccinate for hepatitis B, or are seronegative, treatment with both HBIG (immunoglobulin prepare from human plasma known to contain a high titer of antiboy to HBsAg) an the hepatitis B vaccine is recommene (B). Data on clinical hepatitis C following exposure is lacking. However, the average incience of anti-HCV seroconversion from an HCV-positive source is very low (only 1.8%), suggesting that the risk of transmission from a neelestick injury is very low. In fact, some stuies suggest that the risk of hepatitis C transmission from a soli surgical neele is negligible. No effective prophylaxis for HCV has been ientie. Immunoglobulin an antiviral agents are not recommene for HCV postexposure prophylaxis (C). Reference: Kuhar DT, Henerson DK, Struble KA, et al. Updated U.S. public health service guidelines for the management of occupational exposures to HIV and recommendations for postexposure prophylaxis. Division of Healthcare Quality Promotion, National Center for Emerging an Zoonotic Infectious Diseases, Center for Disease Control an Prevention (CDC); 013. https://stacks.cc.gov/view/ cc/0711.
403
7. B. NSTI is a broa term that encompasses infections lim-
ite to skin an subcutaneous tissue (necrotizing cellulitis) an those involving the fascia (necrotizing fasciitis) an muscle (myonecrosis). They can be extremely ifcult to accurately iagnose early on because fewer than half present with obvious har signs of NSTI, such as bullae, skin necrosis, gas on raiograph, an crepitus. Other signs inclue tense eema, violaceous skin color, severe pain, an neurologic ecit. Several laboratory values have been shown to be useful in istinguishing NSTI from simple cellulitis. The LRINEC (Laboratory Risk Inicator for Necrotizing Fasciitis) score uses the total WBC count, hemoglobin, soium, glucose, serum creatinine, an C-reactive protein levels. A simpler moel uses an amission WBC count greater than 15.4 × 109/L an/or a serum soium level less than 135 mEq/L. This latter moel is more useful for its negative preictive value (99%). A low serum soium level is theorize to be the result of either a sepsis-inuce synrome of inappropriate antiiuretic hormone or arenal insufciency, but this has not been conrme. Risk factors for NSTI inclue iabetes, illicit IV rug abuse, immunosuppression, an liver isease. Seventy percent to 80% of NSTIs are ue to polymicrobial infection. Of those that are cause by a single organism, Klebsiella, S. pyogenes, an C. perfringens are the most common. The NSTI is subivie into two categories; type I infections are cause by polymicrobial infection with aerobic an anaerobic bacteria (e.g., Clostridium an Bacteroides spp.), which work synergistically to prouce infection. Type II infections are cause by group A Streptococcus with or without Staphylococcus. Treatment inclues rapi aministration of broa-spectrum antimicrobial agents, aggressive ui resuscitation, an aggressive surgical ebriement. The mortality rate remains at 0% to 40% an is higher with surgical elays, particularly beyon 4 hours. A rising WBC count an lactate after ebriement are highly suggestive of progression of the NSTI. A secon-look operation is often require an shoul be performe for this patient in orer to ensure that no aitional tissues have become involve since the initial ebriement. Amputation may be necessary, but only a secon-look operation will inicate whether this is the case (A). CT scan in the postoperative setting may not be useful because interpretation can be ifcult seconary to postsurgical changes (D). With septic shock, pressors may be necessary, but this woul not be the enitive treatment (E). Aitionally, no hemoynamic parameters (bloo pressure, central venous pressure) are provie that woul inicate that pressors are neee. Similarly, aing antifungal coverage can be consiere, but this is not a enitive intervention (C). References: Anaya DA, Dellinger EP. Surgical infections an choice of antibiotics. In: Townsen CM, Jr, Beauchamp RD, Evers BM, Mattox KL, es. Sabiston textbook of surgery: the biological basis of modern surgical practice. 17th e. W.B. Sauners; 004:57–8. Dunn DL, Beilman GJ. Surgical infections. In: Brunicari FC, Anersen DK, Billiar TR, etal., es. Schwartz's principles of surgery. 8th e. McGraw-Hill; 005:109–18. Wall DB, Klein SR, Black S, e Virgilio C. A simple moel to help istinguish necrotizing fasciitis from nonnecrotizing soft tissue infection. J Am Coll Surg. 000;191(3):7–31. Wong CH, Khin LW, Heng KS, Tan KC, Low CO. The LRINEC (Laboratory Risk Inicator for Necrotizing Fasciitis) score: a tool for istinguishing necrotizing fasciitis from other soft tissue infections. Crit Care Med. 004;3(7):1535–1541.
404
PArt ii Medical Knowledge
Yaghoubian A, e Virgilio C, Dauphine C, Lewis RJ, Lin M. Use of amission serum lactate an soium levels to preict mortality in necrotizing soft-tissue infections. Arch Surg. 007;14(9):840–846.
8. E. Risk for surgical site infections is relate to several
factors, incluing microbial contamination uring surgery, length of operation, an patient factors such as iabetes, nutritional state, obesity, an immunosuppression (cancer, renal failure, immunosuppressive rugs) (B–D). The National Nosocomial Infection Surveillance Risk Inex is a useful tool to assess the risk of woun infection. This inex inclues (1) American Society of Anesthesiologists physical status score higher than , () class III or IV wouns, an (3) uration of an operation greater than the 75th percentile for that particular proceure (A). Wouns are classie as clean (class I) (e.g., hernia repair, breast biopsy), clean/ contaminate (class II) (e.g., cholecystectomy, elective gastrointestinal surgery), contaminate (class III) (e.g., bowel injury from trauma or inavertent enterotomy), an irty (class IV) (e.g., perforate appenicitis, iverticulitis, necrotizing soft-tissue infections [NSTIs]). Hemoglobin levels have not been shown to increase the risk of woun infection. In a ranomize stuy of patients unergoing colorectal surgery, surgical woun infections were foun in 19% who were permitte to become hypothermic but in only 6% who were actively kept normothermic. In a ranomize stuy of clean surgery (breast, varicose vein, hernia), those who were actively warme 30 minutes before surgery ha only a 5% woun infection rate versus 14% in nonwarme patients. Active control of glucose via continuous infusion was shown to ecrease sternal woun infection in iabetic patients unergoing cariac surgery. The main concern with aggressive glucose control, however, is that it may incite episoes of hypoglycemia. A recent stuy also highlighte the risk of bloo transfusion in woun infection, likely the result of its immunosuppressive effects. References: Campbell DA Jr, Henerson WG, Englesbe MJ, et al. Surgical site infection prevention: the importance of operative uration an bloo transfusion–results of the rst American College of Surgeons-National Surgical Quality Improvement Program Best Practices Initiative. J Am Coll Surg. 008;07(6):810–80. Furnary AP, Zerr KJ, Grunkemeier GL, Starr A. Continuous intravenous insulin infusion reuces the incience of eep sternal woun infection in iabetic patients after cariac surgical proceures. Ann Thorac Surg. 1999;67():35–360. Kurz A, Sessler DI, Lenhart R. Perioperative normothermia to reuce the incience of surgical-woun infection an shorten hospitalization: stuy of Woun Infection an Temperature Group. N Engl J Med. 1996;334(19):109–115. Melling AC, Ali B, Scott EM, Leaper DJ. Effects of preoperative warming on the incience of woun infection after clean surgery: a ranomise controlle trial. Lancet. 001;358(985):876–880.
9. D. Acute mesenteric aenitis presents most commonly in
chilren an young aults. It can frequently be confuse with appenicitis in chilren. Usually, an upper respiratory infection is present or has recently resolve. The abominal pain is usually iffuse, but involuntary guaring on exam is rare. Laboratory values are of little help in establishing the iagnosis. More than 50% have an elevate WBC count. Although infection with the other answer choices can lea to mesenteric lymphaenitis, Y. enterocolitica is the most commonly associate organism in chilren (A–C, E). If the iagnosis
is clear preoperatively (which is usually not the case), treatment is supportive because it is a self-limite isease. Ultrasoun has emerge as a useful tool in chilren to suggest this iagnosis. Finings inclue enlarge, hypoechoic mesenteric lymph noes (at least one more than 8 mm in iameter) an the absence of an iname (ilate) appenix. The iagnosis can also be mae with CT by the emonstration of enlarge, clustere mesenteric lymph noes in the right lower quarant in the absence of acute appenicitis, but there is increasing reluctance to expose chilren to the raiation associate with CT scanning. The iagnosis is sometimes mae uring laparoscopy.
10. D. A rare cause of infection in the rst 48 hours after an operation is woun toxic shock synrome. Toxic shock synrome is an acute onset, multiorgan illness that resembles severe scarlet fever. It was originally escribe in menstruating women in association with tampon use, but it has been increasingly recognize in postsurgical wouns. In the majority of cases, the illness is cause by S. aureus strains that express toxic shock synrome toxin-1, enterotoxin B, or enterotoxin C. It has rarely been escribe in association with S. pyogenes (group A streptococci) (C). The remaining answer choices are not associate with toxic shock synrome (A, B, E). Half of the postsurgical toxic shock synrome cases present early, within 48 hours of operation. Symptoms inclue fever, iarrhea, vomiting, iffuse reness of the skin, an hypotension. This is followe a ay or two later by iffuse esquamation. Physical examination nings of woun infection are often unremarkable. Woun rainage an antibiotics are recommene. Aministration of clinamycin may be helpful because it inhibits exotoxin prouction. Reference: Reingol AL, Dan BB, Shans KN, Broome CV. Toxic-shock synrome not associate with menstruation. A review of 54 cases. Lancet. 198;1(86):1–4.
11. E. Lung abscesses typically present with an inolent
course over several weeks. Patients often complain of fevers, purulent sputum, an cough. Single lung abscesses are frequently monomicrobial an are usually associate with aspiration pneumonia. As such, they are typically foun in segments of the lung that are epenent in the supine position (i.e., the posterior segment of the upper lobes or the superior segments of the lower lobes). An air-ui level on a chest raiograph an purulent sputum are virtually iagnostic of an anaerobic lung infection. However, coinfection with antibiotic-resistant gram-positive organisms is possible in patients with frequent hospitalizations. Most lung abscesses will resolve with antibiotics alone, but aptomycin cannot be use to treat lung infections because it is inhibite by pulmonary surfactant (A). In aition to intravenous (IV) antibiotics, a patient with risk factors for lung cancer (e.g., smoking, recent weight loss) shoul unergo bronchoscopy to rule out an unerlying neoplasm (obstruction leaing to infectious process). Surgical treatment may be necessary for infections that fail to respon to meical management, abscesses greater than 6 cm in size, an abscesses seconary to an obstructe bronchus from a foreign boy or neoplasm. This typically involves either lobectomy or pneumonectomy (C). Percutaneous rain placement can be consiere in patients who are poor surgical caniates (D). Thoracotomy
CHAPtEr 31 Infection and Antimicrobial Therapy an ecortication are treatment options for empyema, not lung abscess (B). References: Manal K. Thoracic infections. In: Yuh DD, Vricella LA, Yang SC, Doty JR. es. Johns Hopkins textbook of cardiothoracic surgery. n e. McGraw-Hill; 014.
12. E. Parapneumonic effusion refers to the accumulation
of pleural ui in response to a respiratory infection. It is generally ivie into three stages: exuative, brinopurulent, an organizing. The rst (exuative) stage is characterize by the evelopment of sterile pleural ui in response to increase capillary permeability. After 5 ays, bacteria begin to enter the ui an inammatory cells follow. This marks the beginning of the brinopurulent phase. In general, new effusions shoul unergo iagnostic thoracentesis to rule out an empyema. If transuative, antibiotic treatment of the pneumonia is all that is require (A). Urgent rainage via tube thoracostomy is recommene for frankly purulent effusions or those with bacteria on Gram stain or culture. The iameter of the chest tube oes not seem to be important so long as smaller caliber tubes are routinely ushe to prevent blockage of the catheter (C). As the brinopurulent phase progresses, loculations begin to form within the collection, making rainage with a single catheter or tube thoracostomy ifcult. Several stuies have been one evaluating the use of intrapleural brinolytics, such as alteplase, to prevent progression to surgery. However, the results are controversial at best, an a 008 Cochrane Review of the practice foun no consistent benet (B). At this stage, vieo-assiste thoracoscopic ebriement an ahesiolysis are viable options, though a certain number of patients will still nee to be converte to thoracotomy (E). After to 3 weeks of untreate infection, broblasts begin to form a pleural peel an the nal (organization) stage is reache. Once this membrane has forme, formal ecortication via thoracotomy is generally necessary. In patients that are unt for surgery, open rainage (e.g., Eloesser ap) may be consiere. However, this subjects patients to months of ressing changes an signicant morbiity (D). References: Cameron R, Davies HR. Intra-pleural brinolytic therapy versus conservative management in the treatment of ault parapneumonic effusions an empyema. Cochrane Database Syst Rev. 008;:CD0031. Davies HE, Davies RJO, Davies CWH, BTS Pleural Disease Guieline Group. Management of pleural infection in aults: British Thoracic Society Pleural Disease Guieline 010. Thorax. 010;65(Suppl ):ii41–ii53. Light RW. Parapneumonic effusions an empyema. Proc Am Thorac Soc. 006;3(1):75–80.
13. B. All
penicillin-erivative antibiotics (β-lactams) inhibit the nal step of bacterial cell wall synthesis by bining transpeptiases or penicillin-bining proteins (A). Cephalosporins work by the same mechanism but are more resistant to egraation by β-lactamases. Tazobactam, sulbactam, an clavulanic aci bin β-lactamases an are frequently combine with penicillin-erivative antibiotics to increase their effectiveness. Examples of this inclue piperacillin-tazobactam an amoxicillin-clavulanic aci. Metroniazole is an antibiotic that only has action against anaerobic bacteria by inhibiting nucleic aci synthesis. It is not effective in aerobic cells because it requires reuction to its active state, which only takes place in anaerobic cells (C). Aminoglycosies an
405
tetracyclines inhibit the 30S ribosome. Linezoli, on the other han, inhibits the 50S ribosome subunit. Several other antibiotics (macrolies, linezoli, chloramphenicol) also inhibit the 50S ribosome; however, it is a slightly ifferent process (D). Clinamycin is a lincosamie antibiotic, which interferes with the amino acyl-tRNA complex (E). Aminoglycosies an tetracycline antibiotics inhibit the 30S ribosome.
14. C. The Third International Consensus Denitions for Sepsis
and Septic Shock, publishe in JAMA in 016, reene the current enition use for sepsis an septic shock. The panel came to the conclusion that the previously use enition of sepsis (+ SIRS criteria an a source of infection) was too nonspecic an generally unhelpful in the ientication of patients at increase risk of mortality from infection (A, B, D, E). Instea, the committee recommene a besie screening tool calle the quick Sequential Organ System Failure score (qSOFA) for ientication of patients that are likely to have a poor outcome as the result of an infection. If a patient meets two of the three criteria (respiratory rate >/min, altere mental status, an systolic bloo pressure 40 shoul receive .5g protein/kg/ay as part of an overall strategy of hypocaloric feeing E. The presence of a 40% burn requires nutritional support with .5 g protein/kg/ay F. Critically ill patients with renal failure shoul receive 1.5 to 1.75 g protein/kg/ay, whereas patients with renal failure on continuous renal replacement therapy require .5 g protein/kg/ay G. Respiratory quotient (RQ): estimates basal metabolic rate; ratio of carbon ioxie prouce by the boy to oxygen consume by the boy 1. RQ > 1.0: overfeeing, can lea to ifculty weaning from ventilator ue to hypercarbia . RQ = 1: carbohyrate utilization 3. RQ = 0.8 to 0.9: protein utilization (average 0.85; mixture of fat, protein, carb metabolization) 4. RQ = 0.7: fat utilization 5. RQ < 0.7: starvation II. Amino Acis: Builing Blocks for Proteins A. Essential: not mae by the boy, must be ingeste; phenylalanine, valine, threonine, tryptophan, isoleucine, methionine, histiine, leucine, lysine B. Nonessential: boy can prouce these: alanine, asparagine, aspartic aci, glutamic aci, serine C. Conitional amino acis: boy can prouce these, but are essential in times of stress: Arginine, cysteine, glutamine, glycine, ornithine, proline, tyrosine III. Nitrogen Balance: To Stuy Protein Metabolism A. Nitrogen balance = protein intake/6.5 – (4-hour urine nitrogen + 4g) B. 4g approximates losses via sweat an feces C. Positive = anabolism; negative = catabolism IV. Starvation A. Certain cells (brain, re bloo cells) primarily use glucose for energy (except when starving) B. During starvation, insulin ecreases an glucagon increases, leaing to an increase in glycogenolysis, lipolysis, an ketogenesis
409
410
PArt ii Medical Knowledge
C. Glycogen stores (glucose supply) are eplete after 1 to ays D. Next, fatty acis (can’t cross bloo-brain barrier) are oxiize to make ketones (for brain) E. With further starvation, muscle (protein) breaks own to provie alanine for gluconeogenesis V. Vitamin an Mineral Deciencies
Deęciency
Manifestation/disease
Vitamin A
Night blindness
Vitamin B1 (Thiamine)
Wernicke’s encephalopathy, Beriberi
Vitamin B3 (Niacin)
Pellagra (diarrhea, dermatitis, dementia)
Vitamin B6 (Pyridoxine)
Anemia, peripheral neuropathy
Vitamin B12 (Cyanocobalamin)
Megaloblastic anemia, peripheral neuropathy
Vitamin C
Impaired collagen cross-linking, scurvy
Vitamin D
Rickets, osteomalacia
Vitamin E
Neuropathy
Vitamin K
Coagulopathy
Chromium
Hyperglycemia, neuropathy
Copper
Anemia, leukopenia, muscle weakness
Iodine
Goiter
Phosphate
Diaphragm muscle weakness, arrhythmia, confusion
Selenium
Cardiomyopathy, weakness
Zinc
Delayed wound healing, hair loss, acne
Essential faĴy acids
Dermatitis, hair loss, easy bruising, delayed wound healing
Essential amino acids
Decreased immune function
VI. Nutritional Deciencies Associate With Surgeries/Surgical Diseases
Surgery/surgical diseases
Related nutritional deęciency
Gastric bypass
Deęciency of vitamin B12, folate, zinc, iron, copper, calcium, vitamin D
Gastric sleeve resection
Deęciency of vitamin B12, folate, zinc, iron
Carcinoid syndrome
Tryptophan deęciency (due to conversion to serotonin) causes pellagra
Blind loop syndrome
Deęciency of vitamin B12, folate, iron, vitamin E
Refeeding syndrome
Hypophosphatemia, hypomagnesemia, hypokalemia
CHAPtEr 32 Nutrition and Metabolism
411
Questions 1. A 45-year-ol male is iagnose with severe gallstone pancreatitis. It is currently hospital ay an he is not requiring vasopressor support or invasive mechanical ventilation. He still reports mil epigastric pain. Which of the following is true regaring the ieal management of his nutrition? A. He shoul be starte on an oral iet as tolerate B. A nasoenteric tube shoul be place with tube fees starte at a trophic rate an avance to goal as tolerate C. Nasojejunal feeing is preferre over nasogastric feeing D. Total parenteral nutrition (TPN) is preferre over enteral nutrition E. Enteric feeing shoul not be consiere until abominal pain has resolve 2. Which of the following is true regaring immunonutrition? A. Alanine is a substrate in the prouction of nitric oxie (NO) B. Glutamine has been shown to reuce raiation injury to the small bowel C. For cancer patients unergoing surgery, immunonutrition reuces postoperative infectious complications D. For patients unergoing major abominal surgery, immunonutrition reuces mortality E. Glutamine has been shown to ecrease ventilator time in critically ill patients 3. A 55-year-ol alcoholic female is amitte to the hospital for a small bowel obstruction. Her serum albumin is 1.8 g/L (normal range is 3.4–5.4 g/L) an prealbumin is 8 mg/L (normal range is 15–36 mg/L). On hospital ay 5, she fails nonoperative management an unergoes an exploratory laparotomy with
lysis of ahesions. She subsequently evelops a postoperative ileus an is starte on total parenteral nutrition on postoperative ay 3. A ay later, she rapily evelops weakness, altere mental status, an hypoxic respiratory failure requiring intubation. Which of the following is true regaring her conition? A. Thiamine eciency is the most likely cause of her symptoms B. Alcoholism is not a risk factor for eveloping this conition C. This conition rarely occurs with enteral nutrition D. This conition coul have potentially been avoie by starting TPN at a slower rate E. She shoul be given a calcium infusion 4. Which of the following amino acis can be synthesize e novo in humans in any physiologic state? A. Tryptophan B. Tyrosine C. Glycine D. Serine E. Any branche-chain amino aci 5. Which of the following is true regaring the use of preoperative TPN to prevent postoperative complications? A. It is useful even if use for as little as 3 ays B. It is efcacious if the patient has lost more than 15% weight before surgery C. There is no evience that it lowers the complication rate D. Slightly overfeeing for 7 ays is recommene as a means to maximize replacement of caloric ecits E. TPN is efcacious even in mil to moerate malnutrition
412
PArt ii Medical Knowledge
6. Which of the following is true regaring nutritional eciencies after a partial gastrectomy with a Billroth II (gastrojejunostomy) reconstruction? A. Calcium absorption will be minimally affecte B. Iron eciency anemia is more common with a Billroth I (gastrouoenostomy) than a Billroth II C. Vitamin B1 eciency will present with a low mean corpuscular volume D. The stomach has no intrinsic absorptive ability E. Carbohyrate absorption is not impaire after surgery 7. Which of the following is true regaring nutrition nees an requirements? A. Preterm infants may nee up to g/kg per ay of protein B. 1 g of fat provies 4 kcals of energy C. A respiratory quotient (RQ) greater than 1.0 suggests overfeeing D. Ventilate critically ill patients require more aily caloric intake than nonventilate critically ill patients E. Obese patients require less aily protein intake compare to nonobese patients 8. A 3-year-ol male was amitte 7 ays ago for multisystem trauma incluing multiple long-bone fractures, subural hematoma, an pulmonary contusions an is still on the ventilator. Which of the following is true regaring tools for assessing nutritional status? A. Use of serial measurements of albumin an prealbumin is the “gol stanar” for trauma patients B. Measurement of nitrogen balance unerestimates nitrogen input C. The Mini Nutritional Assessment is esigne specically for hospitalize patients D. Creatinine height inex may overestimate lean boy mass in trauma patients E. Transferrin is the serum protein that correlates the closest to nitrogen balance
9. Which of the following is true regaring the risk of hypoglycemia following cessation of total parenteral nutrition (TPN)? A. It commonly occurs in patients with liver isease B. Tapering of TPN is recommene so as to avoi this complication C. This complication is relatively common D. It is more likely to occur in a iabetic patient E. It is more likely to occur in patients with renal isease 10. Which of the following is true about the pharmacologic treatment of cancer cachexia? A. There is no evience that ghrelin mimetics are of benet B. Cannabinois are superior to megestrol acetate in stimulating weight gain C. When initiate early, megestrol acetate has been emonstrate to improve survival D. Megestrol is a progesterone erivative E. Anabolic sterois lea to improve long-term weight gain 11. Which of the following is true regaring energy homeostasis uring perios of starvation? A. The largest source of energy after glycogen is eplete is free fatty acis B. Skeletal muscle has the largest store of glycogen available systemically C. Glucose is converte to lactate in the liver D. Re bloo cells metabolize glucose aerobically E. The brain is unable to utilize ketones 12. The most important amino aci use for gluconeogenesis by the liver is: A. Glutamine B. Serine C. Alanine D. Tyrosine E. Asparagine 13. Poor glucose control is a manifestation of eciency of: A. Zinc B. Copper C. Chromium D. Molybenum E. Selenium
CHAPtEr 32 Nutrition and Metabolism
14. Which of the following is true regaring longterm TPN? A. Fat is consiere the nutritional basis of TPN B. It may lea to a mucin gel matrix of cholesterol crystals an calcium bilirubinate in the gallblaer C. Hepatic ysfunction relate to TPN is less likely to be lethal in infants than in aults D. It has not been shown to lea to hepatic brosis E. Carnitine supplementation has been shown to reverse TPN-relate liver amage
413
15. Which of the following amino acis has shown potential for increasing the absorptive capability of the intestine in patients that have unergone large segment small bowel resection? A. Glutamine B. Serine C. Alanine D. Tyrosine E. Arginine
Answers 1. B. Though avancing to an oral iet as tolerate is rec-
ommene in mil acute pancreatitis, this is not the recommene management in moerate to severe pancreatitis (A). Instea, patients with moerate to severe acute pancreatitis shoul have a nasoenteric/oroenteric tube place an enteral nutrition starte in the rst 1 to hospital ays (B). Mil epigastric pain is not a contrainication to enteric feeing (E). With regars to the level at which to fee, three ranomize controlle trials showe no ifference in tolerance or clinical outcomes between gastric an jejunal feeing (C). The use of parenteral nutrition rather than enteral nutrition has also been explore in multiple metaanalyses of ten ranomize clinical trials. These have shown that those receiving enteral nutrition ha lower infectious morbiity, shorter length of stay, fewer surgical interventions, an ecrease mortality (D). Guielines from the Society of Critical Care Meicine an the American Society for Parenteral an Enteral Nutrition now recommen consieration of probiotics in severe pancreatitis for those receiving early enteral nutrition, which is base on a 010 metaanalysis that showe a reuction in infection an hospital LOS. Reference: McClave SA, Taylor BE, Martinale RG, et al. Guielines for the Provision an Assessment of Nutrition Support Therapy in the Ault Critically Ill Patient: Society of Critical Care Meicine (SCCM) an American Society for Parenteral an Enteral Nutrition (A.S.P.E.N.). J Parenter Enter Nutr. 016;40():159–11.
2. C. Immunonutrition is the ability to moulate the
immune system using specic nutrients. This strategy has most often been utilize in critically ill an surgical patients who often require exogenous nutrients through enteral or parenteral routes. The nutrients most robustly stuie for immunonutrition are arginine, glutamine, omega-3 fatty acis, branche chain amino acis, an nucleoties. Arginine an glutamine, two amino acis that have been of particular interest in this el, have unique properties that may explain their mechanism of action. Arginine, via the arginine eaminase pathway, is a unique substrate for prouction of NO (A). Glutamine is the most prevalent free amino aci in the
human boy an is also the major metabolic fuel for enterocytes an other cells within the immune system. Aministration of glutamine has been shown to have no effect on reucing raiation injury (B). A number of large systematic reviews have shown benets of immunonutrients in various subsets of surgical patients. For example, in surgical cancer patients an patients unergoing major abominal surgery, immunonutrition reuces infectious complications an shortens length of stay, but oes not ecrease mortality (C, D). In burn patients, initial clinical trial ata suggests that glutamine may reuce mortality, length of stay, an gram-negative bacteremia though the enitive RE-ENERGIZE trial is ongoing. However, there may be subsets of patients, like the critically ill, who may be harme by immunonutrition. Two large multicenter ranomize controlle trials of critically ill ventilate patients showe that supplementation with glutamine an/or antioxiants may increase 6-month mortality (E). References: Caler PC. Immunonutrition. BMJ. 003;37(7407): 117–118. Suchner U, Kuhn KS, Fürst P. The scientic basis of immunonutrition. Proc Nutr Soc. 000;59(4):553–563. Probst P, Ohmann S, Klaiber U, et al. Meta-analysis of immunonutrition in major abominal surgery. BJS. 017;104(1):1594–1608. Wischmeyer PE. Glutamine in burn injury. Nutr Clin Pract. 019;34(5):681–687. Yu K, Zheng X, Wang G, et al. Immunonutrition vs stanar nutrition for cancer patients: a systematic review an meta-analysis (part 1). J Parenter Enter Nutr. 00;44(5):74–767. van Zanten AR, Hofman Z, Heylan DK. Consequences of the REDOXS an METAPLUS Trials: the en of an era of glutamine an antioxiant supplementation for critically ill patients? J Parenter Enter Nutr. 015;39(8):890–89.
3. D. This patient evelope refeeing synrome, which
is a potentially fatal metabolic isturbance after the reinstitution of nutrition in a malnourishe patient (low serum albumin an prealbumin). Prolonge starvation leas to the severe epletion of a number of minerals, though serum concentrations remain relatively normal ue to compensatory
414
PArt ii Medical Knowledge
intra/extracellular shifts. During refeeing, insulin is release, leaing to stimulation of glycogen, fat, an protein synthesis, which requires phosphate, magnesium, an other cofactors. Phosphate, magnesium, an potassium (through the ATPase symporter) are all taken up into cells, leaing to a suen ecrease in serum levels. Refeeing synrome is cause by these epletions with hypophosphatemia being the most common an most severe isturbance (A). Some common clinical manifestations of severe hypophosphatemia are arrhythmias, metabolic aciosis, seizures, elirium, hyperglycemia, an profoun weakness, sometimes manifesting as iaphragm insufciency requiring mechanical ventilatory support. Risk factors for refeeing synrome inclue anorexia nervosa, malnutrition, chronic alcoholism, cancer, recent surgery, elerly patients with comorbiities, BMI 00 ug/L before resection = poor prognosis B. CA 19-9: Pancreatic cancer 1. >1000U/mL = likely metastatic isease C. CA-15: Epithelial ovarian cancer D. Inhibin-A: epithelial stroma tumors such as mucinous an enometrioi carcinoma an sex cor-stromal tumors such as granulosa cell tumor an Sertoli-Leyig cell tumor E. AFP: hepatocellular carcinoma (HCC), nonseminomatous germ cell tumors F. β-hCG: germ cell tumors G. Calcitonin: meullary thyroi cancer H. Thyroglobulin: papillary thyroi cancer; particularly for etecting recurrence after surgery an raioactive ioine therapy I. Chromogranin A: carcinoi tumors Chemotherapeutic Sie Effects
Agent
Side eěects
Bleomycin
Pulmonary ębrosis
Carboplatin
Myelosuppression
Cisplatin
Nephrotoxicity, neurotoxicity, ototoxicity
Cyclophosphamide
Hemorrhagic cystitis (treat with Mesna), gonadal dysfunction, SIADH
Doxorubicin
Cardiac toxicity (irreversible)
Methotrexate
Nephrotoxicity, nausea (treat with folinic acid)
Oxaliplatin
Peripheral neuropathy
Taxols
Neuropathy
Trastuzumab
Cardiomyopathy (reversible)
Vinblastine
Myelosuppression
Vincristine
Peripheral neuropathy
Hereitary Cancer Synromes
Gene
Syndrome
Associated cancers
APC
Familial adenomatous polyposis (FAP)
Colon, duodenum, gastric, thyroid, desmoid tumors
TP53
Li Fraumeni
Breast (90%), colon, lung, sarcomas, brain, adrenal
DNA Mismatch Repair (MLH1, MSH2, MSH6, PMS2, EPCAM)
Hereditary nonpolyposis colorectal cancer (HNPCC)
Colorectal, endometrial, gastric, ovarian, GU tract, hepatobiliary, small bowel, and CNS
SMAD4, BMPR1A
Juvenile polyposis
Colon, gastric
STK11
Peuĵ-Jeghers
Hamartomas—GI and mucocutaneous. Malignancies—breast, colon, pancreatic, gastric, ovarian, lung, small intestine, endometrial, testicular, esophageal
PTEN
Cowden
Breast, facial lesions, GI hamartomas, thyroid, endometrial
BRCA1
Breast (87% risk), ovarian (40%–60%)
BRCA2
Breast (80%), ovarian (15%–20%), prostate, pancreatic
CHAPtEr 33 Oncology and Tumor Biology
419
Multiple Enocrine Neoplasia (MEN) Synromes
MC Clinical presentation
Organs/tumors involved
What to treat ęrst
Autosomal Dominant
Hypercalcemia
Pancreatic NET Pituitary Hyperparathyroidism
Hyperparathyroidism
RET
Autosomal Dominant
Medullary Thyroid Cancer
Medullary Thyroid CA Pheochromocytoma Hyperparathyroidism
Pheochromocytoma
RET
Autosomal Dominant
Medullary Thyroid Cancer
Medullary Thyroid CA Pheochromocytoma Mucosal Neuromas Marfanoid Habitus
Pheochromocytoma
Syndrome Gene
Protein
Inheritance
MEN 1
11q13
Menin
MEN 2A
10q11.21
MEN2B
10q11.21
420
PArt ii Medical Knowledge
Questions 1. A 44-year-ol male with a history of hypertension well controlle on meications is foun to be anemic an with a positive fecal occult bloo test uring his yearly physical. He notes a family history that inclues eaths ue to colon cancer in of both his mother at age 46 an his maternal granfather at age 51. He unergoes colonoscopy which emonstrates four aenomatous polyps in the ascening colon as well as an aenocarcinoma of the ileocecal junction. This patient is most likely to have which of the following? A. A mutation in the TP53 gene B. A mutation in the PMS gene C. A mutation in the PTEN gene D. A mutation in the STK11 gene E. A mutation in the aenomatous polyposis coli (APC) gene 2. An otherwise healthy 68-year-ol woman is iagnose with locally avance gastric aenocarcinoma. There is no evience of istant metastases. Her tumor is biopsie an note to have HER2/neu overexpression. She is starte on an appropriate chemotherapy regimen. After her secon cycle, she presents with new-onset yspnea on exertion an orthopnea. Which of the following chemotherapeutic agents is likely responsible for her symptoms? A. Bleomycin B. 5-Fluorouracil C. Vinblastine D. Trastuzumab E. Cisplatin 3. A 70-year-ol otherwise healthy male with a history of colon aenocarcinoma that was treate with a formal resection returns two years later with a 3-cm lesion on his liver. Workup conrms a colorectal metastasis with no evience of sprea elsewhere. Which of the following is the most appropriate next step? A. Chemotherapy only B. Surgical resection only C. Surgical resection followe by chemotherapy D. Chemotherapy followe by surgical resection E. Surgical resection followe by raiation
4. An 87-year-ol female presents to the emergency epartment (ED) with weight loss, vomiting, obstipation, an a istene abomen. She has not ha a bowel movement in 3 ays. Past history is signicant for a non-ST segment elevation myocarial infarction (NSTEMI) 6 weeks earlier. A compute tomography (CT) scan with oral contrast shows evience of an obstructing mass in the sigmoi colon. However, the lumen oes appear to be patent. Her vitals are stable. Which of the following is the best recommenation? A. Diverting ileostomy B. Diverting transverse colostomy C. Open sigmoi resection with proximal colostomy D. Colonoscopy with placement of a temporizing stent followe by elective surgery E. Laparoscopic sigmoi resection with proximal colostomy 5. A patient with metastatic sigmoi colon cancer is about to unergo chemotherapy, an the oncologist recommens the use of an anti-EGFR monoclonal antiboy. Which of the following genetic proles is most likely to benet from the aition of this agent? A. K-ras wiltype gene B. BRAF mutation C. NRAS D. PIK3CA mutation E. K-ras mutant gene 6. Which of the following patients shoul be referre to a genetic counselor for BRCA testing? A. Family history of breast cancer in mother at the age of 55 B. Both parents are Sepharic Jews C. Aopte an unknown family history, evelope breast cancer at 55 D. 55-year-ol female with breast cancer in bilateral breasts E. 55-year-ol female with an inammatory breast cancer
CHAPtEr 33 Oncology and Tumor Biology
7. A 55-year-ol male presents to the ED with vomiting an an inability to tolerate oral intake for the last week. CT scan shows a signicantly istene stomach, with a thickene mass near the pylorus. Upper enoscopy shows a large mass in the stomach that partly occlues the istal lumen. Biopsy is consistent with low-grae mucosa-associate lymphoi tissue (MALT) lymphoma. He takes proton-pump inhibitors for aci reux. Which of the following is true regaring his conition? A. Triple antibiotic therapy for eraication of Helicobacter pylori shoul be starte regarless of whether the patient is H. pylori positive or negative B. The patient shoul be given chemotherapy along with triple antibiotic therapy C. Gastrectomy has no role in the treatment of gastric MALT lymphoma D. Raiotherapy has no role in the treatment of gastric MALT lymphoma E. Surgery is recommene for patients who o not respon to triple antibiotic therapy 8. Which of the following is true regaring the interaction between raiation therapy an tumor cells? A. Raiation therapy leas to cancer cell eath by irectly inhibiting aenosine triphosphate (ATP) prouction in the mitochonria B. Larger tumors are more sensitive to raiation therapy C. As the energy use in raiation therapy increases, collateral amage to overlying skin also increases D. The S phase of the cell cycle is most sensitive to raiation effects E. Correcting anemia can increase the efcacy of raiotherapy 9. A 60-year-ol male with cirrhosis presents to clinic with a newly iagnose 4-cm hepatocellular carcinoma (HCC) in segment 6. There is no evience of gross vascular invasion an no regional noal or extrahepatic istant metastases. His international normalize ratio (INR) is 1.8, creatinine is 1.0 mg/L, bilirubin is 3.1 mg/L, an albumin is .6 mg/L, an his compute tomography (CT) scan shows no evience of ascites. Which of the following woul be the best treatment option? A. Transarterial chemoembolization (TACE) B. Liver resection C. Raiofrequency ablation (RFA) D. Irreversible electroporation E. Liver transplantation
421
10. Which of the following is true regaring the evelopment of skin cancers? A. Ultraviolet (UV) raiation both initiates an promotes DNA amage B. UVA is the ultraviolet frequency most responsible for chronic skin amage C. An increase level of skin melanin increases the risk of eveloping basal cell carcinoma D. UV raiation amages the DNA mismatch repair gene E. Mutations in the BCL- gene are a known mechanism for the evelopment of skin cancer 11. A 43-year-ol male is iagnose with a highgrae right lower extremity osteosarcoma an unergoes surgical resection an ajuvant chemotherapy with MAP (methotrexate, oxorubicin, an cisplatin). After the thir treatment cycle, the patient evelops severe nausea, vomiting, an altere mental status. Workup reveals increase liver transaminases, a reuction in glomerular ltration rate (GFR), as well as leukopenia an thrombocytopenia. What meication can potentially reverse these effects? A. Cobalamin B. Folinic aci C. Folic aci D. Folate E. Omeprazole 12. Which of the following statements is true regaring patterns of metastatic sprea? A. The most common metastatic location for breast cancer is the arenal glan B. The most common metastatic location for melanoma is the small bowel C. Metastases to the arenal glan most commonly originate in the lungs D. The most common metastatic location for colon cancer is the lungs E. The transverse colon is frequently the rst location of metastatic sprea of pancreatic cancer
422
PArt ii Medical Knowledge
13. Which of the following statements is true regaring the human protein p53? A. Germline mutations of the p53 gene result in Cowen Synrome B. The unregulate growth seen with human papillomavirus (HPV) is partly ue to bining an inactivation of the p53 protein C. The p53 gene suppresses the translation process in DNA sequencing an cell growth D. Overexpression of this gene leas to uncontrolle cell growth E. Mutations frequently result in benign neoplastic growth rather than malignancy
14. A 77-year-ol male who resies in a subacute care facility has just nishe ajuvant chemotherapy (FOLFOX an Bevacizumab) for metastatic colon cancer. Despite a normal albumin, minimal weight loss, an meticulous local woun care, his nurses have been unable to aequately treat a nonhealing sacral ecubitus ulcer. The woun base looks clean, an he has no signs of systemic infection. Which of the following is true? A. The sacral woun shoul be preemptively ebrie to avoi infection an facilitate woun healing B. Supplemental enteral nutrition will facilitate faster woun healing C. Rescue therapy can be attempte with leucovorin D. The patient shoul be converte to Cetuximab E. Barriers to healing will likely resolve in 6 months
Answers 1. B. This patient has Lynch synrome (hereitary nonpol-
yposis colorectal cancer—HNPCC). The Amsteram criteria ene the criteria necessary for iagnosis. They inclue three or more family members who have been iagnose with an HNPCC-associate cancer (colorectal, enometrial, gastric, ovarian, GU tract, hepatobiliary, small bowel, an CNS—but most commonly colorectal cancer), one of whom is a rst-egree relative of the other two; at least two generations of family members involve; an at least one member iagnose with colorectal cancer prior to the age of 50. In aition, no family members may have been iagnose with FAP. HNPCC is characterize by mutations in mismatch repair genes (MLH1, MSH, MSH6, PMS, EPCAM), resulting in microsatellite instability (MSI), an is inherite in an autosomal ominant fashion. Aitionally, there is a high frequency of cancers in HNPCC arising in the proximal colon when compare to other hereitary colorectal cancer synromes (B). A mutation in the TP53 gene results in Li Fraumeni synrome which is characterize by tumors of the breast (90%), colon, lung, brain, an arenal, as well as sarcomas (A). PTEN mutations result in Cowen synrome, in which patients evelop tumors of the breast, thyroi, an enometrium as well as facial lesions an GI hamartomas (C). Peutz-Jeghers synrome is ue to a mutation in the STK11 gene an is characterize by hamartomas (both GI an mucocutaneous) as well as malignancies of the breast, colon, pancreas, stomach, ovaries, lung, small intestine, enometrium, testicles, an esophagus (D). Familial aenomatous polyposis (FAP) is ue to a mutation in the APC gene an is typically characterize by the appearance of thousans of aenomatous polyps throughout the colon early in life (E). References: Greenel LJ, Mulhollan MW, es. Greeneld’s surgery: scientic principles & practice. 5th e. Lippincott Williams an Wilkins; 010.
Mayer RJ. Lower gastrointestinal cancers. In: Jameson J, Fauci AS, Kasper DL, Hauser SL, Longo DL, Loscalzo J. es. Harrison’s principles of internal medicine. 0th e. McGraw-Hill; 018. Morris A. Epiemiology—clinical risk factors—familial cancer synromes. In: Greenel LJ, Mulhollan MW, es. Greeneld’s surgery: scientic principles & practice. 5th e. Lippincott Williams an Wilkins; 010.
2. D. Commonly utilize chemotherapy regimens in gas-
tric cancer inclue FLOT (5-FU, leucovorin, oxaliplatin, an ocetaxel) as well as capecitabine, cisplatin, an epirubicin. In cases of HER/neu overexpression, trastuzumab may be ae. Trastuzumab is a monoclonal antiboy therapy use in the treatment of HER/neu overexpressing cancers (most commonly breast an GI origins). Trastuzumab can cause reversible cariomyopathy (D). 5-uorouracil (5-FU) is a component of the FLOT regimen for gastric cancer, but cariomyopathy is not a common sie effect (B). Bleomycin is utilize in the treatment of lymphoma, testicular, ovarian, an cervical cancers an can cause pulmonary brosis (A). Vinblastine is not a typical therapeutic agent in the treatment of gastric cancer an can cause myelosuppression (C). Cisplatin is frequently utilize in the treatment of gastric cancer, but its sie effects inclue nephrotoxicity, neurotoxicity, an ototoxicity (E). References: Sah BK, Zhang B, Zhang H, et al. Neoajuvant FLOT versus SOX phase II ranomize clinical trial for patients with locally avance gastric cancer. Nat Commun. 00;11(1):6093. Wagner AD, Syn NL, Moehler M, et al. Chemotherapy for avance gastric cancer. Cochrane Database Syst Rev. 017;8:CD004064.
3. C. Recent literature shows a conferre survival benet
for the resection of hepatic metastases in colorectal cancer. Multiple high-volume centers have emonstrate the 5-year survival for patients with metastatic colorectal cancer to the liver to be 5% to 58% with resection of the metastatic lesion.
CHAPtEr 33 Oncology and Tumor Biology Over the last two ecaes, the perioperative mortality associate with hepatic resection has fallen signicantly, with most high-volume centers reporting a 30-ay perioperative mortality of less than %. The presence of any of the following risk factors ha a negative, an aitive, effect on survival in patients with hepatic metastases from colorectal cancer: (1) noe-positive primary tumor, () isease-free interval less than 1 months, (3) multiple liver metastases, (4) largest hepatic metastasis greater than 5 cm, an (5) serum carcinoembryonic antigen (CEA) level greater than 00 ng/mL. Those with none of these risk factors have the greatest 5-year survival at 60%. Treatment will vary epening on whether it is a synchronous or metachronous lesion. Synchronous lesions can be safely treate with combine colon an liver resection, provie the hepatic resection is limite (75 ( points), previous TIA/stroke ( points), iabetes (1 point), previous vascular isease (1 point), hypertension (1point), CHF (1 point), female (1 point) ) Novel oral anticoagulant (NOAC) rugs a) Direct-thrombin inhibitors (1) Bivaliruin, argatroban, an esiruin: parenteral aministration () Dabigatran: only available oral agent; half-life is 1 to 17 hours; stop ays before surgery; renally metabolize so use with caution in patients with renal isease (a) Reverse with iarucizumab (a monoclonal antiboy that bins abigatran) b) Factor-Xa inhibitors (all factor-Xa inhibitors en in -xaban) (1) Rivaroxaban: half-life is 6 to 9hours, an its therapeutic activity wears off after 4 to 5half-lives; rivaroxaban shoul be iscontinue 1 to ays before a surgical proceure; in patients with a reuce creatinine clearance, it shoul be iscontinue 3 to 5 ays before surgery () Apixaban: factor-Xa inhibitor, half-life of 1 hours; iscontinue ays prior to surgery; metabolize by liver so safe for patients with renal isease (a) Reverse with 4-factor prothrombin complex concentrate (b) Anexanet alfa is a recombinant analog of factor Xa an may be consiere an antiote for factor-Xa inhibitors c) Can restart anticoagulation 6 to 4 hours after a minor proceure, an after to 3 ays for major surgery (barring any perioperative bleeing)
437
438
PArt ii Medical Knowledge
C. Steroi therapy: chronic steroi therapy can affect the hypothalamic–pituitary–arenal axis, leaing to arenal atrophy an a ecrease capability to prouce cortisol leaing to a theoretical risk of hypotension in the perioperative perio; however, perioperative “stress-ose” sterois are not supporte by recent evience 1) “Stress-ose” sterois shoul not be routinely aministere; instea, the patient shoul continue their home ose of sterois perioperatively ) Shoul consier aitional sterois only if the patient evelops refractory hypotension suggestive of arenal insufciency in the perioperative perio
CHAPtEr 35 Preoperative Evaluation and Perioperative Care
439
Questions 1. A 58-year-ol man presents with a reucible inguinal hernia. He is not limite in his aily activities but is bothere by the appearance. He unerwent percutaneous coronary intervention (PCI) with placement of a rug-eluting stent (DES) months ago an is currently taking aspirin an clopiogrel. What is the most appropriate management of this patient? A. Scheule surgery an continue aspirin an clopiogrel B. Scheule surgery an Continue aspirin an stop clopiogrel 5 ays prior to the operation C. Scheule surgery an stop aspirin an clopiogrel 5 ays prior to the operation D. Delay surgery for an aitional 4 months E. Delay surgery for a year post-DES 2. A 38-year-ol woman evelops fever, abominal pain, an multiple loose nonblooy bowel movements following amission for perforate appenicitis. Her WBC count is 1,000 an has normal kiney function. On imaging, there is no evience of eep space abscess or ileus an her stool tests positive for Clostridium difcile. This is her rst episoe. What is the most appropriate treatment? A. Fecal transplant B. Oral an rectal vancomycin C. Oral vancomycin an intravenous metroniazole D. Intravenous metroniazole E. Oral vancomycin 3. Five ays after a laparoscopic Roux-en-Y gastric bypass, a patient evelops fever with rigors, hypotension, tachycaria, an pain in the left shouler. This most likely represents: A. Gas bloat synrome B. Internal hernia C. Woun ehiscence D. Gastric volvulus E. Disruption of the gastric pouch–jejunal anastomosis
4. A 8-year-ol woman unergoes ahesiolysis for an acute small bowel obstruction. During the course of the surgery, she requires a segmental ileal resection with primary anastomosis. On postoperative ay 6, she is note to have thick bile-colore ui emanating from the miline woun. After IV hyration, the next step in the management shoul be: A. CT scan of the abomen B. Water-soluble upper gastrointestinal series with small bowel follow-through C. Fistulogram D. Operative reexploration E. Octreotie 5. The most important preictor of colonic ischemia after repair of a rupture abominal aortic aneurysm is: A. Age B. Presence of preoperative shock C. Time to operation D. Presence of associate cariac isease E. Preoperative patency of inferior mesenteric artery 6. Five ays after surgery for perforate appenicitis, liqui stool emanates from the right lower quarant woun. Which of the following is true about this conition? A. It is most commonly ue to an unrecognize malignancy B. The majority will close spontaneously C. The patient shoul be place immeiately on TPN D. Flui an electrolyte erangements are common E. The patient shoul be returne immeiately to the operating room for surgical repair 7. Five ays after a Billroth II gastric resection for a bleeing ulcer, high fever, hypotension, tachycaria, an generalize peritonitis evelop in the patient. This most likely represents: A. Postoperative pancreatitis B. Acalculous cholecystitis C. Duoenal stump blowout D. Woun ehiscence E. Intraabominal hemorrhage
440
PArt ii Medical Knowledge
8. Which of the following moalities is LEAST likely to assist in the prevention of postoperative pulmonary complications in a 65-year-ol male smoker? A. Postoperative use of an incentive spirometer B. Postoperative eep-breathing exercises C. Postoperative use of continuous positive airway pressure D. Smoking cessation 1 week before surgery E. Placement of a nasogastric tube 9. Which of the following preoperative stuies is most strongly associate with an increase risk of pulmonary-relate postoperative complications? A. Bloo urea nitrogen B. Incentive spirometry C. Chest raiograph D. Serum albumin E. Room air arterial bloo gas 10. A 67-year-ol male recovering from a pelvic exenteration seconary to locally avance rectal cancer is starte on total parenteral nutrition for prolonge ileus via a right-sie peripherally inserte central catheter (PICC) line. Several ays later his arm becomes swollen. Ultrasoun conrms clot in the basilic an axillary veins. What is the appropriate management of his conition? A. Warm compress an nonsteroial antiinammatory rugs (NSAIDs) B. Immeiately remove the line C. Immeiately remove line an then start heparin D. Start heparin an move the line to an alternate site E. Start heparin, keep the line in place, an therapeutic anticoagulation for 3 to 6 months 11. A 76-year-ol iabetic male is amitte to the surgical intensive care unit after a fall. His injuries inclue a right femoral neck fracture an subarachnoi hemorrhage. He continues to have intermittent elevation in his intracranial pressure an is still requiring respiratory support after ays. Which of the following is true regaring nutritional supplementation in this patient? A. Postpyloric feeing may reuce his risk of eveloping pneumonia B. Gastric feeing is associate with a longer length of ICU stay C. Diabetic patients have better outcomes with gastric versus postpyloric feeings D. Postpyloric feeing more closely simulates normal physiologic feeing E. Gastric feeing is associate with increase total nutrition
12. A 55-year-ol obese female with chronic obstructive pulmonary isease (COPD) is unergoing preoperative evaluation for ventral hernia repair. She has a 30 pack/year smoking history, though she quit 1 year ago. Her COPD symptoms are well controlle with her current meication regimen, an her last amission for COPD exacerbation was over years ago. Which of the following is true regaring risk assessment for postoperative pulmonary complications in this patient? A. Higher ASA class is a signicant risk factor B. Preoperative pulmonary function tests (PFTs) shoul be obtaine C. A nasogastric tube shoul be use postoperatively to ecrease pulmonary complications D. Upper miline an lower miline laparotomy confer similar risk for pulmonary complications E. A PaCO of more than 45 mmHg is an absolute contrainication to major abominal surgery 13. A 65-year-ol woman is amitte to the hospital with a large bowel obstruction. Workup reveals a sigmoi cancer, an on hospital ay 4, she unergoes laparoscopy with a plan to perform a resection with a proximal colostomy. During the operation, her en-tial carbon ioxie suenly rops, an she evelops tachycaria to the 10s with occasional premature atrial contractions. Her systolic bloo pressure is 80 mmHg. Which of the following woul be most helpful in establishing the presumptive iagnosis? A. Electrocariogram B. Cariac enzymes C. Transesophageal echocariogram (TEE) D. Arterial bloo gas E. Flexible bronchoscopy
CHAPtEr 35 Preoperative Evaluation and Perioperative Care
14. A 59-year-ol male with a coronary artery bypass grafting 1 year prior for multivessel isease unergoes a right hip replacement surgery. His postoperative course is complicate by pneumonia requiring mechanical ventilation. Electrocariogram shows a stable Q wave in lea II. Heart rate is 80 beats per minute an bloo pressure is 116/8 mmHg. Chest raiograph shows bilateral patchy inltrates. Laboratory exam emonstrates PaO of 70 mmHg, a white bloo cell count of 17,000 cells/μL, an hemoglobin of 7.4 g/L. Which of the following is true regaring the management of his anemia? A. Bloo transfusion will lower his risk of eveloping an acute coronary synrome B. He shoul be transfuse to a hemoglobin goal of 10 g/L C. Re bloo cell transfusion is inepenently associate with lower mortality D. Bloo transfusion is not necessary at this time E. Hemoglobin-base oxygen carriers offer a goo alternative to transfusion in this patient 15. Four ays after a pancreaticouoenectomy for pancreatic aenocarcinoma, a 65-year-ol man evelops a fever an tachycaria. Exam reveals tenerness, eema, an erythema over the angle of the jaw. Which of the following is true regaring this conition? A. It is usually ue to Staphylococcus B. Massage of the area is benecial C. It can be prevente with antibiotics D. The incience has been increasing E. It can be avoie with the use of anticholinergics 16. Which of the following is true regaring venous thromboembolism (VTE) prophylaxis in surgical patients? A. Intermittent pneumatic compression (IPC) prevents DVT by increasing circulating tissue plasminogen activator (tPA) B. Thigh-high IPC is superior to knee-high IPC C. IPC is equivalent to pharmacologic prophylaxis in the majority of patients D. Unfractionate heparin (UFH) is superior to lower-molecular-weight heparin (LMWH) E. LMWH is superior to IPC
441
17. A 69-year-ol patient with a tumor at the rectosigmoi junction unergoes laparoscopic sigmoi colectomy. Postoperative pain is well controlle with patient-controlle thoracic epiural anesthesia. On postoperative ay 1, prophylactic anticoagulation is starte with lowmolecular-weight heparin (LMWH). The blaer is unergoing rainage with an inwelling Foley catheter. Which of the following is true regaring epiural anesthesia? A. Blaer catheterization shoul continue while the thoracic epiural is in place B. LMWH shoul be hel for 4 hours before removal of the thoracic epiural C. The risk of urinary tract infection is the same regarless of whether the urinary catheter is remove on postoperative ay 1 versus postoperative ay 3 D. Risk of urinary retention is not signicantly higher with early removal of the Foley catheter E. Unfractionate heparin shoul not be restarte for at least 4 hours after removal of an epiural catheter 18. A 5-year-ol woman evelops a fever of 104°F 1 hours after an open cholecystectomy. On examination, she has foul-smelling, purulent rainage from her woun. She unergoes the appropriate treatment, an culture of the woun grows gram-positive ros. Which of the following is true regaring this patient an her conition? A. The causative organism is an aerobe B. Diabetes is not consiere to be a risk factor C. Broa-spectrum antibiotics an ui resuscitation resolve the majority of cases D. The organism prouces an enotoxin E. Clinamycin shoul be inclue in the management 19. A 34-year-ol woman unergoes a subtotal thyroiectomy for Graves isease. In the recovery room, she evelops anxiety an progressive respiratory istress with strior. Her incision is bulging an tense on exam. The most important initial step woul be: A. Nebulize racemic epinephrine B. Rapi-sequence intubation C. Neele aspiration of the neck woun D. Ultrasoun examination of the neck E. Rapily opening the incision at the besie
442
PArt ii Medical Knowledge
20. One ay after a left colectomy for recurrent iverticulitis, a patient is note to have an elevation of his serum creatinine. Other laboratories are unremarkable. He has a urine output of 30 to 50 mL/hour. A renal ultrasoun shows no evience of abnormalities with the exception of ascites. Compute tomography (CT) scan emonstrates iscontinuity of the left ureter with contrast extravasation at the level of the pelvic brim. Which of the following about this injury is true? A. Immeiate reoperation shoul not be performe B. Placement of ureteral stents woul have prevente this complication C. A percutaneous nephrostomy shoul be place D. A retrograe stent shoul be place E. A ureteroneocystostomy will likely be the best option 21. Two ays after sustaining signicant crush injury to her bilateral lower extremities from a motor vehicle collision, a 3-year-ol female becomes oliguric an is only proucing scant ark urine. Urine ipstick reveals 4+ bloo, an follow-up urinalysis shows 5 to 10 re bloo cells per high power el. Prevention of acute kiney injury is best achieve by which of the following? A. Urgent 4-compartment fasciotomies B. Loop iuretics C. Vigorous IV ui hyration D. Alkalization of urine with intravenous soium bicarbonate E. Mannitol 22. Which of the following is true regaring PFTs? A. Total lung capacity (TLC) is generally reuce with aging B. A preoperative force expiratory volume in one secon (FEV1) of less than 1.5 L is a contrainication for pulmonary lobectomy C. Diffusion capacity of the lungs for carbon monoxie (DLCO) will stay relatively constant with age so long as there is no intrinsic lung isease D. Percent-preicte postoperative FEV1 of >40% is acceptable for a lobectomy but not for a pneumonectomy E. Chest wall compliance ecreases with age
23. A 45-year-ol male with en-stage renal isease is unergoing placement of a tunnele hemoialysis catheter. During the operation, the anesthesiologist notices a sharp ecline in the continuous capnography an the calculate physiologic ea space is increase. This is followe by massive myocarial infarction an cariac arrest. Which of the following is true regaring this conition? A. Electrocariogram (ECG) will most commonly emonstrate right heart strain B. A congenital heart efect likely contribute to the cariac arrest C. The patient shoul be positione left sie up D. Besie transesophageal echocariography is generally not sensitive enough to etect this complication E. Aspiration from the central line is usually helpful 24. A 65-year-ol man with Barrett esophagus an new-onset ysphagia is being evaluate for iagnostic esophagogastrouoenoscopy (EGD), enoscopic ultrasoun (EUS), an mucosal biopsy. He is on warfarin for mechanical mitral valve an has a history of embolic stroke 10 years ago. What is recommene for his anticoagulation regimen before this proceure? A. Hol warfarin for 3 to 5 ays an brige with low-molecular-weight heparin B. Hol warfarin for 48 to 7 hours, brige with unfractionate heparin, an hol heparin 4 to 6 hours before the proceure C. Perform EGD an EUS while therapeutic on warfarin; if inicate, the mucosal biopsy can be performe at a later ate after holing warfarin D. Continue warfarin without interruption E. Hol warfarin 3 to 5 ays before proceure an restart within 4 hours after the proceure
CHAPtEr 35 Preoperative Evaluation and Perioperative Care
25. A 4-year-ol female with long-staning systemic lupus erythematosus (SLE) complicate by lupus nephritis an ebilitating arthritis is in the ICU following an emergency bowel resection 4 ays earlier. Over the next several hours, she becomes febrile, hypotensive, an complains of abominal pain. She is given ui boluses, but the bloo pressure oes not respon. Her abominal exam is unremarkable. Laboratory values reveal a white bloo cell count of 1,000 cells/L with eosinophilia, serum Na of 133 mEq/L, serum bicarbonate of 0 mEq/L, an serum K of 5.3 mEq/L. Which of the following represents the best management of this conition? A. Two liters of normal saline followe by 4 mg of examethasone B. Exploratory laparotomy C. Vasopressin D. Immeiate aministration of broa-spectrum antibiotics an 100 mg of hyrocortisone E. Flui resuscitation, vasopressor support, an AM cosyntropin test
443
26. Which of the following is true regaring the use of beta-blockers in the perioperative perio for patients unergoing noncariac surgery? A. Starting a beta-blocker within 4 hours of surgery may increase the incience of perioperative stroke B. Beta-blockers shoul be stoppe at least 1 week before surgery C. In low- an intermeiate cariac risk patients, beta-blockers shoul be initiate to 3 weeks before surgery D. Beta-blockers shoul be avoie even in the high cariac risk group E. Perioperative initiation of beta-blocker ecreases the 30-ay mortality
Answers 1. D. The management of antiplatelet meications in
patients unergoing noncariac surgery after PCI poses a common surgical ilemma. In general, patients shoul be stratie by thrombotic risk base on the type an timing of PCI as well as hemorrhagic risk base on the type of surgery. Patients who unerwent plain ol balloon angioplasty within the past weeks, bare metal stent within the past 1 month, an DES within the past 6 months are at high thrombotic risk. In general, surgery shoul be elaye until after this perio (A–C, E). For most abominal operations, continuing aspirin while holing clopiogrel 5 ays prior to surgery is appropriate. However, in the case of time-sensitive surgery such as a colectomy in a patient with colon cancer, surgery shoul not be elaye an the risks/benets of continuing ouble antiplatelet therapy shoul be iscusse with the patient. Of note, there is recent literature supporting the continuation of aspirin an clopiogrel in patients with colon cancer unergoing resection, even within a couple months of DES placement. Reference: Banerjee S, Angiolillo DJ, Boen WE, et al. Use of antiplatelet therapy/DAPT for post-PCI patients unergoing noncariac surgery. J Am Coll Cardiol. 017;69(14):1861–1870.
2. E. Infection with C. difcile is not an uncommon com-
plication following antibiotic treatment. While it has been classically associate with clinamycin, it can occur following treatment with a wie variety of antibiotics. For initial episoes, infection can either be nonsevere (leukocytosis 1.5), or fulminant (shock, toxic
megacolon). Nonsevere an severe infections are treate with either PO vancomycin or PO axomicin (if available) ×10. In cases of fulminant isease, treatment shoul inclue PO vancomycin an consieration of total abominal colectomy with en ileostomy. In cases of fulminant isease with ileus, rectal vancomycin an intravenous metroniazole shoul be ae (B, C, D). First recurrences can be treate with PO vancomycin (usual osing if metroniazole was use for the initial episoe or a prolonge pulse/tapere regiment if a stanar PO vancomycin regimen was use for the initial episoe). Alternatively, axomicin may be use if a stanar PO vancomycin regimen was use for the initial episoe. Subsequent recurrences can be treate with antibiotics or fecal transplant (A). Reference: McDonal LC, Gering DN, Johnson S, et al. Clinical practice guielines for Clostridium difcile infection in aults an chilren: 017 Upate by the Infectious Diseases Society of America (IDSA) an Society for Healthcare Epiemiology of America (SHEA). Clin Infect Dis. 018;66(7):987–994.
3. E. Fever, chills, tachycaria, hypotension, an peritoneal
irritation occurring together within 1 week of any surgery involving a new bowel anastomosis shoul immeiately raise suspicion for an anastomotic isruption. Left shouler pain is often a consequence of left iaphragm irritation an, in this case, correlates with the gastric pouch–jejunal anastomosis. Water-soluble contrast stuies can ai in the iagnosis an inicate how large the leak is because containe leaks can often be manage nonoperatively. However, in
444
PArt ii Medical Knowledge
this patient, hypotension an signs of peritonitis necessitate operative exploration an repair of the anastomosis. Gasbloat synrome results from the inability to relieve gas from the stomach after a funoplication (A). Gastric volvulus can occur after gastric surgery; however, this is extremely rare (D). Internal hernia is less likely given the timeline (most occur beyon a month after surgery) an left shouler pain inicative of iaphragmatic irritation (B). Woun ehiscence woul be suspecte if the skin is erythematous, warm, raining purulent or serous material, an has fallen apart (C).
4. A. This case represents an enterocutaneous stula, likely
resulting from either an anastomotic leak or an unrecognize intraoperative bowel injury away from the anastomosis. Management of enterocutaneous stulas shoul begin with stabilizing the patient via aggressive ui hyration an control of sepsis (if present). If the patient is manifesting signs of sepsis, prompt aministration of IV antibiotics shoul be institute. Sepsis, ehyration, an electrolyte/ nutrient losses are the most evastating early consequences. Prompt return to the operating room is not recommene because the peritoneal cavity will likely have highly vascular ahesions, making reentry treacherous, an early attempts to reclose stulas typically fail (D). Once the patient has been stabilize, the best initial stuy is a CT scan of the abomen. This will ientify whether any intraabominal abscesses are present that might require percutaneous rainage an rule out whether there is a istal obstruction (B, C). Fistulas are loosely categorize as high an low output. High output is ene as outputs of more than 500 mL/ay an low output as less than 00 mL/ay. High-output stulas are less likely to close an often cause signicant ui, electrolyte, an nutritional challenges. Factors that preict whether a stula will close (mnemonic “FRIEND”) inclue foreign boy, raiation to the bowel, inammation/infection (such as inammatory bowel isease), epithelialization of the stula tract, neoplasia at the stula site, an distal obstruction. The mortality rate of enterocutaneous stulas remains signicant at 10% to 15%. Approximately 50% close spontaneously. Conservative treatment shoul be continue for at least 6 weeks before any reoperation is performe. Operating before 6 weeks results in higher mortality an stula recurrence rates. Octreotie has not been shown in ranomize trials to ai in earlier stula closure but oes not ecrease mortality (E). Reference: Sancho JJ, i Costanzo J, Nubiola P, et al. Ranomize ouble-blin placebo-controlle trial of early octreotie in patients with postoperative enterocutaneous stula. Br J Surg. 1995;8(5):638–641.
5. B. Colonic ischemia after repair of a rupture abominal
aortic aneurysm occurs in 1% to 6% of operations but can occur in up to 5% of cases uner certain circumstances. The greatest risk factor is the presence of prolonge hypotension preoperatively. In a patient with stable bloo pressure, age, time to operation, an the presence of cariac isease have little effect on the incience of colonic ischemia after aortic repair (A–C, D). Patency of a patient’s inferior mesenteric artery is not a goo preictor of colonic ischemia because of signicant avenues of collateral ow (E). Symptoms an signs of ischemia inclue blooy iarrhea, abominal pain/ istention, an elevate white bloo cell count. If the patient has evience of peritonitis, urgent reoperation is inicate.
Otherwise, urgent enoscopy is require to view the colonic mucosa. The majority of cases of colonic ischemia can be manage nonoperatively with bowel rest, hyration, an IV antibiotics. If the patient requires colon resection, mortality rates are as high as 75%.
6. B. This case represents a cecal stula. The most common
causes are slippage of the suture or necrosis of the remaining appeniceal stump, leaing to leakage of the enteric contents into the peritoneal cavity (A). Rarely, the stula results from unrecognize Crohn isease, malignancy, tuberculosis or istal colon obstruction. Cecal stulas are low-output stulas an are not associate with losses of large amounts of ui, electrolytes, or nutrients (D). Therefore, TPN is not necessary to maintain aequate nutrition (C) an mortality rates are low in the absence of other serious complications (A). Spontaneous closure is promote in as many as 75% of patients maintaine on low-resiue iets because absorption is mostly complete by the time the contents reach the cecum (B, E).
7. C. Duoenal stump blowout occurs after Billroth II oper-
ations, where back pressure on the uoenal stump results in breakown of the stump closure, leaing to abominal sepsis an peritonitis. Acute pancreatitis is associate postoperatively with Billroth II gastrectomy an jejunostomy, in which increase intrauoenal pressure can cause backow of activate enzymes into the pancreas but is unlikely to cause peritonitis (A). Woun ehiscence is characterize as suen ramatic rainage of relatively large volumes of a clear, salmon-colore ui an is apparent on physical exam (D). Acalculous cholecystitis can also occur postoperatively; however, the clinical presentation woul mainly consist of right upper quarant pain (B). Intraabominal hemorrhage woul be less likely to present with sepsis (E).
8. E. Smoking is a preictor of postoperative pulmonary
complications. The respiratory epithelium is altere in smokers, an poor ciliary activity combine with the prouction of more viscous mucus leas smokers to be more reliant on coughing to clear secretions from their lungs. Several ays after patients have stoppe smoking, there may be a transient increase in sputum volume. The above reasons have typically prevente health professionals from encouraging smoking cessation in the weeks leaing up to surgery. However, a metaanalysis publishe by the American Meical Association has conclue that the concern that stopping smoking only a few weeks before surgery might worsen clinical outcomes is unfoune an clinicians shoul avise smoking cessation as soon as possible (D). Postoperative lung expansion moalities (A–C) reuce postoperative pulmonary complications, although there is no ae benet from using all three. Routine use of a nasogastric tube may increase aspiration risk because the tube stents open the gastroesophageal junction. However, selective use in patients with nausea, bloating, an/or vomiting is probably protective. References: Bluman LG, Mosca L, Newman N, Simon DG. Preoperative smoking habits an postoperative pulmonary complications. Chest. 1998;113(4):883–889. Lawrence VA, Cornell JE, Smetana GW, American College of Physicians. Strategies to reuce postoperative pulmonary complications after noncariothoracic surgery: systematic review for the American College of Physicians. Ann Intern Med. 006;144(8):596–608.
CHAPtEr 35 Preoperative Evaluation and Perioperative Care Myers K, Hajek P, Hins C, McRobbie H. Stopping smoking shortly before surgery an postoperative complications: a systematic review an meta-analysis. Arch Intern Med. 011;171(11):983–989.
9. D. A serum albumin less than 3.5 g/L is the single most
important laboratory preictor of averse pulmonary events after surgery. Bloo urea nitrogen (>1 mg/L) is also useful, although the correlation is not as strong (A). Routine spirometry for all operations oes not seem to a value beyon a careful history an physical examination (B). An exception for the use of spirometry woul be for lung resection. Chest raiograph an arterial bloo gas are iagnostic stuies that woul only be preictive of postoperative complications if there were abnormal nings (C, E). References: Lawrence VA, Cornell JE, Smetana GW, American College of Physicians. Strategies to reuce postoperative pulmonary complications after noncariothoracic surgery: systematic review for the American College of Physicians. Ann Intern Med. 006;144(8):596–608. Qaseem A, Snow V, Fitterman N, et al. Risk assessment for an strategies to reuce perioperative pulmonary complications for patients unergoing noncariothoracic surgery: a guieline from the American College of Physicians. Ann Intern Med. 006;144(8):575–580.
10. E. Thrombosis of supercial an eep veins of the
upper extremity is cause by intravenous catheters in most cases. Upper extremity DVT oes pose a risk of pulmonary embolus, though less risk than pelvic an lower extremity DVT. Management begins with anticoagulation an etermining the necessity for the line; in the above case, there is a continue nee for TPN via a PICC line as the patient has not yet emonstrate a return of bowel function. Stuies have shown that it is not necessary to remove the PICC line espite the DVT. Therapeutic anticoagulation for 3 to 6 months is recommene. Thus, removal of the catheter without anticoagulation is not acceptable because there is a risk of PE (B). Inications to remove a line inclue infection an a contrainication to anticoagulation. If the line is to be remove, anticoagulation is still recommene; however, the recommenation is to wait 5 to 7 ays after the initiation of heparin before removing it (ue to the theoretic fear that pulling the line with a fresh clot might isloge the thrombus) (C, D). Warm compresses an NSAIDs woul be appropriate for supercial thrombophlebitis (A). Reference: Kucher N. Clinical practice. Deep-vein thrombosis of the upper extremities. N Engl J Med. 011;364(9):861–869.
11. A. While there are many theoretic avantages between
each metho of feeing, a 015 Cochrane review comparing postpyloric an gastric feeings showe only two signicant ifferences: lower rates of pneumonia in the postpyloric group an some evience for increase total nutrition elivere in the postpyloric group (E). There was no signicant ifference in length of ICU stay, mortality, or time on the ventilator (B). There was also no signicant ifference in associate complications with tube placement between the two stuy groups. While postpyloric feeing was associate with a longer time to initiation of tube feeing, this i not seem to affect the time it took to reach nutritional goals. Avantages of gastric feeing inclue a better approximation of normal physiology, ease of placement, an convenience (D). It may be a reasonable choice in patients without risk for aspiration, but patients with elaye gastric emptying
445
(which is common in ICU patients), iabetes, an gastroesophageal reux shoul be consiere for postpyloric fees (C). In terms of timing, there is abunant evience that earlier enteral feeing in critically ill patients results in better outcomes. Reference: Alkhawaja S, Martin C, Butler RJ, Gwary-Srihar F. Post-pyloric versus gastric tube feeing for preventing pneumonia an improving nutritional outcomes in critically ill aults. Cochrane Database Syst Rev. 015;(8):CD008875.
12. A. Patient-relate risk factors for the evelopment
of postoperative pulmonary complications inclue: age more than 50 years, COPD, congestive heart failure, American Society of Anesthesiologists (ASA) class greater than , serum albumin less than 3.5 g/L, obstructive sleep apnea, pulmonary hypertension, an current smoking. While a preoperative PaCO greater than 45 oes increase the surgical risk, there is currently no enitive number that prohibits abominal surgery (E). Current American College of Physicians Guielines recommen against the routine use of preoperative chest raiography or PFT (B). Although it is important to ientify patients with COPD, an some COPD patients may benet from preoperative interventions, patients who require aitional testing can be ientie by history of new symptoms or physical examination nings. Thus, PFT shoul be restricte to those who have current symptoms or signs base on history an physical. Location of the surgical incision is an important risk factor for postoperative pulmonary complications, with incisions closer to the iaphragm inferring more risk (D). When patients have been ientie as high risk for pulmonary complications, current evience supports the use of perioperative eep-breathing exercises an incentive spirometry. Routine use of nasogastric ecompression has been associate with increase rates of pneumonia an atelectasis. Current recommenations are for more selective use in patients with nausea, vomiting, or gastric istention (C). Reference: Qaseem A, Snow V, Fitterman N, et al. Risk assessment for an strategies to reuce perioperative pulmonary complications for patients unergoing noncariothoracic surgery: a guieline from the American College of Physicians. Ann Intern Med. 006;144(8):575–580.
13. C. The ifferential iagnosis for a suen rop in
en-tial CO in the operating room (OR) inclues an obstructe airway, acciental extubation, isconnection of the circuit, cariac arrest, an pulmonary embolism (PE). The patient escribe has at least three risk factors for PE incluing malignancy, a heart rate greater than 100, an more than 3 ays of immobilization (Wells criteria). PE is estimate to occur in 1% to % of surgical patients in the perioperative perio. While yspnea, anxiety, tachycaria, an tachypnea are the most common nings in awake patients, physical signs of PE will be limite in patients uner general anesthesia. In this situation, an astute clinician can recognize PE as presenting with hypotension, tachycaria, ecrease en-tial CO, an hypoxemia. In general, laparoscopic proceures have been associate with a low risk of both fatal an nonfatal PE (E). This complication is associate with 10% to 15% mortality in the perioperative perio (D). Electrocariogram changes have been shown to be present in up to 83% of patients, but they
446
PArt ii Medical Knowledge
are generally nonspecic (A). Uncommonly, PE can present with a prominent S wave in lea 1, Q wave in lea 3, an inverte T wave in lea 3; this is suggestive of right heart strain. Despite potential cariovascular consequences of massive PE, an elevate cariac enzyme level occurs in less than 50% of cases an is not specic for PE (B). The two most sensitive tests that can be one to help iagnose PE are a TEE an calculating the physiologic ea space to look for elevations (though it can be time-consuming). TEE has been shown to yiel a iagnosis in an average of 9.6 minutes with a sensitivity of 80% an specicity of 97% an is ieal in the OR setting. Although TEE is relatively poor at visualizing the PE, it is excellent at emonstrating right heart strain, which provies inirect evience of PE. Arterial bloo gas in awake patients with PE may emonstrate a low CO, but this may not be the case for a ventilate patient uner general anesthesia (D). Flexible bronchoscopy is not helpful in iagnosing PE (E). Reference: Desciak MC, Martin DE. Perioperative pulmonary embolism: iagnosis an anesthetic management. J Clin Anesth. 011;3():153–165
14. D. Re bloo cell transfusion has been inepenently
associate with longer intensive care unit (ICU) an hospital stays, increase complications, an increase mortality (C). It is also an inepenent risk factor for multiorgan system failure an systemic inammatory response synrome (SIRS). Most societal guielines agree that a liberal transfusion strategy (goal of 10 g/L) is no better an likely worse than a more restrictive strategy (goal of 7–9 g/L) in the majority of patients. So transfusion is neee as long as the hemoglobin remains above 7 g/L. The Transfusion Requirements in Critical Care (TRICC) trial emonstrate that critically ill patients without active bleeing fare better with a restrictive transfusion strategy. There is also no evience that transfusion lowers the risk of acute coronary synromes. (A, B). While hemoglobin can improve oxygen elivery to tissues, it has not been shown to lower risk of acute coronary synromes, ecrease time on the mechanical ventilator, improve oxygen consumption, or improve outcomes in patients with ault respiratory istress synrome or acute lung injury (A). Because of the negative effects of bloo transfusion, alternative methos of managing anemia are actively being researche, incluing the use of recombinant human erythropoietin (EPO) an hemoglobin-base oxygen carriers. Trials esigne specically looking at aministering exogenous EPO in trauma patients, the EPO-1 an EPO- trials showe reuctions in require bloo transfusions an improve mortality, respectively. While hemoglobin-base oxygen carriers show some promise, they are not currently approve for use in the Unite States (E).
15. A. This patient has postoperative parotitis. This most
commonly occurs in elerly patients with poor oral hygiene, poor oral intake, prolonge nasogastric tube ecompression, an ehyration, all leaing to a ecrease in saliva prouction. The pathophysiology involves obstruction of the salivary ucts with seconary infection an is more common in the iabetic or immunocompromise patient. Most patients will be iagnose with parotitis 4 to 1 ays postoperatively. Signs an symptoms begin with pain an tenerness over the angle of the jaw that can then progress to high fevers an
leukocytosis, as well as signicant eema involving the oor of the mouth. If left uniagnose an untreate, it can lea to life-threatening sepsis. Initial treatment is with high-ose broa-spectrum antibiotics with Staphylococcus coverage (most common organism) an warm compresses (B). If the patient oes not improve, surgical incision an rainage are inicate. In extreme cases involving progressive airway obstruction, emergent tracheostomy may be inicate. Use of measures to stimulate salivary ow, such as sucking on cany, seems to help prevent this complication, but prophylactic antibiotics are generally not inicate (C). Within improve oral hygiene, the incience of this rare complication is eclining (D). Aitionally, the use of anticholinergics will ecrease salivary ow an increase the risk of eveloping postoperative parotitis (E).
16. E. VTE prophylaxis is generally ivie into two
categories: pharmacologic an mechanical. Mechanical prophylaxis inclues static compression evices (like grauate compression stockings) an IPC evices. While grauate compression stockings work primarily by preventing venous stasis in the legs, IPC combines that with its effects on the intrinsic brinolytic system. It was originally hypothesize that intermittent compression cause the release of agents like tPA from the vascular enothelium. However, when these levels are irectly measure, they seem to be relatively constant espite an increase in tPA activity. The currently propose mechanism is relate to measure ecreases in plasminogen activator inhibitor-1 (PAI-1), which functions as a tPA inhibitor (A). Currently, there is no evience that one IPC evice is superior to another in preventing VTE (B). While there are relatively few contrainications to mechanical prophylaxis, traumatic injury to the extremity an evience of ischemia seconary to peripheral vascular isease are both contrainications. Aitionally, patients with conrme DVT in the lower extremity shoul not be place on IPC. Limitations of its usefulness are primarily relate to interruption in treatment an improper application. Comparison between mechanical an pharmacologic VTE prophylaxis shows that in certain low-risk patients there is an equivalent reuction in the incience of DVT an PE, though combination therapy is superior to mechanical prophylaxis alone (C). It can be consiere as sole therapy in low-risk patients an patients with contrainications to pharmacologic agents. In terms of pharmacologic prophylaxis, UFH an LMWH are the two most commonly use agents. LMWH is generally regare as more effective, especially in certain populations (e.g., trauma patients) (D). References: Comerota AJ, Chouhan V, Haraa RN, et al. The brinolytic effects of intermittent pneumatic compression: mechanism of enhance brinolysis. Ann Surg. 1997;6(3):306–314. Ho K, Tan J. Stratie meta-analysis of intermittent pneumatic compression of the lower limbs to prevent venous thromboembolism in hospitalize patients. Circulation, 013;18(9), 1003–100. Morris R, Woocock J. Evience-base compression. Ann Surg. 004;39(), 16–171.
17. D. Epiural anesthesia is an excellent tool to control
postoperative pain an has been shown to ecrease cariac morbiity, an as such, it has been gaining popularity in clinical practice. Routine use of urinary rainage in the setting of
CHAPtEr 35 Preoperative Evaluation and Perioperative Care epiural anesthesia remains controversial. However, postoperative ay 1 removal of the Foley with thoracic epiurals has been shown to signicantly ecrease the incience of urinary tract infections with minimal change to the rate of urinary retention as measure by rates of recatheterization (A–C). Current recommenations for the placement an removal of epiural catheters in patients receiving prophylactic LMWH is intene to prevent an epiural hematoma an subsequent paralysis. For the placement of epiural catheters, LMWH must be hel at least 4 hours before placement, an it shoul not be remove within 1 hours of the last ose. Prophylactic anticoagulation can be restarte 6 hours after placement an no sooner than 4 hours after removal of the epiural (B). Unfractionate heparin may be restarte after 1 hour (E). References: FDA Safety Information an Averse Event Reporting Program. Low Molecular Weight Heparins: Drug Safety Communication – Upate recommenations to ecrease risk of spinal column bleeing an paralysis in patients on low molecular weight heparins. U.S. Foo an Drug Aministration; 013. https://www. fa.gov/meia/87316/ownloa. Henren S. Urinary catheter management. Clin Colon Rectal Surg. 013;6(3):178–181. Horlocker TT. Regional anaesthesia in the patient receiving antithrombotic an antiplatelet therapy. Br J Anaesth. 011;107 Suppl 1:i96–i106. Townsen CM Jr, Beauchamp RD, Evers BM, Mattox KL. Sabiston textbook of surgery: the biological basis of modern surgical practice. 17th e. WB Sauners; 004. Zaouter C, Kaneva P, Carli F. Less urinary tract infection by earlier removal of blaer catheter in surgical patients receiving thoracic epiural analgesia. Reg Anesth Pain Med. 009;34(6):54–548.
18. E. Postoperative necrotizing soft-tissue infection is
a rare but well-escribe complication. The escription of “ishwater pus” is classic for a postoperative clostriial woun infection. The causative organisms are typically Streptococcus pyogenes or Clostridium perfringens. C. perfringens is an anaerobic gram-positive ro that prouces alphatoxin; this is a virulent exotoxin that leas to extensive tissue necrosis an cariovascular collapse. Immunocompromise patients (iabetes, malignancy, chronic liver isease) are at increase risk (A–D). Clinamycin has been shown to limit toxin prouction, which ecreases the virulence, slows tissue estruction, an can potentially reuce inammatory cytokine release. Effective therapy requires rapi aministration of broa-spectrum antibiotics incluing aerobic coverage (C) an source control via emergent operative excision of necrotic infecte tissue, incluing fascia. Conservative management is not appropriate if a necrotizing soft-tissue infection is suspecte (B). Reference: Hakkarainen TW, Kopari NM, Pham TN, Evans HL. Necrotizing soft tissue infections: review an current concepts in treatment, systems of care, an outcomes. Curr Probl Surg. 014;51(8):344–36.
19. E. Postoperative hematomas after neck surgery (thy-
roi, parathyroi, caroti artery) can have catastrophic consequences. Physical examination nings can be eceptively benign. Attempts at intubation may be hampere by tracheal compression an eviation (B). Furthermore, the recent neck issection, combine with the hematoma, causes venous an lymphatic obstruction, leaing to airway eema,
447
further compromising attempts at intubation. Rapily opening the incision at the besie is necessary because urgent ecompression is the fastest way to restore proper respiratory function. Denitive hemostasis must then be obtaine in the operating room. Although ultrasonography is an important iagnostic ai for hematomas (D), clinical suspicion is sufcient in this emergent situation an the urgency of ecompression oes not permit waiting for an ultrasoun examination. Neele aspiration woul not be sufcient (C). While nebulize racemic epinephrine is use for the treatment of strior in conitions like croup, it is not appropriate when the cause of strior is external compression of the airway (A).
20. E. The ureters rst pass meial to the psoas muscle an
travel alongsie the transverse processes of the lumbar vertebrae an cross anterior to the common iliac arteries near the bifurcation into the internal an external iliac arteries. The anatomic position places the ureters at risk for injury uring pelvic surgery, an the situation is particularly precarious when inammation, abscess, an/or phlegmon are present. The highest risk of ureteral injury is uring an abominoperineal resection. During mobilization of the left colon an ligation of the inferior mesenteric artery, visualization an protection of the ureter from injury are imperative. Placement of ureteral stents before the operation may help to ientify the ureters an assist with ientifying an injury intraoperatively, but this oes not seem to correlate with a reuction in the number of injuries (B). The presence of blue ye in the operative el after intravenous aministration of inigo carmine or methylene blue is iagnostic for injury to the ureter. The ecision to immeiately reoperate is base on the elay associate with injury recognition, the severity of the injury, an whether the patient has evelope urosepsis. If iscovere within a week postop, reoperation is generally recommene (A). Beyon 10 ays, the inammation present will make reoperation hazarous. In this latter case, percutaneous nephrostomy an/or retrograe rainage with a ureteral stent is inicate (C, D). The type of repair epens on the location an extent of the injury. For miureter injuries, a ureteroureterostomy is preferre. For pelvic injuries, ureteroneocystostomy is neee. If this is not possible, a psoas hitch or a Boari ap (from the blaer) may be neee. Reference: Bothwell WN, Bleicher RJ, Dent TL. Prophylactic ureteral catheterization in colon surgery. A ve-year review. Dis Colon Rectum. 1994;37(4):330–334.
21. C. Crush injury to the extremities causing signicant
muscle injury is often complicate by rhabomyolysis, which can lea to acute renal failure. Degraation proucts of both hemoglobin an myoglobin are toxic to the nephron in aciic urine. Elevate serum creatine phosphokinase, hyperkalemia, an the presence of heme without a signicant amount of re bloo cells on urinalysis are inicative of rhabomyolysis. Management consists of aggressive IV hyration to maintain a urine output of more than 100 mL/hour an shoul begin with infusion rates of at least 00 cc/hour. Myoglobin concentrates in the renal tubules precipitates when it comes in contact with Tamm-Horsfall protein. This precipitation is enhance uner aciic conitions. Routine aministration of bicarbonate (D) an mannitol (E) in the prevention of acute
448
PArt ii Medical Knowledge
kiney injury from rhabomyolysis is controversial, but, theoretically, alkalinization of the urine increases the solubility of the myoglobin–Tamm-Horsfall protein P complex an shoul increase myoglobin washout. It also prevents lipi peroxiation an renal vasoconstriction an seems to have relatively few negative sie effects if use in patients without high serum bicarbonate an without alkalosis. However, it reuces the amount of ionize calcium so shoul be use with caution in patients with hypocalcemia. Historical treatment of rhabomyolysis has inclue force iuresis with mannitol, but its routine use is being questione now in the literature an may actually increase the risk of eveloping renal failure. A retrospective stuy publishe in The Journal of Trauma in 004 looking at over 000 patients with elevate creatine kinase showe no ifference in renal failure, nee for hemoialysis, or mortality in patients receiving bicarbonate an mannitol versus volume resuscitation alone. Mannitol may ai in ecreasing muscle swelling an compartment pressures, but the mainstay of treatment remains ecompression of muscle compartments (A). However, in the case of crush injury, normal compartment pressures woul not change your strategy for preventing acute kiney injury because tissue amage alone coul cause the release of myoglobin. Loop iuretics are not use in the prevention of acute kiney injury in the setting of rhabomyolysis (B). While retrograe urethrogram woul assist in the iagnosis of misse urethral injury, the urine is positive on the ipstick from myoglobin, not hemoglobin. References: Brown CVR, Rhee P, Chan L, Evans K, Demetriaes D, Velmahos GC. Preventing renal failure in patients with rhabomyolysis: o bicarbonate an mannitol make a ifference? J Trauma. 004;56(6):1191–1196. Holt SG, Moore KP. Pathogenesis an treatment of renal ysfunction in rhabomyolysis. Intensive Care Med. 001;7(5):803–811.
22. E. Pulmonary function testing generally inclues three
separate tests: spirometry, lung volumes, an the iffusion capacity of the lungs. Expecte changes with aging inclue an increase in the functional resiual capacity an the resiual volume, with a corresponing ecrease in the vital capacity. This reciprocal change generally means the TLC is preserve (A). DLCO will also ecrease with age (C). Compliance of the lung can be misleaing because even though the compliance of the lung tissue itself increases with age, the chest wall compliance is signicantly reuce. In general, this means that the overall compliance of the pulmonary system is reuce. Preoperative pulmonary function tests are manatory for the evaluation of potential pulmonary resection. The preoperative values to remember are FEV1 greater than L for pneumonectomy, FEV1 greater than 1.5 L for lobectomy, FEV1 greater than 80% preicte, an DLCO greater than 80% preicte. However, these numbers are not absolute inications, an failure to meet them simply necessitates more workup; this inclues getting a ventilation/perfusion scan to etermine the contribution of the preicte segment (B). If the percent-preicte postoperative FEV1 an DLCO are greater than 60%, then the patient is a caniate for resection of the propose segment without further testing (D). If the percent-preictive postoperative FEV1 an DLCO are less than 60%, exercise tolerance shoul be teste. Reference: Brunelli A, Kim AW, Berger KI, Arizzo-Harris DJ. Physiologic evaluation of the patient with lung cancer being consiere for resectional surgery: iagnosis an management of lung
cancer, 3r e: American College of Chest Physicians evience-base clinical practice guielines. Chest. 013;143(5 Suppl):e166S–e190S.
23. B. Venous air embolism is a rare an typically asymp-
tomatic conition. Though it is often associate with the placement of central venous access catheters, it has been associate with other conitions incluing trauma, hea an neck proceures, an neurosurgical proceures. When suspecte, the patient shoul be immeiately place in Trenelenburg position an left lateral ecubitus or right sie up (Durant maneuver) (C). This maneuver is esigne to trap the air embolus in the right ventricle an prevent it from going into the pulmonary arteries. Physical exam nings inclue jugular venous istention, millwheel murmur, an a sucking soun as air enters the venous system through a catheter. The most sensitive besie test for iagnosis is likely transesophageal echocariography, which can etect even small volumes of air (D). ECG suggestive of right heart strain is associate with pulmonary emboli (A). Treatment inclues correct positioning as previously escribe, increasing inspire oxygen, mechanical ventilation, hyperbaric oxygen, an, as a last resort, close-chest cariac massage to try to force the air out of the pulmonary arteries an into the smaller capillaries of the lung. An attempt can be mae to aspirate the air from the ventricle either through an existing central line or irectly through the chest wall, but the return of air with these proceures is generally low (E). Myocarial infarction is uncommon with venous air embolism an is typically the result of the air entering the arterial system via a congenital heart efect, such as a patent foramen ovale, an occluing the coronary arteries. The volume for a fatal venous air embolism is typically estimate at 3 to 5 mL/kg injecte at a rate of 100 mL/s, but these are largely base on animal stuies. The volume is much lower if the air enters the arterial system. Reference: Gory S, Rowell S. Vascular air embolism. Int J Crit Illn Inj Sci. 013;3(1):73–76.
24. D. This patient is scheule for a low-risk enoscopic
proceure an represents a high risk for thromboembolic events (mechanical valves an previous thromboembolic events), so anticoagulation shoul be continue without interruption. When consiering enoscopic proceures for patients on anticoagulation or antiplatelet therapy, three main things nee to be consiere: the urgency of the proceure, the patient’s risk of thromboembolic events (an in this case, the type of valve), an risk of bleeing uring the propose intervention. If the anticoagulation is temporary (e.g., treatment of venous thrombosis) or iscontinuation will be safer at a later ate (e.g., recent myocarial infarction [MI] with stent placement) an the enoscopy is completely elective (such as screening), the enoscopy shoul be elaye. The type of prosthetic valve matters too. Prosthetic mitral valves have a much higher risk of thrombosis than aortic valves (much higher ow) with cessation of anticoagulation. As such, briging is typically not neee for aortic valves. Low-risk enoscopic proceures can safely be performe on therapeutic anticoagulation or antiplatelet therapy an these meications shoul be continue regarless of the intervention. Examples of these proceures are iagnostic enoscopy with mucosal biopsy, ERCP without sphincterotomy, EUS, enteroscopy, an stent eployment. In all
CHAPtEr 35 Preoperative Evaluation and Perioperative Care high-risk enoscopic proceures (polypectomy, sphincterotomy, therapeutic ilation, ne-neele aspiration, enoscopic hemostasis, tumor ablation, cyst gastrostomy, an treatment of varices), anticoagulation shoul be iscontinue with or without briging. However, aspirin an NSAIDs can safely be continue in all enoscopic proceures. For patients on antiplatelet therapy with agents other than aspirin, they shoul be hel 7 to 10 ays before the proceure unless the thromboembolic risks are high, in which case patients may nee to be switche to aspirin or, in the case of ual antiplatelet therapy, aspirin continue an the other agent iscontinue. If the thromboembolic risk is low, anticoagulation can be stoppe an simply restarte after the proceure. For anticoagulation with warfarin in patients with high thromboembolic risk, brige therapy with LMWH or unfractionate heparin shoul be consiere. Use of LMWH an mechanical valves is controversial because of reporte events of fatal thromboembolism on LMWH in these patients. In general, anticoagulation can be restarte within 4 hours after the proceure (A–C, E). Reference: ASGE Stanars of Practice Committee, Anerson MA, Ben-Menachem T, et al. Management of antithrombotic agents for enoscopic proceures. Gastrointest Endosc. 009;70(6):1060–1070.
25. A. Refractory hypotension in the postoperative perio
in patients with conitions such as SLE that are commonly treate with sterois shoul raise concern for acute arenal insufciency. When the iagnosis is suspecte, treatment shoul begin immeiately before conrmatory tests become available (E). Initial treatment consists of: volume resuscitation, laboratory stuies (electrolytes, glucose, arenocorticotropic hormone [ACTH], cortisol), an aministration of either 4 mg of examethasone or 100 mg of hyrocortisone. Dexamethasone is preferre because it will not interfere with cosyntropin stimulation testing, which shoul be one the next morning to conrm the iagnosis. Glucocorticois can then be tapere to regular maintenance oses. Routine aministration of “stress-ose sterois” for patients on longterm corticosterois is not supporte by evience. It is now recommene that patients on long-term sterois shoul not be given “stress-ose” perioperative corticosterois. They shoul be continue on their regular maintenance ose with the consieration of aitional sterois only if they evelop refractory hypotension suggestive of arenal insufciency. While the cosyntropin stimulation test can be instrumental in etecting acute arenal insufciency, its usefulness as a preoperative measure for assessing risk of postoperative arenal crisis is lacking sufcient ata to support its routine use. While septic shock in the early postoperative perio
449
is possible, this vignette provies insufcient ata to point to this iagnosis (C, D). Exploratory laparotomy is not an appropriate option for the above patient (B). References: Brunicari FC, Anersen DK, Billiar TR, Dunn DL, Hunter JG, Matthews JB, Pollock RE. Schwartz’s principles of surgery. 10th e. McGraw-Hill Eucation; 015. Kelly KN, Domajnko B. Perioperative stress-ose sterois. Clin Colon Rectal Surg. 013;6(3):163–167. Marik PE, Varon J. Requirement of perioperative stress oses of corticosterois: a systematic review of the literature. Arch Surg. 008;143(1):1–16.
26. A. The 008 POISE trial was a ranomize controlle
trial to measure the effects of perioperative initiation of beta-blockers. The control group receive a placebo while the stuy arm was starte on metoprolol on the ay of surgery an receive it for 30 ays postoperatively. The stuy foun that patients who receive metoprolol ha a lower incience of myocarial infarction, cariac revascularization, an clinically signicant atrial brillation. However, patients in the stuy arm also ha increase mortality, stroke, hypotension, an braycaria (A, E). This increase in mortality was not seen in the previously publishe DECREASE trials, which also showe a reuction in myocarial infarction. However, several of these stuies were retracte because of falsie ata an questionable ata collection techniques. Without any other large ranomize trials to counter the POISE trial, it has largely become the basis for current guielines regaring perioperative use of beta-blockers. The 014 ACC/AHA guielines for perioperative beta-blocker therapy can be summarize as: (1) Beta-blockers shoul be continue if patients are on them chronically. () Management of beta-blockers after surgery shoul be base on clinical jugment to avoi negative consequences such as hypotension or braycaria (B). (3) Beta-blockers shoul not be starte on the ay of surgery. (4) It is unclear what the risk of starting beta-blockers is in the to 45 ays before surgery (C). (5) It shoul be consiere in high-risk iniviuals (D). References: POISE Stuy Group, Devereaux PJ, Yang H, et al. Effects of extene-release metoprolol succinate in patients unergoing non-cariac surgery (POISE trial): a ranomise controlle trial. Lancet. 008;371(967):1839–1847. Wijeysunera DN, Duncan D, Nkone-Price C, etal. Perioperative beta blockae in noncariac surgery: a systematic review for the 014 ACC/AHA guieline on perioperative cariovascular evaluation an management of patients unergoing noncariac surgery: a report of the American College of Cariology/American Heart Association Task Force on practice guielines. J Am Coll Cardiol. 014;64():406–45.
Transfusion and Disorders of Coagulation CAITLYN BRASCHI, JOON Y. PARK, AND ERIC R. SIMMS
36
ABSITE 99th Percentile High-Yields I. Coagulation Cascae an Factors A. Factor I = brinogen, factor IA = brin, factor II = prothrombin, factor IIA = thrombin B. Intrinsic pathway of coagulation: initiate by expose subenothelial collagen, prekallikrein, high molecular weight kininogen; also involves factors VIII, IX, XI, XII; if impaire, PTT will be elevate C. Extrinsic pathway of coagulation: involves factor VII; if impaire, PT/INR will be elevate D. Common pathway of coagulation: involves factors I, II, V, X E. Factor VII has the shortest half-life of all coagulation factors F. Protein C an S breakown factors V an VIII G. Factor VIII is the only coagulation factor not mae in the liver (mae in the enothelium); von Willebran factor (vWF) also mae in the enothelium II. Bloo Proucts an Drugs for Coagulation Reversal
Contents/mechanism of action
Indications
Notes
Fresh Frozen Plasma (FFP)
All coagulation factors, vWF, Antithrombin III (ATIII)
Warfarin reversal, DIC, TTP, liver disease, AT III deęciency, Factor V deęciency
INR of FFP is 1.4–1.6; takes 1–2 hours to thaw
Cryoprecipitate
I, VIII, XIII, vWF
DIC, vWD type III, hemophilia A, hypoębrinogenemia
Highest concentration of ębrinogen (Factor I)
Prothrombin complex concentrate (PCC)
3-factor: II, IX, X (not used in clinical practice) 4-factor: II, VII, IX, X, C, S
Warfarin reversal in life-threatening bleed (intracranial hemorrhage), reversal of direct Xa inhibitors (rivaroxaban, apixaban)
Immediate warfarin reversal
Recombinant factor VIII
VIII
Hemophilia A
Transfuse to 100% normal factor VIII levels before major surgery
Recombinant factor IX
IX
Hemophilia B
Transfuse to 100% normal factor IX levels before major surgery
Recombinant factor Xa
X
Reverse direct Xa inhibitor
Not widely available
Vitamin K
Cofactor of carboxylation of coagulation factors II, VII, IX, X, C, S
Nonurgent warfarin reversal
Warfarin reversal begins after 6–10 hours, full eěect after 1–2 days
Tranexamic acid (TXA)
Binds plasmin (inhibits ębrinolysis)
Traumatic hemorrhagic shock with hyperębrinolysis
Must be given within 3 hours of injury for beneęt; if patient does not have hyperębrinolysis, TXA increases risk for thromboembolic events
451
452
PArt ii Medical Knowledge
Contents/mechanism of action
Indications
Notes
Aminocaproic acid
Binds plasmin (inhibits ębrinolysis)
tPA-associated bleed DIC
Protamine sulfate
Binds to and inhibits heparin
Heparin overdose with associated bleed
Only partially eěective against LMWH
Desmopressin (DDAVP)
V2 agonist, causes release of vWF and factor VIII from endothelium and platelets
vWD type I/II, uremia
vWD type III has absent vWF, so DDAVP ineěective
III. Anticoagulants
Mechanism of action
Notes
Heparin
Indirect thrombin and factor X inhibitor via ATIII
Ineěective in ATIII deęciency (give FFP with heparin) Aěects the intrinsic pathway (PTT)
Low-molecular-weight heparin (LMWH)
Indirect factor X inhibitor via ATIII
Superior DVT prophylaxis in cancer patients
Coumadin
Vitamin K antagonist
Contraindicated in pregnancy Extrinsic pathway (PT)
Argatroban, bivalirudin, hirudin, dabigatran
Direct thrombin inhibitors
Used in HIT (argatroban ęrst line; bivalirudin if liver failure); Hirudin is irreversible inhibitor; Dabigatran reversal: idarucizumab
tPA, streptokinase, urokinase
Active plasminogen
Monitor ębrinogen levels
Apixaban, rivaroxaban
Direct factor Xa inhibitor
Rivaroxaban reversed with andexanet alfa
IV. Transfusion Reactions
Reaction
Clinical ęndings
Cause
Febrile, nonhemolytic
Fever, pruritus, shivering as Cytokines from nontransfusion is being given; leukoreduced donor most common transfusion product reaction
Related products
Treatment
All products (rarely plasma)
Stop transfusion (although no long-term eěects, need to evaluate why patient is febrile), control symptoms (antipyretics, antihistamines)
Febrile hemolytic Fevers, chills, hypotension, chest/back pain; DIC, hematuria, renal failure
ABO incompatibility
Usually RBCs
Stop transfusion, give Ěuids, hemodynamic support
Urticarial
Hives
IgE reaction to product component
All products
Symptomatic treatment (antihistamines)
Anaphylactic
Hives, hypotension, wheezing, angioedema, hypoxemia
Recipient anti-IgA All products antibodies aĴack donor IgA antibodies, often in IgA-deęcient patients
Stop transfusion, resuscitation, epinephrine
Sepsis
Fevers, chills, hypotension, leukocytosis
Microorganism in stored Usually platelets product (stored at room temp)
Antibiotics, hemodynamic support
TransfusionRespiratory distress, “Two-hit”: neutrophil related lung hypoxemia, fever, sequestration and injury (TRALI) hypotension, leukopenia, activation by donor bilateral inęltrates on CXR; product within 6 hrs of transfusion
All products
Stop transfusion, ventilatory support
Transfusionassociated circulatory overload (TACO)
All products
Diuresis, ventilatory support
Respiratory distress, hypoxemia, JVD, hypertension, pulmonary edema; 6–12 posĴransfusion
Fluid overload; underlying cardiac or renal dysfunction
CHAPtEr 36 Transfusion and Disorders of Coagulation
V. Thromboelastography (TEG) A. TEG is the best way to etermine which bloo proucts shoul be given to a bleeing patient B. Interpretation: 1. R time—time to initial clot formation—if high, lacking coagulation factors -> give FFP . K time—time to brin cross linking—if high, lacking brinogen -> give cryoprecipitate 3. a angel—rate of clot formation—if low, lacking brinogen -> give cryoprecipitate 4. MA (max amplitue)—maximum clot strength—if low, lacking platelets (contributes most to clot strength) -> give platelet 5. LY30—rate of clot lysis—if high, increase brinolysis -> give TXA an/or aminocaproic aci
453
454
PArt ii Medical Knowledge
Questions 1. A 19-year-ol male is evaluate in the trauma bay following a motorcycle accient. He is foun to be hypotensive with an open book pelvic fracture. CT angiography of the pelvis oes not emonstrate active extravasation. A thrombelastography is performe showing an elevate K time an high LY30. He has receive bloo proucts an is 5 hours post injury. Which of the following is true? A. Cryoprecipitate woul not benet this patient B. K time is a measure of the time to initial clot formation C. This ning is inicative of ecrease brinogen levels D. FFP is inicate E. The patient shoul receive tranexamic aci (TXA) 2. The risk of posttransfusion sepsis is greatest with: A. Packe re bloo cells B. Cryoprecipitate C. Fresh frozen plasma D. Platelets E. Whole bloo 3. Which of the following is correct with regar to unfractionate heparin (UFH) an low-molecularweight heparin (LMWH)? A. LMWH is contrainicate while breastfeeing B. UFH is associate with fewer cases of heparininuce thrombocytopenia (HIT) C. Protamine is more effective in reversing LMWH compare to UFH D. LMWH oes not nee to be ose-ajuste in obese patients E. LMWH is consiere superior in trauma patients with traumatic brain injury (TBI)
4. A 34-year-ol woman with no past meical history presents with 6 weeks of left lower extremity pain an marke swelling an is foun to have a left iliofemoral DVT on CT venogram. She is given a heparin bolus, an a heparin rip is starte. She then unergoes catheter-irecte thrombolysis (CDT). Which of the following is true? A. The half-life of alteplase (tPA) is 4 to 6 hours B. Bleeing from inavertent overose may benet from aministration of aminocaproic aci C. Bleeing risk best is best monitore by following INR D. The rate of intracranial bleeing following CDT is higher than systemic thrombolysis E. The heparin rip shoul be continue uring CDT 5. Which of the following is true regaring the use of intraoperative bloo salvage (autotransfusion)? A. Use of intraoperative bloo salvage may lea to coagulopathy B. Malignancy is an absolute contrainication C. Autotransfusion can still be utilize if sterile water is being use in the el D. Most major abominal surgeries woul benet from its use E. Activate clotting time (ACT) shoul be use intraoperatively to monitor for coagulopathy 6. A 49-year-ol female with a history of von Willebran isease type 3 presents for scheule lobectomy for lung cancer. Which of the following is the correct perioperative management? A. Aminister 1 unit of FFP in the preoperative holing area B. Transfuse recombinant factor IX to 100% normal levels prior to surgery C. DDAVP shoul be given prior to incision D. He can procee without any intervention for von Willebran type 3 E. Preoperative von Willebran factor concentrate shoul be aministere
CHAPtEr 36 Transfusion and Disorders of Coagulation
7. A 75-year-ol woman with a history of atrial brillation on coumain presents to the ED with a painful, enlarging bulge in her abominal wall. She is iagnose with a rectus sheath hematoma. Her INR is supratherapeutic at 5. She enies any recent coughing episoes or trauma. However, she reports starting a new meication. Which of the following meications coul have contribute to her conition? A. Cimetiine B. Carbamazepine C. Rifampin D. Phenobarbital E. Phenytoin 8. Persistent life-threatening bleeing in a patient with Hemophilia A with high titers of inhibitors (factor VIII alloantiboies) is best treate with: A. A higher ose of factor VIII B. Fresh frozen plasma C. Cryoprecipitate D. Recombinant factor VIIa E. DDAVP (esmopressin) 9. A 76-year-ol male is unergoing a laparoscopic colectomy for sigmoi colon cancer. Which of the following is the best prophylaxis for venous thromboembolic events (VTEs)? A. Leg compression evice B. Unfractionate heparin (UFH) until fully ambulatory C. Leg compression evice intraoperatively, UFH until fully ambulatory D. Leg compression evice intraoperatively, LMWH until fully ambulatory E. Leg compression evice intraoperatively, LMWH for 4 weeks after surgery 10. A 50-year-ol male unergoes a resection of a large retroperitoneal leiomyosarcoma. There is an estimate bloo loss of 750 cc. The next ay, the patient is foun to be anemic an is given units of bloo. Halfway through the rst unit, the patient evelops chills an his temperature increases from 37 to 39°C. Which of the following is true in regar to this patient’s conition? A. The transfusion oes not nee to be stoppe B. This occurs more commonly when given packe re bloo cells versus platelets C. Filtration is more effective than leukocyte washing in preventing this conition D. Aspirin is more effective than acetaminophen in treating this conition E. Pretransfusion aministration of acetaminophen an iphenhyramine is the most effective prevention
455
11. Which of the following oes not affect the bleeing time? A. Aspirin B. von Willebran isease C. Hemophilia A D. Severe thrombocytopenia E. Qualitative platelet isorers 12. A eciency of which of the following factors woul increase INR but not prolong the PTT? A. II B. V C. VII D. IX E. X 13. The most important preoperative assessment to etermine the risk of abnormal intraoperative bleeing is: A. Bleeing time B. Activate partial thromboplastin time (aPTT) C. International normalize ratio (INR) D. History an physical examination E. Platelet count 14. Glanzmann thrombasthenia is characterize by: A. Normal bleeing time B. Treatment response to DDAVP (esmopressin) infusion C. Autosomal ominant inheritance D. Defect in platelet aggregation E. Prolonge INR 15. Cryoprecipitate contains a low concentration of which of following? A. Fibrinogen B. Factor VIII C. von Willebran factor D. Fibronectin E. Factor XI 16. Which of the following is most likely to be useful in the treatment of bleeing in the uremic patient? A. Desmopressin B. Cryoprecipitate C. Fresh frozen plasma D. Recombinant human erythropoietin E. Estrogens
456
PArt ii Medical Knowledge
17. A 60-year-ol man with iabetes presents with right upper quarant pain an leukocytosis. The patient has an elevate INR of .5 an a prolonge PTT of 60 secons, a low brinogen level, an a platelet count of 70,000 cells/μL. An ultrasoun scan reveals gas in the wall of the gallblaer. The most important part in management of this patient woul be: A. Aministration of fresh frozen plasma B. Aministration of cryoprecipitate C. Checking the D-imer assay D. Emergent cholecystectomy E. Aministration of platelets 18. Which of the following is true in regar to von Willebran isease (vWD)? A. It is the secon most common congenital efect in hemostasis B. Type 1 vWD is transmitte in an autosomal recessive fashion C. DDAVP (esmopressin) is helpful in type 3 vWD D. Increase partial thromboplastin time (PTT) rules out vWD E. DDAVP is ineffective for type B vWD 19. A 40-year-ol female presents with a swollen left lower extremity, an ultrasoun conrms a eep venous thrombosis (DVT). The patient is starte on therapeutic heparin but espite progressively increasing the ose, the pharmacy is having ifculty achieving a therapeutic partial thromboplastin time (PTT) after 4 hours. Which of the following is the best option? A. Convert from unfractionate heparin to lowmolecular-weight heparin B. Aminister fresh frozen plasma C. Start a irect thrombin inhibitor D. Place an inferior vena cava lter E. Continue to increase heparin ose as neee 20. Which of the following is true regaring prothrombin complex concentrate (PCC)? A. Three-factor an 4-factor PCC refer to varying concentrations of factor II B. It is thawe more rapily than fresh frozen plasma (FFP) C. PCC reverses warfarin to an international normalize ratio (INR) less than 1.5 within 30 minutes D. PCC lowers INR as profounly as recombinant factor VIIa E. It reverses the anticoagulant effect of abigatran
21. A 31-year-ol woman in her thir trimester of pregnancy presents with fever, heaaches, an myalgia. She is a former intravenous rug user. She enies pruritus, but her skin appears jaunice. Bloo pressure is normal. Her laboratory exam is remarkable for elevate aspartate aminotransferase (AST) an alanine transaminase (ALT), hyperbilirubinemia as well as thrombocytopenia, anemia, an severe hypoglycemia. From which of the following conitions is she most likely suffering? A. HELLP (hemolysis, elevate liver enzymes, low platelet count) synrome B. Acute fatty liver of pregnancy (AFLP) C. Intrahepatic cholestasis of pregnancy (ICP) D. Preeclampsia E. Hepatitis E 22. A 35-year-ol man has been in the intensive care unit sepsis ue to enterocutaneous stulas, ventilator epenence, an pneumonia for weeks. He is receiving nutrition parenterally. The INR is .0. The aPTT is normal. The total bilirubin level is normal. The platelet count is normal. Which of the following is the most likely etiology? A. Factor VIII eciency B. DIC C. Vitamin K eciency D. Primary brinolysis E. Chronic liver isease 23. Which of the following electrolyte abnormalities are the most likely to occur with massive bloo transfusion? A. Hypocalcemia, hypokalemia, an metabolic aciosis B. Hypercalcemia, hyperkalemia, an metabolic alkalosis C. Hypocalcemia, hyperkalemia, an metabolic alkalosis D. Hyponatremia, hyperkalemia, an metabolic alkalosis E. Hyponatremia, hyperkalemia, an metabolic aciosis
CHAPtEr 36 Transfusion and Disorders of Coagulation
24. A 75-year-ol male with a history of atrial brillation presents to the ED with an acute onset of left lower extremity pain an pulselessness. Heparin is starte. He is foun to have an occlue popliteal artery. The clot is successfully cleare with thrombolytic therapy. He remains on a heparin rip with plans to convert to warfarin. However, on hospital ay 5 his platelet count rops to 160,000 u/L (from an amission level of 370,000 u/L). Which of the following is true with regar to the rop in platelet count an the concern for heparin-inuce thrombocytopenia (HIT)? A. Because the platelet count is above 100,000 u/L, heparin can be continue B. The risk of recurrent thrombosis at this point is low C. Because the platelet count in’t rop until ay 5, the concern for HIT is low D. HIT is less common in men E. Warfarin shoul be starte 25. A 1-month-ol infant with mil skeletal abnormalities suffers a cariac arrest an passes away. On autopsy, he is foun to have extensive thrombosis in his coronary arteries. Which of the following is the most likely unerlying conition? A. Factor V Leien mutation B. Prothrombin gene mutation C. Antithrombin III eciency D. Homocystinuria E. Protein eciency 26. The most common cause of transfusion-relate eath is: A. Infection B. ABO incompatibility C. Acute lung injury D. Delaye transfusion reaction E. Graft-versus-host reaction 27. A 35-year-ol female evelops postpartum hemorrhage an requires a transfusion of packe re bloo cells an platelets. Twelve hours after transfusion, the patient abruptly evelops rigors an chills. Her temperature increases to 39°C, her bloo pressure rops from 110/70 to 70/40 mmHg, an her heart rate increases from 80 to 10 beats per minute. Urine output rops, although the urine is clear. Despite attempts at resuscitation, the patient expires within 4 hours. The eath is most likely ue to: A. Gram-positive sepsis B. ABO incompatibility C. Acute lung injury D. Anaphylaxis E. Gram-negative sepsis
457
28. A 55-year-ol patient unergoes surgery, uring which bloo transfusions were given. One week later, skin lesions evelop that appear to be purpura. The platelet count ecreases from 50,000 cells/μL to 10,000 cells/μL an an upper gastrointestinal blee evelops. The patient has not been receiving any meication that coul affect platelets. Which of the following is true about this conition? A. It is more common in mile-age men B. Severe bleeing is best manage by platelet transfusions C. It can occur without prior antigenic exposure D. It is an antiboy-meiate reaction E. Platelet counts are typically higher than with heparin-inuce thrombocytopenia 29. Which of the following is true in regar to clopiogrel (Plavix)? A. It functionally mimics the pathophysiology of Bernar-Soulier isease B. It has been linke to fatal episoes of pulmonary hypertension C. It is recommene that clopiogrel be stoppe 3 ays before a major operation D. It inhibits platelet aggregation within hours of oral aministration E. It can inhibit the release of von Willebran factor 30. Which of the following factors has the shortest half-life? A. I B. II C. VII D. IX E. X 31. A 9-year-ol female is unergoing splenectomy for iiopathic thrombocytopenic purpura. Intraoperatively, the surgeon notes a signicant amount of bleeing at the splenic hilum uring mobilization. The surgeon woul like to temporarily stop bleeing with a hemostatic agent. Which of the following woul be the least effective choice for this patient? A. Microbrillar collagen B. Oxiize cellulose C. Thrombin D. Fibrin sealant E. Glutaralehye cross-linke peptie
458
PArt ii Medical Knowledge
Answers 1. C. The use of thromboelastography (TEG) has become
4. B. Alteplase, a tissue plasminogen activator (tPA),
trial: a ranomise controlle trial an economic evaluation of the effects of tranexamic aci on eath, vascular occlusive events an transfusion requirement in bleeing trauma patients. Health Technol Assess. 013;17(10):1–79.
Naiu S, Oklu R. Catheter-irecte thrombolysis of eep vein thrombosis: literature review an practice consierations. Cardiovasc Diagn Ther. 017;7(S3):S8–S37.
more common in the setting of hemorrhagic shock seconary to trauma or cirrhosis. TEG provies real-time information about clotting activity an can guie resuscitation. K time refers to the time to brin cross linking an an elevate K time inicates a eciency of brinogen (B). Therefore, transfusion of cryoprecipitate woul be inicate for this patient (A). Platelets woul be inicate in the event of a low MA. R time on a TEG result refers to the time to initial clot formation, an if this is prolonge, transfusion of FFP is inicate (D). A high LY30 is consistent with hyperbrinolysis an suggests the patient woul benet with TXA aministration. Trauma patients with massive hemorrhage receiving TXA have reuce all-cause mortality. However, this benet is only seen for patients receiving TXA within 3 hours of injury. TXA aministration past the 3-hour mark is associate with worse outcomes (E). Reference: Roberts I, Shakur H, Coats T, et al. The CRASH-
2. D. The risk of posttransfusion sepsis is greatest with
platelet transfusion. The risk is the greatest in transfusion of poole platelet concentrates from multiple onor versus single-onor platelet transfusion. Platelets are store at °C which makes this bloo prouct the most vulnerable to bacterial colonization an growth. If bacteria contamination of aministere bloo proucts is suspecte, the transfusion shoul be stoppe immeiately an bloo cultures obtaine.
3. E. The rate of serious bleeing complications has been
shown to be lower with the use of LMWH compare to UFH. It has also been shown to be associate with improve mortality in trauma patients with TBI. However, LMWH oes not have a completely effective reversal agent available. Only 60% of the anticoagulant effect of LMWH can be reverse with the aministration of protamine (C). Higher rates of major bleeing events have been shows in patients with renal insufciency with the use of both UFH an LMWH. LMWH is renally cleare, however, an therefore shoul be avoie in the setting of reuce creatinine clearance. UFH unergoes excretion via the reticuloenothelial system an enothelial cells (D). Although both UFH an LMWH are associate with the evelopment of HIT, this is more commonly seen after exposure to UFH (B). Either UFH, LMWH, or warfarin can be safely use while breastfeeing (A). Patients with obesity have a larger volume of istribution of lipophilic rugs such as LMWH an as such will require ose ajusting to reach aequate levels for thromboprophylaxis (D). Reference: Crowther MA, Berry LR, Monagle PT, Chan AKC. Mechanisms responsible for the failure of protamine to inactivate low-molecular-weight heparin: inactivation of low-molecular-weight heparin by protamine. Br J Haematol. 00;116(1):178–186.
is the rug most commonly use in CDT an has a very short half-life (5 minutes) (A). tPA triggers the activation of plasminogen into plasmin which then breaks brin cross links to issolve clot. Aminocaproic aci is the treatment of overose or reversal of tPA. Fibrinogen levels shoul be monitore closely following thrombolysis. Low levels of brinogen (usually less than 100 or 150 epening on clinical practice), are inicative of an increase risk of bleeing events (C). Although CDT has lower rates of intracranial hemorrhage than systemic thrombolysis (0%–1% versus 3%–6%, respectively), the patient shoul be monitore closely while unergoing treatment (D). Many of the same absolute contrainications to systemic thrombolysis are true for CDT incluing recent stroke, active bleeing, an intracranial trauma. Systemic heparin shoul be hel uring lytic therapy ue to the risk of bleeing (E). Reference: Fleck D, Albaawi H, Shamoun F, Knuttinen G,
5. A. Intraoperative bloo salvage is recommene for
clean (non-GI, noncontaminate) proceures with an estimate bloo loss of 500 to 1000 mL (e.g., cariac, liver, vascular, orthopeic cases) or more (D). This involves removing the patient’s bloo with a suction catheter uring surgery from the operative el. The bloo is then ltere, washe an returne to the patient. It has been shown to reuce the amount of allogenic transfusion require. It also theoretically increases operating room efciency as there is less time neee to request an prepare allogenic prouct. Absolute contrainications inclue mixture with other uis, particularly sterile water, as this hypotonic solution can lea to hemolysis (C). Malignancy is not an absolute contrainication; however, the risks an benets shoul be assesse on a case-by-case basis (B). Intraoperative bloo salvage only replaces re bloo cells an therefore patients are at risk of coagulopathy an ilution of coagulation factors. ACT monitoring is use in the setting of systemic heparinization, not for the use of re cell salvage. Goal ACT varies by provier an proceure but 150 to 00 secons for routine anticoagulation is commonly use (E). References: American Society of Anesthesiologists Task Force on Perioperative Bloo Management. Practice guielines for perioperative bloo management: an upate report by the American Society of Anesthesiologists Task Force on Perioperative Bloo Management. Anesthesiology. 015;1():41–75. Carless PA, Henry DA, Moxey AJ, O’Connell D, Brown T, Fergusson DA. Cell salvage for minimising perioperative allogeneic bloo transfusion. Cochrane Database Syst Rev. 010;(4):CD001888.
6. E. von Willebran isease (vWD) is the most common
congenital bleeing isorer. Patients with WVD type 3 have the most severe bleeing iathesis among patients with VWD. In this type of VWD, there is an absence of von Willebran factor (vWF). DDAVP causes release of vWF an
CHAPtEr 36 Transfusion and Disorders of Coagulation factor VIII from enothelial stores, an therefore, patients with type 3 VWD are not responsive to DDAVP (C). The perioperative management for patients with VWD unergoing major surgery (e.g., cariothoracic, hepatobiliary, neurologic, open vascular) inclues aministration of vWF (D). For patients with Hemophilia B, recombinant factor IX shoul be aministere to a goal of 100% of normal factor IX levels preoperatively (B). FFP can be use to correct INR in the acute setting (A). Reference: Lavin M, O’Donnell JS. New treatment approaches to von Willebran isease. Hematology Am Soc Hematol Educ Program. 016;016(1):683–689.
7. A. This patient has a supratherapeutic INR while on
coumain. Coumain works by interfering with the gammacarboxylation of vitamin K-epenent coagulation factors (factors II, VII, IX, X, protein C, S), an is metabolize by the cytochrome-P450 in the liver. Several rug interactions can lea to altere coumain metabolism. Meications that inhibit cytochrome-P450 lea to ecrease coumain metabolism an supratherapeutic INR. Inhibitors of cytochromeP450 inclue cimetiine, amioarone, several antibiotics (macrolies, uoroquinolones, metroniazole, isoniazi, sulfonamies), voriconazole, an grapefruit juice. Conversely, inucers of cytochrome-P450 will increase metabolism of warfarin ecreasing its effect. These patients may present with a new venous thromboembolism even though they have been on the same ose of warfarin for years. Examples of cytochrome-P450 inclue carbamazepine, rifampin, phenytoin, an phenobarbital (B–E).
8. D. Hemophilia A is a sex-linke recessive genetic coni-
tion an consiere the most common coagulation isorer, accounting for 80% of all inherite coagulation isorers. With time, as many as 10% to 15% of patients with factor VIII–ecient hemophilia A evelop inhibitors (alloantiboies) against factor VIII. This is usually from previous factor VIII transfusions. In situations in which life-threatening hemorrhage evelops, recombinant factor VIIa is the best option. Another option is porcine factor VIII, but there is approximately a 5% cross-reactivity with inhibitors. Factor VIIa complexes with tissue factor at the site of injury, resulting in an activation of factor X, which then results in clot formation. Factor VIIa bypasses the requirement for factors VIII an IX an thus has been shown to be effective in prevention an treatment of joint hemorrhage, the treatment of life-threatening bleeing, an the prevention of surgical bleeing. Restimulation of antiboies to factors VIII an IX shoul theoretically be less problematic than with the use of plasma-erive proucts. The primary concerns with recombinant factor VIIa are the potential for inucing thrombosis (stroke, eep venous thrombosis) an the high cost. A higher ose of factor VIII woul not efeat prouction of patient antiboies (A). Both fresh frozen plasma an cryoprecipitate contain factor VII but woul be ilute with other factors incluing factor VIII (B, C). DDAVP woul not help a patient with a coagulation efect (E). Other options that have been use but are only a temporary x in patients with signicant bleeing are plasmapheresis an immune absorption. References: DiMichele D. Inhibitors in hemophilia: a primer. Treatment of Hemophilia, 008;(7):1–4.
459
Kenet G, Lubetsky A, Luboshitz J, Martinowitz U. A new approach to treatment of bleeing episoes in young hemophilia patients: a single bolus megaose of recombinant activate factor VII (NovoSeven): recombinant FVIIa (NovoSeven) megaose. J Thromb Haemost. 003;1(3):450–455.
9. E. Patients unergoing surgery shoul be assesse for
VTE risk an categorize as very low, low, moerate, an high-risk patients. The Caprini score can be use to facilitate the estimation. A score of 5 or more places a patient at high risk. Age of 75 years or more = 3 points, cancer = points, an major open or laparoscopic surgery longer than 45 minutes is also points. As such this patient woul be consiere high risk. In low-risk patients, mechanical prevention (compression evice) is recommene. In moerate risk, pharmacologic prophylaxis with either UFH or LMWH is recommene. High-risk patients shoul get both mechanical an pharmacologic prophylaxis. The rug shoul be aministere close to surgery an continue until the patient is fully ambulatory. Recent ata in high-risk patients (such as those with cancer) emonstrate enhance VTE prophylaxis with extene LMWH for 4 weeks after surgery (A–D). Interestingly, recent ata inicate that patients unergoing colectomy for inammatory bowel isease (IBD) are also at very high risk for VTE (though IBD is not inclue in the Caprini score). Reference: Veovati MC, Becattini C, Ronelli F, et al. A ranomize stuy on 1-week versus 4-week prophylaxis for venous thromboembolism after laparoscopic surgery for colorectal cancer. Ann Surg. 014;59(4):665–669.
10. C. The patient is likely manifesting a febrile nonhe-
molytic transfusion reaction (FNHTR), the most common bloo transfusion reaction. It occurs in 0.5% to 1.5% of all cases of bloo transfusion (A). It is ene as a rise in temperature of at least 1.8°C from baseline an is not accounte for by the patient’s clinical conition. However, FNHTR is a iagnosis of exclusion. As such, it is generally recommene to at least temporarily stop the transfusion an assess the patient. In particular, attention shoul be pai to aitional symptoms an signs such as respiratory compromise, cyanosis, back pain, an hypotension; these may suggest a hemolytic reaction, TRALI, or sepsis from contaminate bloo. FNHTR is more common in pregnancy an in patients with immunocompromise states (such as leukemia, lymphoma). It occurs more commonly after the transfusion of platelets but can also occur with PRBC or FFP (B). Pretreatment with acetaminophen was thought to reuce the severity of the complication. However, the only ranomize controlle trial to ate emonstrate no ifference in the rate of FNHTR in patients that were pretreate with acetaminophen an iphenhyramine when compare to a placebo (E). The incience of febrile reactions can be greatly reuce by the use of leukocyte-reuce bloo proucts. Filtration removes 99.9% of the white bloo cells an platelets an is more effective than washing. Leukocyte reuction prevents almost all febrile transfusion reactions. There is ebate in the literature as to whether leukocyte reuction leas to a ecrease in postoperative infections or mortality. Aspirin is not avise given its effects on platelets an bleeing (D). References: Hébert PC, Fergusson D, Blajchman MA, et al. Clinical outcomes following institution of the Canaian universal
460
PArt ii Medical Knowledge
leukoreuction program for re bloo cell transfusions. JAMA. 003;89(15):1941–1949. Wang SE, Lara PN Jr, Lee-Ow A, et al. Acetaminophen an iphenhyramine as premeication for platelet transfusions: a prospective ranomize ouble-blin placebo-controlle trial. Am J Hematol. 00;70(3):191–194.
11. C. Bleeing time tests platelet ahesion an aggrega-
tion an will be normal in erangement of the coagulation pathways. Hemophilia A is associate with a factor VIII eciency, which manifests as an abnormality in the coagulation cascae an presents with a prolonge PTT. Drugs that inhibit platelet function, such as aspirin (which works by inhibiting cyclooxygenase), will increase bleeing time (A). von Willebran isease will result in prolonge bleeing time because of the qualitative or quantitative eciency in Willebran factor, which is require for platelet ahesion to other platelets via the IIb/IIIa receptor (B). Severe thrombocytopenia (quantitative) an platelet ysfunction (qualitative) both prolong bleeing time (D, E). Fibrinogen eciency also prolongs bleeing time because brinogen is require for platelet aggregation.
12. C. The INR etects abnormalities in the extrinsic an
common pathways. The extrinsic pathway is triggere by exposure of the injure vessel to tissue factor an starts with factor VII. It then merges with the intrinsic pathway at factor X (E) an is followe by activation of factors V an II an brinogen (factor I) (A, B). Thus, both the prothrombin time an the PTT will be prolonge in factors I, II, V, an X because they are all part of the common pathway between the intrinsic an extrinsic pathways. Factor IX is part of the intrinsic pathway an a eciency woul prolong PTT only (D).
13. D. The most important element in etecting an
increase risk of abnormal bleeing before surgery is a etaile history an physical examination. A systematic review in 008 emonstrate the poor value of using coagulation tests when it came to ientifying the risk of bleeing uring an operation (A–C, E). Other stuies have likewise shown that routine use of laboratory testing is neither sensitive nor specic for etermining increase risk of bleeing. One nees to inquire about a history of prolonge bleeing after minor trauma, tooth extraction, menstruation, an in association with major an minor surgery. In aition, one must make inquiries into meications an over-the-counter supplements that might affect hemostasis. If a careful history is negative an the planne surgical proceure is minor, then further testing is not necessary. A potential pitfall in relying solely on the history is that the history obtaine might not be sufciently thorough or the patients might not recall or recognize that they ha previous abnormal bleeing after an operation. If a major operation is planne that is not a high-bleeing risk, then a platelet count, a bloo smear, an an aPTT are recommene. If the history suggests abnormal bleeing or the operation is either a high bleeing–risk operation or one in which even minor bleeing may have ire consequences (neurosurgery), then a bleeing time an INR shoul be ae an a brin clot to etect abnormal brinolysis. If there is high suspicion for a history of abnormal bleeing, a hematology consult shoul also be obtaine.
References: Chee YL, Crawfor JC, Watson HG, Greaves M. Guielines on the assessment of bleeing risk prior to surgery or invasive proceures. British Committee for Stanars in Haematology: British Committee for Stanars in Haematology. Br J Haematol. 008;140(5):496–504. Chee YL, Greaves M. Role of coagulation testing in preicting bleeing risk. Hematol J. 003;4(6):373–378. Klopfenstein CE. Preoperative clinical assessment of hemostatic function in patients scheule for a cariac operation. Ann Thorac Surg. 1996;6(6):1918–190. Suchman AL, Mushlin AI. How well oes the activate partial thromboplastin time preict postoperative hemorrhage? JAMA. 1986;56(6):750–753.
14. D. Glanzmann thrombasthenia is an autosomal reces-
sive isorer that results in absence of functional glycoprotein IIb/IIIa (C). Glycoprotein IIb/IIIa is a receptor for brinogen an von Willebran factor an causes platelet ahesion an aggregation. Therefore, bleeing time will be prolonge, but aPTT an INR will be normal (A–E). These patients will not respon to DDAVP because there is no quantitative efect in the enothelial release of von Willebran factor or factor VIII (von Willebran isease) (B). The bleeing tenency for patients with Glanzmann’s is variable. Treatment is with platelets. Repeate use of platelet transfusions can inuce antiglycoprotein IIb/IIIa alloimmunization, renering the treatment ineffective. In this circumstance, recombinant factor VIIa may be useful. References: ’Oiron R, Ménart C, Trzeciak MC, et al. Use of recombinant factor VIIa in 3 patients with inherite type I Glanzmann’s thrombasthenia unergoing invasive proceures. Thromb Haemost. 000;83(5):644–647. Nuren AT. Glanzmann thrombasthenia. Orphanet J Rare Dis. 006;1(1):10.
15. E. Cryoprecipitate contains all items liste as well as
factor XIII. However, it contains low concentrations of factor XI (A–D). Cryoprecipitate was originally create as a treatment for hemophilia; however, it is now more often use in patients receiving massive resuscitation in conjunction with fresh frozen plasma to replenish brinogen levels. Factor XI eciency is also known as hemophilia C or Rosenthal synrome, occurs more often in the Ashkenazi Jewish population, an is treate with fresh frozen plasma (uring bleeing episoes).
16. A. The etiology of abnormal bleeing in uremic patients
is multifactorial, but the most important is impairment of platelet function that may be partly ue to a functional efect in von Willebran factor, which leas to impaire platelet aggregation. DDAVP (esmopressin) seems to enhance the release of von Willebran factor by enothelial cells. A single ose of 0.3 to 0.4 mcg/kg is given intravenously or subcutaneously. It has a rapi onset an relatively short uration (4–6 hours). Dialysis is also effective in the treatment of uremic bleeing by removing toxins that cause platelet ysfunction. Cryoprecipitate has high concentrations of von Willebran factor as well as factor VIII an brinogen an may also be effective; however, it shoul not be rst-line therapy (B). Recombinant human erythropoietin (Epogen [epoetin alfa]) has been shown to help uremic bleeing in several stuies as well (D). In aition to stimulating erythropoiesis, Epogen (epoetin alfa) enhances platelet aggregation. The
CHAPtEr 36 Transfusion and Disorders of Coagulation increase re cell mass also seems to isplace platelets from the center of the bloo vessel an places them closer to the enothelium. Estrogens have been shown to help with bleeing in men an women. The exact mechanism is unknown, but it is theorize that they ecrease arginine levels, which ecreases nitric oxie. This may lea to increases in thromboxane A an aenosine iphosphate (E). FFP oes not have high concentrations of von Willebran factor an thus is not effective for uremic bleeing (C). Reference: Heges SJ, Dehoney SB, Hooper JS, Amanzaeh J, Busti AJ. Evience-base treatment recommenations for uremic bleeing. Nat Clin Pract Nephrol. 007;3(3):138–153.
17. D. This is a classic presentation of emphysematous
cholecystitis complicate by sepsis, which then resulte in DIC. Elerly male iabetic patients are at higher risk of emphysematous cholecystitis, an gas in the gallblaer conrms the iagnosis. DIC leas to a ysregulation of the coagulation cascae, leaing to clotting an resultant bleeing. The consumption of brinogen, platelets, an coagulation factors from the overactivation of the coagulation cascae results ultimately in iffuse bleeing. There is no specic test for DIC, but thrombocytopenia, hypobrinogenemia, prolonge PT an PTT, an the presence of increase brin egraation proucts are sufcient to suggest the iagnosis of DIC (C). Fresh frozen plasma, platelets, an cryoprecipitate are all important components of the treatment, especially for an actively bleeing patient, but the most important part in the management of DIC is to ientify an correct the unerlying source, which in this case is by broa-spectrum intravenous (IV) antibiotics an emergent cholecystectomy (A, B, E). Without removal of the source, DIC will continue to consume transfuse proucts. The mortality rate from DIC ranges between 10% an 50%. Reference: Levi M, Toh CH, Thachil J, Watson HG. Guielines for the iagnosis an management of isseminate intravascular coagulation. British Committee for Stanars in Haematology. Br J Haematol. 009;145(1):4–33.
18. E. The most frequent congenital efect in hemostasis is
vWD (A). Laboratory tests will emonstrate increase bleeing time with a normal prothrombin time (PT). Patients may have a normal or increase PTT because von Willebran factor (vWF) is consiere a stabilizing factor for factor VIII (D). There are three types of vWD: Type I is an autosomal ominant isease characterize by a low level of vWF an consiere the most common form of vWD (B). Type I is treate with DDAVP because this increases circulating vWF release from enothelial cells. Type vWD is also inherite in an autosomal ominant fashion an is characterize by a qualitative efect in which there is an appropriate amount of vWF, but it oes not function properly. Type has multiple variants, some that can be treate with DDAVP or cryoprecipitate. Type b, in particular, when treate with DDAVP can inuce thrombocytopenia an form platelet complexes leaing to a prothrombotic state. DDAVP is contrainicate in type b but may be useful in other type variants. Finally, type 3 is the most severe form because there is no vWF prouce by enothelial cells. It is transmitte in an autosomal recessive fashion. For type 3, the recommene treatment is recombinant vWF an factor VIII because these patients
461
o not make any vWF an therefore DDAVP will have no effect (C). References: Holmberg L, Nilsson IM, Borge L, Gunnarsson M, Sjörin E. Platelet aggregation inuce by 1-esamino-8-D-arginine vasopressin (DDAVP) in Type IIB von Willebran’s isease. N Engl J Med. 1983;309(14):816–81. Tosetto A, Castaman G. How I treat type variant forms of von Willebran isease. Blood. 015;15(6):907–914.
19. B. Heparin resistance is ene as the nee for more
than 35,000 units in 4 hours to prolong the PTT into the therapeutic range or as an activate clotting time (ACT) less than 400 secons espite excessive eman for heparin (>400– 600 IU/kg). Heparin resistance is most commonly the result of antithrombin-III (ATIII) eciency. Heparin bins to ATIII causing a conformational change that results in its activation. Activate ATIII then inactivates thrombin an other proteases involve in bloo clotting, most notably factor Xa. ATIII eciency can be congenital or acquire. Hereitary ATIII eciency is rare (much less common than factor V Leien eciency) an can lea to venous thrombosis. Causes of acquire ATIII eciency inclue pregnancy, liver isease, isseminate intravascular coagulation (DIC), nephrotic synrome, major surgery, acute thrombosis, an treatment with heparin. For this latter reason, measurement of ATIII levels while on heparin is an inaccurate metho of ientifying heparin resistance. Treatment of heparin resistance consists of either aministering FFP or ATIII concentrates. FFP has the highest concentration of ATIII, an therefore patients shoul be initially treate with FFP to replete ATIII in plasma, followe by reaministration of heparin. A irect thrombin inhibitor (argatroban) is a potential alternative; however, it has the isavantage of having no way of being reverse in the case of overosage an bleeing (C). A isavantage of FFP in the cariac surgery setting is that large volumes may be require an it exposes the patient to the risks of transfusions, incluing transfusion-relate lung injury (TRALI). Thus, in the setting of cariac bypass, ATIII concentrate is another alternative (though it is very costly). Low-molecularweight heparin has no effect on ATIII eciency an shoul not be use in this event (A). An inferior vena cava (IVC) lter woul be inicate if the patient began to blee while on heparin but not for heparin resistance. In fact, a lter, though protective against PE, increases the risk for DVT, ue to the stasis it may create (D). Most patients achieve therapeutic PTT within 6 to 18 hours of starting heparin, so simply increasing the heparin ose is not appropriate (E). References: Kearon C, Akl EA, Comerota AJ, et al. Antithrombotic therapy for VTE isease: Antithrombotic Therapy an Prevention of Thrombosis, 9th e: American College of Chest Physicians Evience-Base Clinical Practice Guielines. Chest. 01;141( Suppl):e419S–e496S. Spiess BD. Treating heparin resistance with antithrombin or fresh frozen plasma. Ann Thorac Surg. 008;85(6):153–160.
20. C. PCC is an inactivate concentrate of proteins C an
S, an factors II, IX, an X, with variable amounts of factor VII. PCC with normal amounts of factor VII is known as 4-factor PCC, while PCC with low levels of factor VII is 3-factor PCC (A). Since 3-factor PCC has low levels of factor VII, the aition of fresh frozen plasma is sometimes necessary for full reversal of warfarin an thus, 4-factor PCC is superior. When a nonbleeing patient on warfarin nees
462
PArt ii Medical Knowledge
INR reversal, vitamin K is given, either orally (slower acting) or intramuscularly. If a patient is bleeing with an elevate INR, vitamin K an an exogenous clotting factor formulation are given. The options are FFP, PCC, or recombinant factor VII (less often use). PPC has several avantages over FFP; it oes not nee to be thawe (it is lyophilize [i.e., freeze rie]), it has a more rapi correction of INR, an it can be infuse faster an with less volume (this also makes it ieal for patients with congestive heart failure or chronic kiney isease) (B). Recombinant factor VIIa will lower INR faster than PCC (D). However, the concerns regaring recombinant factor VIIa inclue the potential for inucing thrombosis (stroke, eep venous thrombosis) as well as the high cost. PPC oes not reverse the anticoagulant effect of abigatran; this can be accomplishe with iarucizumab (E).
21. B. AFLP is an uncommon but potentially fatal complica-
tion that occurs in the thir trimester of pregnancy or uring the early postpartum perio. It typically presents with a viral prorome characterize by fever, lethargy, malaise, an nausea an vomiting. It is thought that AFLP may be the result of mitochonrial ysfunction resulting in microvesicular fatty inltration of hepatocytes without signicant inammation or necrosis. The mortality rate previously was very high; however, with prompt iagnosis an treatment, the maternal an perinatal mortality have ecrease to 18% an 3%, respectively. Prompt elivery an intensive supportive care are the cornerstones in management of AFLP. Laboratory abnormalities in AFLP inclue elevations of AST an ALT (usually less than 1000 IU/L), prolongation of PT an PTT, ecrease brinogen, renal failure, profoun hypoglycemia, an hyperbilirubinemia. Laboratory stuies of AFLP are similar to HELLP, but the key ning to help ifferentiate the two is hypoglycemia, which oes not occur commonly in HELLP (A). In aition, patients with HELLP typically have preeclampsia, evience of hemolysis, an thrombocytopenia. Preeclampsia presents with hypertension, proteinuria, an rapi weight gain an can progress to seizures (eclampsia) (D). Patients with ICP report intense pruritus most commonly in the hans an soles of the feet that is unrelieve with antihistamines (C). Hepatitis E is cause by a single-strane RNA virus. In men an nonpregnant women, it tens to be mil. However, it can lea to severe fulminant hepatic failure in pregnant patients in the thir trimester, with a mortality rate of up to 5% (particularly in eveloping countries) (E). References: Ko H, Yoshia EM. Acute fatty liver of pregnancy. Can J Gastroenterol. 006;0(1):5–30. Rahman TM, Wenon J. Severe hepatic ysfunction in pregnancy. QJM. 00;95(6):343–357. Vigil-De Gracia P. Acute fatty liver an HELLP synrome: two istinct pregnancy isorers. Int J Gynaecol Obstet. 001;73(3):15–0.
22. C. Several stuies have emonstrate that patients
in the ICU have a high incience of coagulopathy an that vitamin K eciency is the most common cause (B, D, E). The ifferential iagnosis for an elevate INR with a normal aPTT woul inclue a factor VII eciency, warfarin aministration, the acute phase of liver isease, an vitamin K eciency. Vitamin K is not stable in patients receiving total parenteral nutrition; therefore, in this case, the prolonge PT correlates with vitamin K eciency. Prolonge parenteral nutrition often leas to cholestatic liver isease, which in
turn leas to the liver’s inability to use vitamin K appropriately. Factors II, VII, IX, an X as well as proteins C an S all require vitamin K an will be ecient in these patients (A). Twenty percent of hospitalize patients given intravenous nutrition over a 3-week perio evelope elevations of INR. Vitamin K shoul be given at least 6 to 1 hours before a proceure in patients with aequate liver function. IM route of aministration is preferre because an IV push may result in anaphylaxis. In patients with hepatocellular isease, FFP or whole bloo is require. Platelets an cryoprecipitate are unrelate to prolonge prothrombin time. References: Chakraverty R, Davison S, Peggs K, Stross P, Garrar C, Littlewoo TJ. The incience an cause of coagulopathies in an intensive care population. Br J Haematol. 1996;93():460–463. Crowther MA, McDonal E, Johnston M, Cook D. Vitamin K eciency an D-imer levels in the intensive care unit: a prospective cohort stuy. Blood Coagul Fibrinolysis. 00;13(1):49–5. Duerksen DR, Papineau N. Clinical research: is routine vitamin K supplementation require in hospitalize patients receiving parenteral nutrition? Nutr Clin Pract. 000;15():81–83. Fiore LD, Scola MA, Cantillon CE, Brophy MT. Anaphylactoi reactions to vitamin K. J Thromb Thrombolysis. 001;11():175–183. Shearer MJ. Vitamin K in parenteral nutrition. Gastroenterology. 009;137(Suppl. 5):S105–S118.
23. C. The correct answer is hypocalcemia, hyperkalemia,
an metabolic alkalosis (A, B, D, E). Severe hypocalcemia with massive bloo transfusion is uncommon an oes not typically manifest unless the patient is receiving more than 1 unit of packe re bloo cells (PRBCs) every 5 minutes. The hypocalcemia is the result of citrate toxicity because the citrate in the transfuse bloo bins to circulating calcium in the patient. Because citrate is metabolize in the liver, hypocalcemia can be more severe in patients with hepatic ysfunction. Aitionally, the citrate is metabolize to bicarbonate leaing to metabolic alkalosis. Potassium concentration of store PRBC is higher than human plasma potassium level. This is thought to occur as a result of re bloo cell lysis uring storage, releasing potassium in the supernatant. The concentration of potassium in PRBC increases linearly an is approximately equal to the number of ays of PRBC storage. Reference: Vraets A, Lin Y, Callum JL. Transfusion-associate hyperkalemia. Transfus Med Rev. 011;5(3):184–196.
24. D. HIT occurs in approximately 1% to 1.% of patients
receiving heparin. A scoring system has been evise to assess risk of HIT, known as the 4 “T”s (Thrombocytopenia, Timing, Thrombosis, an other causes for Thrombocytopenia). Variables that shoul heighten suspicion of HIT inclue a platelet count rop greater than 50%, occurrence between ays 5 an 10 (it takes time for antiboies to evelop), nair of platelet count greater than 0,000 (nair below 10,000 is less likely HIT), no other reason for platelet count rop, an new skin necrosis or VTE (C). Thus, more important than the absolute nair is the percentage rop (A). HIT is cause by antiboies that attack the heparin-platelet factor 4 (PF4) complex. Heparin-PF4 antiboies (sometimes calle “HIT antiboies”) in the resultant multimolecular immune complex activate platelets via their FcγIIa receptors, causing the release of prothrombotic platelet-erive microparticles, platelet consumption, an thrombocytopenia. The microparticles in turn promote excessive thrombin generation, frequently resulting in thrombosis. Patients receiving any type
CHAPtEr 36 Transfusion and Disorders of Coagulation of heparin at any ose an by any route of aministration are at risk of eveloping HIT antiboies. It oes occur less commonly in men an occurs more frequently in the elerly. However, not all of those with HIT antiboies will necessarily evelop the clinical synrome. If this is suspecte, heparin shoul be iscontinue, an the patient shoul be starte on a irect thrombin inhibitor (E). If anticoagulation is not initiate, the chance of another thromboembolic event is approximately 5% to 10% per ay (B). Diagnosis is performe by an ELISA antiboy test. If these results are equivocal, then a conrmatory serotonin release assay shoul be performe. References: Ahme I, Majee A, Powell R. Heparin inuce thrombocytopenia: iagnosis an management upate. Postgrad Med J. 007;83(983):575–58. Jang IK, Hursting MJ. When heparins promote thrombosis: review of heparin-inuce thrombocytopenia. Circulation. 005;111(0):671–683. Warkentin TE, Haywar CP, Boshkov LK, et al. Sera from patients with heparin-inuce thrombocytopenia generate platelet-erive microparticles with procoagulant activity: an explanation for the thrombotic complications of heparin-inuce thrombocytopenia. Blood. 1994;84(11):3691–3699. Wheeler HB. Diagnosis of eep vein thrombosis. Review of clinical evaluation an impeance plethysmography. Am J Surg. 1985;150(4A):7–13.
25. D. Although all the answer choices can increase the
risk of venous thromboembolism, homocystinuria is the most common inherite conition preisposing patients to arterial thrombosis an affects 5% to 10% of the population. It is an autosomal recessive isease. Homocystinuria is most commonly cause by a eciency of cystathionine beta-synthase resulting in an elevate level of homocysteine in plasma an urine. The toxic effect of an elevate level of homocysteine in the brain results in mental retaration as well as seizures. Skeletal abnormalities (marfanoi habitus)may occur seconary to the interference of collagen cross-linking. Patients are at increase risk of thrombosis ue to the isruption of vascular enothelium by homocysteine leaing to platelet activation an aggregation. Patients ientie early to have this conition will benet with aministration of pyrioxine (vitamin B6) to inuce cystathionine beta-synthase activity. Factor V Leien mutation is the most common inherite conition increasing the risk of venous thromboembolism followe by prothrombin gene mutation (A, B). Patients that o not have a response to the aministration of unfractionate heparin may have antithrombin III eciency (C). Protein C eciency is a rare cause of venous thromboembolism (E). References: D’Angelo A, Selhub J. Homocysteine an thrombotic isease. Blood. 1997;90(1):1–11. Greico AJ. Homocystinuria: pathogenetic mechanisms. Am J Med Sci. 1977;73():10–13. Rosenaal FR. Risk factors for venous thrombosis: prevalence, risk, an interaction. Semin Hematol. 1997;34(3):171–187.
26. C. The leaing causes of allogeneic bloo transfusion
(ABT)–relate mortality in the Unite States (in the orer of reporte number of eaths) inclue transfusion-relate acute lung injury (TRALI), ABO an non-ABO hemolytic transfusion reactions, an transfusion-associate sepsis (A, B, D). Graft-versus-host reaction is not a common cause of ABT (E). Aitionally, it has been emonstrate that
463
nonleukocyte-reuce bloo transfusions have been associate with increase mortality when compare with leukocytereuce bloo transfusions. Reference: Vamvakas E, Blajchman M. Transfusion-relate mortality: the ongoing risks of allogeneic bloo transfusion an the available strategies for their prevention. Blood. 009;113(15):3406–3417.
27. E. Bacterial contamination of bloo is the most fre-
quent cause of eath from transfusion-transmitte infectious isease an is the thir most common cause of eath overall in a large series (after acute lung injury an ABO incompatibility) (B, C). A key feature of ABO incompatibility (hemolytic reaction) is the evelopment of re urine (hemoglobinuria). Patients also often complain of back pain an a sense of oom. Acute lung injury manifests with rapi onset of yspnea an tachypnea aroun 6 hours after transfusion. Anaphylactic reaction rarely occurs (D). Bacterial contamination now accounts for 1 in every 38,500 cases of bloo transfusion. This increase ha coincie with a ramatic ecrease in viral infections. The highest risk of bacterial infection is from poole platelet transfusions because many microorganisms can live an propagate uner the storage conitions of platelets (0–4°C). Gram-negative sepsis is the most lethal (A), an Yersinia is one of the most common organisms. Gram-negative sepsis can become clinically apparent within 9 to 4 hours after bloo transfusion. Cytomegalovirus is the most common infectious agent transmitte, but because it is so ubiquitous, it is generally not a threat to most patients. The exception to that rule is the transplant recipient. References: Benjamin RJ. Transfusion-relate sepsis: a silent epiemic. Blood. 016;17(4):380–381. Bihl F, Castelli D, Marincola F, Do RY, Braner C. Transfusiontransmitte infections. J Transl Med. 007;5(1):5. Kuehnert M, Roth V, Haley N, et al. Transfusion-transmitte bacterial infection in the Unite States, 1998 through 000. Transfusion. 001;41(1):1493–1499.
28. D. Transfusion purpura is an uncommon cause of
thrombocytopenia an bleeing after transfusion. A small minority of patients lack the HPA-1a antigen on their platelets that is present in almost all humans. Transfusion purpura requires that the patient has been previously sensitize to the HPA-1a antigen; this happens usually by a prior pregnancy or previous bloo transfusion. When these patients later receive bloo proucts that contain a small number of platelets with the ubiquitous HPA-1a, they prouce alloantiboies that attack both the onor’s an the patient’s own platelets (C). This usually presents 5 to 1 ays after a transfusion an leas to profoun thrombocytopenia an bleeing that can last for weeks. Mortality occurs in 10% to 0% ue to hemorrhage. Although sensitization can occur after prior bloo transfusions, it has become less common with leukocyte-reuce re cells an therefore this issue is most common in women who have been pregnant (A). Diagnosis is mae by emonstrating platelet alloantiboies with an absence of the corresponing antigen on the patient’s platelets. Treatment is primarily with intravenous immunoglobulin (IVIG). Plasmapheresis an corticosterois are also potential options. Treatment with platelet transfusions can exacerbate the isease process (B). The presentation can easily be confuse with heparin-inuce thrombocytopenia without appropriate
464
PArt ii Medical Knowledge
testing. A platelet count of fewer than 15,000 cells/μL is more suggestive of transfusion purpura (E). References: Hillyer CD, Hillyer KL, Strobl FJ, Jefferies LC, Silberstein LE, es. Handbook of transfusion medicine. Acaemic Press; 001:38. Lubenow N, Eichler P, Albrecht D. Very low platelet counts in post-transfusion purpura falsely iagnose as heparin-inuce thrombocytopenia: report of four cases an review of literature. Thromb Res. 000;100(3):115–15.
29. D. Clopiogrel (Plavix) irreversibly inhibits platelet
aggregation within hours of aministration an its effects last 5 to 7 ays (the half-life of platelets is 1 week) (C, D). It works by inirectly inhibiting the activation of the glycoprotein IIb/IIIa complex (E). It oes this by antagonizing the ADP receptor which, when activate, inserts glycoprotein IIb/IIIa receptors on the platelet’s surface. This is functionally similar to Glanzmann thrombasthenia, which is characterize by a GpIIb/IIIa receptor eciency on platelets preventing brin from linking platelets together. Bernar-Soulier isease is characterize by GpIb receptor eciency on platelets which prevents vWF from linking the platelet to expose collagen on amage tissue (A). Clopiogrel has been shown to ecrease the rate of a combine enpoint of cariovascular eath, myocarial infarction, an stroke in patients with acute coronary synromes. Use with aspirin increases the risk of bleeing. Clopiogrel has been associate with the evelopment of thrombotic thrombocytopenic purpura, even with short-term use (00 mmHg heale 90% of the time with HBOT (C). TOT applies 100% oxygen irectly to the woun an has been stuie far less than HBOT. Though there are some clinical trials suggesting efcacy in some patients, it is accepte that TOT oes not penetrate to bone ue to its mechanism of action (B). References: Kranke P, Bennett MH, Martyn-St James M, Schnabel A, Debus SE, Weibel S. Hyperbaric oxygen therapy for chronic wouns. Cochrane Database Syst Rev. 015;(6):CD00413. Moon H, Strauss MB, La SS, Miller SS. The valiity of transcutaneous oxygen measurements in preicting healing of iabetic foot ulcers. Undersea Hyperb Med. 016;43(6):641–648. Mutluoglu M, Cakkalkurt A, Uzun G, Aktas S. Topical oxygen for chronic wouns: a PRO/CON ebate. J Am Coll Clin Wound Spec. 013;5(3):61–65.
2. E. Diabetic foot ulcers are very common, with the lifetime risk of a patient with iabetes approximately 5% (A). Diabetic foot ulcers are mainly cause by peripheral neuropathy leaing to scrapes/cuts of the foot that may go unnotice for several ays (B). There is also autonomic neuropathy leaing to failure of sweating. This manifests as ry skin at risk for mechanical breakown which can initiate ulcer formation. Aitionally, autonomic ysregulation of the microcirculation results in poor ow to istal extremities preventing aequate woun healing. Preventative care is paramount in the prevention of iabetic foot ulcers an inclues maintaining normoglycemia an aily exams for occult scrapes/cuts of the foot along with aily moisturizer use. Diabetic foot ulcers shoul be manage with a combination of ebriement an off-loaing. Sharp ebriement, enzymatic ebriement, biological ebriement, an autolytic ebriement are all acceptable methos of removing ebris an necrotic tissue (C, D). Ofoaing is also critical to woun healing, with nonremovable total-contact casts being the gol stanar. Contrainications to this type of cast are ischemia, ongoing infection, osteomyelitis, an poor skin quality. References: Alexiaou K, Doupis J. Management of iabetic foot ulcers. Diabetes Ther. 01;3(1):4. Boulton AJM, Armstrong DG, Albert SF, et al. Comprehensive foot examination an risk assessment: a report of the task force of the foot care interest group of the American Diabetes Association, with enorsement by the American Association of Clinical Enocrinologists. Diabetes Care. 008;31(8):1679–1685.
3. B. A large boy of evience supports the utilization of
a moist woun environment to promote faster woun healing an less scar formation. Therefore, the woun ressing shoul create an ieal amount of moisture by either aing moisture to ry wouns or absorbing it from highly exuative wouns. Despite the progress mae in woun ressings, many outate strategies like wet-to-ry ressings are overly use. Wet-to-ry ressings, most often gauze, are rst allowe to ry on the woun an then are remove, resulting in nonselective ebriement of both slough an healthy tissue (C). Aitionally, moisture-retentive ressings like hyrocollois an transparent lms have a lower infection rate than gauze ressings (D). Some wouns, espite moistureretentive ressings, remain too ry an require supplementation. Hyrogels are useful in this case as they are hyrating to the woun be, but care shoul be taken not
to use in combination with enzymatic ebriing agents, as they may become inactive (E). Other wouns have a large amount of exuate which nees to be controlle in orer to prevent maceration an improve woun healing. Both foam ressings an alginate ressings, which can absorb 0 times their weight, can be use in this scenario (B). Enzymatic ebriing agents, like collagenase (e.g., Santyl) an meicalgrae honey, are also commonly use to remove slough an necrotic tissues (A). Reference: Niezgoa JA, Baranoski S, Ayello EA, et al. Woun treatment options. In: Baranoski S, Ayello EA, es.Wound care essentials. 5th e. Wolters Kluwer Health; 00:184–41.
4. C. All humans have three main types of cell junctions:
anchoring junctions, communicating (gap) junctions, an tight junctions. The rst group (anchoring junctions) is further subivie into esmosomes, hemiesmosomes, an aherens junctions (D). Hemiesmosomes an esmosomes both connect with intermeiate laments in the cytoskeleton, but the former connects cells to the unerlying extracellular matrix, an esmosomes connect ajacent cells to one another (A, E). Aherens junctions serve the same purpose but use actin laments as their cytoskeletal anchor. Anchoring junctions, as a whole, provie structural integrity to a tissue mae up of iniviual cells. Communicating junctions allow irect chemical communication between ajacent cells. This is facilitate by six iniviual subunits, calle connexins, which form a central pore, calle a connexon. When two connexons from ajacent cells come in contact, a channel is forme allowing communication between the two cells. The nal group, tight junctions, refers to a group of proteins that allow the selective iffusion of molecules base mainly on size, molecular charge, an polarity. These primarily act as selective barriers such as in the ifferent layers of the skin (B).
5. B. Delaye woun healing is a multifactorial problem
with many ientiable risk factors incluing malnutrition, vitamin eciencies, smoking, obesity, iabetes, an hypoxemia. However, few systemic factors have been shown to spee up woun healing. Short perios of starvation can have negative effects on postoperative woun healing (A). This seems to occur primarily by prolongation of the inammatory phase because there are inaequate builing blocks for cell proliferation, protein synthesis, an creation of new DNA. The notion that malnutrition plays a key role in the evelopment of chronic wouns le to multiple stuies aime at etermining if nutritional supplementation can prevent chronic wouns or spee recovery. A Cochrane review one in 014 looking at 3 ranomize controlle trials evaluating the effect of enteral an parenteral nutrition on the prevention an treatment of pressure ulcers foun no clear benet of any intervention (D). By knowing the halflives an current serum measurements of certain proteins, we are able to estimate the synthetic ability of the liver over a given time perio. Albumin (14–0 ays), transferrin (8–9 ays), an prealbumin (–3 ays) all give a snapshot into someone’s nutritional status but nee to be combine with the entire clinical picture (C). Granulation tissue, if present, is preictive of aequate woun healing (E). References: Greenel LJ, Mulhollan MW, es. Greeneld’s surgery: scientic principles & practice. 5th e. Lippincott Williams an Wilkins; 011.
CHAPtEr 37 Wound Healing
469
Langer G, Fink A. Nutritional interventions for preventing an treating pressure ulcers. Cochrane Database Syst Rev. 014;(6):CD00316. Stechmiller JK. Unerstaning the role of nutrition an woun healing. Nutr Clin Pract. 010;5(1):61–68.
Mangram AJ, Horan TC, Pearson ML, et al. Guieline for the prevention of surgical site infection. Infect Control Hosp Epidemiol. 1990;0:47–80.
6. C. A eciency of trace elements an essential fatty acis
hemostasis/inammation (combine in the 3-phase moel), proliferation, an maturation (or remoeling). The hemostasis/inammation phase is initiate with the isruption of capillaries resulting in hemorrhage. This immeiately causes vasoconstriction to assist with the formation of a platelet plug. After 10 to 15 minutes, local tissue factors an platelets begin to facilitate vasoilation an increase vascular permeability. The inltration of ui an cells (mainly neutrophils) causes the woun to become erythematous (A). In aition, the woun is warm an eematous (inuration) (E). At this point, changes in tissue pH an local tissue estruction cause the woun to be painful (B). The rst cells to arrive after formation of a platelet plug are neutrophils, which on’t seem to be critical to healing an mainly help with phagocytosis of bacteria an estruction of ea tissue. Neutrophil preominance persists for 48 hours, at which point they are largely replace by macrophages, which will remain in the woun until the completion of healing (D). Macrophages are arguably the most important cell in healing because of their effects on angiogenesis, matrix eposition, an remoeling via the release of cytokines an growth factors. By ay 4, the proliferative phase begins an enothelial cells an broblasts begin to appear in the woun. By ays 5 to 7, there is no longer a signicant population of inammatory cells. The previously create matrix of type III collagen is slowly replace with type I collagen, angiogenesis takes place, granulation tissue begins to form, an woun contraction commences. This phase persists for 3 to 4 weeks an nally gives way to the remoeling phase. At this point, vascularity ecreases an collagen is continuing to be synthesize, but it is being broken own at the same rate an collagen cross-linking occurs. References: Brunicari FC, Anersen DK, Billiar TR, Dunn DL,
is a relatively rare entity in patients taking foo by mouth. However, it has occurre with increase frequency with the avent an wiesprea use of TPN, particularly in patients with a history of short gut synrome. Copper is primarily associate with anemia resistant to iron supplementation, leukopenia, an neurologic efects (A). Vitamin C eciency, or scurvy, causes elaye woun healing, bleeing gums, loose teeth, an abnormal bone eposition in chilren (B). Selenium eciency is associate with a fatal cariomyopathy (E). Zinc an essential fatty aci eciency (linoleic aci an alpha-linolenic aci) have many similar features incluing elaye woun healing, increase infections, iarrhea, an a rash. However, the essential fatty aci rash tens to be scalier an is associate with ry skin, an the rash from zinc is primarily locate in the perioral area an intertriginous skin of the ngers an toes. While alopecia an thrombocytopenia can be foun with both conitions, it is more closely associate with free fatty aci eciency. Conversely, the impaire taste, night blinness, an loss of appetite are more closely relate with zinc eciency (D). References: Jeppesen PB, Høy CE, Mortensen PB. Essential fatty aci eciency in patients receiving home parenteral nutrition. Am J Clin Nutr. 1998;68(1):16–133. Kumar V, Fausto N, Abbas A, es. Robbins and Cotran pathologic basis of disease. 7th e. WB Sauners; 004. O’Leary JP, Tabuenca A, Capote LR. The physiologic basis of surgery. 4th e. Wolters Kluwer Health/Lippincott Williams & Wilkins; 008.
7. E. Surgical site infections have been shown to increase
the cost of hospitalizations an length of hospital stays prompting the Surgical Care Improvement Project (SCIP) to aress this major economic buren to moern health care. While preoperative bathing with antiseptic solution has been shown to ecrease bacterial colonization of skin, it has not been proven to be associate with ecrease rates of surgical site infections (C). Multiple preparations for preoperative skin antisepsis have been esigne; however, the two most commonly in use are ioine-base an chlorhexiine-base in either an aqueous or alcohol solution. Ioine works primarily by passing through the bacterial cell membrane an replacing intracellular ions with molecular ioine an oxiizing various structures within the bacterium (D). It is also, however, toxic to normal tissues, so it is generally combine with a carrier molecule (e.g., povione) to reuce the systemic availability of molecular ioine an reuce its toxicity. In contrast, chlorhexiine works by its ability to estabilize cellular membranes. A Cochrane review one in 015 comparing ioine-base an chlorhexiine-base preoperative antiseptic techniques foun the latter to be superior in preventing surgical site infections (A). However, it is generally not recommene for use above the chin because of ototoxicity an potential for causing amage to the cornea in higher concentrations (B). References: Dumville JC, McFarlane E, Ewars P, et al. Preoperative skin antiseptics for preventing surgical woun infections after clean surgery. Cochrane Database Syst Rev. 015;(4):CD003949.
8. C. Woun healing is typically ivie into 3 or 4 phases:
Hunter JG, Matthews JB, Pollock RE. es. Schwartz’s principles of surgery. 10th e. McGraw-Hill Eucation; 015. O’Leary JP, Tabuenca A, Capote LR. The physiologic basis of surgery. 4th e. Wolters Kluwer Health/Lippincott Williams & Wilkins; 008.
9. C. Hypertrophic scarring an keloi formation are both
examples of pathologic excessive healing. Both are cause by the increase eposition of collagen (A). Formation of kelois has a large genetic component that is inherite in an autosomal ominant fashion. It is also more prominent in arker-skinne iniviuals. Hypertrophic scarring is generally cause by a elay in woun healing or by excessive tensile forces on a new woun an is at a particularly high risk of forming after burns (D). They o not sprea outsie of the borers of the original woun, unlike kelois (E). Hypertrophic scars ten to recee with time, but if they persist, they ten to respon better to surgical excision as compare to kelois. Excision of kelois shoul be performe with caution, as they ten to reoccur an become bigger. If excision is planne, it shoul be accompanie by an ajunctive treatment such as sterois or low-ose raiation to prevent recurrence (B). Several other ajuncts have also been shown to reuce scarring incluing silicone banages, occlusive ressings, an extremity compression evices.
470
PArt ii Medical Knowledge
References: Gauglitz GG, Korting HC, Pavicic T, Ruzicka T, Jeschke MG. Hypertrophic scarring an kelois: Pathomechanisms an current an emerging treatment strategies. Mol Med. 011;17(1–):113–15. Greenel LJ, Mulhollan MW, es. Greeneld’s surgery: scientic principles & practice. 5th e. Lippincott Williams an Wilkins; 011. O’Leary JP, Tabuenca A, Capote LR. The physiologic basis of surgery. 4th e. Wolters Kluwer Health/Lippincott Williams & Wilkins; 008.
10. B. This patient most likely has scurvy cause by a e-
ciency in vitamin C an is uncommon in the moern age. It is typically seen in patients with severe malnutrition often from unerevelope countries without access to fresh fruits an vegetables. Patients present with loose or missing teeth, open sores, pigmente spots on the extremities, bleeing mucous membranes, vague myalgias, an fatigue. It is a key cofactor in the hyroxylation of lysine an proline uring collagen synthesis; as such, collagen cross-linking is extremely iminishe in patients with vitamin C eciency. It can even cause the involution of previous scars because remoeling continues, but patients are unable to synthesize new collagen. Vitamin C is also involve in iron absorption (C). Vitamin A is another essential vitamin in woun healing an assists with epithelialization, proteoglycan synthesis, an normal immune function (A). It has also been shown to reverse the effects of sterois on woun healing. Vitamin D is consume in the iet an prouce in the skin. It then unergoes activation (hyroxylation) by the liver an kiney to play an essential role in calcium metabolism (E). Exogenous vitamin E has been shown in animal trials to cause elaye woun healing via an inammatory mechanism similar to corticosterois (D). Reference: O’Leary JP, Tabuenca A, Capote LR. The physiologic basis of surgery. 4th e. Wolters Kluwer Health/Lippincott Williams & Wilkins; 008.
11. C. The most common types of collagen locate in the
boy inclue types I to V, though there are many more that are clinically relevant in certain iseases. Type I collagen makes up 90% of the boy’s collagen an is foun to some egree in most tissue, incluing skin, bones, tenons, arterial walls, an scars. It is implicate in isease like osteogenesis imperfecta (D). Type II collagen makes up about 50% of the protein in hyaline cartilage (carTWOlige). Type III collagen is foun in bone, cartilage, an multiple types of connective tissue, an abnormalities have been foun in Dupuytren contracture an the formation of aneurysms. Type IV collagen is foun primarily in the basement membrane (type four
is oor) an has been associate with Alport an Goopasture synrome (A). Type V collagen is closely associate with type I an is in most of the same tissues but with the aition of placental tissue. While there exist clinically signicant collagens outsie of these main ve, such as type VII (epiermolysis bullosa) an type XVII (bullous pemphigoi), they are not nearly as prevalent (E). Ehlers-Danlos is a spectrum of connective tissue isorers that can affect multiple types of collagen (B). However, the most common is type V (seen in classic type Ehlers-Danlos). Reference: De Paepe A, Malfait F. Bleeing an bruising in patients with Ehlers-Danlos synrome an other collagen vascular isorers: review. Br J Haematol. 004;17(5):491–500.
12. D. While healing of the gastrointestinal tract goes
through the same basic steps as healing of the skin, there are several key ifferences an unique features. Skin wouns unergo a relatively steay increase of the tensile strength of the woun over time. In contrast, the increase collagenase activity in the small bowel allows collagen breakown to excee collagen eposition on ays 3 to 5 after an anastomosis (B). This is why anastomotic leaks in the gastrointestinal tract occur with increase frequency in this critical time perio. However, the gastrointestinal tract is quicker to reach maximal tensile strength when compare with the skin. The submucosa provies most of the tensile strength for an anastomosis because of the coarse, interwoven bers that make it up. However, the mucosa an serosa are also important, an both help provie a quick, leakproof barrier over the rst several ays (C). One can appreciate this effect in action by noting the relatively higher leak rates with portions of the GI tract that lack serosa such as the esophagus. Multiple ajuncts an techniques have been trie to ecrease the rate of anastomotic leaks, an while there may be a tren towar fewer leaks with a staple anastomosis in certain circumstances, there still isn’t conclusive evience that one is superior to the other in all cases (A). While omental wrapping has been shown to improve outcomes in certain situations, a evitalize “omental free ap” will necrose an will not help with the anastomosis (E). References: Brunicari FC, Anersen DK, Billiar TR, Dunn DL, Hunter JG, Matthews JB, Pollock RE. es. Schwartz’s principles of surgery. 10th e. McGraw Hill Eucation; 015. Egorov VI, Schastlivtsev V, Turusov RA, Baranov AO. Participation of the intestinal layers in supplying of the mechanical strength of the intact an suture gut. Eur Surg Res. 00;34(6):45–431. Thornton FJ, Barbul A. Healing in the gastrointestinal tract. Neurosurg Clin N Am. 1997;77(3):549–573.
Conf idence is ClinicalKey Evidence-based answers, continually updated
The latest answers, always at your ngertips A subscription to ClinicalKey draws content from countless procedural videos, peer-reviewed journals, patient education materials, and boos authored by the most respected names in medicine
our patients trust you ou can trust ClinicalKey Euip yourself with trusted, current content that provides you with the clinical nowledge to improve patient outcomes
et to now ClinicalKey at storeclinicaleycom 2019v1.0